Vous êtes sur la page 1sur 320

8th_GRDM_00.

qxd:Layout 1 8/11/09 4:32 PM Page i

8TH GRADE
MATH REVIEW
8th_GRDM_00.qxd:Layout 1 8/11/09 4:32 PM Page ii

Similar Titles

Math Word Problems in 15 Minutes a Day


Algebra in 15 Minutes a Day
Basic Math in 15 Minutes a Day
Express Review Guides: Algebra I
Express Review Guides: Algebra II
Express Review Guides: Basic Math & Pre-Algebra
Express Review Guides: Fractions, Percentages & Decimals
Express Review Guides: Math Word Problems
8th_GRDM_00.qxd:Layout 1 8/11/09 4:32 PM Page iii

8TH
GRADE
MATH
REVIEW

NE W Y O RK
8th_GRDM_00.qxd:Layout 1 8/11/09 4:32 PM Page iv

Copyright © 2009 LearningExpress, LLC.

All rights reserved under International and Pan-American Copyright Conventions. Published in the United
States by LearningExpress, LLC, New York.

Library of Congress Cataloging-in-Publication Data:


8th grade math.
p. cm.
ISBN 978-1-57685-712-0
1. Arithmetic—Study and teaching (Elementary) 2. Mathematics—Study and teaching (Elementary)
3. Eighth grade (Education) I. LearningExpress (Organization) II. Title: Eighth grade math.
QA135.6.A14 2009
510—dc22
2009022760

Printed in the United States of America

9 8 7 6 5 4 3 2 1

First Edition

ISBN 978-1-57685-712-0

For information on LearningExpress, other LearningExpress products, or bulk sales, please write to us at:
2 Rector Street
26th Floor
New York, NY 10006

Or visit us at:
www.learnatest.com
8th_GRDM_00.qxd:Layout 1 8/11/09 4:32 PM Page v

Contents

HOW TO USE THIS BOOK ix

PRETEST 1

INTRODUCTION: MIND VERSUS CALCULATOR 17

SECTION 1 OPERATIONS ON NUMBERS 21

LESSON 1 Operations with Integers 23

LESSON 2 Operations with Fractions 33

LESSON 3 Decimals and Percentages 41

LESSON 4 Exponents 49

LESSON 5 Order of Operations 57

LESSON 6 Estimating and Checking 63

LESSON 7 Strategies 69

LESSON 8 Putting It All Together 75

v
8th_GRDM_00.qxd:Layout 1 8/11/09 4:32 PM Page vi

–CONTENTS–

SECTION 2 ALGEBRA 81

LESSON 9 Patterns, Variables, and Equations 83

LESSON 10 Introduction to Functions 91

LESSON 11 Operations with Polynomials 101

LESSON 12 Graphing Linear Equations and Inequalities 107

LESSON 13 Finding Solutions 117

LESSON 14 Estimating and Checking 125

LESSON 15 Strategies 131

LESSON 16 Putting It All Together 137

SECTION 3 GEOMETRY 145

LESSON 17 Angles and Triangles 147

LESSON 18 Polygons and Circles 159

LESSON 19 Volume and Similar Shapes 173

LESSON 20 Coordinate Geometry—Distances 183

LESSON 21 Estimating and Checking 191

LESSON 22 Strategies 197

LESSON 23 Putting It All Together 203

SECTION 4 MEASUREMENT 213

LESSON 24 The Metric and Customary Systems 215

LESSON 25 Estimating and Checking 225

LESSON 26 Strategies 231

LESSON 27 Putting It All Together 237

vi
8th_GRDM_00.qxd:Layout 1 8/11/09 4:32 PM Page vii

–CONTENTS–

SECTION 5 DATA AND PROBABILITY 243

LESSON 28 Data: Describing, Displaying, and Interpreting 245

LESSON 29 Probability, Permutations, and Combinations 255

LESSON 30 Estimating and Checking 263

LESSON 31 Strategies 269

LESSON 32 Putting It All Together 275

POSTTEST 283

GLOSSARY 301

vii
8th_GRDM_00.qxd:Layout 1 8/11/09 4:32 PM Page viii
8th_GRDM_00.qxd:Layout 1 8/11/09 4:32 PM Page ix

How to Use This Book

E ighth grade can be an exciting year full of changes and challenges. It’s also an important year academ-
ically. This year is your last chance to brush up your academic skills before high school. You’ll also be
required to take several standardized tests that measure your reading, writing, and math skills. Math
skills are of course necessary in a large number of occupations, and learning them helps you develop the ability
to think logically and abstractly. And because you’ll need to develop that ability for many of your high school
classes and most of your college classes, learning math skills is as important as learning to read and write. That’s
why almost half of the standardized tests you’ll be taking in the future will cover math—much of it the math you
learn in eighth grade.
Up to eighth grade, you’ve probably developed some skills in algebra and geometry, even if your teachers
didn’t use the words algebra and geometry. When you get to high school, you’ll have courses called Algebra and
Geometry; in fact, some eighth grade math courses are called “Introduction to Algebra.” But, regardless of the
name of your eighth grade math course, you’ll need to have a firm grasp of the skills in the basics of different
kinds of math. You’ll need to know how to add, subtract, multiply, and divide numbers—skills you’ve been
working on since first grade—and you’ll also have to know the general laws that govern these number opera-
tions. You’ll have to know how to recognize mathematical patterns and how to use them to solve problems; if
you’ve never heard the word algebra associated with this skill, you will in eighth grade. You’ll also have to recog-
nize shapes and know what kinds of properties different shapes have: This is part of geometry. And you’ll have to
know how to measure things in appropriate units. One final area that you’ll have to understand is collecting
data; you should also know how to display and interpret those data.
The lessons are divided into five sections. Each section focuses on one of the mathematical areas just
mentioned. The areas are outlined at the beginning of each section and reviewed at the end of the section in the

ix
8th_GRDM_00.qxd:Layout 1 8/11/09 4:32 PM Page x

–HOW TO USE THIS BOOK–

Putting It All Together lesson. Each section begins with a vocabulary list. Each of the words in the vocabulary
list will appear in boldface when it is first used in the section. If you’re not sure of the word’s meaning, look it
up in the Glossary at the back of the book. Then, use whatever memory devices work best for you to remember
the meaning. When you have finished with all the lessons in the section, you should go back to the beginning of
the section and make sure that you know all the words.
As you work through the lessons in this book, you will build critical thinking skills in five main areas of
mathematics: number operations, algebra, geometry, measurement, and data collection and interpretation.
Each of the 32 short lessons should take about 30 to 45 minutes to complete. Some might take longer, and some
might take less time, depending on how well you know the material. The beginning lessons of each section will
start with the basics, and the last three lessons will move into more complex problem-solving strategies. Even
though each lesson is an effective skill builder on its own, we recommend that you go through this book in or-
der, from Lesson 1 through Lesson 32—and don’t skip the Introduction. Each lesson builds on skills and ideas
discussed in the previous lessons within each section, and sometimes a lesson will depend on ideas learned in
previous sections. So, as you move through this book and your math knowledge increases, the practice problems
may become longer and more difficult.
Each lesson includes several exercises for you to practice the skills you have learned. To be sure you’re on
the right track, you’ll find answers and explanations for the practice questions—but first try to solve the prob-
lems without looking at the answers.
This book also includes a pretest and a posttest. To help you measure your progress, take the pretest before
you begin Lesson 1. The pretest will give you a sense of your strengths and weaknesses so you can focus on spe-
cific sections. After you finish the lessons, take the posttest. You’ll be able to see how much your math skills have
improved. You’ll also be able to find out if there are areas in which you still need practice.
Will you know everything you need to know for eighth grade math after you finish this book? No! You’ll
still have to pay attention in class and work lots of problems. But you’ll have a very good foundation for moving
ahead and preparing yourself for math—and all the courses that use math—in high school and college.

x
8th_GRDM_01.qxd:Layout 1 8/11/09 12:46 PM Page 1

Pretest

B efore you begin, find out how much you already know about math—and how much you need to learn
by taking this pretest. These 37 multiple-choice questions and three open-ended assignments cover all
of the topics in this book. If your score is high, you might move through this book more quickly than
you expected. If your score is low, you may need more than 30 to 45 minutes to get through each lesson. On the
following page there is an answer sheet, or you can just circle the correct answers. If you don’t own this book,
write the numbers 1–37 on one lined sheet of paper, and write your answers next to the numbers. Then answer
38–40 in the space given or on separate paper. Take as much time as you need for this test. Then use the answer
key at the end of the test to check your answers. The key tells you which lesson covers the strategy in that ques-
tion. Good luck!

1
8th_GRDM_01.qxd:Layout 1 8/11/09 12:46 PM Page 2
8th_GRDM_01.qxd:Layout 1 8/11/09 12:46 PM Page 3

–LEARNINGEXPRESS ANSWER SHEET –

1. a b c d 15. a b c d 29. a b c d
2. a b c d 16. a b c d 30. a b c d
3. a b c d 17. a b c d 31. a b c d
4. a b c d 18. a b c d 32. a b c d

5. a b c d 19. a b c d 33. a b c d

6. a b c d 20. a b c d 34. a b c d
7. a b c d 21. a b c d 35. a b c d
8. a b c d 22. a b c d 36. a b c d
9. a b c d 23. a b c d 37. a b c d
10. a b c d 24. a b c d
11. a b c d 25. a b c d
12. a b c d 26. a b c d
13. a b c d 27. a b c d
14. a b c d 28. a b c d

3
8th_GRDM_01.qxd:Layout 1 8/11/09 12:46 PM Page 4
8th_GRDM_01.qxd:Layout 1 8/11/09 12:46 PM Page 5

–PRETEST–

Section 1: 5. Mary is driving from Los Angeles to San


Multiple-Choice Questions Francisco at an average speed of 65 miles per
hour. She left Los Angeles at 2:15 P.M. and will
1. Evaluate: (24) (4–3) travel a total distance of 390 miles. She also
a. 14 took a 45-minute break from driving for
b. – 43 dinner and fueling her car. At what time will
c. 23 Mary arrive at her destination?
d. – 14 a. 8:45 P.M.
b. 9:00 P.M.
2. The 3 600 falls between which two consecutive c. 9:30 P.M.
integers? d. 9:45 P.M.
a. 6 and 7
b. 7 and 8 6. The diameter of this circle is 12 inches.
c. 8 and 9 Which is closest to the area of the circle?
d. 9 and 10 (Use 3.14 for pi.)

3. Determine the value of x if (5x – 4)/2 > 7 + x.


a. x < 6
b. x > 6
c. x > 4
d. x < 4 12 in.

4. A, B, and C are three cities that form a right


triangle. If the distance from A to B is 10 miles
and the distance from B to C is 24 miles, what’s
the distance from A to C in miles? a. 19 square inches
A b. 38 square inches
c. 113 square inches
x
d. 452 square inches
10

7. Solve for x in the following equation:


B 24 C
(2x3 ) – 5 = –1
a. 9
a. 34 b. 6
b. 25 c. 1
c. 26 d. –6
d. 14

5
8th_GRDM_01.qxd:Layout 1 8/11/09 12:46 PM Page 6

–PRETEST–

8. A party hat in the shape of a cone has a radius 11. Last year, 60 students from the eighth grade
of 4 inches, and its height is 15 inches. What is attended the class trip to the local science
the approximate volume of this hat? (Use 3.14 museum. This year, 69 students made the same
for pi.) trip. What is the percentage increase in the
a. 188 cubic inches number of students who went to the science
b. 251 cubic inches museum?
c. 50 cubic inches a. 9%
d. 754 cubic inches b. 12%
c. 18%
9. A line passes through the points (4,7) and d. 15%
(6,2). What is the slope of this line?
a. 25 12. Alan has received an average grade of 85 on
b. – 25 four math exams so far this semester. If he
c. 25 scores a 60 on the fifth and final exam of the
d. – 25 semester, what will his average grade be for the
semester?
10. Triangle ABC is similar to Triangle XYZ. What a. 70
is the difference between the length of XZ and b. 80
the length of YZ? c. 75
d. 85
X

A 13. Juan has a fair six-sided die. If Juan rolls the die
twice in a row, what is the probability that the
die will show a 3 both times?
16 cm a. 61
1
b. 12
12 cm 12 cm
c. 91
1
d. 36

B 9 cm C Y Z

a. 8 centimeters
b. 3 centimeters
c. 4 centimeters
d. 6 centimeters

6
8th_GRDM_01.qxd:Layout 1 8/11/09 12:46 PM Page 7

–PRETEST–

14. Which equation represents the relationship a. The survey should have been repeated
between the values of x and y in the following every week throughout the semester.
table? b. The students who participated in the
survey may have lied about their favorite
x y sports.
c. The sample is not representative of the
0 –3
student body; participants should have
1 –1 been chosen at random among all the
students at the high school.
2 5
d. Some students might like more than one
3 15 sport.
4 29
16. Which property allows us to determine that the
a. y = 2x – 3 sum of the expression 3x + 4y + 5z is
b. y = 3x2 – 4 equivalent to the sum of the expression 5z + 4y
c. y = 2x2 – 3 + 3x?
d. y=x+3 a. Associative property
b. Commutative property
15. Jason conducted a survey to find out the most c. Additive identity property
popular sport among the students at his high d. Distributive property
school. He asked each of the 50 members of the
high school football team what their favorite 17. Quadrilateral WXYZ is an isosceles trapezoid.
sport was. The results are shown in the If the length of WX = 16 cm and the length of
following table: YZ = 28 cm, and the area of the trapezoid is
176 square centimeters, then what is the length
SPORT NUMBER OF STUDENTS
of h?
W 16 cm X
Football 40

Baseball 3
h
Basketball 2

Hockey 2 Y 28 cm Z

Soccer 3 a. h = 10 centimeters
b. h = 8 centimeters
Looking at the results, Jason concluded that c. h = 6 centimeters
football was by far the most popular sport at d. h = 12 centimeters
his high school. Which statement best
describes the reason why his conclusion may be
incorrect?

7
8th_GRDM_01.qxd:Layout 1 8/11/09 12:46 PM Page 8

–PRETEST–

18. What is the mode of the following data 21. The circumference of Earth at the equator is
sample? approximately 24,902 miles. If Superman were
{1, 5, 8, 3, 1, 8, 8, 3, 6, 8, 2, 3, 4, 8, 7, 8, 1, 8} to travel around Earth twice at the equator,
a. 1 which of the following would be the correct
b. 3 representation of the total distance he would
c. 5 have traveled, in scientific notation?
d. 8 a. 4.9804 × 104 miles
b. 2.4902 × 105 miles
19. The water tower in the town of Watertown can c. 2.4902 × 108 miles
hold 12 million milliliters (mL) of water when d. 49,804 × 10–4 miles
filled to capacity. How many liters would this
be when expressed in scientific notation? 22. A rectangular box is 50 centimeters long, 20
a. 12 × 104 liters centimeters wide, and 30 centimeters high.
b. 12,000,000 × 10–3 liters What is the volume of the box in cubic meters?
c. 1.2 × 104 liters a. 300 m3
d. 1.2 × 107 liters b. 3 m3
c. 0.3 m3
20. Shelley and her sister decided to buy a DVD d. 0.03 m3
together, each paying half of the price. On
Wednesday, the DVD had a sticker price of $18, 23. Solve for the value of m in the following
which had been discounted 25% of the original equation.
6
price. However, the two sisters couldn’t buy it m = 0.24
that day and had to return to the store on a. .4
Saturday. The discount had ended and the b. 2.5
DVD had returned to its original sticker price. c. 4
The sisters decided to buy it anyway and paid d. 25
8.25% sales tax on top of the sticker price. How
much did each sister pay toward the cost of the 24. What number would you get if you added 3 to
DVD? the absolute value of –9 and then divided the
a. $9 resulting number by 2?
b. $9.75 a. –3
c. $13 b. –6
d. $26 c. 3
d. 6

25. There are 35 frogs in a pond. If the number of


frogs in the pond tripled over the next month,
how many frogs would there be in the pond at
the end of the month?
a. 65
b. 36
c. 310
d. 610
8
8th_GRDM_01.qxd:Layout 1 8/11/09 12:46 PM Page 9

–PRETEST–

26. The volume of the cylindrical can shown here 29. Melissa’s Bicycle Shop sold 50 bikes in April,
is 960 cubic inches, and its height is 12 inches. and Melissa recorded the following informa-
What is the area of the base of the can? (Use tion about the prices of the bicycles she sold:
3.14 for pi.)
APRIL SALES

Mode $120

Median $125

h = 12 inches Mean $130

Range $100

How much did Melissa receive from bicycle


sales in April?
r
a. $6,000
a. 80 square inches b. $6,250
b. 16 square inches c. $6,500
c. 160 square inches d. $5,000
d. 300 square inches
30. David has a bag with nine ping-pong balls in it,
27. John owns a music and video game store. Each and each ball has a number (1 through 9)
music CD costs $10, and each video game costs written on it, with no number being repeated.
$15. If John sold a total of 80 CDs and video He selects a ball out of the bag at random, and
games and earned $925 last week, how many then, without replacing the first ball, he picks
music CDs did he sell? out a second ball. What is the probability that
a. 55 both of the balls David picks out of the bag will
b. 40 have even numbers on them?
c. 25 a. 21
d. 30 b. 41
c. 61
28. Which choice is the best estimate of the d. 81
product?
36.083542 × 6.98379333 31. The graph of a line contains the points (6,3)
a. 180 and (1,7). Which of the following must be true
b. 216 about the graph of this line?
c. 300 a. The slope of the line is negative.
d. 252 b. The slope of the line is positive.
c. The line intersects the x-axis.
d. The line intersects the y-axis.

9
8th_GRDM_01.qxd:Layout 1 8/11/09 12:46 PM Page 10

–PRETEST–

32. Which of the following is NOT a true statement? 35. The ratio of right-handed students to left-
a. |–7| – |–3| = 4 handed students in Michael’s eighth grade class
b. |–7| – (|–1| + 4) = 5 – |–3| is 4 to 1. If there are a total of 120 students in
c. |–5| + (2 – |–8|) = –|–1| the class and 14 of them are left-handed girls,
d. 4 – (–|–7| + |–4|) = –5 how many of them are left-handed boys?
a. 10
33. This scatter plot shows the relationship b. 16
between temperature and the number of ice c. 12
creams sold. Which statement best describes d. 8
this relationship according to the data in this
graph? 36. The table shows the favorite animals of the
students in Carol’s eighth grade class at George
Ice Cream Sales
200 Washington Junior High School.
Ice creams sold

150
(thousands)

NUMBER OF
100 FAVORITE ANIMAL STUDENTS
50
Cat 29
0
0 20 40 60 80 100 120 Dog 35
Temperature (Fahrenheit)
Lion 4

a. As temperature increases, sales of ice cream Monkey 2


decrease.
Goldfish 15
b. As temperature decreases, sales of ice cream
increase. Other 15
c. Ice cream sales are not affected by changes
Total 100
in temperature.
d. As temperature increases, sales of ice cream
Which graph accurately represents the data in
increase.
the table?
34. ∠1 and ∠2 are complementary angles. If the
measurement of ∠1 is 5 times the measure-
ment of ∠2, what is the measurement of ∠2?
a. 18°
b. 15°
c. 36°
d. 30°

10
8th_GRDM_01.qxd:Layout 1 8/11/09 12:46 PM Page 11

–PRETEST–

a. Cat Section 2:
Dog Short-Response Questions
Lion

Monkey
Directions: In the space below each problem, show
Goldfish
how you worked the problem. You need not write
Other
complete sentences, but you should show the steps
b. Students’ Favorite Animals you take to solve the problem. Your solution should
Number of students

40
30 include a drawing or diagram if you used one. Be sure
20
that your final answer and all the steps are clear.
10

0
Cat Dog Lion Monkey Goldfish Other
TYPICAL SCORING RUBRIC
Animal Type
SCORE LEVEL DESCRIPTION
c. Cat

Dog
0 No answer is given/The answer
given is incorrect in terms of
Lion strategy and result.
Monkey
Goldfish 1 The answer given is partially
correct.
Other

2 The answer given is completely


d. correct/The strategy used in
Students’ Favorite Animals
Number of students

40 solving the problem is clearly


30 explained, the work is shown,
20 and the result is accurate.
10

0
Cat Dog Lion Monkey Goldfish Other 38. Joe, Jim, Jack, and Jeff went out to dinner
Animal Type
together, and their final bill was $120,
including tax and tip. Joe paid 61 of the bill, Jim
37. The rectangle shown here has a perimeter of 34 paid 32 times as much as Joe, Jack paid 54 of the
cm, and its area is 60 cm2. Which of the combined amount paid by Joe and Jim, and
following can be the length and height of this Jeff paid the rest of the bill. What was the
rectangle in centimeters? proportion of the amount Jeff paid to the total
amount of the bill?

a. x = 30, y = 4
b. x = 12, y = 10
c. x = 10, y = 6
d. x = 12, y = 5

11
8th_GRDM_01.qxd:Layout 1 8/11/09 12:46 PM Page 12

–PRETEST–

39. ∠A and ∠B are complementary angles. ∠C Section 3:


and ∠D are supplementary angles. Solve for x Extended-Response Question
in the following equation:
∠C + ∠D Directions: Answer the questions in the following
= ∠A + ∠B
3 + 2x problem. Be sure to tell the information you are given
that will help you answer the questions, and how you
propose to get the answers. Describe all the steps in
getting there. Include pictures or diagrams if they
help you get the answer or help you at any stage. Be
sure all drawings and mathematical operations are
shown clearly and in the right order.

TYPICAL SCORING RUBRIC


SCORE LEVEL MATHEMATICAL KNOWLEDGE, STRATEGIC KNOWLEDGE, AND EXPLANATION

0 No answer is given. There is no clear strategy provided for the answer. No attempt is made to
give an explanation of the solution process.

1 Shows little or limited understanding of the mathematical concepts involved. Strategy used to
answer is wrong, irrelevant, or inappropriate for the question. Gives minimal explanation of
the solution process and does not adequately explain the steps taken.

2 Shows some understanding of the mathematical concepts and principles relevant to the
question. Some evidence of a strategy to solve the problem is presented. An explanation of
the solution process is given, but it is inadequate or does not match the solution of the
problem; it may also be vague or difficult to understand.

3 Shows a high level of understanding of the mathematical concepts and principles relevant to
the question. The student successfully identifies most of the important elements of the
problem and the strategy presented to solve the problem is almost comprehensive. The
student gives a nearly complete explanation of the solution process, clearly explains what was
done, and addresses the question of why it was done to some extent.

4 Shows full understanding of the mathematical concepts and principles relevant to the
question. The student successfully identifies all of the important elements of the problem, and
the strategy presented to solve the problem is comprehensive. The student gives a complete
explanation of the solution process and clearly explains what was done and why it was done.

12
8th_GRDM_01.qxd:Layout 1 8/11/09 12:46 PM Page 13

–PRETEST–

40. Nancy’s cell phone service provider charges her phone for at least 500 minutes each month, find
$30 per month for the first 300 minutes of air the equation that would be used to accurately
time, and 35 cents for each additional minute calculate the dollar amount of her cell phone
each month. Nancy also pays a fixed monthly bill, t, in any given month. (Assume that n is the
fee of $2 for text messages (regardless of usage number of minutes she uses that month.)
level) and an 8% tax, which is applied to the What would her cell phone bill be if she
total amount of her bill, including the text spoke on the phone for 700 minutes last
message fee. Assuming that Nancy talks on the month? (Round to the nearest whole number.)

13
8th_GRDM_01.qxd:Layout 1 8/11/09 12:46 PM Page 14

–PRETEST–

Answers

If you missed any of the questions, you can find help with that kind of question in the lesson(s) shown to the
right of the answer.

QUESTION ANSWER LESSON QUESTION ANSWER LESSON

1 a 4 20 c 7

2 c 6 21 a 4, 6

3 b 12 22 d 19, 24

4 c 20 23 d 10, 14

5 b 7 24 d 5

6 c 18 25 b 4

7 b 10 26 a 15

8 b 19 27 a 12

9 d 11 28 d 6

10 c 22 29 c 31

11 d 3 30 c 30

12 b 31 31 a 11

13 d 29 32 d 5

14 c 9 33 d 29

15 c 28 34 b 22

16 b 0 35 a 7

17 b 18 36 a 28

18 d 28 37 d 22

19 c 4, 25

14
8th_GRDM_01.qxd:Layout 1 8/11/09 12:46 PM Page 15

–PRETEST–

Short Answer 39. Solution: To solve the problem, we need to


38. Solution: To find the amount that Jeff paid, we know and use the definitions of complemen-
need to first calculate the amount that each of tary and supplementary angles. Two angles are
the other diners in the group paid, using our complementary if they add up to 90°, and two
knowledge of fractions and proportions. angles are supplementary if they add up to
180°. Substituting these numbers into the
Joe paid 61 of the bill, which is 61 × $120 =
equation, we get
$20. 180
3 + 2x = 90
Jason paid 23 as much as Joe, which is 23 × Solving for x:
$20 = $30. 60 + 2x = 90
Jack paid 54 of the combined amount paid 2x = 30
by Joe and Jason. The combined amount x = 302
paid by Joe and Jason is $20 + $30 = $50. x = 15
So, Jack paid 54 × $50 = $40. Thus, the correct answer is 15°.

To find what Jeff paid, we need to first add the


To the Grader
amounts that the three other friends paid:
Scoring: If the student gives no answer or fails to pro-
$20 + $30 + $40 = $90. Subtracting this
vide a strategy for solving the problem (or if the strat-
amount from the total bill gives us the portion
egy provided is completely wrong/irrelevant), the
paid by Jeff: $120 – $90 = $30.
corresponding score on the rubric should be 0. If the
Finally, we need to convert this amount to a student shows an understanding of supplementary
$30
fraction: $120 = 41 . The correct answer is 41 , and complementary angles and provides a strategy
which is 25%. for solving the problem, but applies the strategy in-
correctly or makes a computation error to arrive at a
To the Grader wrong answer, the score given should be 1. If the stu-
Scoring: If the student gives no answer or fails to pro- dent shows an understanding of supplementary and
vide a strategy for solving the problem (or if the strat- complementary angles, explains the strategy clearly,
egy provided is completely wrong/irrelevant), the and arrives at the correct result by applying the strat-
corresponding score on the rubric should be 0. If the egy to the given question, the answer should be
student shows an understanding of fractions and scored 2 according to the rubric.
proportions and provides a strategy for solving the
problem, but applies the strategy incorrectly or makes
a computation error to arrive at a wrong answer, the
score given should be 1. If the student shows an un-
derstanding of fractions and proportions, explains
the strategy clearly, and arrives at the correct result by
applying the strategy to the given question, the an-
swer should be scored 2 according to the rubric.

15
8th_GRDM_01.qxd:Layout 1 8/11/09 12:46 PM Page 16

–PRETEST–

Extended Response Thus, the final equation for the cell phone
40. Solution: Begin by identifying the elements of bill is:
the problem and showing an understanding of
the relationships among the elements. There t = {[(n – 300) × 0.35] + 30 + 2} × 1.08
are two unknown variables in the equation: t,
which is the total amount of the cell phone bill, which can be simplified to:
and n, the number of minutes used in a given
month. There are also fixed elements in the t = {[(n – 300) × 0.35] + 32} × 1.08
equation: the $30 base monthly fee for the
first 300 minutes, the $2 text message fee, Plugging n = 700 into this equation, we find
the 35 cents/minute charge for air time beyond that Nancy’s cell phone bill would be $185.76.
300 minutes, and the 8% tax rate. To find the Rounded to the nearest whole number, the an-
solution, add all of these elements up in the swer is $186.
appropriate manner in order to arrive at
the total cell phone bill for any given month. To the Grader
If the student correctly identifies the formula for cal-
■ First, find the number of excess minutes culating the cell phone bill but makes an arithmetic
used by Nancy in a given month. This would mistake in the calculation of the bill in the month in
be n – 300, where n is the number of min- which Nancy used up 700 minutes, 1 point should be
utes used. Note that the information given deducted.
in the question that “Nancy talks on the Other computation errors that scorers should
phone for at least 500 minutes each month” watch out for include:
is relevant only to the extent that it lets us
know that n will be greater than 300. Since ■ Ignoring the tax completely: t = [(n – 500) × 0.35]
the threshold for the extra usage charge is + 32
300 minutes, the number 500 is not itself ■ Applying the tax rate before adding the text mes-
relevant and will not be a part of the correct sage fee: t = {[(n – 300) × 0.35] + 30} × 1.08 + 2
solution. ■ Using the 500-minute figure as the threshold
■ Then, mutiply the number of excess min- instead of the appropriate 300-minute figure:
utes by 0.35 (35 cents expressed in dollars) t = {[(n – 500) × 0.35] + 32} × 1.08. If a student
to find the usage surcharge for a given uses n – 500 instead of n – 300 in the equation, 1
month. Thus, we get (n – 300) × 0.35. point should be deducted.
■ Then, add the $30 base monthly fee and the
$2 text messaging fee to this number, and
we arrive at [(n – 300) × 0.35] + 30 + 2.
■ Finally, apply the 8% tax rate to this amount
by multiplying it by 1.08.

16
8th_GRDM_01.qxd:Layout 1 8/11/09 12:46 PM Page 17

Introduction:
Mind versus Calculator

I f you’re a typical eighth grade student, you’ve already been using a calculator for several years. Calculators
are great labor-saving devices, especially when you want precise answers on calculations with large numbers
or with numbers that have lots of digits, such as 14.64563 and .035. In fact, you’ll need a calculator to work
many of the problems in this book. But sometimes it’s better not to use a calculator.

■ When you’re doing basic operations with integers (whole numbers), it’s a little faster to do them in your
head, and, if you know your number facts well, it’s also more accurate. (You can’t make a typing error in
your head.)
Example: 132 – 109. You have to hit 8 keys (1, 3, 2, –, 1, 0, 9, =), in the correct order, to get the right
answer. You’ll do a lot better in eighth grade math and beyond if you can do calculations like this in
your head.
■ When you’re learning new procedures that involve several steps, you don’t want to be punching numbers
into a calculator at any point in the procedure, because then it’s harder to learn the steps.
Example: You’re just learning how to manipulate simultaneous linear equations to solve them
(Lesson 13). You have the following equations: 2x + 5y = 17 and 19x – 41y = –15.5. For reasons you’ll
understand later, you have to multiply every term in the first equation by 19 and every term in the
second by –2. If you do your calculations correctly, you’ll get two new equations: 38x + 95y = 323
and –38x + 82y = 31. If you have to punch calculator keys to do these calculations, you’ll lose the

17
8th_GRDM_01.qxd:Layout 1 8/11/09 12:46 PM Page 18

–INTRODUCTION: MIND VERSUS CALCULATOR–

flow of the procedure, and it may take you Now we still have to subtract 17 from 340. One
a lot longer to learn the procedure. (You’ll way to do this is to first subtract 10 and then subtract
find the solution to this problem in 7. Some people find it easier to subtract 20 and then
Lesson 13.) add 3 back (20 – 3 = 17). Some people even find it
easier to visualize a column of subtraction with the
So you might agree that it’s easier and faster to borrowing procedure you probably learned in second
do some calculations in your head, but you might grade. You just need to find a method that’s easy for
also be thinking that you had enough trouble learn- you and stick with it.
ing your 12 times tables and if you have to learn 19 Here are some calculations to try. Do them first
times tables your head might explode. Well, you’re in in your head and then check them with a calculator:
luck: You don’t need to memorize your 19 times tables
to multiply 19 times 2, or 19 times 5, or even 19 times 43 × 7 = ? 268 – 149 = ? 268 – 49 = ?
17. You just have to do some substitution: 18 × 19 = ? 83 – 66 = ? 101 – 83 = ?

Just think of 19 as (20 – 1). Then, 2 × 19 = 2 ×


(20 – 1) = 40 – 2 = 38. (This is possible because Estimating
of the distributive property of multiplication,
one of the properties of numbers discussed Even when you get very good at calculating in your
later.) And 5 × 19 = 5 × (20 – 1) = 100 – 5 = 95. mind, you’ll still make mistakes. So you ought to check
Then, 17 × (20 – 1) = 340 – 17 = 323. every calculation you do, if the problem you’re doing is
important—as it is when you’re taking a test or figur-
Now maybe you’re wondering how you’re sup- ing out how much you can buy with a certain amount
posed to know that 340 – 323 is 17, or even that 17 × 20 of money. Even before you check, though, you should
is 340. Let’s start with 17 × 20. You know that 20 is the be able to know approximately what the right answer is.
same as 2 × 10, so one way to do it is Let’s take the calculation 2 × 15.5 = 31. You know that 2
× 15 = 30 and that 2 × 16 = 32, so 31 seems reasonable.
17 × 20 = 17 × (2 × 10) Then, if you check your answer with a calculator and
= 17 × 2 × 10 miss the decimal point so that you get an answer of
= (20 – 3) × 2 × 10 310, you know that you should do the calculation with
= (40 – 6) × 10 the calculator over again—more carefully this time.
= 34 × 10 And even when it’s necessary to use a calculator
= 340 for your first calculation, you need a good estimate, be-
cause there’s no guarantee that you’re going to hit the
But an even easier way to do it is 17(10) + 17(10) = right keys. Let’s use the numbers 14.64563 and .035.
170 + 170 = 340—easier if you know what 17 + 17 is, Can you estimate 14.64563 × .035? Will it be bigger
of course. than 15 or smaller than 15? If you said smaller, give
yourself a pat on the back: If you multiply any positive
number by a positive number that’s less than 1, you
Math is a lot easier if you know your 2 times ta- will get a smaller number. And what if you divide any
bles up to at least 20: 2 × 15 = 30, 2 × 16 = 32, positive number by a positive number that’s less than
2 × 17 = 34, 2 × 18 = 36, and 2 × 19 = 38. 1? That’s right, you’ll get a larger number. Now let’s go
back to estimating 14.64563 × 0.035. We’ll round the
first number to its nearest whole value, which is 15.
18
8th_GRDM_01.qxd:Layout 1 8/11/09 12:46 PM Page 19

–INTRODUCTION: MIND VERSUS CALCULATOR–

Then, notice that 35 is pretty close to 33, and 33% Properties of Numbers
(0.33) is about one-third. And one-third of 15 is 5. But
0.035 is not the same as 0.33—it’s just one-tenth of .33. Here are some properties of numbers. You need to
(See Lesson 3 if you don’t know why.) And one-tenth know these to do most math from here on out. You
of 5 is .1 × 5 = 105 = 0.5. In fact, the answer worked on may already know these, but you may not know their
the calculator is 0.51259705, which is very close to 0.5. names. And you can be pretty sure that any math
course you take beyond eighth grade will ask you to
know the names, so you might as well learn them
Math is much easier if you know that a positive
now.
number multiplied by a number smaller than 1
gets smaller and a positive number divided by
a number smaller than 1 gets larger.
COMMUTATIVE PROPERTY OF ADDITION

a+b=b+a
Now let’s divide 14.64563 by 0.035. First, we
know we’ll get a number that’s bigger than 14—but
how much bigger? Remember that 0.035 is about COMMUTATIVE PROPERTY
one-tenth of one-third, and 13 × 101 1
= 30 . So if we divide OF MULTIPLICATION
1
by 30 , it’s the same as multiplying by 30. (See Lesson 2 a×b=b×a
if you don’t know why.) Again rounding 14.64563 to
15, we can get a good estimate of 450. The answer, us-
ing the calculator, is 418.4, rounded to the nearest ASSOCIATIVE PROPERTY OF ADDITION
tenth. This is in the same ballpark as 450, and because
a + (b + c) = (a + b) + c
we rounded up to 15 and we rounded down to .033,
our answer should be somewhat lower than 450, and
we have a reasonable answer at 418.4.
ASSOCIATIVE PROPERTY OF MULTIPLICATION

Exercise 1 a × (b × c) = (a × b) × c

Try to give reasonable estimates for the following cal-


culations (without a calculator):
DISTRIBUTIVE PROPERTY
1. 0.46 × 1.7777 = ? a × (b + c) = a × b + a × c

a × (b – c) = (a × b) – (a × c)
2. 4.28697 ÷ 15.13 = ?

3. 0.002 × 2.965 = ?

4. 12.1 ÷ 0.9878 = ?

5. 179.6 × 0.155 = ?

6. 1.43 = ?

19
8th_GRDM_01.qxd:Layout 1 8/11/09 12:46 PM Page 20

–INTRODUCTION: MIND VERSUS CALCULATOR–

1
Answers 5. 179.6 × 0.155 ≈ 180 × 0.15 ≈ 180 × 1.5 × 10 ≈ 27
Exact answer: 27.838
Exercise 1 Note: It’s easy to do multiplication of round
1. 0.46 × 1.7777 ≈ 0.5 × 1.8 ≈ 0.9 numbers by 1.5: Just multiply by 1 (1 × 180 =
Exact answer: 0.817742 180) and by 12 : (12 × 180 = 90) and add the
4
2. 4.28697 ÷ 15.13 ≈ 4.3 ÷ 15.1 ≈ 16 ≈ 0.25 answers (180 + 90 = 270).
Exact answer: 0.2833 6. 1.43 = 1.4 × 1.4 × 1.4 ≈ 1.96 × 1.5 ≈ 3
2 6
3. 0.002 × 2.965 ≈ 1000 × 3 ≈ 1000 ≈ 0.006 Exact answer: 2.744
Exact answer: 0.00593
4. 12.1 ÷ 0.9878 ≈ 12 ÷ 1 ≈ 12
Exact answer: 12.2494

20
8th_GRDM_02.qxd:Layout 1 8/11/09 12:53 PM Page 21

1
S E C T I O N

OPERATIONS
ON NUMBERS

M ath is a subject where you have to build on what you already know. You lay down a firm founda-
tion, and then you build on it. Probably the first thing you learned in math was how to count,
and then how to add and subtract whole, positive numbers. Then you learned how to multiply
and divide whole, positive numbers. Then you went on to operations on fractions and decimals, then to nega-
tive numbers. By eighth grade, you’re expected to know how to do operations on numbers fluently, just as you’d
be expected to know how to speak a language fluently before you could be an interpreter in a country where
people speak that language.
By the end of this section, you should be fluent with these skills:

■ Addition, subtraction, multiplication, and division of positive and negative integers


■ Addition, subtraction, multiplication, and division of fractions, or rational numbers
■ Addition, subtraction, multiplication, and division using decimals and percentages

21
8th_GRDM_02.qxd:Layout 1 8/11/09 12:53 PM Page 22

–OPERATIONS ON NUMBERS–

■ Operations with positive and negative exponents


■ Order of operations
■ Strategies for solving problems with operations on numbers

WORDS YOU SHOULD KNOW

absolute value exponent order of magnitude

algorithm greatest common factor order of operations

approximately equal to (≈) improper fraction percent (%)

canceling integer prime number

composite number interest quotient

decimal irrational number radical

dividend least common multiple ratio (“:”or “to”)

divisor mixed number rational number

elapsed time negative exponent reciprocal

equivalent fraction negative number scientific notation

22
8th_GRDM_02.qxd:Layout 1 8/11/09 12:53 PM Page 23

1
L E S S O N

OPERATIONS
WITH INTEGERS

L E S S O N S U M M A RY
To do the more complex operations of geometry and algebra, you
need to be able to do the basic number operations on integers,
the whole numbers that you probably started working with in
kindergarten.

I n this lesson, you will discover the ins and outs of integers. Sure, you’ve been working with these whole
numbers since you started school, but now’s the time you really need to know how to work with them. We’ll
start out by reviewing the basic operations with integers—addition, subtraction, multiplication, and divi-
sion—and then we’ll cover how to factor whole numbers and how to do the basic operations with negative
numbers.

Addition, Subtraction, Multiplication, Division

When you started out with addition and subtraction, you probably counted on your fingers to get answers to
simple problems like 4 + 5 = 9 or 9 – 4 = 5. When you got to problems that involved numbers higher than 10,
you had to learn other ways to do the problems. Maybe you still used your fingers, but then you had to keep
track of how many times you had gone through a cycle of your own fingers, which isn’t always easy and opens

23
8th_GRDM_02.qxd:Layout 1 8/11/09 12:53 PM Page 24

–OPERATIONS WITH INTEGERS–

up the possibility of making mistakes. So you had to (1 × 1), you add 1 to the 10’s column to get 4 + 1 + 1
learn other algorithms for adding and subtracting. = 6. Or you can just add 11 + 50 = 61. Because 47 has
Maybe these methods were called “carrying,” “bor- no 100’s, 1,000’s, or 10,000’s, your answer is 35,661.
rowing,” or “regrouping” by your teachers, but they This is an easy addition problem, and there’s no need
all required you to be familiar with how integers are to use a calculator, but how about a similar subtrac-
constructed. tion problem?
Subtraction is a little trickier, but you can use
Place Value place value to do it in your head, too. Let’s check the
All integers greater than 9 can be represented as sums problem we just did by subtracting 47 from 35,661. If
of products, and all those products are some single- you can do this problem in your head by using the
digit number times a multiple of 10. For example, 47 “borrowing” algorithm you learned in second grade,
can be thought of as great: Continue to use it. If you’re one of the many
who can’t, then try this: round 61 down to 60 and
4 × 10 = 40 round 47 up to 50. Everyone knows that 60 – 50 = 10.
+ 7 × 1* = 7 Then you can add back the 1 you rounded down
47 (61 – 60) and the 3 you rounded up (50 – 47) to get a
difference (remember that subtraction is always the
*In Lesson 4, you’ll learn that 1 = 100. difference between two numbers) of 10 + 1 + 3 = 14.
The other numbers in your subtraction problem are
This representation holds no matter how large the all unchanged, so the answer is 35,614.
number. For example, 35,614 can be represented as You can probably think of other algorithms that
(3 × 10,000) + (5 × 1,000) + (6 × 100) + (1 × 10) + work. It’s not important how you do addition and
(4 × 1). subtraction of relatively small numbers as long as you
can make the calculations quickly and accurately. In
3 × 10,000 = 30,000 eighth grade you should find a way to do this, because
+ 5 × 1,000 = 5,000 you have to get ready for operations that require
+ 6 × 100 = 600 more thought and unfamiliar steps. Try Exercise 1
+ 1 × 10 = 10 without a calculator. If possible, do the problems in
+ 4×1= 4 your head; if not, use a pencil and paper. Answers are
35,614 on page 30.

The numbers on the right are very easy to add Exercise 1


in a column because most of addition involves adding
0’s to single non-zero numbers. And you know any 1. 27 + 63 =
number plus 0 is the same number you started out
with. Knowing this way of representing numbers is 2. 369 – 284 =
very helpful when it comes to doing addition, sub-
traction, and multiplication when you can’t use a 3. 302 – 97 =
calculator or when using a calculator might take
valuable time. For example, let’s say you want to add 4. 75 + 88 =
47 to 35,614. You can start with the 1’s column and add
4 + 7, which is 11. Because 11 is the same as (1 × 10) + 5. 452 + 259 =

24
8th_GRDM_02.qxd:Layout 1 8/11/09 12:53 PM Page 25

–OPERATIONS WITH INTEGERS–

6. 33 – 18 = 0’s at the end of 30 (one). As a check, notice that 30 ×


500 = 15,000.
7. 1,909 + 111 = You can also use place value to do multiplica-
tion and division calculations that don’t involve 0’s at
8. 4,000 – 222 = the ends of numbers. For example, 9 × 14 can be
thought of as (9 × 10) + (9 × 4) = 90 + 36 = 126. Us-
9. 14,641 – 14,639 = ing algorithms for division of integers is precise only
if you have a number that goes evenly into another,
but you can also use it for estimation. For example,
Multiplying and Dividing you know that 8 ÷ 4 = 2 but that 8 ÷ 3 = 2 with a re-
Large Numbers mainder of 2. So you can figure out, using mental
math, that 72 ÷ 24 = 3, because if you subtract 24
When you’re doing a complex physics problem in high from 72 three times, you get 0, meaning that 3 equal
school, you shouldn’t have to stop in the middle of it shares of 24 is a fair way of dividing up 72 things. But
to enter 20 × 300 in your calculator. If you feel that to divide 72 by 26, for example, you might start with
you’d need to do that, you should take the time now to 72 – 26 = 46 and 46 – 26 = 20. Since you can’t take 26
learn how to do calculations like that in your head. things from 20, you know that the answer is some-
Let’s start with 20 × 300, which can be rewritten where between 2 and 3—which at least gives you an
as (2 × 10) × (3 × 100). When we multiply by 10, we estimated value, which will be discussed in Lesson 6.
can easily do it just by adding a 0 after the integer we Again, do the following exercise without a cal-
start with: For example, 3 × 10 = 30 or 476 × 10 = culator. Answers are on page 30.
4,760. Likewise, when we multiply times 100, we just
add two 0’s; when we multiply by 1,000, we add three Exercise 2
0’s; and when we multiply by 1,000,000 (quick! count
the 0’s in a million!), we add six 0’s. So when we have 1. 80 × 110 =
2 × 10 × 3 × 100, we can think of that as 2 × 3 × 10 ×
100 = 6 × 10 × 100 = 6 × 1,000 = 6,000. 2. 8 × 3,000 =
To sum up, the product of any two integers that
end in one or more 0’s is the product of the non-zero 3. 700 ÷ 70 =
parts followed by the total number of 0’s at the end of
the two integers. Thus, 2 × 60 = 120 (12 followed by one 4. 45,000 ÷ 90 =
0), and 2 × 60,000 = 120,000 (12 followed by four 0’s),
and 20 × 6,000 = 120,000 (12 followed by four 0’s). One 5. 7,500,000 ÷ 2,500 =
tricky case is when the product of the non-zero parts
ends with a zero: you still have to add the right number 6. 60 × 900,000 =
of 0’s after that product. For example, 500 × 6,000 =
3,000,000 (5 × 6 = 30, followed by five 0’s). This book won’t review various pen-and-pencil
Division for these types of numbers works al- algorithms for division and multiplication. Many
most the same way, except in reverse. For example, methods are available, and it’s recommended that you
15,000 ÷ 30 = 500, which is 15 ÷ 3 = 5, followed by know at least one for multiplication and at least one
two 0’s, which is the difference between the number for division.
of 0’s at the end of 15,000 (three) and the number of

25
8th_GRDM_02.qxd:Layout 1 8/11/09 12:53 PM Page 26

–OPERATIONS WITH INTEGERS–

Factoring Factoring is huge when it comes to solving alge-


braic problems. It’s pretty safe to say that you’ll have a
Factors are the numbers that, when multiplied to- hard time with algebra unless you understand what
gether, result in a given number. For example, 2 and factors are and how to find them.
12 are factors of 24 because 2 × 12 = 24. Other factors Fortunately, to find all the factors of an integer,
of 24 are 3, 8, 4, 6, 1, and 24. Knowing the factors of a there are a few simple rules that you can use, even on
number makes division easier, and it will also help in large numbers, to get started. The simplest one is that
working with fractions, as you’ll see in Lesson 2. if a number is even (i.e., it ends in 0, 2, 4, 6, or 8), it is
divisible by 2. And there are rules for divisibility by
other numbers as well.
Another way of saying this is that if x is a factor
of y, then y is divisible by x.

A NUMBER IS
DIVISIBLE BY IF EXAMPLE

3 its digits add up to a multiple of 3 4,764 is divisible by 3 because 4 + 7 + 6


+ 4 = 21, which equals 3 × 7

4 its last two digits are divisible by 4 17,372 is divisible by 4 because 72 is


divisible by 4 (4 × 18 = 72)

5 its last digit is either a 0 or a 5 165 is divisible by 5; 560 is divisible by 5

6 it is divisible by both 2 and 3 4,764 is divisible by 6 because it’s


divisible by 2 (it’s even) and by 3 (see
above)

8 it is divisible by 2 and then by 4 17,372 is NOT divisible by 8 because


(or by 4 and then by 2) 17,372 ÷ 2 = 8,686 but 86 is not divisible
by 4. However, 376 is divisible by 8
because half of 376 is 188, and 88 is
divisible by 4 (4 × 22 = 88).

9 its digits add to a multiple of 9 15,066 is divisible by 9 because 1 + 5 + 6


+ 6 = 18, which is a multiple of 9

10 its last digit is a 0

26
8th_GRDM_02.qxd:Layout 1 8/11/09 12:53 PM Page 27

–OPERATIONS WITH INTEGERS–

Unfortunately, there’s no rule for 7; if you have a process, called prime factorization, we begin by di-
big number and it’s not divisible by 7, it’s good to use viding the number by any one of its factors. For ex-
a calculator or pencil-and-paper division algorithm ample, with the number 48, we could begin with
to see if the number is divisible by 7. division by 2, giving us factors of 2 and 24:

Exercise 3 48

Tell which, if any, numbers less than 10 the following


numbers are divisible by. Answers are on page 30. 2 × 24

Because 2 is a prime number, we can leave it alone for


1. 333
now, but we can divide 24 by any of its factors, say 4
and 6:
2. 444
48

3. 7,494
2 × 24
4. 1,645
4 × 6
5. 89
Now, we divide 4 and 6 each by 2, and at the bottom
Prime Factorization of our diagram we have nothing but prime numbers:
A prime number is a positive integer whose only fac-
48
tors are 1 and the integer itself. It’s obvious that 2 is a
prime number because 1 and 2 are the only possible
2 × 24
factors. Likewise, 3 is a prime number. But 4 is not, be-
cause 2 × 2 = 4 and 1 × 4 = 4. In fact, no even number
greater than 2 is prime, because they can all be divided 4 × 6

by 2. The next prime numbers in order are 5, 7, 11, 13,


17, 19, and 23. Can you figure out which is the next 2 × 2 2 × 3

prime number in order? It can’t be 25 (divisible by 5) or


27 (divisible by 3 and 9), so it must be 29, which isn’t di- We now have five numbers with no factorization be-
visible by any of the rules you know and isn’t divisible neath them: four 2’s and one 3. So the prime factor-
by any of the prime numbers that come before it. ization of 48 is 2 × 2 × 2 × 2 × 3. Another way to write
2 × 2 × 2 × 2 is 24. The way the prime factorization of
48 is usually written is 24 × 3.
You can tell a number is prime if it isn’t divisible Now, try some prime factorizations on your own.
by 2, 3, or 5, or by any of the other prime num-
bers that are smaller than that number.

It’s often good to know how to factor a number


so that all the numbers are prime numbers, especially
in operations with fractions (see Lesson 2). To do this

27
8th_GRDM_02.qxd:Layout 1 8/11/09 12:53 PM Page 28

–OPERATIONS WITH INTEGERS–

Exercise 4 One thing to keep in mind is that the terms


Write the prime factorizations of the following inte- greater than (>) and lesser than (<) are based on the
gers in the form just shown. Answers are on page 31. number’s position along the number line. That is, 9 > 4
and 4 > 0, but also 0 > –4 and –4 > –9. This last fact
1. 45 may seem opposite, but if you think of the number
line as a thermometer, you know that 0° is warmer
2. 46 than –4°, which in turn is warmer than –9°. The
number line works the same way.
3. 64 We call the distance of a number from 0 on the
number line its absolute value. For example, the ab-
4. 91 solute value of 6 is 6, and the absolute value of –6 is
also 6. The absolute value of a number n is denoted
5. 104 |n|. It’s always true that |n| = |–n|. Take a minute to try
some examples to convince yourself of this fact.
The next thing to notice is that the sign “–” is
Operations with used in different ways in math. In fact, in the last
Negative Integers equation, –4 – 3 = –7, the equation can be read “neg-
ative four minus three equals negative seven.” So “–”
You may have learned to do addition and subtraction can mean either “negative,” as in a negative number,
with a number line: Just count to the right to add and or it can mean “minus,” as in a typical subtraction
count to the left to subtract. For example, if you add 3 calculation. But “–” can be used to mean something
+ 2, start at 3 and count up 2; if you want to subtract else, too. Note that –1 is the “opposite” of 1 (we don’t
2 from 3, start at 3 and count 2 left. Of course, you often use the positive notation +1; we just use “1”);
learned this using only positive integers. Now, look at it’s the same distance from 0 on the number line, but
the following number line with negative numbers; it’s on the opposite side. Likewise –3 is the “opposite”
from 0, when you go left, you get negative numbers: of 3. And this idea will come in handy when you’re
–1, –2, –3, and so on. doing operations with negative numbers. When you
have a double negative, –(–4), for example, it’s the
opposite of –4, or 4. This is confusing, no doubt, but
–9 –8 –7 –6 –5 –4 –3 –2 –1 0 1 2 3 4 5 6 7 8 9
operations with negatives become a little easier if
But even with negative numbers, we can use the same you can just remember the following rules. First,
basic strategy for adding and subtracting. Let’s say we look at the what happens when you have plus and
want to add 3 to –4. We start at –4 and count 3 to the minus together.
right, ending at –1. So –4 + 3 = –1. Now, let’s subtract 3
from –4. We start at –4 and count 3 left, and –4 – 3 = –7.

OPERATION WHAT HAPPENS EXAMPLES

1. Addition/subtraction: positive and negative . . . negative 6 + (–3) = 6 – 3


together become . . .

2. Addition/subtraction: positive and positive . . . positive 6 + (+3) = 6 + 3


together or negative and negative together 6 – (–3) = 6 + 3
become . . .

28
8th_GRDM_02.qxd:Layout 1 8/11/09 12:53 PM Page 29

–OPERATIONS WITH INTEGERS–

Now, here are some rules for how to find the result of operations once you’ve resolved the signs.

OPERATION RESULT EXAMPLES

3. Adding/subtracting: 2 numbers with like signs Sum of the absolute values of 43 + 27 = 70


the 2 numbers; sign is the only –43 – 27 = –70
sign in the operation. –43 + (–27) = –43 – 27 = –70

4. Adding/subtracting: 2 numbers with Difference of the absolute 43 – 27 = 16


opposite signs values of the 2 numbers; sign –43 + 27 = –16
is that of the number with the 43 + (–27) = 43 – 27 = 16
larger absolute value. –(–43) – 27 = +43 – 27 = 16
27 + (–43) = 27 – 43 = –16

5. Multiplication/division: positive and Product or dividend is 6 × (–3) = –18


negative together negative. –6 × 3 = –18
18 ÷ (–6) = –3
–18 ÷ 3 = –6

6. Multiplication/division: positive and positive Product or dividend is positive. (–6) × (–3) = 18


together or negative and negative together 6 × 3 = 18
–18 ÷ (–3) = 6
18 ÷ 3 = 6

Here’s another way to think of operations with Exercise 5


negative numbers. You can think of positive numbers as
assets and negative numbers as debts, and you can 1. 17 – (–9) =
think of total value as a combination of assets and
debts. For instance, if you have $4 in cash but have 2. –9 – 17 =
debts of $3 and $6 to two of your friends, the total value
of your “wealth” is $4 + (–$3) + (–$6) = –$5. Now, let’s 3. 14 – (–33) =
consider the friend to whom you owe $6. You help her
with her math homework, so she decides to forgive $2 4. (–45) ÷ 5 =
of your debt. Now you can subtract that $2 of debt, and
your new net value is –$5 – (–$2) = –$5 + $2 = –$3. You 5. –104 ÷ (–13) =
decide to celebrate by spending $2 on a big bag of chips,
meaning that you’ve deducted $2 from your assets. 6. (–7) × (–14) =
Now your new worth is –$3 – ($2) = –$5 again.
Another good way to think of a negative number 7. –3 – (6 + 3) =
is that it’s a positive number that has been multiplied
by –1. For instance, (–1) × 18 = –18, because of the rule 8. 11 × (–7 + 4) =
for the sign of the product of multiplication, just listed.
Of course, it’s often handy to think of multiplication 9. –29 + 36 – (–67) =
by (–1) simply as a sign switcher. In the following exer-
cise (answers are on page 31), and especially in some
algebraic operations that you’ll see later in this section
and in Section 2, you might use this trick as a way for
remembering how to do the operation.

29
8th_GRDM_02.qxd:Layout 1 8/11/09 12:53 PM Page 30

–OPERATIONS WITH INTEGERS–

Summary Exercise 2
1. 8,800
You’ve now reviewed the basic operations with posi- 8 × 11 = 88, followed by two 0’s
tive and negative integers. You’ll need to know these 2. 24,000
very well as you go on to operations with fractions 3. 10
(Lesson 2) and decimals and percentages (Lesson 3). 7 ÷ 7 = 1, followed by 2 – 1 = 1 zero
4. 500
45 ÷ 9 = 5, followed by 3 – 1 = 2 zeros
Answers 5. 3,000
75 ÷ 25 = 3, followed by 5 – 2 = 3 zeros
Exercise 1 6. 54,000,000
1. 90 6 × 9 = 54, followed by 1 + 5 = 6 zeros
Example mental calculation: 3 + 7 = 10; 60 +
20 = 80; 80 + 10 = 90 Exercise 3
2. 85 1. 3, 9
Example mental calculation, counting down: 3+3+3=9
369 – 300 = 69; 300 – 290 = 10; 290 – 284 = 6; 2. 2, 4
69 + 10 + 6 = 85 Even number; 44 is divisible by 4; 8 doesn’t work
3. 205 because 444
2 = 222, which isn’t divisible by 4.

Again, counting down: 302 – 300 = 2; 300 – 100 3. 2, 3, 6


= 200; 100 – 97 = 3; 2 + 200 + 3 = 205 Even number; digits add to 24, which is a
4. 163 multiple of 3.
75 + 75 = 150; 8 + 5 = 13 4. 5
5. 711 Last digit is 5; no other rules apply.
450 + 250 = 700; 2 + 9 = 11 5. None
6. 15
33 – 23 = 10; 23 – 18 = 5
7. 2,020
9 + 11 = 20; 1,900 + 100 = 2,000; 2,000 + 20
= 2,020
8. 3,778
4,000 – 200 = 3,800; 3,800 – 20 = 3,780;
3,780 – 2 = 3,778)
9. 2
The easiest method here is to notice that
14,639 is just 2 smaller than 14,641!

30
8th_GRDM_02.qxd:Layout 1 8/11/09 12:53 PM Page 31

–OPERATIONS WITH INTEGERS–

Exercise 4 Exercise 5
1. 3 × 5
2
1. 26; 17 – (–9) = 17 + 9 = 26; rule 2
45
2. –26; |–9| + |–17| = 26; rule 3
3. 47; 14 – (–33) =14 + 33; rule 2
5 × 9 4. –9; rule 5
5. 8; rule 6
3 × 3 6. 98; rule 6
2. 2 × 23 7. –12; –3 – (6 + 3) = –3 – 9 = –12; rule 2
23 is a prime number. 8. –33; 11 × (–7 + 4) = 11 × (–3) = –33; rules 4
3. 26 and 5
64 9. 74
–29 + 36 – (–67) = –29 + 36 + 67 = 7 + 67 = 74
8 × 8

4 × 2 2 × 4

2 × 2 2 × 2

4. 7 × 13
13 is a prime number.
5. 23 × 13
Shortcut: 104 is 13 more than 91 (see answer 4),
so 104 = 8 × 13. Then 8 factors into 2 × 2 × 2.

31
8th_GRDM_02.qxd:Layout 1 8/11/09 12:53 PM Page 32
8th_GRDM_02.qxd:Layout 1 8/11/09 12:53 PM Page 33

2
L E S S O N

OPERATIONS
WITH FRACTIONS

L E S S O N S U M M A RY
To do the more complex operations of geometry and algebra, you
have to review the basic number operations on rational numbers,
or fractions. You may already be familiar with these, but here you’ll
get a thorough review of all the operations, which are certainly
harder to learn than those with integers.

I n this lesson, you will discover the rules, and some of the reasons that the rules work. You undoubtedly
started working with fractions a couple of years ago, and if you’ve found them confusing, you’ll need to read
the explanations in this lesson. We’ll start out by reviewing proper fractions, improper fractions, and mixed
numbers. We’ll then go on to multiplication and division of fractions and then to addition and subtraction of
fractions, which most people find more difficult than multiplication and division. Finally, you’ll be ready to do
some operations on multilayer fractions, using all that you’ve learned in the first two lessons.
As you go through this chapter, you will notice that in division operations, the symbol “÷” will be replaced
by the symbol “/” and that division may also be represented as a fraction like this: ab . This last expression, as well
as “a/b,” may be read as “a out of b,” “a over b,” or “a divided by b.” Also, fractions will be represented sometimes
as ab and sometimes as a/b. Integers can be represented as fractions, using 1 as the denominator: 7 = 17 = 7/1.

33
8th_GRDM_02.qxd:Layout 1 8/11/09 12:53 PM Page 34

–OPERATIONS WITH FRACTIONS–

Proper Fractions, Improper (As in operations with integers, you can use different
Fractions, and Mixed Numbers algorithms of your own invention to get the same an-
swers as the algorithms offered here—just be sure
A proper fraction is a fraction that is less than 1. Its that they always get the same results.)
denominator is greater than its numerator. An im-
proper fraction is greater than or equal to 1: Its de- Exercise 1
nominator is less than or equal to its numerator. A Convert the mixed numbers to improper fractions and
mixed number is a combination of an integer and a convert the improper fractions to mixed numbers. An-
fraction and represents the sum of those two num- swers are on page 39.
bers: 185 is the sum of 1 and 85 , and it lies between 1 9
1. 116
and 2 on the number line. The mixed number –185,
however, is the sum of –1 and – 85, and it lies between
2. 811
13
–1 and –2 on the number line.
Every improper fraction has an equivalent 3. –383
mixed number. (Every proper fraction also has an
17
equivalent mixed number, but the integer part of the 4. 6
mixed number is always 0.) Let’s look at the improper
89
fraction 197 . We learned long ago that in 1 whole there 5. 4
are 7 sevenths, so in 2 wholes there are 14 sevenths.
6. –14
13
Three wholes would contain 21 sevenths, so we know
that 197 will be less than 3. In fact, it is 5 sevenths (19 –
14 = 5) greater than 2, so its equivalent mixed num-
ber is 257.
Multiplying and
Another (and quicker) way to think of finding
Dividing Fractions
the mixed number is to divide the numerator by the
Multiplying fractions is straightforward: Just multiply
denominator; if there is a remainder, that number
numerator by numerator and denominator by de-
will be the numerator of the fractional part of the
nominator to get your result.
mixed number. So if we have the improper fraction 134,
we can divide 13 by 4 and get 3 with a remainder of 1. 3 7 3 × 7 21
× = =
Therefore, the equivalent mixed number is 341. 8 9 8 × 9 72
On the other hand, converting a mixed number
In general,
is easier in that it doesn’t involve any division or re-
mainders. The first thing to remember is that the im- a c a×c
proper fraction will have the same denominator as × =
b d b×d
the fractional part of the mixed number. Then, to
Division, you may remember, is the opposite of
find the numerator, multiply the denominator of the
multiplication in operations with integers. Well, it’s
fractional part times the integer part and then add
the opposite in operations with fractions, too, but
the numerator of the fractional part. For instance, to
with a little twist. What you do is invert the divisor
convert 341, we multiply 4 times 3 and add 1 to get 13.
and then multiply:
The denominator is 4, so the improper fraction is 13 4.
As you can see, reverse conversion is a good way 3 7 3 9 3 × 9 27
to check that you did your first conversion correctly. ÷ = × = =
8 9 8 7 8 × 7 56

34
8th_GRDM_02.qxd:Layout 1 8/11/09 12:53 PM Page 35

–OPERATIONS WITH FRACTIONS–

In general, should do that to see if we have a reasonable answer),


it will be 739 48. So is this a reasonable answer? Well, our
a c a×d mixed numbers are pretty close to 4 and 2, and 4 × 2
÷ =
b d b×c is 8, so our answer seems to be in the ballpark.
Here are some general rules for estimation: When What if instead we want to divide 461 by 187?
you multiply by a number less than 1 (i.e., a proper Again, our numbers are close to 4 and 2, and 4 ÷ 2 = 2,
fraction), you will get a number with absolute value so we’d expect our answer to be about 2. Let’s see if it
smaller than what you began with. When you divide by is: 256 ÷ 158 = 256 × 15
8
= 200
90 , which is little over 2. Can you
a number less than 1, you will get a number with ab- estimate what 18 divided by 461 would be? (Answer:
7

solute value larger than what you began with. The op- about 21.) Can you quickly tell exactly what 187 divided
posite is true if you multiply or divide by a number by 461 is, given the calculations you’ve already made?
greater than 1. And of course you know what you get if Yes, you can: It’s the same as the previous calculation,
you multiply or divide by a number that is equal to 1. except that the numerator and denominator are re-
90
versed: 200 . A fraction that’s reversed like this is called
Exercise 2 the reciprocal.
(Answers are on page 39.)
Exercise 3
1. 1
3 × =
4
7 (Answers are on page 39.)

2. 6
31 × 98 = 1. 465 ÷ 461 =

3. 14
13
5
÷ 12 = 2. 1
15 × 821 =

4. 1
2 × 115
13 = 3. 1032 × 932 =

9
5. 10 ÷ 81 = 4. 5
3 ÷ 445 = 132
25
. What is 854 ÷ 132?

9
6. 10 ÷ 37 = Equivalent Fractions and
Simplest Form
Multiplying and Dividing Two fractions ba and xy are said to be equivalent if a di-
Mixed Numbers vided by b yields the same answer as x divided by y.
As usual, there are multiple algorithms available for Another way of thinking of equivalent fractions is
multiplying and dividing with mixed numbers, but that they’re equivalent if ba times any number n gives
the method shown here—and for most people the the same result as xy times n.
easiest one—is to convert the mixed number to an Now remember that any number multiplied or
improper fraction and then follow the usual multipli- divided by 1 is still the number you started with. We
cation/division algorithms. For example, if we want can of course use a calculator to find out if two frac-
to multiply 461 by 187, we will first change those mixed tions are equivalent; for example, 18 54
25 and 75 both yield
numbers to the improper fractions 256 (4 × 6 + 1 = 25) 0.72 on the calculator. Sometimes, though, it’s much
and 158 (8 × 1 + 7 = 15). This gives a number that you more convenient just to notice that 3 × 18 = 54 and
find by either mental math or a calculator to be 37548 . If that 3 × 25 = 75. This is the same as multiplying 18 25 by
we need to change it back to a mixed number (and we 1, because if you multiply (or divide) both numerator

35
8th_GRDM_02.qxd:Layout 1 8/11/09 12:53 PM Page 36

–OPERATIONS WITH FRACTIONS–

and denominator by the same number, you’re really Note: it should be clear from this exercise that
3 2 7
multiplying by 1. 4> > 12
3 > 21. This allows us to state a rule for finding
out whether one fraction is larger than another.
18 3 18 × 3 54
× = =
25 3 25 × 3 75
Usually, though, we want smaller numbers in To find out which of two fractions has a larger
our fractions if possible; they’re easier to talk about absolute value, convert both to equivalent frac-
and easier to estimate with. (Most of us can imagine tions having the same denominator.
how far halfway across the street is, or even 32 of the
way across the street, but it’s a lot harder to estimate
138 138 Canceling
276 of the distance or 207, even though the last two 6
Let’s say you want to multiply 25 by 56. Here’s one way
fractions are equivalent of the first two.) What we’re
to do it:
usually doing when we’re finding equivalent fractions
is trying to put the fraction in its simplest form, so 6 5 30
we’ll more often be dividing numerator and denomi- × =
25 6 150
nator by the same number. So how do we know a
This leaves you with some big numbers in your frac-
fraction is in its simplest form? Let’s take 138
276. Do you 30
tion. You can probably figure out that 150 = 51 (by di-
remember the divisibility rules from Lesson 1? We
viding numerator and denominator by 30), but if you
know that numerator and denominator are both
had numbers like 138 and 207 it might take a bit
even, so both can be divided by 2. You can use a cal-
longer. But you don’t have to wait until the numbers
culator or a pencil-and-paper method to find both
get big to start getting a fraction in its simplest form:
quotients, which are 69 and 138. Because both of
You can cancel numbers out while you’re in the mid-
these numbers are divisible by 3 (see the divisibility
dle of the calculation.
rules), divide both by 3 to get 23
46. Some experimenta-
For example, in the calculation above, notice
tion will show you that both numerator and denomi-
that you have a 6 on the top and a 6 on the bottom. If
nator are divisible by 23, and you end up with 21.
you divided them both by 6, you’d have 11, and re-
138 2 69 3 23 23 1 member that 1 times any number is itself, so you’d be
÷ = ÷ = ÷ = 5
left with simply 25 . But notice also that 5 and 25 have
276 2 138 3 46 23 2
a common factor: 5. Dividing both by 5 leaves you
Now use your knowledge of the divisibility rules
with 51, long before you get to the big numbers.
and the procedure of dividing numerator and denom-
inator by the same number to put the following frac- 1 1
tions in their simplest form. Answers are on page 40. 6 5 1
—– × – = –
25 6 5
5 1
Exercise 4
Canceling works no matter how many fractions
9 you’re multiplying or dividing by each other.
1. 12

8
2
2. 12 1 4 1
1 – × —– ÷ – = – × —– × – = –2
1 8 6 3 8 7
7
2 21 7 2 21 6 3
3. 12
1 3 3
1
6
4. 12

36
8th_GRDM_02.qxd:Layout 1 8/11/09 12:53 PM Page 37

–OPERATIONS WITH FRACTIONS–

1 1 1× 3 1× 2 3 2 5
After getting the fractions in their proper form, we + = + = + =
can cancel 7 in the third numerator and 21 in the sec- 2 3 2× 3 3× 2 6 6 6
4 1 4 × 8 1 × 7 32 7 39
ond denominator to 1 and 3, 2 in the first denomina- + = + = + =
tor and 8 in the second numerator to 1 and 4, the 7 8 7 × 8 8 × 7 56 56 56
remaining 3 in the second denominator and the 3 in Another way to find a common denominator is
the first numerator to 1 and 1, and the remaining 4 in to find the least common multiple. Here’s how you
the second numerator and the 6 in the third denomi- find it if you want to subtract a number given in 8ths
nator to 2 and 3, for an answer of 32. If we multiplied from a number given in 6ths. First, list a few multiples
all these numbers together, we’d have 168
252, which is the of 8 and a few multiples of 6:
2
same as 3, but it might take a while to prove that.
Now that we’ve simplified multiplication and 6, 12, 18, 24, 30, 36, 48
division by canceling, we’ll move on to something 8, 16, 24, 32, 40, 48
that’s much harder: addition and subtraction of
fractions. The lowest number in both lists is 24, so it should be
used as the common denominator. This takes a bit
more effort up front, but may result in less work
Adding and later. Here’s the same calculation carried out first us-
Subtracting Fractions ing 48 in the denominator, and then using 24.

5 11 5 × 8 11 × 6 40 66 106
The thing that makes adding and subtracting frac- + = + = + =
tions hard is that you must have a common denomi- 6 8 6 × 8 8 × 6 48 48 48
nator. In other words, if you want to add 21 and 31, you 5 11 5 × 4 11 × 3 20 33 53
+ = + = + =
can’t just add 1 + 1 in the numerator and 2 + 3 in the 6 8 6 × 4 8 × 3 24 24 24
denominator. That would give you 52, which is obvi- Notice that the second calculation used smaller
ously wrong because 52 is smaller than 21, and if you’re numbers, and it’s already in simplest form; it doesn’t
adding a positive number to 21, you have to end up need to be reduced, although it can still be converted
with a larger number! (You know that 52 < 21 because to a mixed number.
4 5
10 < 10)
Exercise 5
Do the following operations as indicated. All an-
You must always have a common denominator swers should be in simplest form with no improper
to add or subtract fractions. fractions. Answers are on page 39.

5
1. 6 – 83 =
So how do you find a common denominator? A
quick way to do it is to multiply both numerator and 7
2. 10 + 54 =
denominator of the first fraction by the denominator
of the second and both numerator and denominator 14 8
3. 9 – 15 =
of the second fraction by the denominator of the first.
Sound confusing? It’s not really that bad. Here are 3
4. 4 + 87 =
some examples:

37
8th_GRDM_02.qxd:Layout 1 8/11/09 12:53 PM Page 38

–OPERATIONS WITH FRACTIONS–

Multilayer Fractions The first fraction is still 49, but the numerator of
20
the second fraction is now (–1) , which equals –20. (Re-
Now we’re going to test your knowledge of fractions member the rules for adding and dividing positives
by having you do a few operations with fractions and negatives?) So now our calculation will look like
where the numerator and denominator are fractions this:
themselves. You’ll undoubtedly see lots of fractions
9 −20 9 20 27 40 13 1
like this when you get to high school algebra, but + = − = − = − = −1
4 6 4 6 12 12 12 12
you’ll see a few of them in eighth grade as well. Here’s
an example: Review the rules for calculating with negative
numbers if you don’t understand all these steps.
3 5×4
4
1 + 4−3 = ?
3 6 Exercise 6
(Answers are on page 40.)
Let’s see if we can go through the steps of find-
3−1 5×4
ing an answer. Let’s take the first multilayer fraction. 1. 4 + 1 + 4 + 1 =
Remember that one meaning of a fraction bar is 1 3
4 5
“division by.” So we can evaluate this first fraction
as 34 ÷ 31 = 43 × 13 = 49. Now we’re down to just the two 3 3
2. 4 − 10 =
layers for the first fraction, so let’s go on to the next 3 −3
fraction. In the numerator of the numerator we have 2 2

5 × 4, or 20, and the denominator is 1, so the whole


numerator is simply 20. That means the second frac-
tion is just 20
Summary
6 . So our problem now is a fairly easy ad-
dition problem:
You’ve now reviewed the basic operations with frac-
9 20 27 40 67 7 tions, which are some of the numbers that fall be-
+ = + = =5
4 6 12 12 12 12 tween integers. Next we’ll go on to other ways to talk
about numbers that fall between the integers: deci-
For this calculation we used 12 as the common de-
mals and percentages.
nominator based on least common multiples of 4 and
6. We multiplied the top and bottom of the first frac-
tion by 3 and multiplied the top and bottom of the
second by 2. Then we added the two fractions and
changed the improper fraction to a mixed number by
dividing 67 by 12 and finding the remainder. (Review
the earlier parts of this lesson if you don’t understand
any part of this procedure.)
Now we will change just one number in our
original problem so that you can review your negative
numbers and use them in fraction operations:

3 5×4
4 4−5
1 + =?
3 6

38
8th_GRDM_02.qxd:Layout 1 8/11/09 12:53 PM Page 39

–OPERATIONS WITH FRACTIONS–

Answers Exercise 3
24
1. 1150
6 ÷ 6 = 6 × 25 = 150 = 1150
29 25 29 6 174 24
Exercise 1
1. 25
16; (16 × 1) + 9 = 25; denominator remains the 2. 17
30
15 × 2 = 30
1 17 17
same.
1
13 ; (8 × 13) + 11 = 115; denominator remains
2. 115 3. 10328
3 × 3 = 9 = 1039
32 29 928 1
the same.
3. – 278; The numerator is (3 × 8) + 3 = 27; sign 4. 132 7
25 , or 525
25
and denominator remain the same. This answer is the reciprocal of 132 , because the
4. 256; There are 6 sixths in 1 whole, 12 sixths in 2 division problem is reversed. If you calculate it
wholes, 56 left over. out by changing mixed numbers to improper
5. 2241; (894 ) = 22 with a remainder of 1; denomina-
fractions, you’ll get 445 divided by 35, or 445 times 53,
tor remains the same. or 132
25 , which is the same answer for more work.
1 14
6. –113 ; (13) = 1 with a remainder of 1; denomina-
tor and sign remain the same. Exercise 4
3
1. 4
Exercise 2 Divide numerator and denominator by 3.
2
1. 4
21
2. 3
48 Divide numerator and denominator by 4.
2. 279
7
For those who already know simplest form, this 3. 12
fraction will reduce to 16
93.
Numerator and denominator share no
168 common factors; if you’ve forgotten prime
3. 65
As a mixed number this is 238
65.
factorization, revisit Lesson 1.
1
4. 115
26
4. 2
As a mixed number this is 411
26.
Divide numerator and denominator by 6.
72
5. 10
As a mixed number in simplest form, this is 751. Exercise 5
11
6. 27
70
1. 24
9 Use 24 as the common denominator.
Note that 10 divided by a proper fraction is
much larger than 109
divided by an improper 2. 121
fraction. Use 10 as the common denominator.
1
3. 145
Use 45 as the common denominator.
4. 185
Use 8 as the common denominator.

39
8th_GRDM_02.qxd:Layout 1 8/11/09 12:53 PM Page 40

–OPERATIONS WITH FRACTIONS–

Exercise 6 2. 43 3
10 3 2 3 ⎛ 3⎞
− = × − ×⎜− ⎟ =
1. 3 −3 4 3 10 ⎝ 2 ⎠
3−1 5×4 2 20 2 2
4 +1 4 +1 5 2 4 20 5
1
+ 3 = 1 + 53 = × + × =
5 1 5 3
4 5 4 5 6 ⎛ 9 ⎞ 1 9 10 9 19
− − = + = + =
8 100 24 100 124 4 12 ⎜⎝ 20 ⎟⎠ 2 20 20 20 20
+ = + = =8
5 15 15 15 15 15 6
Notice that we reduced 12 to 21 once we realized
that the other denominator was 20, which
Notice that we could have reduced 100 20
15 to 3 , but
would have given us a least common multiple
our common denominator would still have
of 60 if we hadn’t reduced.
been 15.

40
8th_GRDM_03.qxd:Layout 1 8/11/09 1:05 PM Page 41

3 DECIMALS
L E S S O N

AND
PERCENTAGES

L E S S O N S U M M A RY
To do the more complex operations of geometry and algebra, and
especially those that are involved in science, you need to have a
thorough understanding of decimals and percentages. This lesson
will guide you through the meaning of decimal numbers and per-
centages, and it will show you how to do all the basic calculations
with them.

I n this lesson, you will discover how to work with numbers expressed as decimals. Of course, you’ve been us-
ing decimals ever since you learned to count money. Now you’ll review what they mean, and with a better
understanding of them you can go on to solve problems that will help you in eighth grade and throughout
your life. We’ll start out by reviewing the basic operations—addition, subtraction, multiplication, and divi-
sion—and then we’ll cover percentages and some applications of percentages.

41
8th_GRDM_03.qxd:Layout 1 8/11/09 1:05 PM Page 42

–DECIMALS AND PERCENTAGES–

Decimals: value. In fact, 42.06300000 would have the same


Addition, Subtraction, mathematical value.
Multiplication, Division Now let’s look at several numbers and put them
in order from greatest to least: 1.78, 1.708, 1.087, –1.78,
A decimal number is really a sum of fractions, but 1.7804, –1.0708, and –1.0871. First, we know that all
each fraction has a “power” of 10 in its denominator: the positive numbers are greater than any of the neg-
1, 10, 100, 1,000, and so on. The decimal number atives. So we can choose from among 1.78, 1.708,
43.67104 could be represented as 1.7804, and 1.087. Of these, 1.78, 1.7804, and 1.708
have 7 in the tenths place, so they’re all bigger than
43 6 7 1 0 4 1.087, which will come in fourth. Then we can elimi-
+ + + + +
1 10 100 1, 000 10, 000 100, 000 nate 1.708 as the largest because it has a 0 in the hun-
You might think this is a cumbersome way of repre- dredths place, while the other two numbers have 8’s.
senting a number, and you’d be right. It’s easier to say So how do we compare the two remaining numbers?
“43 and 67,104 hundred thousandths,” and it’s even 1.78 has no thousandth place at all, and 1.7804 has a
easier just to say “43-point-six-seven-one-oh-four.” zero. Which is bigger so far? Neither; so let’s go one
Hence, we have decimals, which make our mathemat- more place to the right. Again, 1.78 has nothing there,
ical lives much easier. but 1.7804 has a 4, which means it is larger. So now
we have the order of the positives: 1.7804 > 1.78 >
Place Value 1.708 > 1.087. When arranging the negatives, remem-
Like integers, decimal numbers have place value ber from Lesson 1 that with negative numbers, the
based on position. The decimal places start at the number with the smaller absolute value is actually
right of the decimal point, and like the place values in higher (that is, further to the right on the number
integers, they go down as they go further to the right. line). So the order of our negatives is –1.0708 >
Here’s a number with all its place values labeled: –1.0871 > –1.78.
thousands thousandths
When you add or subtract decimal numbers,
hundred tenths
thousands tens hundred you follow all the rules and procedures for adding
thousandths
and subtracting integers, except the decimal point is
3,476,037.731098 millionths
brought straight down. For example,
ones ten
millions hundreds thousandths
ten hundredths
thousands
1.7804
To determine which of two numbers is larger, + 1.087
you look at it starting from the left, and as soon as a 2.8674
number in one place is larger than one in the corre-
sponding place in the other number, you know which You can see that in pencil-and-paper calculations, it’s
number is larger. Let’s take the numbers 42.063 and important to keep your columns straight, or you
42.0366. They’re both larger than 42 and less than 43, could end up adding or subtracting the wrong num-
but to find out which is larger, you’ll see that the bers, as well as getting your decimal in the wrong
tenths place of each is 0, but that the hundredths place. Of course, you can always use calculators as
places are not equal. Therefore, 42.063 > 42.0366 well, but make sure that you know approximately
even though 42.0366 has more numbers in it. In what your answer should be so that you’ll know if you
other words, the number of digits in a decimal num- made a big mistake. For instance, in this calculation,
ber is no indication of its size. We could write 42.063 you should know that your answer will be about 1.8 +
as 42.0630 and it would have the same mathematical 1 = 2.8, so if you get 28 or 0.28 on your calculator,
42
8th_GRDM_03.qxd:Layout 1 8/11/09 1:05 PM Page 43

–DECIMALS AND PERCENTAGES–

you’ll know that you made an inputting mistake with Exercise 2


the calculator. Now let’s try a subtraction exercise.
1. 14.5469 × 100 =
Exercise 1
Take six of the seven numbers in our previous order- 2. 14.5469 ÷ 100 =
ing exercise and find the difference between each con-
secutive pair. First estimate the difference and then 3. 3.18 ÷ 10 =
use a calculator or a paper-and-pencil method to get
the exact answer. Answers are on page 47. 4. 0.097 × 1,000 =

1. 1.7804 – 1.78 5. 0.097 ÷ 1,000 =

2. 1.78 – 1.087 6. 12 ÷ 10,000 =

3. 1.087 – (–1.0708) Placing the Decimal in Multiplication


and Division
4. (–1.0708) – (–1.0871) When you multiply or divide with numbers that con-
tain decimal places, you can use whatever algorithm
5. (–1.0871) – (–1.78) you feel comfortable with and then follow rules for
placing the decimal. Let’s start with the multiplica-
Moving the Decimal Point When tion rules, and with the example 6.5 × 1.4. You can
Multiplying and Dividing by start out doing this operation as 65 × 14, which is 910,
Powers of 10 and then place the decimal by counting the number
Before we get to a general algorithm for multiplica- of digits to the right of the decimals in the numbers
tion, let’s review a handy rule. Because decimals are you’re multiplying and then counting that number of
based on powers of 10, it’s easy to multiply or divide places from the right in your answer. In this case, this
by 10, or 100, or 1,000. We saw this with integers method would give you 9.10. Again, though, you
when we multiplied 10 × 40 to get 400: You just add a should always check your answer with an estimate; in
zero. Dividing 10,000 by 100, you just take away two this case your answer should be somewhere between
0’s to get 100. Another way to think of this process is 6 (6 × 1) and 14 (7 × 2), because 1.4 is about halfway
to think of moving the decimal point to the right between 1 and 2 and 6.5 is exactly halfway between 6
(multiplication) or to the left (division): 10 × 40.000 and 7. That estimate would allow you to throw out
10, 000.000
= 400.00, and 100
= 100.00000. We can do the 91.0 or .910 if you had mistakenly arrived at one of
same thing when the numbers being multiplied and those numbers, on paper or on the calculator.
divided have digits to the right of the decimal point. Division can work in a similar way, but it’s
For example, 42.238 × 10 = 422.38, and 42.238
10 = 4.2238. rarely practical. Let’s take 68.8 and divide it by 1.72.
Here are some problems to practice on. Answers are Again, calculate 688
172 any way you can do it best. The
on page 48. answer comes out to 4. Then you take the difference
between the number of digits to the right of the deci-
mal in the divisor (1.72) and subtract the number of
digits to the right of the decimal in your original
number (68.8). So you move the decimal one place to

43
8th_GRDM_03.qxd:Layout 1 8/11/09 1:05 PM Page 44

–DECIMALS AND PERCENTAGES–

the right: 4.0 would become 40, which is the correct calculator to compute 15 2 ; you’ll get 7.5, meaning that
answer. But because division involving decimals 0’s would go on as far as your calculator shows. This
doesn’t often come out to whole numbers, and be- is called a terminal decimal, because it just stops after
cause the algorithm is so complicated, it’s better just a while. In fact, all rational numbers are either termi-
to use an estimate to place the decimal once you’ve nal or repeating. For instance, 74, computed on the cal-
68.8
come up with the digits in the answer. For the 1.72 culator, is 0.571428571428, with the six-number
70
problem, the answer should be about 2 = 35, so once sequence “571428” repeating forever. Any time you
you’ve got the 4.0000, you know that 40 must be the divide with decimals, then, you’ll get a repeating se-
right answer. quence, although sometimes your calculator won’t be
powerful enough to tell you what the sequence is. Try
48.103456
Exercise 3 52.09876
, for example; unless you have a great calcula-
Estimate the following, and then find the exact an- tor, you won’t be able to find the repeating sequence.
swer with a paper-and-pencil method or a calculator.
Answers are on page 48.
Percentages
1. 6.6 × 6.6
You’ve often heard people using percentages to de-
2. 0.014 × 3.9902 scribe things.

3. 3.9902 ÷ 0.014 ■ “Frazier’s a terrible shooter. He’s making only


55% of his free throws.”
4. 0.014 ÷ 3.9902 ■ “About 20% of our school population is African
American; 15 years ago it was 45%.”
5. 6.6 ÷ 6.7231 ■ “I heard that 65.6% of all students at four-year
colleges are using student loans.”
6. 4 × 1,053.125 ■ “She gets a commission of 7.5% on each sale.”
■ “I got a 64% on the science test, but it was the
More about Decimal Division highest grade in the class.”
Decimal notation may be used to represent any num-
ber. Integers are simply decimal numbers with noth- In other words, percentages can help us make sense
ing but 0’s to the right of the decimal point. Also, of the world around us. But what are they? Mathe-
decimal numbers may be divided into two categories: matically, percentages are a method for expressing
rational numbers and irrational numbers. You may fractions as a decimal or as a whole number. In
think that one of these sets is just easier to get along other words, Frazier’s free-throw percentage means
55
with, but in fact, they have definitions that aren’t re- that he is making 100 free throws. But he may not
lated to how reasonable they are. A rational number even have shot 100 free throws—so how can he
is the quotient (that is the result of division) of two have a fraction with 100 in the denominator? Well,
2 15 1
numbers. Here are some rational numbers: 15 , 2 , 3, a percentage can be computed simply by dividing
3
and 1. Of course, they could also involve very large the denominator of any fraction into the numera-
48.103456
decimal numbers, such as 52.09876 . Irrational num- tor and then multiplying by 100. For example, if
bers are all the rest of the real numbers. Use your cal- Frazier has shot 20 free throws and made 11, his free-
11
culator to divide 2 by 15: You’ll see that 3’s end your throw percentage would be exactly 55%: 20 = 0.55,
answer as far as your calculator goes. Now use your
44
8th_GRDM_03.qxd:Layout 1 8/11/09 1:05 PM Page 45

–DECIMALS AND PERCENTAGES–

multiplied by 100. So you can calculate a percentage 0.5714 × 100 = 57.14%. If he misses it, he’ll have
for something that hasn’t happened 100 times. In made 11
21, and his percentage will be 52.38%.
fact he has a free-throw percentage after only one
shot. (It will be either 0% or 100%.) Exercise 4
Let’s compute Frazier’s shooting percentage af- Here’s a sequence of free throws shot and made by
ter his 21st free throw if he makes it, and after his 21st Billy Superstar. X represents a made shot and O a
free throw if he misses it. We’ll round to the nearest miss. Using a calculator when you have to, fill in his
hundredth, which means we’ll have to round our dec- percentage (rounded to the nearest tenth of a per-
imal to 4 places (or ten-thousandths). If he makes it, cent) with each success or failure. Answers are on
he will have made 12 21, and his percentage will be page 48.

SHOT # 1 2 3 4 5 6 7 8 9 10 11

X/O X O X X O X X X X O X

Introduction to instance, if you have a ratio of 53, you’ll have a problem


Ratios and Percentages to solve that looks like this:

3 ?
You may recall from earlier math that a ratio is just a =
5 100
relation between two quantities. In Exercise 4, you
were concerned with the ratio of the number of free You know that to make the denominators of these
throws made and the number attempted. The ratio is two fractions equal, you’ll have to multiply the top
expressed as a fraction; after Superstar’s 11th free and bottom of 53 by 20, which results in 100
60
. Now you
3
throw, the ratio of number made to number at- know that 5 = 60%. Let’s try a ratio that’s a little
86
tempted was 11 8
. And we can always express a ratio as a larger: 200 . In this case, you want to change your de-
72.7
percentage. In the Superstar case the percentage is 100 , nominator of 200 to 100, so you divide both 86 and
43
which is the same ratio, except that the comparison is 200 by 2, to get 100 , or 43%.
to 100. We use this as a standard so that we can com- Now, let’s apply what we know to the two basic
pare things more easily; otherwise, we’d have diffi- types of percentage/ratio problems.
culty comparing the free throw accuracies of Frazier,
who has taken 20 shots and made 11, and Superstar, Finding a Percentage of a
who has taken only 11 but made 8. At this point, it Given Quantity
appears Superstar is the superior shooter because his Suppose someone says, “About 20% of our school
percentage is 72.7% to Frazier’s 55%. population is African American; 15 years ago it was
You can always use a calculator to compute per- 45%.” If there are 850 students at your school, how
centages, but there’s also another way to do it, at least many are of African American descent? To figure this
with some numbers. Remember percentages are out, all you have to do is multiply the number of stu-
based on fractions with denominators of 100. Some- dents by the decimal equivalent of 20%, which is 0.20
times it’s easy to figure a percentage on a ratio if its (because percentages are always a decimal multiplied
denominator has a divisor in common with 100. For by 100). So, 0.20 × 850 = 170, and there are about that

45
8th_GRDM_03.qxd:Layout 1 8/11/09 1:05 PM Page 46

–DECIMALS AND PERCENTAGES–

number of students of African American descent cur- Now let’s use those figures to find percentage
rently in the school. Fifteen years ago, there were decreases and percentage increases. Here are the fig-
about 520 students in the school: How many African ures as we have them so far.
Americans attended the school then? The answer is
0.45 × 520 = 234.

TOTAL AFRICAN AMERICANS NON–AFRICAN AMERICANS

Then 520 234 286

Now 850 170 680

% of Decrease/Increase + 44.23% – ?% + ?%

It’s clear that the number of African Americans Exercise 5


has decreased, but by what percentage? To find the (You may use a calculator where you think it will
answer, we find the amount of the decrease: 234 – 170 help. Answers are on page 48.)
= 64. We then form a ratio based on the original
64
number: 234 . Then we use our percentage algorithm to 1. Last quarter about 32% of girls at your school
get (64 ÷ 234) × 100 = 27.4%. Now it’s also clear that made the honor roll. There are 103 girls at the
the number of non–African Americans has increased. school. How many girls were on the honor roll?
First, find the total number of the increase: 680 – 286
= 394. Then, we use the ratio 394 286 and apply our per- 2. All cell phones at BigBox Circuits were selling
centage algorithm to get 137.76%. at a 16% discount this week. How much would
Note that this last percentage is over 100%. a cell phone that normally costs $95 cost you
Sometimes percentages over 100% don’t make sense: this week?
How could someone have made more than 100% of
her free throws, or how could there be more African 3. You got 24 bull’s-eyes at the shooting range this
American students than the total number of stu- week. Last week you made only 19. By what
dents? But increases can be 100% or more. In fact, if a percentage did your number of bull’s-eyes
number exactly doubles, its percentage increase will improve?
be exactly 100%. (Why? Work some examples to see.)
However, you usually can’t have a decrease of more Finding a Quantity Given
than 100% in a real-world problem, because a de- a Percentage
crease of 100% will always leave you at 0. (Why? Now you know how to figure the value of a discount
Work some examples to see.) Notice also that if you when you know the original price. But let’s say you
multiply a percentage that’s less than 100 times any just bought a table for $21. You bought it at a sale that
number, you’ll get a smaller number. If you divide a offered 40% off all merchandise. So here you already
number by a percentage that’s less than 100, you’ll get know the amount you paid with the discount. How
a larger number. You know this also from the rules of can you figure out the original price, without the dis-
multiplying and dividing by fractions. count? First, you have to realize that if you got a 40%
discount, you were paying 60% of the original price.

46
8th_GRDM_03.qxd:Layout 1 8/11/09 1:05 PM Page 47

–DECIMALS AND PERCENTAGES–

Now, you can divide your $21 price by 0.60, and you’ll 3. Bill bought a new basketball at 25% off. He
get the original price of $35. paid $18; how much would the basketball have
When you’re given a percentage but don’t know cost without the discount?
the total or the original number, you usually divide by a. $72
the percentage, because you know that the original b. $54
number was larger, and dividing by a percentage less c. $24
than 100% will give you a larger number. But in the d. $15
previous problem, what if you’d forgotten to subtract
the 40% from 100%, divided 21 by 0.40, and got an
answer of $52.50? That’s a larger number than $21, so Summary
how would you know you were wrong? You can check
this answer just the way you can check most answers: In this chapter you’ve learned how to do some opera-
Work backwards. If the original price really was tions with decimal numbers and with percentages.
$52.50, a 40% discount would be .40 × $52.50 = $21. Maybe it’s helped you understand integers and frac-
And with a $21 discount you’d be paying $52.50 – tions a bit better as well. Let’s hope so, because you’ll
$21.00 = $31.50, quite a bit more than you actually need a thorough understanding of the number sys-
paid. So you know that you need to start over. (You tem in eighth grade! Now it’s on to exponents.
should also notice here that $52.50 and $31.50 will al-
most certainly be answer choices on multiple-choice
tests, so you’ll need to know exactly what the problem Answers
is asking for.)
Exercise 1
Exercise 6 1. 0.0004
Estimate and answer should be the same.
1. You just bought your mother a set of earrings 2. 0.7 (estimate), 0.693 (exact)
on sale for $25. The original store price was 3. 2.2 (estimate—about 1.1 on each side of 0 on
$40. How much of a discount did you get? the number line), 2.1578 (exact; remember that
a. 60% two negatives result in a positive in this
b. 37.5% problem)
c. 25% 4. 0.02 (estimate), 0.0163 (exact)
d. 23.1% 5. 0.7 (estimate), 0.6929 (exact)

2. You paid 20% of the original cost for a blanket.


The blanket cost you $120; how much was the
original cost?
a. $600
b. $420
c. $270
d. $150

47
8th_GRDM_03.qxd:Layout 1 8/11/09 1:05 PM Page 48

–DECIMALS AND PERCENTAGES–

Exercise 2 Exercise 4
1. 1,454.69 (decimal point 2 places to the right) After 1: 100%
2. 0.145469 (decimal point 2 places to the left) After 2: 50%
3. 0.318 (decimal point 1 place to the left) After 3: 66.7%
4. 97 (decimal point 3 places to the right) After 4: 75%
5. 0.000097 After 5: 60%
Here we have to think of the original number After 6: 66.7%
as having at least three 0’s in front of it so that After 7: 71.4%
we can move the decimal point 3 places to the After 8: 75%
1,000 → 0.000097; also note that “/” is the
left: 0000.097 After 9: 77.8%
same as “÷.” After 10: 70%
6. 0.0012 After 11: 72.7%
Same reasoning as the preceding; note also that
the original number doesn’t need to have a Exercise 5
decimal point, as in this case it was assumed. 1. 33
0.32 × 103 ≈ 33; notice that using fractional
Exercise 3 parts of people doesn’t make sense in this
1. 42 (estimate, 6 × 7), 43.56 (exact) problem.
1 1
2. .04 (estimate: 10 of 4 is 0.4, and 100 of 4 is .04), 2. $79.80
0.0558628 (exact) 0.16 × $95.00 = $15.20, which is the 16%
1
3. 400 (estimate: 4 ÷ 10 is the same as 4 multiplied discount. Because fractional parts of dollars do
by 10 [review Lesson 2 if necessary], and make sense, subtract $15.20 from $95.00 to see
1
4 ÷ 100 is the same as 400), 285.01 (exact, how much you’ll be paying after the discount:
but rounded) $79.80.
4. .0035 (estimate: 144 is 3.5; note that we had to 3. 26.3%
move the decimal one place to the right when You got 24 – 19 = 5 additional bull’s-eyes. Set
dividing), 0.00351 (exact, but rounded) up a ratio of your improvement to your
5. slightly less than 1 (estimate: 6.6 and 6.7 are original number to get 195
≈ 26.3%.
very close, but 6.6 is slightly smaller), 0.982
(exact, but rounded) Exercise 6
6. 4,000 (estimate: 4 × 1,000), 4,212.5 (exact) 1. b. Price reduction was 40 – 25 = $15, and
15
40 = 37.5%.
2. a. 20% of the original cost was $120, and
120
.20 = $600.
3. c. Bill paid 75% of the original price, and
$18
0.75 = $24.

48
8th_GRDM_03.qxd:Layout 1 8/11/09 1:05 PM Page 49

4
L E S S O N

EXPONENTS

L E S S O N S U M M A RY
3
23, 24, 2–3, 2 4 : How are these different? Knowing operations with
different kinds of exponents—positive, negative, and fractional—
will make algebra easier. Here you’ll find all the information you
need to compute using those different kinds of exponents.

I n this lesson, we’ll review what a positive exponent means, and you’ll learn about negative and fractional ex-
ponents. Then you’ll learn how to divide and multiply when you have expressions with exponents, and
you’ll learn how to use a formula that has an exponent in it to figure out how a bank account will grow. Fi-
nally, you’ll learn something called “scientific notation.”

49
8th_GRDM_03.qxd:Layout 1 8/11/09 1:05 PM Page 50

–EXPONENTS–

Positive Exponents Exercise 1

You learned in earlier grades that an exponent indi- 1. 27 =


cates that a number is multiplied times itself a certain
number of times, depending on the size of the expo- 2. (–3)2 =
nent. For example, 32 means 3 × 3. This also means
that 32 has a value of 9. Another way of saying 32 is “3 is 3. (–4)2 =
raised to a power of 2” and still another way is “3
squared.” What is the value of 25? It’s 2 × 2 × 2 × 2 × 2. 4. (–2)5 =
This string of multiplications has a value of 32, and
one can say that “2 to the fifth power is 32.” 5. (–3)3 =
What happens if you have a negative number
raised to a positive power? Let’s take the number –2 6. 26 =
and raise it to the third power (this can also be read as
“–2 cubed”). So (–2)3 is (–2) × (–2) × (–2). How will 7. (–2)6 =
we evaluate this expression? Let’s start with the first
two terms: (–2) × (–2). Because we have two nega- 8. 53 =
tives, our answer will be positive 4 (review Lesson 1 if
necessary). So now we have 4 × (–2) = –8, so (–2)3 =
–8. If we want to know what (–2)4 is, we’ll just multi- Negative Exponents
ply (–8) × (–2) and get positive 16. Do you see a pat-
tern developing? Operations with positive exponents are straightfor-
ward, but those with negative exponents involve frac-
tions. That’s because a negative exponent is defined as
Negative numbers raised to an even, positive a fraction. Any number x raised to the (–n) power,
1
power are positive, and negative numbers where n is a positive integer, is defined as x n . So 2–3 is
raised to an odd positive power are negative. defined as 18, and 3–2 is defined as 91. Do you see the re-
lationship of positive exponents to negative expo-
nents? Correct: A number raised to a negative power
A number, by the way, can also be raised to the is the reciprocal of the same number raised to the
first power; in that case, it’s just the number itself. For same positive power. We’ve mentioned reciprocals
example, 51 = 5, 81 = 8, and (–7)1 = –7. The number before as “reverse fractions” (see Lesson 2), but let’s
–1 is a special case: (–1)n is 1 if n is positive and –1 if n look at them a little more closely. What is 5 × 15? What
is negative. Quick—what’s (–1)483? Answer: –1. is 92 × 29? What is –81 × (–8)? The answer to all three
Positive exponents are pretty easy to deal with, questions is the same: 1.
so let’s go right to the exercise. (Use calculators only Now here’s another question about exponents:
to check.) What is 51 × 5–1? It’s 5 × 51 = 1. And what’s 51 × 52? It’s
5 × 5 × 5 = 125, which is the same as 53. This last cal-
culation seems to indicate that we can just add expo-
nents while multiplying, so that 51 × 52 = 5(1 + 2) = 53.
But then how about 51 × 5–1? Does it also equal 5(1 + –1)
= 50? Well, it does if you define 50 to equal 1, which is

50
8th_GRDM_03.qxd:Layout 1 8/11/09 1:05 PM Page 51

–EXPONENTS–

what mathematicians did a long time ago, so that How do we find square roots? There is a way to
multiplication and division would work out more calculate them, and if you’re interested, you can look
easily. In fact, any number raised to the 0 power is it up on the Internet. But in eighth grade, you’ll just
equal to 1. To see why this makes sense, we’ll skip be expected to memorize a few common ones and use
ahead to division of numbers raised to different pow- a calculator to find others. You may also be expected
ers. As you might expect from the multiplication rule, to give a good estimate of ones you don’t know. But
3
in division you subtract exponents, so that 52 = 5(3 – 2) first, let’s take a look back at the statement in the last
5 1 1 1

= 51. Also, 53
= 1, because any number divided by paragraph that 4 2 = 2, 99 2 = 3, and 25 2 = 5. This state-
53 53 ment is only half true. We know that 2 × 2 = 4, so it is
itself is 1. Another way of thinking about 53 is 5(3 – 3) 1
true that 4 2 = 2. But (–2) × (–2) also equals 4, so –2 is
= 50 = 1. So does it make sense to you to make 50 = 1?
also a square root of 4.
Doesn’t matter: That’s the way it is, and your life will
be a lot easier if you accept that fact.
Now let’s practice some multiplication and divi-
Every positive number has two square roots
sion (no calculators needed!). Answers are on page 55.
that are exactly the same except that they have
1
opposite signs. That is, n 2 = n and – n ,
Exercise 2
which can also be written as ± n .
43
1. =
41

Note that negative numbers don’t have a square


2. 62 × 6–1 =
root, because a positive times a positive and a nega-
tive times a negative both have positive products.
3. 114 ÷ 113 =
Negative numbers’ square roots are imaginary, and
(−1)44 you’ll learn about imaginary numbers in high school.
4. =
(−1)50 For now, all you need to know about them is that they
exist.
5. 35 ÷ 36 = 1
Now let’s consider the cube root of 8, or 88 3 : Is
there more than one of these? It turns out that 2 × 2 ×
6. 212 × 2–9 =
2 = 8, so 2 is a cube root of 8. How about –2? Well,
(–2) × (–2) × (–2) = 4 × (–2) = –8, so (–2)3 = –8,
and –2 is not a cube root of 8. In fact, every odd-
Roots and Fractional Exponents
numbered root has only one solution, because an
odd number of negatives multiplied together pro-
A root of a number n is a number that, multiplied by
duces a negative number, and an even number of
itself a specified number of times, will result in a
negatives multiplied together produces a positive
product of n. A number m that will equal n if multi-
number. For example,
plied by itself once (that is, m × m = n) is defined as
1
nn 2 , and is commonly called the square root of n. 1
1 53 = 5 × 5 × 5 = 125 and 125 3 = 5
Here are some well-known examples of roots: 4 2 = 2,
1 1 (–5)3 = (–5) × (–5) × (–5) = –125, and
9 2 = 3, and 25 2 = 5. A square root may also be 1
–1255 3 = –5
written as 25 , for example. It’s also possible to have
1 54 = 5 × 5 × 5 × 5 = 625
cube roots (n 3 ), which may also be written like this:
3 (–5)4 = (–5) × (–5) × (–5) × (–5) = 625, so
n. 1
625 4 = ±5
51
8th_GRDM_03.qxd:Layout 1 8/11/09 1:05 PM Page 52

–EXPONENTS–

The pattern is that odd-numbered roots are not Exercise 3


doubled, but that negative numbers may have odd- (Answers are on page 55.)
numbered roots. Even-numbered roots exist only for
1
positive numbers, but two of each exist for each posi- 1. 64 3 =
tive number.
How do you estimate something like 3 300 ?
2
2. –125 3 =
You can do it through trial and error and get fairly
close. Let’s start with a cube we know: 10 × 10 × 10 =
2
3. 125– 3 =
1,000, which is much larger than 300. On the other
hand, we just calculated 53 and got 125, so 5 is too
5
4. 16 4 =
small. We next calculate 63 and get 216, and 73 is 343,
1
5. 16 4 × 16– 4 =
5 1
so 300 3 lies somewhere between 6 and 7. That’s prob-
ably close enough if you don’t have a calculator, but if
2 2
you feel like doing some extra multiplying (or if your 6. 27 3 × 27 3 =
teacher has specified that you find the cube root of
5
300 to the nearest tenth), you can go on with your es- 7. 8– 3 ÷ 8–1 =
timation. Because 300 is closer to 343 than to 216,
1
let’s next try 6.63 (287.5) and 6.73 (300.8). Since 6.7 8. (–64)– 3 =
got you a lot closer to 300 than 6.6, and because 6.83
3
will result in a higher number, 6.7 is the cube root of 9. (–16)2 =
300 to the nearest tenth.
Of course, not all fractional exponents are posi-
tive, and not all have 1 in the numerator. Here are a Using an Interest Formula
couple more definitions with examples:
Ms. Jung puts $4,000 into a savings account that pays
2
n means the cube root of n squared: ( n ) or
2
3
3
her an annual rate of 3%, compounded annually.
3 2
n This means that at the end of the first year, she will
Example: 3 2
82 = ( 3 8 ) = 4 get .03 × $4,000 = $120.00. Then she’ll have $4,120.00
2 1 in the bank for her second year. At the end of the sec-
n– 3 means
ond year she’ll get 3% of her new balance, or .03 ×
2
n3
– 23
Example: 8 = 41 $4,120.00 = $123.60, and her new balance will be
$4,243.60.
Be sure that you understand these examples be- There’s a formula that will tell her how to figure
fore trying the following exercises. Because the num- out how much money she’ll have at the end of each
bers are low and all roots are integers, you won’t need year, and in Section 2 you’ll see how to arrive at that
a calculator as long as you remember that 22 = 4, 23 = 8, formula. The formula is
24 = 16, 25 =32, 32 = 9, 33 = 27, 34 = 81, 42 = 16, 43 = 64,
52 = 25, 53 = 125, and 54 = 625. A = P(1 + i)n

In this formula A stands for the amount of money at


the end of any year, P is the money you start with (of-
ten called the principal), i is the annual interest rate,
and n is the number of years.

52
8th_GRDM_03.qxd:Layout 1 8/11/09 1:05 PM Page 53

–EXPONENTS–

We’re going to do some calculations with this Exercise 4


formula, with different interest rates and with differ- Using the formula A = P(1 + i)n, figure out how much
ent numbers of years. You should get a good idea of money a depositor would have with an initial deposit
how numbers are affected by being raised to different of P and an annual interest rate of i for n years. For
powers, and what difference an interest rate makes. the last three problems, also figure out how long it
You can use your calculator on these, because we’ll be would take the original deposit to increase by 50%.
dealing with lots of decimal places, but you’ll have to Round dollar amounts to the nearest penny; you’ll
know how to decide that your answer is reasonable. definitely need a calculator. Answers are on page 55.
First, let’s use Ms. Jung’s initial deposit and interest
rate. How much money will she have after five years? 1. P = $300, i = 4%, n = 3
After seven years?
2. P = $3,000, i = 8%, n = 3
five years: A = $4,000(1 + .03)5 = $4,637.10
seven years: A = $4,000(1 + .03)7 = $4,919.50 3. P = $1,000, i = 3.6%, n = 10

Did you notice that 1.035 is about 1.16, and that 4. P = $3,000, i = 9%, n = 3
1.037 is about 1.23? If a number is just a little over 1, it
grows slowly as its exponent increases. Compare the 5. P = $10,000, i = 1.8%, n = 30
exponential growth of 1.03 to that of 2: 25 = 32, and
27 = 128—a much larger percentage increase. 6. P = $1,000,000, i = 6%, n = 5
Now, let’s figure out how long it will take Ms.
Jung’s money to double. That is, after how many years
will she have $8,000? There is a mathematical method Scientific Notation
for solving this problem quickly, but eighth graders
have to do it by trial and error. The first thing to no- One common use of exponents is in the description
tice is that we don’t have to multiply by $4,000 for of very large and very small numbers, and in the cal-
every trial: We just need 1.03n to be greater than or culation of their products and quotients. These sorts
equal to 2. If you try 1.03 to the 20th power, it is in of calculations are done every day in scientific set-
the 1.8 range, and 1.0325 is well over 2. If you try tings, and not only by highly educated scientists;
1.0323 and 1.0324, you’ll see that at the end of the 24th they’re also done frequently in experiments assigned
year, her money will be slightly more than double to high school chemistry and physics students.
what she started with. The principle is simple: You can take any num-
What if the interest rate were 10% instead of ber and adjust it so that it’s a number with exactly one
3%? If you use your calculator to figure some values digit to the left of the decimal point and then multiply
of 1.10n, you’ll see that she doubles her money in just it by 10 to some power. For example, 496.12 in scien-
eight years and that in 24 years (the time it took to tific notation is 4.9612 × 102, and 0.49612 is 4.9612 ×
double her money at 3%), Ms. Jung would have 10–1. This notation helps us to see quickly how big or
nearly ten times her original deposit. Is it any wonder small a number is without counting how many digits
that some people would fall for a thief ’s scheme that there are before or after the decimal point. And it also
would promise a 10% return instead of putting helps you to compare numbers to one another: Be-
money into a relatively safe investment that paid 3% cause one number is “× 102” and the other is “10–1”
annually?

53
8th_GRDM_03.qxd:Layout 1 8/11/09 1:05 PM Page 54

–EXPONENTS–

you can easily tell that there are three powers of 10 4.0 × 104 ⇒ 40,000
(that is, 1,000) between the two numbers. 1.021 × 10–4 ⇒ 0.0001021
So now let’s do some calculations. Addition and 7.089 × 105 ⇒ 708,900
subtraction are not often done using scientific nota- 8.8188 × 10–1 ⇒ 0.88188
tion, because if there’s a big difference between num-
bers, adding the little number to the big number Now let’s look at multiplication of numbers in
won’t usually make a lot of difference. Therefore, if scientific notation. Example: 1.0 × 106 × 3.1 × 10–6.
you want to add two numbers that are in scientific The first thing you do is multiply the numbers, in this
notation, and the powers of 10 are different, it’s easi- case 1 and 3.1. So now you have 3.1 × 106 × 10–6. Next
est to convert those numbers to standard decimal add the exponents: 6 + (–6) = 0, so we have 3.1 × 100
notation. = 3.1 × 1 = 3.1. Now let’s do 1.0 × 106 ÷ 3.1 × 10–6.
Here we’ll divide 1 by 3.1 using a calculator to get
Example: 1.0 × 106 + 3.1 × 10–6 0.3226, rounded to four places, so now we have
0.3226 × 106 ÷ 10–6. Remember that when we divide
To convert a number in scientific notation to standard like numbers with exponents, we subtract. So we have
notation, write down your first number and fill in the 10 to the power of 6 – (–6) = 12, and our quotient is
rest of it by counting the number of spaces right to the 0.3226 × 1012. Unfortunately, this isn’t quite in scien-
decimal point. The first number in the example is tific notation, because there’s no digit to the left of the
written in standard notation as 1,000,000. If the expo- decimal point (0 doesn’t count). To get exactly 1 digit
nent of 10 is negative, you count 1 less than the ab- to the left of the decimal point, we’ll have to multiply
solute value of that exponent left to the decimal point. by 10, but we can’t change the overall value of our
The second number in standard notation is .0000031. number. Therefore, if we multiply the number by 10
This sounds complicated, but it’s really just multiply- (101), we’ll also have to multiply by its reciprocal (10–1).
ing by one-millionth (see Lesson 3 if you need to re- So we have (0.3226 × 101) × (1012 × 10–1) = 3.226 × 1011.
view). Just looking at these two numbers, it’s obvious One more multiplication and we can go on to
that their sum is 1,000,000.0000031. In scientific nota- Exercise 5:
tion, this would be 1.0000000000031 × 106.
5 × 103 × 8 × 108 = 40 × 1,011 =
Let’s pause here to convert some other numbers
(40 × 10–1) × (108 × 10–1) = 4 × 107
from scientific to standard notation and vice versa. The
number 3.4 × 1010 is equivalent to 34,000,000,000;
Be sure that you understand each step of this cal-
there are ten places following the 3 (remember that
culation.
the single number to the left of the decimal point is
your reference point). The number 6 × 10–3 is
Exercise 5
equivalent to 0.006; there are two 0’s places to the
(No calculators, except to check. Answers are on
right of the decimal point, not three. Note that in
page 56.)
fractional form 6 × 10–3 can be represented as 1,000 6
,
which is the same as 0.006 in decimal form. This 1. 3 × 105 + 3 × 10–5 =
may help you to check whether you’ve got the con-
version algorithm right. 2. 3 × 105 +3 × 105 =
Take a look at the following conversions. Make
sure you understand each before trying Exercise 5. 3. 4.2 × 104 × 3 × 10–2 =

4. 8.1 × 105 ÷ 9 × 10–5 =

54
8th_GRDM_03.qxd:Layout 1 8/11/09 1:05 PM Page 55

–EXPONENTS–

Summary Exercise 3
1. 4
In this lesson, you’ve reviewed the meaning of positive 4 × 4 × 4 = 64
and negative exponents, the meaning of roots and frac- 2. 25
tional exponents, and how to do basic number opera- Cube root of –125 is –5, and –52 = 25.
1
tions with all forms of exponents. You’ve also reviewed 3. 25
scientific notation and learned something about the Cube root of 125 is 5, and 52 = 25; negative
behavior of formulas that include exponents. sign in the exponent means the reciprocal
of 25.
4. 32
Answers Fourth root is 2; 25 = 32.
5. 16
Exercise 1 Adding exponents, 45 – 41 = 44 = 1; 161 = 16.
1. 128 6. 81
1

2. 9 Adding exponents, 32 + 32 = 34; 27 3 = 3, and


3. 16 34 = 81.
2

4. –32 7. 41 (adding exponents, – 35 – (–1) = – 32; 8– 3


1
5. –27 = 3 8 2 = 41)
6. 64 8. – 41
7. 64 – 641 has a cube root of – 41.
8. 125 9. Answer doesn’t exist; the square root of 16 is
imaginary.
Exercise 2
3
1. 16 ( 41 = 4(3 – 1) = 42) Exercise 4
4
1. $337.46 (1.043 ≈ 1.12)
2. 6 (6 × 6–1 = 6(2 + –1) = 61)
2
2. $3,779.14 (1.083 ≈ 1.26)
3. 11 (111)
3. $1,424.29
4. 1 (1–6)
1 –1 1.03610 ≈ 1.42; note that 3.6% is .036, not .36!
5. 3 (3 )
4. $3,885.09
6. 8 (23)
1.093 ≈ 1.30; money would increase by 50%
(would be 1.5 times the original deposit,
meaning 1.09n ≥ 1.5) after five years.
5. $17,077.86
1.01830 ≈ 1.71; money would increase by 50%
(would be 1.5 times the original deposit,
meaning 1.018n ≥ 1.5) after
23 years.
6. $1,338,225.58
1.065 ≈ 1.34; money would increase by 50%
(would be 1.5 times the original deposit,
meaning 1.06n ≥ 1.5) after seven years.

55
8th_GRDM_03.qxd:Layout 1 8/11/09 1:05 PM Page 56

–EXPONENTS–

Exercise 5
1. 300,000.00003, or 3.0000000003 × 105
3 × 105 = 300,000; 3 × 10–5 = .00003
2. 6 × 105, or 600,000
300,000 + 300,000 = 600,000
3. 1.26 × 103, or 1,260
12.6 × 102 = 12.6 × 10–1 × 102 × 101 = 1.26 × 103
4. 9 × 109
0.9 × 1010 = 0.9 × 101 × 1010 × 10–1 = 9 × 109

56
8th_GRDM_03.qxd:Layout 1 8/11/09 1:05 PM Page 57

5
L E S S O N

ORDER OF
OPERATIONS

L E S S O N S U M M A RY
“Please excuse my dear Aunt Sally.” If you haven’t heard this sen-
tence before, you will in this lesson, which will review the order of
operations in a number sentence and will also give you some tips
about how to practice with harder calculations.

I n the sentence “Please excuse my dear Aunt Sally,” the first letter of each word stands for the order that op-
erations should be in:

■ Parentheses
■ Exponents
■ Multiplication and Division, left to right
■ Addition and Subtraction, left to right

You may want to create your own personal sentence with the letters PEMDAS to remember the order of
operations.

57
8th_GRDM_03.qxd:Layout 1 8/11/09 1:05 PM Page 58

–ORDER OF OPERATIONS–

Why Is an Order of The original problem now looks like this:


Operations Necessary?
22 + 1 – 6 × 3 = ?
You may be wondering why you even need to follow
the order of operations. Does it really matter? Let’s Calculate the exponent: 22 = 4. Now you can further
look at what happens when you ignore PEMDAS and simplify the problem:
attack a problem by instinct:
4+1–6×3=
9+8×2–3×2
17 × 2 – 3 × 2 The next stage of PEMDAS, the MD, indicates that
34 – 3 × 2 you should do all the multiplication and division
31 × 2 from left to right. There is no division to worry about,
62 but there is multiplication: 3 × 3. This equals 9. So
your problem is now
Is this answer correct? Because you ignored PEMDAS,
this isn’t the right answer. No worry—now proceed in 4+1–9=?
the correct order. There are no parentheses or expo-
nents, so we’ll start with any multiplication or divi- You’re almost there! Perform all addition and sub-
sion first from left to right. traction. Remember to do this from left to right.

9+8×2–3×2 4 + 1 = 5 and 5 – 9 = –4
9 + 16 – 3 × 2
9 + 16 – 6 So 22 + (6 – 5) – (3 + 3) × 3 = –4.

Now complete the addition and subtraction from left Exercise 1


to right. Use the order of operations to find the answers. Show
your work. Answers are on page 62.
9 + 16 – 6
25 – 6 1. 8 + 15 × 3 =
19
2. 7 + 24 ÷ 6 × 10 =
Wow—without using the correct order of operations,
the first answer wasn’t even close to the actual value! 3. (36 + 64) ÷ (18 – 20) =
Remember, take your time and carry out each opera-
tion in the correct order.
Let’s look at another example: Parentheses Other
Grouping Symbols
22 + (6 – 5) – (3 + 3) × 3 = ?
The first step is to perform any math operations
Begin by completing any work inside the parentheses. located inside parentheses. We’ve been using parenthe-
ses so far in this book without explanation. Parentheses
(6 – 5) = 1 and (3 + 3) = 6 are one type of grouping symbol that can be used to

58
8th_GRDM_03.qxd:Layout 1 8/11/09 1:05 PM Page 59

–ORDER OF OPERATIONS–

make a mathematical statement clear. Other sym-


bols include [ ] and { }. We’ll use only [ ] in this If your calculator has parentheses keys, then it
book, and we’ll use them to show a grouping larger will most likely perform the correct order of op-
than what is in parentheses. For example, consider erations. Check your calculator with these ex-
the expression [6 + 3 × (6 – 1)] +5. Here we really amples to see if it performs the correct order of
have two sets of grouping symbols, an inner and an operations.
outer set. We always start with the innermost set of ■ To evaluate 6 – 100 ÷ 5, enter the num-
grouping symbols. bers as they appear. Your calculator
should show a result of –4.
[6 + 3 × (6 – 1)] + 5 ■ To evaluate 48 ÷ (4 + 2), again enter the
[6 + 3 × 5] + 5 numbers as they appear. (Don’t forget
[6 + 15] + 5 about the opening and closing parenthe-
21 + 5 = 26 ses!) Your calculator should show a result
of 8.
This expression could also have been written [6 + 3
(6 – 1)] + 5; that is, we could have left out the multi-
plication sign, because two numbers next to each
other separated only by a parenthesis always imply Absolute Value
multiplication. In fact, you’ll see later that a number In Lesson 1, you learned the meaning of absolute
and a variable don’t even need to have any sign to im- value. When you are using the order of operations,
ply multiplication: The expression 2n means 2 × n. the absolute value symbol is treated at the same level
Also, remember that you can always use paren- as parentheses.
theses or other grouping symbols so that the order of Try this out:
operations is not even particularly important, except
5 × |–13 + 3|
in the way you put the calculations into your calcula-
tor. For example, you could write 9 + 8 × 2 – 3 × 2 as:
First, evaluate the expression inside the absolute value
symbol:
9 + (8 × 2) – (3 × 2)

5 × |–10|
This grouping would force the person doing the cal-
culation to do it in the same order as if it had no
Now, evaluate the absolute value: |–10| = 10, so
parentheses: first, multiplication and then, addition
5 × 10 = 50.
and subtraction. But if you know order of operations
well, you can input a string of numbers and opera-
tions into a calculator without using parentheses.

59
8th_GRDM_03.qxd:Layout 1 8/11/09 1:05 PM Page 60

–ORDER OF OPERATIONS–

Exercise 2 And speaking of canceling, what if we started


(Answers are on page 62.) out with this expression?

1. 100 ÷ 5 + |–5 × 3| = 3× 8 20
9+8× −
16 5 − 10
2. 5 × –|9| = Here we could cancel out the 8 in the numerator
of the first fraction and change the 16 in the denomi-
3. 99 ÷ |–33| = nator to 2 (refer to Lesson 2 if you don’t understand
this). But can we do any canceling on the second frac-
4. 17 – (8 ÷ |1 – 3|) × 3 = tion? The answer is no, because the operation in the
denominator of that fraction is subtraction rather
5. |14 – (2 – 12)| ÷ 12 = than multiplication.

Remember that you can only cancel numbers


Fractions
that are being multiplied or divided, not added
or subtracted.
Fractions often don’t have parentheses or other
grouping symbols, but they should still be evaluated
as if they do. Let’s do this calculation:
So after evaluating the fractions in this expres-
5 + 1 20 sion, we have
9+8× −
16 5 − 3
3 20 3
Even though the fractions don’t have parentheses 9 + 8 × − = 9 + 8 × − (−4) = 9 + 12 − (−4)
2 −5 2
around them, we treat them as if they do. Then, after
evaluating the operations within the fractions, we = 9 + 12 + 4 = 25
have

6 20 Exponents
9+8× −
16 2
When we talk about evaluating exponents, we mean
At this point, we can also put the fractions into their
that you should carry out calculations involving ex-
simplest form: 2
ponents, such as 26 = 64 or 8– 3 = 14. But there’s an-
3 other rule for operations involving fractional
9 + 8 × − 10 exponents, especially those that don’t result in ra-
8
tional numbers. Some of the most common irrational
If you remember how to multiply fractions (and use
numbers you’ll encounter in eighth grade and high
canceling), our expression becomes
school math are 2 and 3 . It’s usually best to leave
roots like these in their exponent forms without cal-
9 + 3 – 10 = 2
culating them, especially if you’re doing paper-and-
pencil work. For example,

1
6 × 19 − 16 + 8 − 3 × 3 2

60
8th_GRDM_03.qxd:Layout 1 8/11/09 1:05 PM Page 61

–ORDER OF OPERATIONS–

If you work this out on your calculator, you’ll get an Then we’ll add and subtract: 49 + 5 + 1 – 26 = 29.
answer of 13.196 followed by a lot of other numbers.
But if you first simplify the expression, you’ll have
Remember that in addition and subtraction, a +
6× 3 + 8− 3× 3 = 3 3 + 8 and a – result in a negative; a – and a – result in
a positive.
Now solving the problem using the calculator is much
easier. Why can you subtract 3 3 from 6 × 3 ? For
Here are some practice problems to end this chapter
the same reason you can subtract 3 × 2 from 6 × 2: It’s
with. The numbers are low, so you shouldn’t need a
the distributive property of multiplication: (b × a) –
calculator. Answers are on page 62.
(c × a) = a(b – c).

Exercise 3
Practice with Negatives
1. 33 – 8 × 3 – 6 ÷ 2 =

When you’re evaluating a series of operations, keep-


2. 14 × 3 – (4 – 1 – 6)2 + (1 – 4)3 =
ing positives and negatives straight is one of the hard-
est parts. One thing that might help with some of
3. –8 – (43 ÷ 24) =
these calculations is to think of a negative sign as
multiplication by –1 and change the sign to a +. For
4. –8 – [(–4)3 ÷ 24] =
example, –3 = (–1)(3). Likewise, 3 = (–1)(–3). This is
useful when evaluating fractions by canceling.
5. –8 ÷ [(–4)3 ÷ 24]2 =
1 1
– —
3–8
—— — — –5 (–1)5 5 6. (4 – 2 × 6 + 15)–1 =
3 = 3 – —– = 3 – ——— = 3 – – = 3 – 1 = 2
1 + 2(–3) –5 (–1)5 5
1 1
7. 6–1 × 3 + 104 ++22 (–3) =
The –1’s in the numerator and denominator cancel
each other out, and so do the 5’s, so your fraction is
simply 11, or 1. Summary
Now let’s work a series of operations that in-
volve double and triple negatives. In this chapter you’ve learned how to do some com-
⎛ 1⎞ plicated operations in the right order. Even though
7 2 + ⎜ − ⎟ (−3 − 7) − (−1) + (−26) = ? your calculator will do many of these operations for
⎝ 2⎠
you, it’s important that you know how to do them on
If we take operations that are inside parentheses paper as well, because you need to check your work
first, we get –3 – 7 = –10, so we have on the calculator, and on paper it’s much easier to see
⎛ 1⎞ where you’ve made an error if you’ve made one. (And
7 2 + ⎜ − ⎟ (−10) − (−1) + (−26) = ? no matter how good you are, you’ll make errors
⎝ 2⎠
sometimes.)
Now evaluate 72 and then multiply (– 12) (–10) to get

49 + 5 – (–1) + (–26)

61
8th_GRDM_03.qxd:Layout 1 8/11/09 1:05 PM Page 62

–ORDER OF OPERATIONS–

Answers Exercise 3
1. 0
Exercise 1 27 – 8 × 3 – 6 ÷ 2 = 27 – 24 – 3 = 0
1. 53 2. 6
In this problem, there are no parentheses or 14 × 3 – (–3)2 + (–3)3 = 14 × 3 – 9 + (–27) =
exponents, so evaluate multiplication first: 42 – 9 –27 = 6
8 + 45. Now perform the addition to get 53. 3. –12
2. 47 –8 – (64 ÷ 16) = –8 – (4) = –12
No parentheses or exponents, so evaluate 4. –4
multiplication and division, from left to right. –8 – [(–64) ÷ 16] = –8 – (–4) = –4
First, do the division: 7 + 4 × 10. Next, perform 5. –21, or –0.5
multiplication: 7 + 40 = 47. –8 ÷ [(–64) ÷ 16]2 = –8 ÷ (–4)2 =
8
3. –50 –8 ÷ 16 = – 16 . Reduce to the simplest form of
First, evaluate what’s in the parentheses, from the fraction to get – 21, or divide –8 by 16 to get
left to right: 100 ÷ –2. Next, do the division: – –0.5.
50. 6. 71
4 – 12 + 15)–1 = 7–1 = 71
Exercise 2 7. –1
3
1. 35 1 1 6 (–1)(3)
6–1 × 3 + 12 6
(–3) = –6 × 3 + —– 12 × –3
The absolute value symbol serves as a grouping – 4 —–
2 1 1
symbol, and grouping symbols are evaluated 2
first: |–5 × 3| = 15. Now, divide 5 into 100 to = 12 + ⎛⎜ −3 ⎞⎟ = –1
⎝ 2⎠
get 20. Finally, 20 + 15 = 35.
2. –45
–|9| is equal to the opposite of the absolute
value of 9, or –9; 5 × –9 = –45.
3. 3
|–33| is equal to 33. What times 33 equals 99?
The answer is 3.
4. 5 (First, evaluate |1 – 3|, the innermost
grouping, which is |–2| = 2. Then, the
expression in parentheses, 8 ÷ 2 = 4. Now you
have 17 – 4 × 3 = 17 – 12 = 5.)
5. 2
First, evaluate the innermost grouping,
(2 – 12) = –10. Then, the absolute value:
|14 – –10| = |24| = 24. Then, 24 ÷ 12 = 2.

62
8th_GRDM_04.qxd:Layout 1 8/11/09 12:59 PM Page 63

6
L E S S O N

ESTIMATING
AND CHECKING

L E S S O N S U M M A RY
When you’re trying to figure something out in real-life situations,
you often need to have a good estimate, and you have to know
whether your estimate is higher or lower than the exact answer. In
some of those real-life situations (and on all math tests), it’s also im-
portant to know the exact answer. Also you have to know how to
check to make sure your answer is exactly right. In this lesson,
you’ll learn how to do both: estimate and check.

W e’ve been rounding numbers in most of the lessons so far, but in this lesson you will review
rounding and learn when it’s appropriate to round up and when it’s better to round down. We
will also do some rounding with very large and very small numbers. Then we’ll do some work
with reversing operations to check answers, and we’ll explain how to check that you have the right answers to
word problems.

63
8th_GRDM_04.qxd:Layout 1 8/11/09 12:59 PM Page 64

–ESTIMATING AND CHECKING–

Rounding Now we’ll do a number on which we want to


overestimate: 764,931 to the hundreds place.
The algorithm for rounding is straightforward, but it
takes a lot of words to say. Review these steps to the 1. 764,931
algorithm, and be sure you understand the proce- 2. 764,900
dure. 3. 765,000 (Remember, we change the 9 to 0 and
add 1 to the thousands place.)
1. Find the digit in the place you’re rounding to; 4. Step 2 produces the closer answer, but we want
this is your target digit. to overestimate here, so we choose 765,000.
2. Rewrite the number, replacing all digits to the
right of the target digit with 0’s. (Note: this is One last example before we stop: 10.998 to the
also your lower estimate, if you need one.) nearest hundredth.
3. Add 1 to the target digit. If your target digit is 9,
replace the 9 with a 0, and add 1 to the digit to 1. 10.998
the left. (Note: this is also your higher estimate, 2. 10.990
if you need one.) 3. 11.000 (Here when we add 1 to the tenths place,
4. Which number is closer to your original num- we get another 10, so we replace the 9 with 0
ber? and add 1 to the ones place.)
5. Round to the number in step 1 or the number 4. 11.000 is closer.
in step 2 that you need. Normally this will be
whichever is closer to your original number. The rule you probably learned is that you round up if
you have a 5 or higher in the place to the right of
Now let’s do some examples. Let’s start with where you’re rounding. This rule would have worked
32,872 and round to the nearest thousand. For this with all these examples as well.
one, we’ll pick the closer rounding.
When You Don’t Necessarily Want
1. 32,872 the Closer Estimate
2. 32,000 Sometimes you don’t want the closer estimate; you
3. 33,000 want the safer one.
4. 33,000 is closer. Let’s say you’re grocery shopping, and you have
$20, no credit card, and no calculator. You live in a
Next, let’s do a number with a decimal: 6.0475 state that charges 1% tax on packaged food but 8%
to the hundredths place; assume that we want to un- on readily consumable food, such as a bottle of soda.
derestimate here. It’s going to be pretty hard to keep track of the tax
without a calculator. So how will you know when
1. 6.0475 you’ve reached your $20 limit without going over
2. 6.0400 (and maybe being sent to the back of the checkout
3. 6.0500 line)? You want to estimate to the nearest dollar for
4. 6.0500 is closer, but because we wanted to un- convenience, but what if you get several items that
derestimate, we will choose 6.0400. have prices like $3.46 or $1.27? If you round to the
nearest dollar and get to $19.00 in your estimation

64
8th_GRDM_04.qxd:Layout 1 8/11/09 12:59 PM Page 65

–ESTIMATING AND CHECKING–

(still not including tax, but that’s only 19 cents for Estimating Very Large and
your packaged food, right?), your $20 may not cover Very Small Numbers
your items. Therefore, you’ll probably want to overes- Remember scientific notation? For example, 3.6 × 10–7
timate every item, so that you know you have enough = .00000036, and 3.6 × 107 = 10,000,000. (If you don’t
money, and you’ll estimate $3.46 as $4.00, $1.27 as remember why, look back at Lesson 4.) Scientific no-
$2.00, and so on. tation comes in handy for estimation as well. If you’re
In some situations, you’ll want to underesti- traveling a distance of 18,134 miles each way, for ex-
mate. For example, let’s say you participate in a recy- ample, about how long will your round trip be? In
cling drive and your group’s goal is to recycle 2,000 scientific notation, the one-way trip will be about
pounds of newspapers. Your group has a somewhat 1.8 × 104 miles. So if you double it, you’ll still be in
accurate scale on which to weigh each bag. You want the same order of magnitude, the 104 order of mag-
to weigh each bag of papers (sorry, battery’s dead on nitude, at about 3.6 × 104 miles. But what if you have
your calculator) and add up the total to make sure to make this trip every month for a year—how many
that you’re over 2,000 pounds. You do have a note- total miles will that be? You’d be making 12 round
book to record the totals, so you’ll probably want to trips of 3.6 × 104 miles:
round to the nearest 10 pounds for each bag. Your
scale isn’t 100% accurate, though, and you are highly 12 × 3.6 × 104 ≈ 43 × 104 = 4.3 × 105
motivated to make your 2,000-pound goal. (Your to-
tal collection will be weighed later on an accurate (If you don’t remember how to go from 43 × 104 to
scale.) Will you round to the lower approximation, 4.3 × 105, return to Lesson 4.) Now you’re at a differ-
the higher approximation, or the one that’s closer? If ent order of magnitude in your travel plans, and you’d
you said lower approximation, you’d be making the better plan for a lot more travel expense.
best decision. Because you’d much rather be over
2,000 pounds than under, you should keep collecting
paper until you’re sure that you’re over 2,000 pounds. Checking

Exercise 1 Generally, when you’re taking a math test, the only


Round using any algorithm you think is best. (Just get excuse you should ever have for getting an answer
the right answer!) Answers are on pages 66–67. wrong is running out of time. The reason for this
is that you can always check your answers. With a
1. Round 18.724 to the nearest tenth. multiple-choice exam, it’s even easier to check: With
only four possible answers, you’ll often have time to
2. Round 1,447,083 to the next highest hundred thoroughly check every answer! But you have to
thousand. know how to check. With straight computations, you
can always just reverse your operations: If you were
3. Round 4,623.0697 to the nearest thousandth. asked to multiply, for instance, you can divide your
answer by one of your original numbers and get the
4. Round 17.9999 to the nearest hundredth. other original number:

5. Round 330,992 to the nearest hundred. 51 × 13 = 663 (?)


663 ÷ 13 = 51 (✓)
6. Round 45,501 to the nearest thousand.

65
8th_GRDM_04.qxd:Layout 1 8/11/09 12:59 PM Page 66

–ESTIMATING AND CHECKING–

You should do this kind of checking even if you’re us- Exercise 2


ing a calculator, because you might have accidentally
hit the wrong keys. Even if you’ve estimated the an- Jose is a salesperson who makes a flat weekly
swer to be in the 650 range, you usually need to check salary of $300 plus a 15% commission on the
to see if your answer is exact on a test, or if you need total sales he makes each week. What is the
to know the exact answer in real life. lowest dollar amount of sales he must make if
In real life it’s difficult to check your answer for he wants his weekly paycheck to be $600 or
accuracy, because you rarely have all the facts. If your more?
calculation depends on measurements, for example,
there’s always some kind of limit on how accurate Exercise 3
your measurements will be. Even machines that
measure aren’t perfect, and human beings are even BigBox Electronics marked a certain computer
less perfect. And if some of the facts are hidden from down 20% when business was bad. Then, when
you, you can make serious miscalculations. business started picking up again, it marked
In eighth grade, however, you’ll normally get all the same computer up by 50%. It’s now selling
the facts you need in word problems (though you may for $1,200. How much did the computer cost
also get some extra, irrelevant facts!), and you can as- before the store originally marked it down?
sume they’ll be accurate. You can then check your an-
swer by going through all the facts in the problem to
see if your answer fits them. Here’s a sample: Summary

There are 189 students and 19 teachers at In this lesson you’ve learned how to do some estima-
Willoughby Middle School who are going on a tions that help you figure out whether you have a rea-
field trip. The school needs to hire buses for the sonable answer and even help you decide which
event, and each bus has a capacity of 28 people. algorithm to use. You’ve also learned how to check
How many buses will the school need to take all your answers, especially for word problems. In the
the students and teachers? next lesson, you’ll get some advice on strategies to use
when you are doing word problems.
Suppose Kerry Calculatorguy, who knows how to
calculate but doesn’t know much about life, says the
school will need seven buses because 208 28 = 7.4, which Answers
rounds to 7. Marty Mathguy says no, it will take eight
buses. Who is right, and why? In this situation, the Exercise 1
school plans to take 208 people (189 + 19) on the field 1. 18.720
trip. If it hires seven buses, it can take 7 × 28 = 196 peo- 2. 1,500,000
ple, leaving 12 people without a ride! In this case, you 3. 4,623.0700
need to round up to the larger number. Therefore seven Adding 1 to the 9 in the thousandths place
buses is not a good answer, and the school needs eight makes 10, so you have to add 1 to the
buses. hundredths place.
Now, try to solve a couple of word problems, 4. 18.000
and check your answers to see that they’re right. An- 17.9999 ⇒ 17.9900 (lower) ⇒ 18.0000 (higher;
swers are on page 67. adding 1 to the hundredths place forces the
tenths and even the ones place to go up.)
66
8th_GRDM_04.qxd:Layout 1 8/11/09 12:59 PM Page 67

–ESTIMATING AND CHECKING–

5. 331,000 Exercise 3
330,992 ⇒ 331,000 $1,000. Our strategy on this one will have to be
6. 46,000 to work backward, because we know the final
price, but not the original price. Before the
Exercise 2 computer got marked up to $1,200, it
$2,000 obviously had a price. 100% of that price plus
Jose gets $300 every week if his sales are $0. 50% of that price add to $1,200, so 150% of
Because he wants an extra $300 ($600 – $300 that price = $1,200. That price was obviously
= $300) every week, the question becomes, smaller than $1,200, so let’s try dividing 1,200
“$300 is 15% of what amount?” In Lesson 4 by 150%, or 1.5: $1,200 ÷ 1.5 = $800. Using
you learned how to answer this type of that answer, we know that $800 was smaller
question, but suppose you don’t exactly than the original price, because it was 20% off
remember. You do know that $300 is 15% of the original price. And if it was 20% off, that
some larger amount, though, and that dividing means that it cost 80% of the original price.
$300 by 0.15 will give you a larger amount. Can we find the original price by multiplying
$300/0.15 on the calculator is $2,000, which is the $800 by 0.80? No, that would give us a
a larger amount, so stick with that and check smaller number. Let’s divide the $800 by 0.80
it out. to get $1,000.
We’ll go through the problem to see if That answer seems reasonable, because
you got the right total. First, Jose wants to be at $1,000 is lower than $1,200 and the markup
or over $600 in total salary, so let’s try $2,000 to was a higher percentage than the markdown.
see what he makes. His $300 regular salary is But let’s check to make sure. If the original
added to 15% of $2,000 for his total, and 15% price was $1,000, a 20% markdown would be
of $2,000 (on the calculator or in your head) is $200, and the new price would be $800. A 50%
$300. So he does make exactly $600 if his sales markup from $800 would be $400, which gives
are $2,000. It stands to reason that any sales you a final price of $1,200. You can be
over $2,000 will increase his salary, but let’s confident that your answer is correct.
make one more calculation just to be sure.
Suppose his sales total is $2,500; 15% of $2,500
is $375, so his weekly salary would be $675,
well over $600. You can now be pretty
confident that your answer is right.

67
8th_GRDM_04.qxd:Layout 1 8/11/09 12:59 PM Page 68
8th_GRDM_04.qxd:Layout 1 8/11/09 12:59 PM Page 69

7
L E S S O N

STRATEGIES

L E S S O N S U M M A RY
Certain types of problems occur over and over again, in both math
class and real life. If you have an idea about how to start tackling
certain types of problems, you’ll usually solve them more quickly
and accurately. This lesson will teach strategies that you can apply
to more than just one problem.

L et’s say that you’re driving from one city to another. (Okay, you’re not old enough to drive yet, but if you
get good grades in math, you’ll probably be able to drive in a couple of years.) You know your average
speed and the distance to the other city, so how long will it take you to get there? Or let’s say that you
want to get there in a certain amount of time and that you can adjust your average speed up or down: Can you
figure out how much your speed needs to be adjusted?
This lesson will show you some strategies for solving different classes of problems, including rate prob-
lems and number problems that don’t fit into regular mathematical patterns, such as time and coordinate grid
systems. It will also give you some tips on how to answer test questions that require you to write down how you
solved problems.

69
8th_GRDM_04.qxd:Layout 1 8/11/09 12:59 PM Page 70

–STRATEGIES–

General Rate Problems


Problem-Solving Procedure
One general type of problem that you’ll often see on
Different books and different teachers will often give math tests, mostly because it’s so common in real
different general procedures for solving problems. life, is a problem involving a rate. A rate is a ratio of
You should read this one and understand it, but if two quantities, and it contains the word per. Exam-
you have one that works better for you on the prob- ples include miles per hour, feet per second, dollars
lems in this lesson and the other strategies lessons in per pound, pages per second, and problems solved
this book, then keep on using it. per minute. The word per really means divided by, so
The first thing to do, after reading the descrip- mathematically a speed of 65 mph could be read as
tion of the problem carefully, is to ask yourself these 65 miles divided by 1 hour. Usually, but not always,
four questions: these problems involve time. Here’s a problem in-
volving rate.
1. What do I know?
2. What do I need to know? The Williams family is driving to Levittown
3. What units will my answer be in? from Green Village. The distance between the
4. How will I get there? two towns is 188 miles on Highway 46, which
allows a car to legally average 47 miles per hour.
The answer to the last question will normally involve How long will it take the Williams family to get
some kind of problem-solving procedure. Here are to Levittown?
some of those methods, but there are many others.
The first step to solving this problem is to recog-
■ Estimate and check. nize that you know the distance between the cities
■ Use a number operation or series of number and you know the rate you’re going to drive at. You
operations. need to know how long it will take to get there. What
■ Work backward. units will your answer be in? Sometimes, a problem
■ Use a formula. will specify the units, but this one doesn’t. It does ask
“how long,” though, so your answer had better be in
Ideally, you’ll write the answers to these ques- hours, minutes, seconds, days, or some other unit of
tions on a piece of paper if they’re too complicated to time. Because your speed is given in miles per hour,
keep entirely in your head. While you’re in school, of hours is the most convenient unit.
course, your teachers are usually going to demand The algorithm for solving most rate problems is
that you write down all your work, and you won’t re- to set up an expression like this:
ceive full credit even for a correct answer without
showing your work. Later in this lesson, you’ll see distance time
distance ÷ = distance × = time
some examples of the kind of short responses and ex- time distance
tended responses that many tests, especially standard- miles hour
miles ÷ = miles × = hours
ized tests, require. hour miles
This setup works because division by a fraction is the
same as multiplication by the reciprocal (see Lesson 2
if you’ve forgotten this). Also, notice how miles will
cancel out and leave just hours.

70
8th_GRDM_04.qxd:Layout 1 8/11/09 12:59 PM Page 71

–STRATEGIES–

So let’s apply this method to our problem: count backward 3 hours to 2:00 P.M. and then another
45 minutes to 1:15 P.M.
47 miles 1 hour When you do subtract times, you have to re-
188 miles ÷ = 188 miles × = 4 hours
hour 47 miles member that there are 60, not 10 or 100, minutes in
It will take four hours to get to Levittown. We an hour, so you can’t use your usual regrouping algo-
still need to check the answer, of course. Let’s do an rithms. And if it’s convenient to get decimal or frac-
hour-by-hour check. After one hour, you’ve gone 47 tional equivalents of an hour or a minute, you have to
miles, and you’ve gone 94 after two hours. After use ratios. For example, let’s say you’re given 1 hour
three hours, you’ve gone 141 miles, and exactly at and 20 minutes as the time frame in a rate problem,
the end of four hours, you’ve reached 188 miles, so and your rate is given in miles per hour. Then you’ll
your answer is correct. probably want your time in hours, so you’ll have ei-
This same procedure works even if what you ther a decimal or a fraction left over after the 1 hour.
need to know isn’t time, but distance. You’ll get a Because there are 60 minutes in an hour, 20 minutes
chance to try one in Exercise 1 later. would be 2060 of an hour, so you could use either 1.333
hours or 131 hours in your problem. You could convert
the 1 hour and 20 minutes, but then you’d have to
Operations with Time convert your miles per hour to miles per minute, and
there’s usually no reason to do that.
The way we measure time is almost independent Other artificial number systems include the lati-
from the rest of our system of mathematics. There are tude and longitude system that is used to measure
24 hours in a day, 60 minutes in a second, and 60 sec- global positions, the coordinate grid system that we
onds in a minute. Why? That’s just the way it is. To use for the graphs of functions (covered in Section 2),
make it even more confusing, we usually think in angle measures (see Lesson 17), and any non-metric
math terms again when we’re talking about parts of a measurements (see Section 4). Dealing with these and
second, which are usually measured in tenths and any other systems requires careful thought to bring
hundredths. For example, Usain Bolt holds the world those systems in line with the one that you know how
record for completing the 200-meter run with a time to do number operations in.
of 19.30 seconds.
It doesn’t make sense to add together the times Exercise 1
of the day; what would be the meaning of 2 P.M. plus (Answer is on page 73.)
4:30 P.M.? Sometimes, we can subtract times, or at
least find the differences between times to calculate The Jacobs family got to Green Village from
elapsed time. For example, if a task takes 2 hours and Palm Falls in 1 hour and 40 minutes. If they
20 minutes to complete and you start it at 11:30 A.M., went on a highway that they know allows them
what time will you finish? To find the answer to this to drive at an average speed of 55 miles per
question, it’s usually best just to count forward. It’s hour, how far is Green Village from Palm Falls,
easy to figure that it will be 1:30 P.M. 2 hours after to the nearest mile?
11:30 A.M., so then you can just add 20 minutes to
1:30 to get 1:50 P.M. Sometimes, of course, you have to
count backward rather than forward. If you know
that the finishing time of a task is 5:00 P.M. and you
know that it took 3 hours and 45 minutes, you’d first

71
8th_GRDM_04.qxd:Layout 1 8/11/09 12:59 PM Page 72

–STRATEGIES–

Problems That Require Exercise 2


Written Responses The following is a short-response question. Answer
the question and show all your work. Answer is on
As time goes on, more and more people have realized page 73.
that communication is an important mathematics
skill. And why wouldn’t it be? If you can’t explain the An office equipment store sold two-thirds of
math you’ve done, why should anyone accept it as its pencils in one day for 60 cents each. At the
correct? So more and more tests will be asking you to end of the day it had 14 pencils left. How
write open-ended short responses and extended re- much money did the store collect on pencils
sponses. Open-ended short responses usually call for that day?
a two- or three-step solution, and sometimes you can
even answer them with math sentences only, without Exercise 3
using English sentences. Extended responses always Write an extended response that describes how you
require easily understandable English sentences, solved the following problem. Answer is on page 73.
which should describe every step in the procedure
and sometimes why you chose a certain algorithm or Kim wants to purchase a portable music player,
procedure. The specifics of what the test graders ex- a DVD player, and video game. The music
pect will vary from test to test, but here are some rules player is listed at $40 and a sign says “20%
to keep in mind: discount. Price will be reduced at register.” The
DVD player has a sticker of $56, and its price
■ Plan your answer on a separate sheet of paper or has already been marked down 20%. The video
off to the side. game costs $25 and is not on sale. Tax in Kim’s
■ Make every step clear, and clearly show the an- state is 8.25%, and tax is charged on the full
swer, including units. The people grading these nondiscounted price. How much money does
tests, whether it’s your teacher or someone else, Kim need for her purchases?
have to grade quickly and can’t be looking
through a disorganized mess to find the right
answer. Summary
■ Always include a diagram if you use one—and
you usually should, especially on an extended This lesson has introduced you to strategies for cer-
response. tain types of problems and has given you some tips
■ Always include any table or other set of lists that on writing short and extended responses. You’ll next
helps you. get practice using what you’ve learned here.
■ If the problem involves a computation—and
nearly every one does—check every step.
■ On a short response, you usually don’t have to
show how you checked your work, but on an ex-
tended response, you usually have to show what
you did to check.

72
8th_GRDM_04.qxd:Layout 1 8/11/09 12:59 PM Page 73

–STRATEGIES–

Answers Exercise 2
The correct answer is $16.80. Your answer
Exercise 1 should include the steps you used to get to the
92 miles answer: first getting the number of pencils sold
This is a rate problem, so start by setting it up (because 14 is 31, 32 is twice that number, or 28),
like this: and second, multiplying 28 by $0.60 to get
$16.80. As a check, you can figure out that if
distance time 14 pencils is 31, then the store must have started
distance ÷ = distance × = time
time distance with 33, or 42 pencils and sold 42 – 14 = 28.
First, we need to get a usable number for 40 (You could also use this method as your initial
minutes. Because 40 minutes is 4060 of an hour,
method and check another way.) You don’t
2 normally have to show why you think your
we can call it 3 of an hour. Now you can just fill
in the numbers: answer is reasonable (as long as you get it
right!) in a short response, but if the directions
55 miles 2
? miles ÷ = 13 hours had told you to include an estimate, you could
hour observe that 60 cents is a little over half a
If you already know how to solve an equation dollar, so your answer should be a little over
without looking ahead to Section 2, then you $14.
can just do that. But you can also figure it out
with the following reasoning: In 1 hour a car
traveling at 55 miles an hour would obviously
go 55 miles. So in 132 hours it would go 132 × 55
miles. For the calculator, you’d have to convert
to the decimal, 1.6667, and you’d get 91.6669
miles. If you leave the time in fractional form,
you’ll get 275
3 hours, and figuring that on the
calculator would give you a slightly different
answer, but both would round to 92 miles.
Note: Doing the problem in two different ways
is another good way of checking that you’ve
done your calculations right.

73
8th_GRDM_04.qxd:Layout 1 8/11/09 12:59 PM Page 74

–STRATEGIES–

Exercise 3 To find out the original costs, which Kim


The correct answer is $124.14. There are has to pay tax on, we put $40 for the music
several ways to do problems like this one, but player and $25 for the video game. Because the
let’s start by making two separate price lists. $56 is at a 20% discount, you know that $56 is
$56
Your extended response should begin by saying 80% of the original price, and 0.80 = $70.
something like “I first need to find the total (Remember to use the $ sign before all money
amount for the three items without tax, and amounts; it helps the reader/grader keep track
then I need to find the nondiscounted prices of what you’re doing.) Now you can make your
that the tax is computed on. Then I need to second list, which adds up to $135. You
find the total tax and add it to the pretax multiply the tax rate, 0.0825 (NOT .825 or
prices.” 8.25), times $135 to get (rounded to cents)
So you’ll need the prices Kim pays at the $11.14.
register in one list and the nondiscounted With tax, Kim’s purchase costs $124.14.
prices, which you need to calculate tax, in the
other list. Both lists should appear in your
answer. We know two of the prices she’ll pay at
the register, because they’re given directly in
the problem: $56 and $25. We have to figure
out the register price of the music player, which
will be $40 discounted by 20%. Because 0.20 ×
$40 = $8 and $40 – $8 = $32, we now know all
the prices she’ll pay at the register, so we have
to add them up. Now you can make your first
list and add it: $32 + $56 + $25 = $113.

74
8th_GRDM_04.qxd:Layout 1 8/11/09 12:59 PM Page 75

8 PUTTING
L E S S O N

IT ALL
TOGETHER

L E S S O N S U M M A RY
This lesson pulls together what you learned about number opera-
tions in Lessons 1–7. You’ll use those operations to answer prob-
lems and to write short and long responses that tell how you
solved problems.

Y ou’ve now reviewed the number operations you’ll need to be successful in eighth grade. You’ve also
learned strategies for solving word problems based on real-life situations. Now, it’s time to see how
well you know them.

75
8th_GRDM_04.qxd:Layout 1 8/11/09 12:59 PM Page 76

– PUTTING IT ALL TOGETHER –

What You’ve Learned Practice

Here’s a quick summary of the lessons we’ve covered Short Answers


in this section. Complete the operations asked for. (Calculators are
Lesson 1: Operations with Integers. This les- permitted, but checking answers without them is rec-
son reviewed the basic operations of addition, sub- ommended. Answers are on pages 77–79.)
traction, multiplication, and division. Factoring and
operations with negative integers were other topics 1. 83 – (–4 – 6) =
covered.
2
Lesson 2: Operations with Fractions. You 2. –27 3 =
learned the number operations that can be done with
representatives of numbers that fall between integers 3. List the prime factors of 108. Show how you
on the number line. Equivalence of fractions and the arrived at this answer.
meaning of fraction notation were also covered.
Lesson 3: Decimals and Percentages. You 4. Denise sold 45 of her orchestra fundraiser
learned number operations that can be done with still cookies for $3 per box today. She has six boxes
other representatives of numbers that fall between in- left over. How much money has she taken in
tegers on the number line. You were also introduced for the day?
to percentages and how to solve several kinds of
problems involving percentages and other ratios. 3 ⎛ 2⎞
5. × ⎜ 2 − ⎟ + 180 =
Lesson 4: Exponents. You learned how to evalu- 4 ⎝ 3⎠
ate and combine numbers with exponents. You also
learned scientific notation, a powerful method for 6. Arrange these improper fractions in order
representing numbers, especially very large and very from largest to smallest: 34, 175 , 89, 69, 134, 57. Then
small ones. write them all as mixed numbers.
Lesson 5: Order of Operations. “Please excuse my
dear Aunt Sally”: She’s really very fond of long strings of 7. 3–4 × 34 ÷ 33 + 32 =
number operations, and she especially likes challenging
people with fractions and negative numbers. 8. 713 – 413 =
Lesson 6: Estimating and Checking. You learned
some methods for estimating solutions to problems. 9. 313 – 841 = (Give your answer as a mixed
You also learned how to check your answers to word number and as a decimal equal to the nearest
problems involving number operations. hundredth.)
Lesson 7: Strategies. You studied a general pro-
cedure for solving word problems, as well as more 10. Add 14% of 17 to 17% of 14. Round to the
specific strategies for working problems with rates nearest tenth.
and elapsed time. You also got some tips on writing
short responses and extended responses, which you
will have to do at the end of this lesson, and at the end
of all the other sections in the book.

76
8th_GRDM_04.qxd:Layout 1 8/11/09 12:59 PM Page 77

–PUTTING IT ALL TOGETHER –

11. Julia has made 13 of 17 free throws for the car averages 60 miles per hour over the same
basketball season. What is her percentage of route as the truck. Will the car catch the truck
free throws made, to the nearest tenth of a before it reaches its destination? If so, what
percent? How many consecutive free throws time will it be?
does she have to make to raise her percentage _____________________________________
above 85%?
_____________________________________
1 7 _____________________________________
8
12. 1
− −201 =
2 2 _____________________________________
_____________________________________
Short Response _____________________________________
On a separate piece of paper or in the space provided, _____________________________________
write a response that explains how you arrived at
_____________________________________
your answer. (No calculator is allowed except to
check.)

2
13. Which is larger: 3– 3 or 5– 3 ?
2 Answers
_____________________________________ 1. 93
_____________________________________ 83 – (–4 – 6) = 83 – (–10) = 83 + 10 = 93.
_____________________________________ 2. 81
–27 3 = ( 3 −27 ) by the definition of a fractional
2 4

_____________________________________ exponent. The cube root of –27 is a number


_____________________________________ that multiplied by itself 3 times = –27, so you
know the number has to be a negative one.
Extended Response Using either your calculator or a guess-and-
On a separate piece of paper or in the space provided, check method, you find out that the cube root
write a response that fully explains how you arrived at of –27 is –3. Then (–3)4 = (–3) × (–3) × (–3) ×
your answer. Show all diagrams or tables that you (–3) = 81.
made as well as all calculations. 3. 22 × 33
Methods for getting the prime factors will vary;
14. A delivery truck leaves a printing company at here’s an example:
5:30 A.M. to deliver magazines to a customer
108
whose warehouse is 260 miles away. The truck’s
average speed is 50 miles per hour. At 5:45 A.M.,
someone at the printing company spots an 2 × 54

error on the magazine’s cover. It is also


discovered that the truck driver’s cell phone 6 × 9

battery is dead. By 6:15 A.M. the error is


corrected and the covers are reprinted. The 2 × 3 3 × 3
new covers are put into a car at 6:20 A.M. The

77
8th_GRDM_04.qxd:Layout 1 8/11/09 12:59 PM Page 78

– PUTTING IT ALL TOGETHER –

4. $72 8. 3
Denise has 1 – 45 = 51 of her boxes left over. We You may notice that 7 – 4 = 3 and 31 – 31 = 0, so the
know that 51 = 6 boxes. So 45 must be 4 times as answer is 3. If you don’t notice that, you could
many, or 24 boxes, and this is the number she’s change to improper fractions and subtract:
sold. Each box cost $3, so her total sales figure 731 – 431 = 223 – 133 = 39 = 3.
is $3 × 24 = $72. 9. –41112 and –4.92
5. 2 3 ⎛ 2⎞ 3 ⎛ 6 2⎞ The best (but not the only) way to calculate this
× 2 − ⎟ + 180 = × ⎜ − ⎟ + 180
4 ⎜⎝ 3⎠ 4 ⎝ 3 3⎠ is to convert to improper fractions and then use
a common denominator.
2 3 ⎛ 4⎞
= × ⎜ ⎟ + 1 = 1 + 1 = 2. 331 – 841 = 103 – 334 = 40 99 59 11
12 – 12 = –12 = –412.
4 ⎝ 3⎠
Use your calculator to find the decimal value.
6. 17 13
5 > 4> 69 > 57 > 43 > 89; 325 > 314 > 163 > 152 > 131 > 181 As a check, 3 – 8 ≈ –5.
There are several ways to do this. One way is to 10. 4.8
change them all to mixed numbers first, since 0.14 × 17 = 0.17 × 14 = 2.38; 2 × 2.38 = 4.76.
the question asks you to convert them anyway. 11. 76.5%; 10 free throws
Now your numbers are 131, 352, 118, 136, 314, and 125. Her current percentage is 13 17 = .764706 × 100 =
The two numbers beginning with 3 obviously 76.5%, rounded to the nearest tenth of a
are the largest, and 52 > 14. You can prove that 52 > percent. You could use trial and error alone to
1
4 by either (1) doing the divisions 2 ÷ 5 (= 0.4) find out how many free throws in a row she has
and 1 ÷ 4 (= 0.25) on your calculator, or to make, but you might also observe that it’s
(2) comparing equivalent fractions: easy to calculate percentages with 25 attempts:
2 × 4 1× 5 8 5 just multiply the number made by 4. So if she
? ⇒ >
5× 4 4×5 20 20 still has only 4 misses at 25 free throws, her
You can order the mixed numbers beginning percentage will be 21 25 = 84%. Then you can use
with 1 in a similar way. Converting to the your calculator to try out 22 23
26 and 27, which is the
60 48
common denominator of 120, you get 120 > 120 first percentage over 85. So she has to make 10
40 15 23
> 120 > 120 . Using your calculator to get decimal consecutive free throws to get a ratio of 27 ,
equivalents, you get 0.5 > 0.4 > 0.33 > 0.125. which will get her over 85%.
Thus, 163 > 152 > 131 > 118. 12. 19
20
1 1 7
7. 9 27 1 2 7 ⎛ 2⎞ 1 ⎛ 7 ⎞
8 20
3–4 × 34 ÷ 33 + 32 = 30 ÷ 33 + 32 = 3–3 + 32 = 27 1 − −1
= × − ×⎜− ⎟ = −⎜− ⎟
1
2 2
8 1 20 ⎝ 1 ⎠ 4 ⎝ 10 ⎠
1
+ 9 = 9 27. In this string of operations, it’s easier
5 14 19
to multiply and divide before evaluating the = + =
exponents because you can consider 20 20 20
⎛ −4 4 ⎞
3–4 × 34 ÷ 33 as a fraction ⎜ 3 ×3 3 ⎟ that can be
⎝ 3 ⎠
evaluated before anything else.

78
8th_GRDM_04.qxd:Layout 1 8/11/09 12:59 PM Page 79

–PUTTING IT ALL TOGETHER –

2
13. 3– 3 is larger. CAR TIME TRUCK
Responses will vary; here’s a sample.
( 32 hour) 40 miles 7:00 75 miles (1 21 hours)
I started out with 2 numbers that have
negative exponents. I know that a 70 7:30 100
negative exponent means that the
100 8:00 125
number with the exponent is in the
1
denominator, so the numbers are 5 23 and 130 8:30 150
1 1 1
2 . I know that
3 is bigger than 5
3 3
160 9:00 175
because its decimal equivalent is bigger,
and because it has the same numerator 190 9:30 200

but a smaller denominator. It seems 220 10:00 225


logical that the cube root of 3 must be
250 10:30 250
bigger than the cube root of 5, and I
checked it on my calculator to be sure. So
1
after calculating the exponents, 23 is still At 7:00 the car had driven 40 minutes, and I
3
larger. Just to be sure I checked on my know 60 mph is 1 mile per minute, so the car
calculator and found that it was larger. had gone 40 miles. The truck was at 75 miles
14. The car will catch the truck at 10:30, which will after an hour and a half.
be 10 miles before the truck reaches its As you can see from the table, the car
destination. Responses will vary; here is a catches the truck at 10:30 at 250 miles. This
sample. Note that this sample does not use is 10 miles before the truck reaches its
algebra to get the right answer. If you did, give destination.
yourself a pat on the back; the next lesson will
then be mostly a review for you!
I know the speeds of the truck and the
car, and I know the truck leaves at 5:30
and the car leaves at 6:20. The other times
don’t matter. I know that at the end of
each hour the truck will be 50 miles
farther along, so at the end of each half-
hour it will be 25 miles farther. The car
will go 30 miles each half hour. So I made
a table to see if the car could catch the
truck.

79
8th_GRDM_04.qxd:Layout 1 8/11/09 12:59 PM Page 80

– PUTTING IT ALL TOGETHER –

IF YOU MISSED THEN STUDY

Question 1 Lessons 1, 5

Question 2 Lesson 4

Question 3 Lesson 1

Question 4 Lessons 2, 7

Question 5 Lessons 4, 5

Question 6 Lesson 2

Question 7 Lessons 4, 5

Question 8 Lesson 2

Question 9 Lessons 1, 2

Question 10 Lessons 3, 6

Question 11 Lesson 3

Question 12 Lesson 2

Question 13 Lessons 4, 6

Question 14 Lessons 6, 7

80
8th_GRDM_05.qxd:Layout 1 8/12/09 10:19 AM Page 81

2
S E C T I O N

ALGEBRA

W hen you were filling in the blanks in sequences like 2, __, 6, 8, you probably didn’t realize that
you were starting to learn algebra. But when you were learning to figure out which rule to fol-
low, even in those simple sequences, you were really beginning to get at the core of algebra.
Like the rest of mathematics, your knowledge of algebra rests on the foundation that you’ve been laying down
since you started studying math, and the algebra that you’ll learn in high school and beyond will be based on
your eighth grade knowledge.
By the end of this section, you should have these basic skills:

■ solving equations with one unknown


■ understanding patterns, relations, and functions

81
8th_GRDM_05.qxd:Layout 1 8/12/09 10:19 AM Page 82

–ALGEBRA –

■ adding, subtracting, multiplying, and factoring polynomials


■ graphing linear and nonlinear functions and inequalities
■ solving systems of equations and systems of inequalities
■ strategies for solving problems with algebraic methods

WORDS YOU SHOULD KNOW

coefficient inequality relation

constant linear equation sequence

coordinate graph monomial system of equations

equation nth term term

evaluate polynomial unknown

expression proportion variable

function quadratic equation

82
8th_GRDM_05.qxd:Layout 1 8/12/09 10:19 AM Page 83

9 PATTERNS,
L E S S O N

VARIABLES, AND
EQUATIONS

L E S S O N S U M M A RY
In this lesson you will learn how to find rules that describe se-
quences. You will also learn about formulas that can describe the
behavior of real-life situations, and you will learn how to solve
equations that have an unknown quantity in them.

I n this lesson, we’ll start to look at ways to solve puzzles, which can be one of the most enjoyable parts of
math. We’ll begin by filling in holes in sequences and trying to find rules that describe those sequences.
We’ll also have a look at some formulas—we’ve already met one (the compound interest formula in Lesson 4),
and you undoubtedly know many more, such as area and volume formulas. Finally, we’ll define what an equa-
tion is and go over how to solve equations.

83
8th_GRDM_05.qxd:Layout 1 8/12/09 10:19 AM Page 84

–PATTERNS, VARIABLES, AND EQUATIONS–

Patterns (Note: A sequence with a + or – rule is called an arith-


metic sequence, and a sequence with a × or ÷ rule is
Try to fill in the missing numbers in the following 8- called a geometric sequence.)
term sequence:
Finding a Rule
7, 12, 17, __, 27, __, __, 42 What if we wanted to find the 54th term in the se-
quence 7, 12, 17, __, 27, __, __, 42 if it was expanded?
Looking at the first three numbers in the sequence, It wouldn’t be convenient to write out 54 blanks and
you can easily see that 5 has been added to each, so fill them all in, so we need some kind of a shortcut.
the first missing number (the 4th term in the se- Let’s start by looking at the first number we
quence) is 22, the missing 6th term is 32, and the filled in. It was the 4th term and the correct solution
missing 7th term is 37. We can find each term from was 22. If we want to make a rule that will find the
the value of the term that comes before it in the se- 54th term (or the 65th term, or the 198th term, etc.),
quence. All we have to do is find the rule and apply it. we want our rule to apply to every term. Because we
The rule has a number operation (in this sequence +) don’t necessarily know which term we’ll want at any
and a number (in this sequence 5); therefore, our rule given time, we have to call the term something gen-
is +5. Fill in the missing numbers and find the rule eral, like tn. The letter t stands for term, and the letter
for each of the following sequences. Answers are on n stands for the place in the sequence. Now how did
page 89. you find the value of the 4th term, tn? Right, you
added 5 to the 3rd term. So, using n = 4 in this case,
Exercise 1 you added 5 to the (n – 1)th term. In other words,
you simply add 5 to whichever term came before the
1. –4, 8, __, 32, 44, __ one you’re looking for—in this case n – 1 = 3, the 3rd
term. To find the 6th term, you added 5 to the 5th
2. 14, 11, __, 5, __ term. But how do we find the value of the (n – 1)th
term? Let’s look at our sequence with the term num-
3. 2, 6, 18, __, 162, __ bers above them:

4. –9, __, –3, 0, __

5. –4, 8, __, 32, –64, __

6. 40, __, 10, 5, __

Term n=1 n=2 n=3 n=4 n=5 n=6 n=7 n=8 n=9

Value 7 12 17 22 27 32 37 42 47

84
8th_GRDM_05.qxd:Layout 1 8/12/09 10:19 AM Page 85

–PATTERNS, VARIABLES, AND EQUATIONS–

It looks like our formula will have to start with the 1st term, t1, which in this case is 7, and go up by 5 for
every term. If we’ve gone up 3 times, we’ve added 5 three times, which is the same as multiplying 5 by 3. So the
formula for any term will be tn = t1 + (n – 1) × 5. Try this formula with some of the values in the table to con-
vince yourself that it works. For example, the 6th term t6 = 7 + (6 – 1)5 = 7 + 25 = 32. Now, what would the 54th
term be? It would be t54 = 7 + (54 – 1)5 = 272.

Exercise 2
Find the formula for the nth term tn for each of the following sequences. Answers are on page 89.

1.

Term n=1 n=2 n=3 n=4 n=5 n=6 n=7 n=8 n=9

Value 15 11 7 3 –1 –5 –9 –13 –17

2.

Term n=1 n=2 n=3 n=4 n=5 n=6 n=7 n=8 n=9

Value –3 6 15 24 33 42 51 60 69

Finding Rules That Involve More Than One Term


The rules for sequences we’ve looked at so far were for a single operation on a single term. Now let’s look at a
slightly more complicated sequence.

Term n=1 n=2 n=3 n=4 n=5 n=6 n=7 n=8 n=9

Value 0 1 1 2 3 5 8 13 21

Take a minute to figure out the rule for this one. (Hint: the rule involves more than just the preceding term.) If
you look at just the 4th, 5th, and 6th terms, for example, you can see that the 6th term is the sum of the 4th and
5th terms, or that tn = t(n – 1) + t(n – 2). That definition works for everything from t3 up, but not for the 2nd term,
because there’s only one term before t2. You’ll learn more about how to specify this sequence, called the Fi-
bonacci sequence, in Lesson 10.

85
8th_GRDM_05.qxd:Layout 1 8/12/09 10:19 AM Page 86

–PATTERNS, VARIABLES, AND EQUATIONS–

Exercise 3
Find the formula for t6 and tn in each of the following in terms of other terms in the sequence. Answers are on
page 90.

1.

Term n=1 n=2 n=3 n=4 n=5 n=6 n=7 n=8 n=9

Value 4 2 2 0 2 –2 4 –6 10

2.

Term n=1 n=2 n=3 n=4 n=5 n=6 n=7 n=8 n=9

Value –1 –1 0 1 3 6 11 19 32

3.

Term n=1 n=2 n=3 n=4 n=5 n=6 n=7 n=8 n=9

Value 0 1 1 2 5 29 866 750,797

Variables Formulas
There are formulas for many things, and you can
In the work you’ve just been doing, you’ve been using make up formulas from just about any set of facts
variables. The term tn can stand for any term in the that you know. You’ve probably learned that the
sequence, depending on the number of the term. Do perimeter of any polygon is the total distance of its
you remember the interest formula that we worked sides. You know a square has four sides, and that each
with in Lesson 4? It was A = P(1 + i)n, and it had four one is equal to all the others. So we can make a for-
variables: A, the amount of money after n years; P, the mula by calling one side s, which will be the variable
principal; i, the interest rate; and n, the number of in this formula. Then we can write the formula for
years. This formula to find A will work for any num- the area, which we’ll call A, as
bers you want to insert for the variables. However,
some numbers aren’t practical to use in this formula. A = s + s + s + s = 4s
For example, it probably wouldn’t make sense for n to
be 1,000, because you won’t be around to collect your In the expression 4s, in which 4 is multiplied by s, 4 is
money 1,000 years from now. Likewise, a bank could called the coefficient of s. In other words, we can add
offer interest higher than 100%, but it never would. variables, as long as the variable stands for the same
(In practice, if anyone offers interest higher than 10%, quantity, just like we could add any other numbers.
or .10, there’s a good chance that person is a crook!) For example, 2.5 + 2.5 + 2.5 + 2.5 = 4 × 2.5 = 10.

86
8th_GRDM_05.qxd:Layout 1 8/12/09 10:19 AM Page 87

–PATTERNS, VARIABLES, AND EQUATIONS–

Solving an Equation for an


In the expression 4s, in which 4 is multiplied by
Unknown Quantity
s, 4 is called the coefficient of s.

An equation is simply a mathematical sentence that


says one thing is equal to another. Here are some
Let’s look at our formulas for rate problems, in-
equations:
troduced in Lesson 7, where the rate is in terms of
distance per time:
1+3=4 18–1 = 18
1
2 × 3(–4 + 2) = –12
1+m=4 x–1 = 18
1
2 × 3(–y + 2) = –12
Rate = Distance
Time
Distance = Rate × Time
Each equation in the second row above has an
Time = Distance ÷ Rate
unknown—a variable for which the value isn’t im-
mediately known. To find out the value of the un-
We can call the three variables here R for rate, D for
known variable, you usually have to follow an
distance, and T for time. It’s important that you see
equation-solving algorithm. For these equations,
that you can get three different formulas if you have
though, you just have to look at the equation above
three different variables—one for each variable.
the one with the unknown. You’ll see that m replaces
3 in an equation exactly the same otherwise. Thus, we
Exercise 4
can say that m = 3 in that equation. Likewise, x = 18
Find the value of the missing variable using one of
and y = 4 in the other two equations.
the rate/distance/time formulas above, given the other
Now, we’ll look at some algorithms that can
variables. Be sure to include units in your answer, and
help you solve equations like these.
round your answer to the nearest 10th of a unit if
necessary.
Same Operation to Both Sides
When you solve an equation for an unknown, you re-
1. R = 65 mph, D = 26 miles
ally just get another equation that has your unknown
on one side and something else on the other side that
2. T = 12 days, D = 2,800 miles
does not contain your unknown. For example, m = 6
is a solution, but m = 7 + 8m is not a solution. So the
3. R = 40 feet/second, T = 12 seconds
algorithm we’ll look at is to do basic number opera-
tions to both sides of an equation until we end up
4. D = 385 miles, R = 55 mph
with nothing except the unknown on one side of the
equation; the other side of the equation then will be
5. D = 150 yards, T = 18 minutes
the solution. Let’s start out with 1 + m = 4. You’ll no-
tice right away that there’s one other term on the
6. T = 8 seconds, R = 186,000 miles/second
same side as m, so we’d like to “get rid of ” it. One way
we can make the 1 disappear from the left side is to
add –1 (or subtract 1, depending on how you want to
think of it) to both sides.

87
8th_GRDM_05.qxd:Layout 1 8/12/09 10:19 AM Page 88

–PATTERNS, VARIABLES, AND EQUATIONS–

Divide each side by 19.


It is important to perform the exact same oper- 19p ÷ 19 = 19 ÷ 19
ation on both sides of the equation at the same p=1
time—in other words, whatever you do to one Check.
side, you have to do to the other. 9(1) + 17 = 36 – 10(1)
26 = 26 ✔

So for this equation we have Now it’s time to solve some equations on your own.

1+m–1=4–1 Exercise 5
m=3 Give answers in fraction form, using mixed numbers
and fractions reduced to lowest terms. Be sure to check
We’ve found a solution now, so we should check it. your answers in the original equation before looking
You can check your answer by substituting 3 into the them up in the Answers on pages 90–91.
equation for m: 1 + 3 = 4. Because 1 + 3 does equal 4,
your solution is correct. And later you’ll find out that 1. 6x – 4 = 32
all equations whose unknown is raised to a power of
1 (m1 = m) have only a single solution. In this lesson, 2. 16a – 13 = 4a + 3
that’s the only kind of equation you’ll have to solve.
Now let’s try two more equations, each of which 3. s + 13 = 6s + 12 – 4s
isn’t quite so easy to solve. Our first will be 2z + 5 = 7.
4. 4x = 44 – 24
Subtract 5 from each side. 2z + 5 – 5 = 7 – 5
2z = 2 5. 2 × 3(–y + 2) = –12
Divide each side by 2. 2z ÷ 2 = 2 ÷ 2
z=1 6. 32y = 46
Check. 2×1+5=7
7=7✔ 7. 34t – 37 = 27

Notice that we check by substituting into the original 8. y+8


3 – 7=0
equation. Next, we’ll get a little more complicated:
This time there will be an unknown on each side: 9p 9. 3b – 18 = b 2– 1
+ 17 = 36 –10p.

Subtract 17 from each side.


9p + 17 – 17 = 36 – 10p – 17
9p = 19 – 10p
Add 10p to each side.
9p + 10p = 19 – 10p + 10p
9p + 10p = 19
19p = 19

88
8th_GRDM_05.qxd:Layout 1 8/12/09 10:19 AM Page 89

–PATTERNS, VARIABLES, AND EQUATIONS–

Proportions and Cross-Multiplication Exercise 6


There is a special class of equations that are easy to Solve the equations for the unknown and check.
solve if you know the shortcut. Consider the equation
x
1. 21 = 79
x 10
=
4 8 2. 4
= 108
x
Multiply both sides of the equation first by 4 and then
15
by 8: 3. 4 = 8y

x 10
4× = ×4
4 8 Summary
40
8× x = ×8
8 In this lesson you found the rules that defined some
8x = 40 sequences, and you learned how to describe them
with variables. You also learned the role of variables
Now we’re left with 8x = 40, and dividing both sides in formulas, and you learned how to solve some
by 8 leaves x = 5. In fact, any time you have exactly equations with one unknown.
one fraction on one side of an equation and exactly
one fraction on the other, you can cross-multiply,
where you multiply one numerator by the other de- Answers
nominator and vice versa. This usually makes the
equation easier to solve. Exercise 1
1. –4, 8, 20, 32, 44, 56; rule is +12
x 10
= 2. 14, 11, 8, 5, 2; rule is –3
4 8
3. 2, 6, 18, 54, 162, 486; rule is ×3
8x = 40
4. –9, –6, –3, 0, 3; rule is +3
x=5
5. –4, 8, –16, 32, –64, 128; rule is ×(–2)
6. 40, 20, 10, 5, 2.5; rule is ÷2
Here’s another example:

3x + 3 x − 2 Exercise 2
= 1. tn = 15 – 4(n – 1)
4 2
The rule is –4, and the first term is 15. You are
2(3x + 3) = 4(x – 2)
subtracting 4 (n – 1) times for the nth term,
6x + 6 = 4x – 8
which is the same as multiplying –4 times (n –
6x + 6 – 4x = 4x – 8 – 4x
1). As a check, let’s do n = 7: t7 = 15 – 4(7 – 1)
2x + 6 – 6 = –8 – 6
= 15 –4(6) = 15 – 24 = –9.
2x = –14
2. tn = –3 + 9(n – 1)
x = –7
The rule is +9, and the first term is –3. You are
adding 9 (n – 1) times for the nth term, which
is the same as multiplying 9 times (n – 1). As a
check, let’s do n = 8: t8 = –3 + 9(8 – 1) = –3 +
9(7) = –3 + 63 = 60.

89
8th_GRDM_05.qxd:Layout 1 8/12/09 10:19 AM Page 90

–PATTERNS, VARIABLES,
–CHAPTER TITLE–
AND EQUATIONS–

Exercise 3 5. y = 4
1. t6 = t4 – t5; tn = tn – 2 – tn –1 2 × (–3y + 6) = –12 ⇒ –6y + 12 = –12 ⇒ –6y +
As a check, t9 = t7 – t8 = 4 – (–6) = 10. 12 – 12 = –12 – 12 ⇒ –6y = –24 ⇒ (–6) –6y –24
= (–6) ⇒
2. t6 = t4 + t5 + 2; tn = tn – 2 + tn – 1 + 2 y = 4.
As a check, t9 = t7 + t8 + 2 = 11 + 19 + 2 = 32. 6. y = 69
3. t6 = t 24 + t 25 ; tn = t n 2– 2 + t n2–1 Because y has a fraction for a coefficient, you
As a check, t7 = t 25 + t 26 = 52 + 292 = 25 + 841 = could just remember how to divide by a
866. fraction (multiply by its reciprocal). Or we
could just multiply each side of the equation by
Exercise 4 3 to get 2y = 138 and then divide each side by 2
1. 0.4 hours, or 24 minutes to find y = 69.
T = D ÷ R = 26 ÷ 65 = 0.4 hours. 7. t = 48
2. 233.3 miles/day First, add 37 to each side to get 34t = 64. Then,
2,800 miles
R = TD = 12 days = 23331, or 233.33333 miles divide by 34 to get t = 48.
per day. 8. y = 13
3. 480 feet This one and the next one are tricky. Whenever
D = R × T = 40 × 12 = 480 feet. you have an unknown in the numerator of a
4. 7 hours fraction, you’ll have to multiply both sides by
T = D ÷ R = 38555 = 7 hours.
the denominator at some point. In this case,
5. 8.3 yards per minute. we’ll add 7 to each side first and then we’ll have
R = TD = 150 yards = 813, or 8.33333 yards per y+8
3 . Then, multiplying each side by 3, we’ll get
18 minutes
minute. y + 8 = 21, so y = 13.
6. 1.488 × 106 miles 9. b = 7
D = R × T = 186,000 × 8 = 1,488,000 miles, or To do this one, keep in mind that when you
1.488 × 106 miles. multiply or divide one side by a certain
number, you have to multiply or divide the
Exercise 5 whole other side by the same quantity. We need
1. x = 6 to get the 2 out of the denominator on the
6x – 4 + 4 = 32 + 4 ⇒ 6x = 36 ⇒ x = 6 right, so we multiply both sides by 2. On the
2. a = 131 left, that will give us 2(3b – 18) = 6b – 36. On
16a – 13 + 13 = 4a + 3 + 13 ⇒ 16a – 4a = 4a + the right, because the 2 in the denominator
16 – 4a ⇒ 12a =16 ⇒ a = 16 4 1 cancels out with the multiplication of 2, you’re
12 = 3 = 13.
3. s = 1 left with just b – 1. Then simple addition and
s + 13 – s = 6s + 12 – 4s – s ⇒ 13 = s + 12 ⇒ 13 subtraction will solve the equation.
– 12 = s.
Note that it’s okay to have your unknown on Exercise 6
the right-hand side if it’s more convenient, as it 1. x = 27
was here. 7x = 189 ≈ x = 27.
4. x = 7 2. x = 3.2 or 315
4x = 44 – 16 ⇒ 4x = 28 ⇒ 4x ÷ 4 = 28 ÷ 4 ⇒ 10x = 32 ≈ x = 3.2.
x = 7. 3. y = 30
4y = 120 ≈ y = 30.

90
8th_GRDM_05.qxd:Layout 1 8/12/09 10:19 AM Page 91

10
L E S S O N

INTRODUCTION
TO FUNCTIONS

L E S S O N S U M M A RY
In this lesson you will learn where functions come from, what a
function is, how to evaluate a function for a given value, and how
to compare different kinds of functions. You’ll also be learning
about the coordinate grid system throughout the lesson, and you’ll
be introduced to various kinds of functions.

I n this lesson, we’ll look at a couple of sample functions and then define what a function is. We’ll look at how
to evaluate a function and map it onto the coordinate grid system. Finally, we’ll look at functions with direct
and inverse variation.

Some Example Functions

Phone Company X charges you $30 per month for up to 20 hours of calling time made locally, then 5 cents per
minute thereafter, until you hit 40 hours. After that you pay 10 cents per minute. How can you figure out your
phone bill? (Assume no long-distance calls.)

91
8th_GRDM_05.qxd:Layout 1 8/12/09 10:19 AM Page 92

–INTRODUCTION TO FUNCTIONS–

One way would be to make a table that shows Any function that involves real numbers can
inputs (number of minutes of local calling) and out- also be graphed on a coordinate grid system. You’ve
puts (how much you’d pay). To construct that table, used these before in math, and you may have graphed
you’d need some rules to convert input to output. For some functions without calling them functions. They
input of 0–20 hours, it would be easy: $30. For 21–40 consist of two axes, usually called the x and y axes,
hours, it would be the $30 plus 5 cents per minute which are at right angles and which are horizontal
times however many minutes you talked. (Remember and vertical number lines.
that there are 60 minutes in an hour, so for each hour
you’ll be paying $0.05 × 60 = $3.00.) How much per y
hour would you pay after 40 hours? It would be $0.10
× 60 = $6.00/hour.
Here we’re going to use variables because the
number of minutes you talk and your phone bill will 5
4
vary from month to month. Let’s call the amount of
3
your phone bill A and measure it in dollars. We’ll call 2
your talking time, in hours, T. Here’s a table that 1 x
shows some sample values: –5 –4 –3 –2 –1 0 1
–1
2 3 4 5

–2
–3
Input (T, in hours) 0–20 25 30 35 40 45 50 –4
–5
Output (A, in $) 30 45 60 75 90 120 150

Another way of thinking about the relationship


between your hours on the phone and your bill is as a
collection of ordered pairs. For each number of hours
of calling time (including those not in the table, such
The input variable is often called the independ-
as 23 hours or 27.9 hours), there is a specific and
ent variable, because it exists outside the function,
knowable dollar amount. That is, for each input T
and the output variable is called the dependent vari-
there is an output A. The relation between your hours
able, because it’s dependent on the input. The inde-
on the phone and your bill can be called a function. A
pendent variable is typically plotted on the x axis and
function is defined as any set of inputs and outputs in
the dependent variable on the y axis. Also, the de-
which each input is associated with a unique output.
pendent variable is often called a function of the de-
Another property of a function is that it has a do-
pendent variable. If the independent variable is called
main. To put it simply, the domain consists of all the
x, as it often is, the function is called f(x) and pro-
numbers that are valid for a given function. The do-
nounced “F of X.” In the phone-bill function we’re
main is usually self-evident; for example, the domain
working on now, the A (the amount of the phone
of the function defined by the phone bill is obviously
bill) is another name for f(T). Here is a graph of the
restricted to numbers 0 or greater; you can’t have
phone-bill function with our points filled in and con-
negative calling time.
nected by lines.

92
8th_GRDM_05.qxd:Layout 1 8/12/09 10:19 AM Page 93

–INTRODUCTION TO FUNCTIONS–

A Exercise 1
150 Fill in the input/output tables for each function:
Choose numbers from the domain and substitute
120 them for the variable. (The first table is filled in for
you.) Then match the numbered graphs that follow
90
to the functions. Answers are on pages 99–100.

60 1. f(x) = 5x – 3(x + 1), for all real x

30
INPUT OUTPUT

–2 –7
T
10 20 30 40 50 0 –3

2 1

20 17

It makes sense to connect the dots with a line, because


every value between our input points will also have
2. f(x) = x + (x – 1) + (x – 2) for all positive integers
an output.
x
Now let’s try to write a rule for this function in
mathematical terms. If T is 20 or less, A is 30, so if T ≤
INPUT OUTPUT
20, A = 30. If T > 20 and ≤ 40, which can be written as
40 ≥ T > 20 or 20 ≤ T ≤ 40, A = f(T) = f(20) + 3(T – 20).
If T > 20, then A = f(T) = f(40) + 6(T – 40). Here’s the
mathematically correct version of our function:

= f(T) = 30 for all T, 0 ≤ T ≤ 20


= f(20) + 3(T – 20) for all T, 20 < T ≤ 40
= f(40) + 6(T – 40) for all T, 40 < T

Here are some examples of other functions: 3. f(x) = |x| – 1 for all x < 0

f(x) = 3x + 5(x – 3) for all real x INPUT OUTPUT

f(x) = x(x – 1)(x – 2) for all positive integers


=1 x>2
for x = 1 and x = 2
f(x) = |x| – 1 for all x < 0
f(m) = 0.5m + 33 for all real m
f(x) = 59 x + 32 for all x > –273

93
8th_GRDM_05.qxd:Layout 1 8/12/09 10:19 AM Page 94

–INTRODUCTION TO FUNCTIONS–

Graph 1 Graph 3
f(x) f(x)

x x

Graph 2 4. f(m) = 0.5m + 2 for 2 < m < 4

INPUT OUTPUT

5. f(C) = 59C + 32, for all x > –273

INPUT OUTPUT

94
8th_GRDM_05.qxd:Layout 1 8/12/09 10:19 AM Page 95

–INTRODUCTION TO FUNCTIONS–

6. f(x) = 21x2, for all real x Graph 5


f(x)
INPUT OUTPUT

Graph 4

Graph 6
f(c)

40
30
20
10
c
10 20 30 40

95
8th_GRDM_05.qxd:Layout 1 8/12/09 10:19 AM Page 96

–INTRODUCTION TO FUNCTIONS–

Evaluating a Function Exercise 2


Evaluate the given functions as indicated; assume the
If you did Exercise 1 correctly, you have already value asked for is in the function’s domain. You may
learned to evaluate a function for a given input. For use your calculator on questions 3 and 4. Answers are
example, in the function on page 100.

f(T) = 30, for all T, 0 ≤ T ≤ 20 1. f(x) = 3x2 – 4x + 6; f(–2) =


= f(20) + 3(T – 20) for all T, 20 < T ≤ 40
= f(40) + 6(T – 40) for all T, 40 < T 2. f(a) = 3a + 4(2 – a); f(8) =

we can evaluate the function for any T that is 0 or 3. f(x) = 0.23x3; f(6) – f(5) =
above. To do that, we simply substitute any value of T
into the function. For T = 10, f(T) = 30, because 30 is 4. f(x) = 2x; f(43) =
the value of f(T) for any T that’s 20 or less. What if T
is 28? To find out, we’ll use the function equation for 5. f(n) = n(n +2 1); f(9) – f(8) =
T between 20 and 40:
6. f(n) = (–1)n(3 + n); f(4) + f(5) =
f(T) = f(20) + 3(T – 20)
f(28) = 30 + 3(28 – 20) = 30 + 3(8) = $54
Comparing Functions
In the same way, we can evaluate the function at T =
47 by using the function for T > 40: Recall the function that your phone company is using
to bill you every month. Because you’ve been averaging
f(T) = f(40) + 6 (T – 40)
50 hours of calls per month, your phone bill has been
f(47) = 90 + 6(47 – 40) = 90 + 6(7) = $132
averaging $150. You think you could save money by us-
ing other plans, so you investigate. The WeServ Com-
We can also add and subtract the values of functions
pany has a plan that charges you a $100 flat fee plus 2
at different points, with function notation. For in-
cents per minute for all calls of local origin. The AtYour
stance, if you used your phone for 28 hours one
Service Company has a simple charging system: It
month and 47 hours the next, your total bill for those
charges 4 cents per minute for all calls up to 100 hours
two months would be f(28) + f(47) = $54 + $132 =
with only a $10 administrative fee per month. Could
$186. Also, if you wanted to know how much bigger
you save money with either of these plans? Let’s write
your bill was the second month than the first, you
the functions and evaluate them at 50 hours per
could subtract: f(47) – f(28) = $132 – $54 = $78.
month to see what your best plan would be.
You cannot evaluate a function at a value that is
not in the function’s domain.
WeServ Plan: At 2 cents per minute, the hourly
charge would be $.02 × 60 = $1.20 per hour.
Because you pay the flat fee no matter how many
hours you use, the function would just be g(T) =
100 + 1.2T. For T = 50, g(T) = 100 + 1.2 × 50 =
$160. So, if you’re sure that you’ll stay at 50 hours
per month, the WeServ plan is not better than
your current plan. Here’s a graph of the plan.
96
8th_GRDM_05.qxd:Layout 1 8/12/09 10:19 AM Page 97

–INTRODUCTION TO FUNCTIONS–

g(t) Exercise 3
Use the graphs of your current plan, the WeServ plan,
150
and the AtYourService plan, or the function defini-
tions to answer the following questions. Answers are
100 on page 100.

1. Which plan would be best for someone who


used the phone 75 hours per month? Which
20 30 40 50
would be worst?

2. Which plan would be best for someone who


used the phone 35 hours per month? Which
would be worst?

3. Which plan would be best for someone who


used the phone 75 hours in one month but
Note: Graph is not plotted to scale.
only 10 hours the next? Which would be worst?

AtYourService Plan: At 4 cents per minute, the


hourly charge would be $.04 × 60 = $2.40 per Types of Functions
hour. Thus, your function would be h(T) = 10
+ 2.4T. For T = 50, h(T) = 10 + 2.4 × 50 = $130. Any relationship that maps input to a unique output
At 50 hours per month, the AtYourService plan is a function, but there are some ways of defining
is definitely better. Here’s a graph of the plan. functions that you must recognize in eighth grade.
These are
h(T)

■ discrete and continuous functions


140
■ linear functions
■ absolute value functions
100
■ quadratic functions
■ exponential functions

T
20 30 40 50 Discrete and Continuous Functions
If you were to graph the closing price of the Dow
Jones Industrial Average stock index over a period
of time, you would have a graph with days as the in-
put variable (graphed on the x axis), and the price,
in dollars, on the y axis. The graph would be a series
of points, because it wouldn’t tell you what the clos-
ing price was between April 20 and April 21, for ex-
Note: Graph is not plotted to scale. ample; there’s only one closing price per day. A

97
8th_GRDM_05.qxd:Layout 1 8/12/09 10:19 AM Page 98

–INTRODUCTION TO FUNCTIONS–

function with only disconnected, or discrete, points


is called a discrete function. (Discrete is the opposite
of concrete.) You could connect the dots to show a
trend, as the newspapers do, but the line between y = |x|

the points wouldn’t really have meaning. In the tele-


phone bill functions earlier in this lesson, however,
there really is a different price for every fraction of an
hour and even of a minute. Graphs that give a unique
value for every input variable are called continuous
functions.

Linear Function
A linear function is a function that can be repre-
sented by a single line. All three example telephone
company phone bill functions are continuous, but
only the WeServ plan and the AtYourService plan are
linear functions, because they can be represented by
one straight line. There are three connected but dif- Quadratic Equations
ferent lines in Phone Company A’s plan. You’ll learn a Unfortunately, not all the phenomena around us can
lot more about linear functions in Lessons 12 and 13. be described by linear functions. For example, it was
Their general form is f(x) = ax + b, where x is the in- found centuries ago that the path any object takes
put variable and a and b are constants. as it’s shot or thrown into the air can be predicted
by a quadratic function. For example, the function
Absolute Value Function f(t) = 21(–9.8)t2 + 49t + 2.5 describes the height of a
An absolute value function consists of two straight rocket shot upward at any time t in seconds after be-
lines that meet at some point. Their general form is ing shot. The graph looks something like this.
f(x) = ax + b, where x is the input variable and a and b
are constants. f(t)

10
t
0

98
8th_GRDM_05.qxd:Layout 1 8/12/09 10:19 AM Page 99

–INTRODUCTION TO FUNCTIONS–

What distinguishes a quadratic function is that 2. Graph 1


it has an input variable that is raised to the second There are no lines in this graph, because the
power, or squared. Its general form is f(x) = ax2 + bx function is only defined for integers, not the
+ c. You’ll learn how to solve certain types of quad- numbers between them. A function where the
ratic equations in Lesson 13. points are not connected is called a discrete
function.
Exponential Equations
INPUT OUTPUT
The interest equation that you first saw in Lesson 4 is
an example of an interest equation. The input vari- 1 0
able is the number of years, and it’s in an exponent in
2 3
the definition of the function: f(n) = P(1 + i)n, where
n is the number of years. The graph is always a curve, 3 6
and sometimes it’s not too steep, as in the interest 4 9
function, and sometimes it’s very steep, as in the
growth of a bacteria colony.
3. Graph 4
Notice that only negative numbers on the x
Summary axis have values. Also, the line stops just short
of 0, because the function is not defined for 0.
In this lesson, you’ve learned what a function is, how INPUT OUTPUT
to evaluate a function for a given value, and how to
–1 0
compare different kinds of functions depending on
what kind of output you’re interested in. –2 1

–3 2

Answers –5 4

Exercise 1 4. Graph 5
Answers to the input/output tables can vary.
Notice that the line is rather short, because the
1. Graph 3
domain is so small.
INPUT OUTPUT
INPUT OUTPUT
–1 –5
2.5 3.25
0 –3
3 3.5
1 –1
3.5 3.75
2 1
4 4

99
8th_GRDM_05.qxd:Layout 1 8/12/09 10:19 AM Page 100

–INTRODUCTION TO FUNCTIONS–

5. Graph 6 Exercise 3
Note: this is the function that converts Celsius 1. WeServ and AtYourService would be equally
temperatures to Fahrenheit. good. Your plan would be much more
expensive. Current plan: f(75) = f(40) + 6(T –
INPUT OUTPUT
40) = 90 + 6(75 – 40) = 300; WeServ: g(75) =
–5 23 100 + 1.2(75) = 100 + 90 = 190; AtYourService:
h(75) = 10 + 2.4(75) = 10 + 180 = 190. Note
0 32
that for any total above 75 hours, the WeServ
10 50 plan would be the best, because it’s going up at
a lower rate.
20 68
2. Your plan is best. The WeServ plan would be
worst. Current plan: f(35) = 75; WeServ: g(35)
6. Graph 2 = 100 + 1.2(35) = 100 + 42 = 142;
Note that all values of f(x) are greater than or AtYourService: h(35) = 10 + 2.4(35) = 10 + 84
equal to zero and that the values of x are the = 94.
same as the values of –x. 3. f(75) = 310, f(10) = 30, f(75) + f(10) = 340;
g(75) = $190, g(10) = 100 + 1.2(10) = $112,
INPUT OUTPUT
g(75) + g(10) = 302; h(75) = 190, h(10) =
–2 2 10 + 2.4(10) = 34, h(75) + h(10) = 224.

0 0

1 0.5

2 2

Exercise 2
1. f(x) = 26; f(x) = 26; f(–2) = 3(–2)2 –4(–2) + 6 =
3(4) – 4(–2) + 6 = 12 + 8 + 6 = 26
2. f(8) = 0; f(8) = 3(8) + 4(2 – 8) = 3(8) + 4(–6) =
24 – 24 = 0
3. f(6) – f(5) = 20.93; f(6) = 0.23(6)3 = 49.68, f(5)
= 0.23(5)3 = 28.75, f(6) – f(5) = 20.93
4. f(43) = 1.68 (rounded to the nearest hundredth);
( )
3
f ( 3 ) = 2 4 = 4 2 3 ≈ 1.68
4
5. f(9) – f(8) = 9; f(9) = 9(92+ 1) = 9(10
2
)
= 45 ; f(8) =
8(8 + 1) 8(9)
2
= 2 = 36; f(9) – f(8) = 45 – 36 = 9
6. f(4) + f(5) = –1; f(4) = (–1)4(3 + 4) = 1(7) = 7;
f(5) = (–1)5(3 + 5) = (–1)(8) = –8

100
8th_GRDM_05.qxd:Layout 1 8/12/09 10:19 AM Page 101

11 OPERATIONS
L E S S O N

WITH
POLYNOMIALS

L E S S O N S U M M A RY
You will need to know how to do operations with polynomials and
other algebraic expressions in order to work with functions. Most
of these are very much like the number operations you studied in
Section 1, but there are a few new twists.

I n this lesson, you’ll be working with polynomials. For a definition of a polynomial, you first need some
other definitions. Constant is another word for a number. You’ve already learned variable and coefficient in
Lesson 9, and we’ve been using the word term, which can be used for any variable, constant, or combination
of them. An expression is a collection of one or more terms; these terms are added or subtracted if the number
of terms exceeds one. A polynomial is an expression with more than one term. An expression that has only
multiplication or division, even if it has several variables, is called a monomial. Clear as mud, right? Here are
some examples that might help to distinguish these terms:

–1
Terms: 8.72, 3y, abc, 7, 5m
n , (1.04)x
4

x
Expressions: 6, 6xy, 7x + 2, 3ab – 2cd
Polynomials: 7x + 2x, 3ab – 2cd
Monomials: 6xy, 18abc
4b 2 c 2

101
8th_GRDM_05.qxd:Layout 1 8/12/09 10:19 AM Page 102

–OPERAT IONS WITH POLYNOMIALS–

Note that all terms are expressions and that all Addition and Subtraction
polynomials are expressions. Note also that polyno-
mials always contain + or – signs. You’ll also need to The key to addition and subtraction of polynomials is
know the commutative, associative, and distributive that you can only add or subtract like terms. For ex-
properties of addition and multiplication. ample, you can add 2y to 5y to get 7y, but you can’t
add 2y to 5y2 at all. Remember from Lesson 4 that you
can add 2 5 to 5 5 to get 7 5 , but you can’t add
COMMUTATIVE PROPERTY OF ADDITION 2 5 to 5 3 5 , unless, of course, you calculate the
a+b=b+a square and cube roots first. As an example, let’s add the
polynomial x2 + 3x – 5 to the polynomial 4x2 – 8x + 4.

COMMUTATIVE PROPERTY Group like terms: x2 + 4x2 + 3x – 8x – 5 + 4


OF MULTIPLICATION Combine like terms: 5x2 – 5x – 1
a×b=b×a
This is pretty easy, right? As long as you’re care-
ful, it won’t be too difficult. Let’s move to subtraction:
ASSOCIATIVE PROPERTY OF ADDITION Now we’ll subtract the polynomial 4x2 – 8x + 4 from
the polynomial x2 + 3x – 5. This will involve an extra
a + (b + c) = (a + b) + c
step. (Actually, it’s just a step that we could skip in ad-
dition because there are no sign changes in addition.)

ASSOCIATIVE PROPERTY OF MULTIPLICATION


Write subtracted expression in parentheses:
a × (b × c) = (a × b) × c x2 + 3x – 5 – (4x2 – 8x + 4)
Change signs of expression in parentheses:
x2 + 3x – 5 – 4x2 + 8x – 4
DISTRIBUTIVE PROPERTY Group like terms: x2 – 4x2 + 3x + 8x – 5 – 4
a × (b + c) = a × b + a × c Combine like terms: –3x2 + 11x – 9

a × (b – c) = (a × b) – (a × c)
How do you check that operations on polyno-
mials are correct? One way to do it is to substitute a
We’ll begin with addition and subtraction in number such as 2 or 3 into both polynomials and see
polynomials, followed by multiplication and division. if the sums or differences are the same.
Then we’ll look at factoring and simplifying polyno-
mials, and we may even make some of the number x2 + 3x – 5 evaluated at x = 2: 4 + 6 – 5 = 5
operations seem easier to do along the way. 4x2 – 8x + 4 evaluated at x = 2: 16 – 16 + 4 = 4
Note: You’ll have to know the order of operations. (sum) 5x2 – 5x – 1 evaluated at x = 2: 20 – 10 – 1
=9✓
■ Parentheses (difference) –3x2 + 11x –9 evaluated at x = 2:
■ Exponents –12 + 22 – 9 = 1 ✓
■ Multiplication and Division, left to right
■ Addition and Subtraction, left to right

102
8th_GRDM_05.qxd:Layout 1 8/12/09 10:19 AM Page 103

–OPERAT IONS WITH POLYNOMIALS–

Exercise 1 Special Cases


Do the operations A(z) + B(z) and A(z) – B(z) for There are two special cases of polynomial multiplica-
each of the following. Answers are on page 106. tion that you ought to know. They’ll be very impor-
tant when you get to high school algebra. The first is
1. A(z) = 4x2 + 3z – 2; B(z) = –2x2 + 3 the square of a two-term polynomial, or binomial.

2. A(z) = z3 – z2 + 7z; B(z) = 2z3 + z2 – 4z – 4 (a + b)2 = (a + b)(a + b)


= a2 + ab + ab + b2
3. A(z) = 3z2 – z + 4 + (3z – z); B(z) = –z3 – 4 = a2 + 2ab + b2

4. A(z) = 4(z2 – 2z) – 5; B(z) = 2x2 + 6z + 5 and


– 2(z2 + 1)
(a – b)2 = (a – b)(a – b)
= a2 – ab – ab + b2
Multiplication
= a2 – 2ab + b2
We can multiply a polynomial by a monomial or by a
polynomial. Multiplication by a monomial is easy, if
Try it with some positive integers to convince yourself
you just remember the distributive property: a(b + c)
it works (actually, it works for any two numbers). For
= ab + ac. Thus, 4xy2(3x – 4y) = 12x2y2 – 16xy3. (Re-
example, if a = 2 and b = 3, then (2 + 3)2 = 4 +
member from Lesson 4 that you multiply a base with
2(2)(3) + 9 = 4 + 12 + 9 = 25 = 52. And (2 – 3)2 = 4 –
its exponents by adding the exponents.)
2(2)(3) + 9 = 4 – 12 + 9 = 1 = (–1)2.
When you multiply a polynomial by another
The next special case involves the sum and dif-
polynomial, you also use the distributive property, but
ference of two numbers: Their product always equals
you have to multiply each term in the first polynomial
the difference of their two squares.
by each term in the second. As an example, let’s multi-
ply (2a + 3b) times (a – b + 2):
(a + b)(a – b ) = a2 – ab + ab + b2
= a2 – b2
(2a + 3b)(a – b + 2)
= 2a2 – 2ab + 4a + 3ab – 3b2 + 6b
Again, use some substitution for a and b to convince
= 2a2 – 3b2 + 4a + 6b + ab
yourself that this always works. Let’s do a couple of
these for practice before you get to the exercises.
First, we multiplied each term in the second polyno-
mial by 2a: 2a × a + 2a × (–b) + 2a × 2. Then, we mul-
( ) = x + 2xy ( )
2

tiplied each term in the second polynomial by 3b: 3b x+ y 5 2


5 + y2 5× 5
× a + 3b × (–b) + 3b × 2. Then, we added like terms = x + 2 xy 5 + 5 y
2 2

( x + y 5 )( x − y 5 ) = x − 5 y
(the only like terms were –2ab and 3ab). The answer 2 2
could be given in any order, but usually we put the
higher powers first so they’re easy to see.

103
8th_GRDM_05.qxd:Layout 1 8/12/09 10:19 AM Page 104

–OPERAT IONS WITH POLYNOMIALS–

Exercise 2 Be sure you understand how to get these answers be-


(Answers are on page 106.) fore you go on. Now let’s divide a polynomial by a
monomial. We’ll have to divide every term in the
1. 5p2q(3p2q + 1) = polynomial by the monomial.

2. (2a + b)15a = 3a 6b 3 3 6b 2
(3a + 6b 3 ) ÷ 5ab = + = +
5ab 5ab 5b 5a
3. (4c + d)(3c – 2d) =
Adding Fractions with
2
4. (2x + y)(3x – xy + y ) = 2 Polynomial Denominators
Let’s first look at the fractions ab and dc. To add them
5. (4a + b)2 = we get the common denominator of bd and then
multiply the first fraction by (dd) and the second by
6. (6 – y)(6 + y) = (bb). Then we solve the addition problem with ad bd + bc
.
In Lesson 2, we didn’t give a formula for adding
7. (x4 + y3)2 = fractions: Do you think it’s easier to use a formula or
just go about finding a common denominator?
8. (x4 – y3)(x4 + y3) = (Hint: There’s no right answer; it’s whatever works
better for you!)
9. (3 + 5)(8 + 90) = Let’s add two fractions, one with a polynomial
denominator and one with a monomial denominator.
Our first step will be to get a common denominator,
Division which will be the product of the two denominators:
y2(y – 3). We’ll multiply the first fraction by yy –– 33 and
y2
We will concentrate on dividing by monomials and the second by 2 .
y
adding fractions with polynomials in their denomi-
nators. 6 − y y + 3 ⎛ y − 3⎞ 6 − y y + 3 ⎛ y2 ⎞
+ = +
y2 y − 3 ⎜⎝ y − 3 ⎟⎠ y 2 y − 3 ⎜⎝ y 2 ⎟⎠
Division by Monomials
− y 2 + 9 y − 18 y 3 + 3 y 2
First, look at a couple of divisions of monomials by = + 2
monomials; all you have to do is divide constants and y3 − 3y2 y − 3y2
subtract exponents. y 3 + 2 y 2 + 9 y − 18
=
y3 − 3y2
26 x 2 y 8 z 3
= 2 x −1 y 4 z 2
13x 3 y 4 z If none of this looks familiar, go back to adding frac-
6 tions in Lesson 2.
18a 2b −5c ÷ 3a 3b 4 c = 6a −1b −9 =
ab 9

104
8th_GRDM_05.qxd:Layout 1 8/12/09 10:19 AM Page 105

–OPERAT IONS WITH POLYNOMIALS–

Exercise 3 grade, either, unless you’re in an advanced class)


(Answers are on page 106.) unless it contains one of the special cases listed un-
der the multiplication section: (a + b)2, (a – b)2, or
225 x 5 y 4 z
1. = (a + b)(a – b).
125 x 3 y 4 z 2

2x5 – 4y Exercise 4
2. 4xy
=
Simplify the following expressions. Answers are on
y y page 106.
3. y – 1 – y – 3 =
1. 16b2 – 25c2

Factoring and Simplifying 2. 4z3 – 20z


Algebraic Expressions
3. 4y – 9xy – 6y2 + 20
You’ve already learned some ways to simplify alge-
braic expressions. For instance, the expression 43 x + 43 x
4. 3x2y – 12y
+ 23 x is simplified (that is, easier to evaluate) if you
just add it up to get 3x. But you can also factor an al-
5. 6 – 3m
m+4
+2
gebraic expression.
Factoring an algebraic expression consists of
6. 2c – 5c2 + 10cd – 5d2 – 2d
three steps:

1. Combine like terms through addition or sub-


Summary
traction, if possible.
2. Factor out any monomial factors, if there are
In this lesson you’ve extended your knowledge of
any. Example: 3x3 – 12x2 + 12x = 3x(x2 – 4x + 4).
number operations to include algebraic expressions,
Sometimes, just one part of the polynomial
which include variables as well as constants. You’ll
has a common factor, and you can just factor that
need this extended knowledge to tackle the algebra in
out. Example: 3y + 4x2 – 12x + 6x3. Rearranging
the next four lessons as well as throughout eighth
the terms, we get 3y + 6x3 + 4x2 – 12x = 3(y + 2x3)
grade.
+ 4(x2 – 3x).
3. Then see if you can find any polynomial factor.
In the example of step 2, you might recognize
that (x2 – 4x + 4) is the same pattern as (a2 –2ab
+ b2), which was produced by (a – b)2. That
means that (x2 – 4x + 4) = (x2 – 2)2, and the
whole expression simplifies to 3x(x2 – 2)2.

Because finding polynomial factors hasn’t been dis-


cussed in this book, you won’t need to do step 3 in
any of the exercises (and probably not in eighth

105
8th_GRDM_05.qxd:Layout 1 8/12/09 10:19 AM Page 106

–OPERAT IONS WITH POLYNOMIALS–

Answers Exercise 3
5 4 2 0 −1 2
1. 225 x y z = 9 x y z = 9 x
Exercise 1 125 x 3 y 4 z 2 5 5z
1. A(z) + B(z) = 2x2 + 3z +1
2. 2 x − 4 y = 2 x − 4 y = 2 x − 1
5 5 4

A(z) – B(z) = 4x2 + 3z – 2 – (–2x2 + 3)


4 xy 4 xy 4 xy 4 y x
= 4x2 + 3z – 2 + 2x2 – 3 = 6z2 + 3z – 5
2. A(z) + B(z) = 3z3 + 3z – 4 3. y y ⎛ y − 3 ⎞ ⎛ y ⎞ ⎛ y −1⎞ ⎛ y ⎞
− =⎜ −
A(z) – B(z) = z3 – z2 + 7z – (2z3 + z2 – 4z – 4) y − 1 y − 3 ⎝ y − 3 ⎟⎠ ⎜⎝ y − 1 ⎟⎠ ⎜⎝ y − 1⎠⎟⎠ ⎜⎝ y − 3 ⎟⎠
= z3 – z2 + 7z – 2z3 – z2 + 4z + 4 = –z3 – 2x2 y2 − 3y y2 − y
+ 11z + 4 = −
y2 − 4 y + 3 y2 − 4 y + 3
3. A(z) + B(z) = –z3 + 3z2 + z
−2 y
A(z) – B(z) = 3z2 – z + 4 + 2z – (–z3 – 4) = 3z2 = 2
y −4y +3
– z + 4 + 2z + z3 + 4 = z3 + 3z2 + z + 8
4. A(z) + B(z) = 4x2 – 8z – 5 + 2x2 + 6z + 5 – 2x2
– 2 = 4x2 – 2z – 2 Exercise 4
A(z) – B(z) = 4x2 – 8z – 5 – (2x2 + 6z + 5 – 2x2 1. 16b2 – 25c2 = (4b)2 – (5c)2 = (4b + 5c)(4b – 5c)
– 2) = 4x2 – 8z – 5 – 2x2 – 6z – 5 + 2x2 + 2 2. 4z3 – 20z = 4(z3 – 5z) = 4z(z3 – 5)
= 4x2 – 14z – 8 3. 4y – 9xy – 6y2 + 20 = 4y + 20 – 9xy – 6y2
= 4(y + 5) – 3(3xy + 2y2)
Exercise 2 or
1. 5p2q(3p2q + 1) = 15p4q2 + 5p2q = 4(y + 5) + 3(–3xy – 2y2)
2. (2a + b)15a = 30a2 + 15ab 4. 3x2y – 12y = 3y(x2 – 4)
3. (4c + d)(3c – 2d) = 12c2 – 8cd + 3cd – 2d2 = 5. 6 – 3m +2 m + 4 3m + 2 6m + 24 3m + 2
m+4 = 6 m+4 – m+4 = m+4 – m+4 =
12c2 – 5cd – 2d2 3m + 22
m+4
4. (2x + y)(3x2 – xy + y2) = 6x3 – 2x2y + 2xy2 + 6. 2c – 5c2 + 10cd – 5d2 – 2d
3x2y – xy2 + y3 = 6x3 + x2y + xy2 + y3 = 2c – 2d – 5c2 + 10cd – 5d2
5. (4a + b)2 = 16a2 + 8ab + b2 = 2(c – d) – 5(c2 – 2cd + d2)
6. (6 – y)(6 + y) = 36 – y2 = 2(c – d) – 5(c – d)2
7. (x4 + y3)2 = x8 + 2x4y3 + y6
8. (x4 – y3)(x4 + y3) = x8 – y6
9. (3 + 5)(8 + 90) = 24 + 270 + 40 + 450 = 784 =
(8)(98)

106
8th_GRDM_06.qxd:Layout 1 8/11/09 1:43 PM Page 107

GRAPHING

12 LINEAR
L E S S O N

EQUATIONS
AND
INEQUALITIES

L E S S O N S U M M A RY
In this lesson you will learn how to graph linear equations, which
are equations with one unknown. You’ll be able to use what you
learned in Lessons 9 and 11 to help you manipulate equations and
to make better use of these functions. You’ll also learn how to
solve inequalities.

A linear equation with one unknown is an equation that can be put in the form ax + b = 0, with a and
b constants and a =/ 0. (Why can’t a be equal to zero? Then you wouldn’t have an unknown, and the
equation wouldn’t be true unless b was equal to zero! It’s not very helpful to know that 0 = 0.) As
you learned in Lesson 9, the solution to this equation is x = ab. The general equation for a line is y = mx + b. Note
that there’s now another unknown: y. But that’s okay, because there are two axes on graphs.
In this chapter you’ll learn how to use graphs to draw the lines that represent equations with one un-
known. You’ll also learn about the meanings of those lines. Knowing these things will help you solve lots of
classroom and real-life problems, and it will also help you on standardized tests as long as you’re in school.
You’re only in eighth grade now, but the ACT, SAT, and other college entrance tests have lots of questions on the
topics in this lesson.

107
8th_GRDM_06.qxd:Layout 1 8/11/09 1:43 PM Page 108

–GRAPHING LINEAR EQUATIONS AND INEQUALITIES –

Slope-Intercept Form the same way, we find the difference between the x co-
ordinates: 3 – (–2) = 5.
Remember from Lesson 10 that the x-axis is the hori-
y
zontal axis of a graph and the vertical dimension is
called the y-axis. Any ordered pair of numbers (p,q)
can be represented on a two-dimensional graph, with (–2,4) run = 5
p representing the number on the x axis and q repre-
senting the number on the y axis. For example, here’s
rise = –7
a graph with the points (–2,4) and (3,–3) plotted.
x
y

(3,–3)

(–2,4)
4
3
2
1 x
–4 –3 –2 –1 0 1 2 3 4
–1
–2
–3 The general formula for finding the slope of a
(3,–3)
–4 line with points (x1,y1) and (x2,y2) is

y 2 − y1
slope =
x 2 − x1

Our slope for the line between those two points will
be –75 = – 57. We started with two points: How do you
Finding the Slope know which one is (x1, y1) and which is (x2, y2)? The
We will graph a linear equation that has both of those answer is that it doesn’t matter. What you do have to
points on it, but first let’s go over the meaning of the keep in mind, though, is that you have to subtract in
constants in the equation y = mx + b. The constant m the same direction for both the x’s and the y’s. For in-
is called the slope of the line, and the slope measures stance, if to find the slope in this example, we had be-
the steepness and also the direction of the steepness. gun by subtracting 4 – (–3) we would get a rise of 7.
Mathematically, the slope is expressed as the ratio be- Then we’d have to subtract the x’s in the same order,
tween the amount of change in y to the change in x. and we’d get –2 –3 = –5 and our slope would be –57 =
It’s often referred to as “rise over run,” or rise
run . Take a
– 57 —the same.
look at the graph again. To find the slope of a line
connecting those two points, we’ll count the distance Finding the Intercepts
between the y values, which are 4 and –3. The value of The slope-intercept form has another essential: the in-
this number is called the rise. Mathematically, we find tercept. The word intercept means the point at which
this number by subtracting 4 from –3: –3 – 4 = –7. In the line crosses the y-axis. (This, of course, is the y in-
tercept; the x intercept, which isn’t part of the point-

108
8th_GRDM_06.qxd:Layout 1 8/11/09 1:43 PM Page 109

–GRAPHING LINEAR EQUATIONS AND INEQUALITIES –

slope form, is where the line crosses the x-axis.) The Some Special Cases
first number in that coordinate will always be 0, because Of course, nothing you learn would be complete
the y axis is defined as the line where x = 0. So if you without exceptions, and the slope-intercept form has
have the equation y = 3x + 2, where is the y intercept? a couple of these as well: y = b and x = c, where both
The obvious solution—and the right one—is to let x = b and c are constants. Look at the lines for y = 1 and
0 in the equation. Then you have y = 3 × 0 + 2, or y = x = –2 below.
2. In fact, in the equation y = mx + b the constant b is
always the y intercept. Since y = 3x + 4 is already in y
slope-intercept form, we can graph it without too much
trouble. First, we plot the point (0,2), and from that
point we get a slope of 3 by counting up 3 and 1 to the
x = –2
right. That point would be (0+1,2+3), or (1,5). Now we
can just draw the line through those two points.
y=1
y x

y 2 − y1
If you use the slope formula slope = x 2 − x1 , you can
pick out any two points on the line y = 1, such as
(–6,1) and (1,1), and you’ll get a slope of 0—which
seems right, because the line has no slope. On the
equation x = –2, however, any two points you pick
y will give you a denominator of 0 in the slope for-
mula. Because division by 0 is not allowed, we say
How would you find the x intercept of that the slope for lines in which x is equal to a constant is
line? You set y = 0 and solve the equation for x. (See undefined.
Lesson 9 if you need to refresh your memory on solv-
ing equations.)

0 = 3x + 2
0 – 2 = 3x + 2 – 2
–2 = 3x
–2 3x
3 = 3
– 32 = x

So the x intercept is – 32, and its coordinates are (– 32,0).

109
8th_GRDM_06.qxd:Layout 1 8/11/09 1:43 PM Page 110

–GRAPHING LINEAR EQUATIONS AND INEQUALITIES –

Exercise 1 Finding an Equation


Find the slope of a line that goes through the follow- Given Two Points
ing pairs of points. Answers are on page 113.
Sometimes you won’t be given an equation for a line
1. (5,2) and (3,0) but just two points. If you are given just the two
points, you can not only draw the line but write the
2. (–7,3) and (1,3)
equation as well. Here’s how:
3. (21 ,3) and (– 21 ,7) y 2 − y1
1. First, calculate the slope using x 2 − x1 .
4. (6,–2) and (6,2) 2. Then, select whichever point has the friendliest-
looking numbers (fairly small and no fractions).
y−y
5. (65,– 34 ) and (0,0) 3. Then, set up the equation slope = x − x11 , where
x1 and y1 are the coordinates of the point you
6. (–9,–15) and (–1,–7) selected.
4. To get the equation in point-intercept form, use
Find coordinates of the y intercepts and the x inter-
algebra to simplify the equation to the form y =
cepts for the following equations.
mx + b.
7. y = 3x – 3 5. Use the point you didn’t use to find the equa-
tion to check: Fill in its values for x and y and
8. 2x = y + 45 make sure the equation works.

9. 6y – x – 12 = 0 Let’s use the points (–2,4) and (3,–3)—the ones


used in the graphs in the previous section in the intro-
duction to slope. The slope we found was – 57. Now we
can select either point to set up the equation. Let’s use
(3,–3)—both points have friendly-looking numbers.

y− y
slope = x − x1
1

7 y − (−3)
−5 = x −3
y+3
− 57 = x + 3

At this point we’ll use cross-multiplication (see Les-


son 9).

y+3
− 7 × (x − 3) = × (x − 3)
5 x −3
− 7 x + 21 = y + 3
5 5
7 21
− x + − 3 = y + 3− 3
5 5
−7x+6 = y
5 5

110
8th_GRDM_06.qxd:Layout 1 8/11/09 1:43 PM Page 111

–GRAPHING LINEAR EQUATIONS AND INEQUALITIES –

Our y intercept, then, is (0, 56 )—if the equation is cor- Just as a check, let’s look at some other points
rect. Now we need to check this equation using the on that line. How about the point where x = 5? Sub-
other point, (–2,4). stituting x = 5 into the equation – 75x + 65 = y, we get
y = –545. (Make sure you know why before you go on!)
7 6 Looking at the graph, you see that the point (5,–545)
− (−2) + = 4
5 5 appears to be on the line.
14 6
+ =4
5 5 Exercise 2
Find the slope-intercept equation for graphs with the
If you let y = 0 in the slope-intercept form, you get
pairs of points given in Exercise 1. Then sketch a
7 6 graph for each equation.
− x+ =0
5 5
7 6 1. (5,2) and (3,0)
− x=−
5 5
2. (–7,3) and (1,3)
7 ⎛ 5⎞ 6 ⎛ 5⎞
− x ×⎜− ⎟ = − ×⎜− ⎟
5 ⎝ 7 ⎠ 5 ⎝ 7⎠ 3. (12,3) and (–12,7)
6
x=
7 4. (6,–2) and (6,2)

Take a look back at the line drawn between these two 5. (56,–43) and (0,0)
points. It’s hard to tell exactly, but it does look like
(0,65) and (76,0) are the y and x intercepts, respectively. 6. (–9,–15) and (–1,–7)

General Rules for Graphing an Equation

Here’s a summary of general rules for drawing a graph of a line, depending on what you’re given.

GIVEN PROCEDURE

Equation in slope-intercept form y = mx + b 1. Start with y intercept b. Point is (0,b).


2. Use slope to find another point.
3. Connect the two points.
y 2 – y1
Two points 1. Find slope using x 2 – x1 .
y−y
2. Using one of the points, use the equation slope = 1
and
x − x1
solve for y to obtain y = mx + b.
3. To graph equation, follow procedure above. However, if all
you’re asked to do is graph the equation, you only have to join
the two points.

Slope and one point Use steps 2 and 3 of the two points procedure if given two points.

111
8th_GRDM_06.qxd:Layout 1 8/11/09 1:43 PM Page 112

–GRAPHING LINEAR EQUATIONS AND INEQUALITIES –

Solving an Inequality Now, we can graph the inequality just as if it were


An inequality is much like an equation, except it an equation. We count up 2 and over 3 from (0,–1) to
has one of the following signs in place of an equal get our second point (3,1) and draw the line.
sign: > meaning greater than; < meaning less than;
≥ meaning greater than or equal to; or ≤ meaning y

less than or equal to. These are solved just like


equations, with one exception: If you multiply or
divide by a negative number, you have to reverse the
sign. For example:

x + 7 > 5x + 3
x
–4x + 7 > 3
–4x > –4
x<1

We can graph this solution on a one-dimensional


number line. If we have an inequality with two vari-
ables, we can find its solution on a coordinate graph.
Let’s start with the inequality 2x – 3y > 3. First, we’ll
put it in point-slope form:
When you graph a linear equation in two un-
2x – 3y > 3
knowns, the only solutions to the equations are on the
–3y > –2x + 3
line of the graph. But there are many pairs of x and y
y < 23x – 1
that satisfy the inequality, and all fall on just one side of
the equation line. Do they fall above or below it? The
Note that we had to reverse the direction of the in-
quickest way to figure this out is to try a point on one
equality in the last step.
side of the line and see if it satisfies the inequality. The
point (0,0) is usually the easiest one to try. If we plug
(0,0) into the original inequality, we get 2(0) – 3(0) >
3, which says that 0 > 3, which isn’t true, so (0,0) isn’t
part of our solution, and the solutions must lie on the
other side of the line. Just to check, let’s try a point on
that side: (0,–2). We get 2(0) – 3(–2) > 3, or 6 > 3,
which is true. Thus, the solution looks like this:

112
8th_GRDM_06.qxd:Layout 1 8/11/09 1:43 PM Page 113

–GRAPHING LINEAR EQUATIONS AND INEQUALITIES –

y Summary

In this lesson you learned how to graph a linear equa-


tion in two unknowns, and you learned how to do it
given different kinds of input. You also learned how
to solve and graph inequalities.

x
0
Answers

Exercise 1
1. 1; 30 –– 52 = –2
–2
or 25 –– 03 = 22
2. 0;13–––73 = 80
7−3
3. –4; 1 1 = −41 = −4
−2−2
4. Undefined; denominator of the slope formula
The inequality was > rather than ≥, so the solu- is 0.
−4 4 4
0− 3
tion does not include the equation line. An equation 5. –153;
5
= 3
5
= 3
5
= 43 × −56 = − 58
line that’s not included in an inequality solution is 0− 6 −6 −6

usually drawn as a dashed or dotted line. 6. 1; –7 – –15 8


–1 – –9 = 8
7. y intercept: (0,–3); x intercept: (1,0)
Exercise 3 The equation is already in slope-intercept
Put the following inequalities in slope-intercept form form, so letting x = 0, y = –3. Letting y = 0 and
and graph the solution region. solving for x, we get x = 1.
8. y intercept: (0,–45); x intercept: (22.5,0)
1. x ≥ y + 5 To get the equation in slope-intercept form,
subtract 45 from both sides to get y = 2x – 45.
2. 2x – y > 4 To find the x intercept, use the original equation
and let y = 0 and divide both sides by 2.
3. y + 2 – 23x ≥ 0 9. y intercept: (0,2); x intercept: (–12,0)
To get the equation in slope-intercept form,
add x + 12 to each side to get 6y = x + 12, then
divide both sides by 6 to get y = 16x + 2. To get
the x intercept, let y = 0 in the original
equation and add x to each side to get x = –12.

113
8th_GRDM_06.qxd:Layout 1 8/11/09 1:43 PM Page 114

–GRAPHING LINEAR EQUATIONS AND INEQUALITIES –

Exercise 2 2.
1. y = x – 3 y
y–0
Slope is 1, so 1 = x – 3 and x – 3 = y.
2. y = 3
y–3
Slope is 0, so 0 = x – 1 and y – 3 = 0.
3. y = –4x + 5
y−3
Slope is –4, so −4 = 1 and –4x + 2 = y –3,
x− 2
and y = –4x + 5.
4. x = 6 x
0
Slope is undefined, so you can’t use the general
equation, but it’s clear that x = 6 for all values
of y.
5. y = –85x
Slope is –85, and you already know the intercept
from the point (0,0).
6. y = x – 6
y – (–7)
Slope is 1, so 1 = x – (–1) and x + 1 = y + 7.
3.
1. y
y

x
x 0
0

114
8th_GRDM_06.qxd:Layout 1 8/11/09 1:43 PM Page 115

–GRAPHING LINEAR EQUATIONS AND INEQUALITIES –

4. 6.
y y

x x
0 0

5.
y Exercise 3
1. y ≤ x – 5
Subtract 5 from each side. Slope is 1 and the y
intercept is –5. Test (0,0) in the original
inequality and get 0 ≥ 5 (not true).

y
x
0

x
0

115
8th_GRDM_06.qxd:Layout 1 8/11/09 1:43 PM Page 116

–GRAPHING LINEAR EQUATIONS AND INEQUALITIES –

2. y < 2x – 4 3. y ≥ 23x – 2
First, subtract 2x from each side and then Subtract 2 – 23x from each side. Test (0,0) in the
divide by –1, reversing the sign. Test (0,0) in original inequality and get 2 > 0 (true).
the original inequality and get 0 > 4 (not true).

x
0
x
0

116
8th_GRDM_07.qxd:Layout 1 8/11/09 1:47 PM Page 117

13
L E S S O N

FINDING
SOLUTIONS

L E S S O N S U M M A RY
In previous lessons you’ve learned how to solve equations with
one unknown and how to graph linear equations and inequalities
with two unknowns. In this lesson you will learn how to find a
unique solution to two equations with two unknowns and how to
find the solution region for two inequalities. You’ll also learn how
to find solutions to some forms of quadratic equations.

I n this lesson, we’ll try to solve two equations in two unknowns. How much do you want to bet we succeed?
Most of these pairs of equations have a unique solution, but some have no solution, and others have an in-
finite number of solutions. Then, things will get more complicated when we look at pairs of inequalities in
two unknowns and find a region of solutions. Finally, you’ll learn how to solve certain types of quadratic equa-
tions using the factoring skills you learned in Lesson 11.

117
8th_GRDM_07.qxd:Layout 1 8/11/09 1:47 PM Page 118

–FINDING SOLUTIONS–

Two Equations, Two Unknowns Now think about the two equations x + 4y = 20
and 6y – 7 = 23 – 1.5x. How many solutions do they
There are many situations in real life, and many in have? If we put the first into slope-intercept form, we
math textbooks, that encourage you to solve two get y = – 0.25x + 5. Now put the other into slope-
equations with two unknowns. For example, there is intercept form. It’s exactly the same—the lines are
the extended response problem of Lesson 8, which identical. And as you might expect, any (x,y) that sat-
can be solved by guess-and-check or by setting up isfies one equation also satisfies the other. Let’s try a
two equations to find the two unknowns of time and couple just to be sure. The point (0,5) is obviously a
distance. You’ll see some others in Lessons 14 and 15. solution (see Lesson 12 if you don’t know why), so
In this lesson, though, you’re just going to practice let’s try it in both equations:
solving the equations.
x + 4y = 20
Parallel Lines on a Graph 0 + 4(5) = 20 ✓
You may remember that parallel lines are lines that 6y – 7 = 23 – 1.5x
never meet. (See Lesson 17 if you don’t know about 6(5) – 7 = 23 – 1.5(0) ✓
parallel lines.) It seems logical, and in fact it’s almost
true, that two lines that have the same slopes will Let’s try another point. We want to work with inte-
never meet. For instance, look at the graphs of y = 3x gers if we can, so we’ll start with x = 4. Then we see
+ 2 and y = 3x – 1. what y is, when x = 4 in the equation x + 4y = 20, and
y turns out to be 4. Thus, a solution to x + 4y = 20 is
y
(4,4), so we’ll see if it’s also a solution of
18
16 6y – 7 = 23 – 1.5x
14
12
y = 3x + 2 6(4) – 7 = 23 – 1.5(4)
10
y = 3x –1
24 – 7 = 23 – 6 ✓
8
6
4 In summary, two equations in two unknowns
2
x have
–16 –14 –12 –10–8 –6 –4 –2 2 4 6 8 10 12 14
–2
–4
–6
–8 ■ No solutions if their slopes are the same
–10
–12
■ An infinite number of solutions if their
–14 slopes are the same and their intercepts are
the same
■ Exactly one solution otherwise

Both lines have the same slope but different y inter-


cepts. For two equations to have a common solution,
though, they have to intersect, and that point of inter-
section is the solution. If two lines intersect at (3,–6),
then the solution is x = 3 and y = –6.

118
8th_GRDM_07.qxd:Layout 1 8/11/09 1:47 PM Page 119

–FINDING SOLUTIONS–

Exercise 1 Now we substitute that value of x into the first equa-


Tell whether the following pairs of equations have tion and solve for y:
one, none, or an infinite number of solutions. An-
swers are on page 123. 7x – 2y = 6
7(–5y) – 2y = 6
1. 3x + 2y = –6 and 9x = –18 – 6y –37y = 6
6
y = – 37
2. y = 0.75x – 12 and 3x – 4y = 24
Now we know what y is, so we can substitute y into
3. y = 18 – 2x and 2y = 36 – 2x our equation for x: x = –5y. Then

4. Explain which values of a and b give the ⎛ 6 ⎞ 30


x = −5 ⎜ − ⎟ =
following pair of equations no solutions, ⎝ 37 ⎠ 37
infinitely many solutions, and exactly one
solution. 30 6
and the solution is ( 37 , − 37 ).
Have we finished? No, we have to check. We can
y = a + bx and 4y – 3 = x + 4
substitute these values into either of the original
equations, so let’s choose 7x – 2y = 6.

Solving a System of Two Equations 30 ⎛ 6⎞


It’s possible to find the solution to two equations just 7× = 2×⎜− ⎟ = 6
37 ⎝ 37 ⎠
by graphing the lines and finding where they inter-
210 12
sect, but unless the coordinates of the intersection are + =6
37 37
fairly small integers, that isn’t a very reliable method. 222
So you have to learn to solve them with algebra to get =6 ✓
37
exact answers. We’ll learn two ways to do that: substi-
tution and elimination. Notice that it was convenient to leave the first value in
fractional form rather than converting it to a decimal
Substitution with a calculator.
Consider the system of equations

Unless converting to a decimal leaves you with


7x – 2y = 6 and x + 5y = 0
a number that is exact to hundredths (example:
3
To use the substitution method, we must find what 4= 0.75), it’s best to leave it as a fraction. If you
one of the variables is in terms of the other. Because x need a decimal answer at the end, you can
has a coefficient of 1 and will be easy to solve, let’s convert it then.
solve the second equation for x. We get:

x = –5y The substitution method is especially easy to


use if you can get a value for either x or y without do-
ing too much algebraic manipulation. In this system,
we got to x = –5y with one operation.

119
8th_GRDM_07.qxd:Layout 1 8/11/09 1:47 PM Page 120

–FINDING SOLUTIONS–

Elimination convenient. Also, you can multiply both sides of one


Look at the system of equations of the equations by one number and then multiply
one or both sides of the other by a different number to
3x – 2y = 15 and 4x + 2y = –9 help eliminate one of the variables. Here’s an example:

Notice that if you simply add these two equations to 2a + 10b = 6 and 7a + 4b = 14
one another, there will be no y in the sum:
Our procedure with this pair will be to multiply the
3x – 2y = 15 first equation by 7 and multiply the second by –2.
+4x + 2y = –9 Doing these operations will leave a with a coefficient
7x = 6 of 14 in the first equation and a coefficient of –14 in
the second.
The y’s have been eliminated. Now it’s easy to see that
x = 67. Now we can use that x value in either equation 14a + 70b = 42
to find what y is when x = 67. –14a – 8b = –28
62b = 14
3x − 2 y = 15
⎛ 6⎞ 7
And b = 14 ÷ 62, or 31. Now we return to the substitu-
3 ⎜ ⎟ − 2 y = 15
⎝ 7⎠ tion method to get
18
−2 y = 15 −
7 ⎛ 7⎞
2a + 10 ⎜ ⎟ = 6
105 18 ⎝ 31 ⎠
−2 y = −
7 7 ⎛ 7⎞
87 2a = 6 − 10 ⎜ ⎟
−2 y = ⎝ 31 ⎠
7 186 70
87 2a = −
y=− 31 31
14 116
2a =
31
So our solution point is tentatively (67,– 14
87
). We’ll check
58
using the first equation to get a=
31
⎛ 6 ⎞ ⎛ 87 ⎞ Thus (58 7
3 ⎜ ⎟ − 2 ⎜ − ⎟ = 15 31,31) is the tentative solution. Because we used
⎝ 7 ⎠ ⎝ 14 ⎠ the first equation to get the value for a, we should use
18 87 the second to check:
+ = 15
7 7
105 7a + 4b = 14
= 15
7 7 × 58 4 × 7
31 + 31 = 14
You can also use elimination even when one 406 + 28
31 = 14 ✓
variable doesn’t drop out as easily as in the previous
example. For one thing, you don’t have to add the two
equations; you can also subtract them if that’s more

120
8th_GRDM_07.qxd:Layout 1 8/11/09 1:47 PM Page 121

–FINDING SOLUTIONS–

Exercise 2 For the first equation, try (0,0) to see if it satisfies the
Solve the following pairs of equations by either substi- inequality: 0 ≥ 2(0) – 2 is true, so (0,0) is in the solu-
tution or elimination. Answers are on pages 123–124. tion region. We can’t use (0,0) for the second inequal-
ity because it’s on the line, so let’s use (1,0): 0 < – 32(1)
1. 6x + 5y = –20 and –6x – 10y = 25 is not true, so (1,0) is not in the solution region. Now
we know enough to shade in the solution region on
2. x + 3y = 1 and 2x – 2y = 4 the graph.

3. 6x – 4y = 21 and 7x – 8y = 27 y

Systems of Inequalities

Systems of inequalities are most easily solved by


graphing the lines and then finding which side of
x
each line has the region that satisfies each original in-
equality, as in Lesson 12. For example, let’s graph
these two inequalities, already in slope-intercept
form:

y ≥ 2x – 2
y < – 32x

The two lines are shown here.

The solution region is the one with the darker shad-


y
ing. As a check, let’s look at a couple of points in the
region with the dark shading. First, (–2,0) and then,
(0,–1):

y ≥ 2x – 2 y ≥ 2x – 2
0 ≥ 2(–2) – 2 (true) –1 ≥ 2(0) – 2 (true)
x y < – 32x y < – 32x
0 < – 32(–2) (true) –1 < – 32(0) (true)

121
8th_GRDM_07.qxd:Layout 1 8/11/09 1:47 PM Page 122

–FINDING SOLUTIONS–

Exercise 3 Here’s another way to solve the same equation.


Use substitution to check whether the following Remember from Lesson 11 that the difference of
points lie in the solution region for the following sys- two squares a2 – b2 can be factored as (a + b)(a – b).
tem of inequalities (answers are on page 124). If we start with x2 = 16, we can subtract 16 from each
side to get
2x – y ≤ 6
3x + 2y ≥ 12 x2 – 16 = 0
(x + 4)(x – 4) = 0
1. (0,0)
With 0 on the right side of the equation, it stands to
2. (5,0) reason that the equation would be satisfied if either (x
+ 4) or (x – 4) is equal to 0. So we just set up the
3. (3,3) equations:

4. (0,6) (x + 4) = 0 x = –4
(x – 4) = 0 x=4

Quadratic Equations We get the same two answers either way. In fact,
any time you can factor the expression on the left-
Quadratic equations come in the form ax2 + bx + c = hand side of ax2 + bx + c = 0, you can set each factor
0, where a, b, and c are constants and a ≠ 0. (If a = 0, equal to 0 to get the two solutions to the equation.
it’s a linear equation.) Quadratic equations normally Here’s an easy example: 4x2 + 12x = 0. Here there’s a
have two solutions, that is, two values of x that will common factor of 4x that can be factored out to give
make the equation true. However, they can have a
single root, as in the equation x2 = 0; it’s clear that no 4x(x + 3) = 0
number but 0 would work.
In the simplest quadratic equations, b and c are Now we set both factors equal to 0:
equal to 0, so you have an equation like this: x2 = 16.
Because both sides of an equation are equal, you can 4x = 0 (x + 3) = 0
take the square root of each side: x=0 x = –3

1 1
(x 2 ) 2 = 16 2 So the two solutions seem to be 0 and –3. Let’s check
1
to be sure:
2 × ( 12 )
x = 16 2
x = ±4 4x2 + 12x = 0
4(0)2 + 12(0) = 0 ✓
Remember that even-numbered roots can be either
4(–3)2 + 12(–3) = 0
positive or negative: In this case, 4 × 4 = 16 and (–4) ×
4(9) – 36 = 0 ✓
(–4) = 16.

122
8th_GRDM_07.qxd:Layout 1 8/11/09 1:47 PM Page 123

–FINDING SOLUTIONS–

Exercise 4 Answers
Solve the following quadratic equations. Keep in
mind the rules you learned in Lesson 11: a2 – b2 = Exercise 1
(a + b)(a – b) and (a + b)2 = a2 + 2ab + b2. Be sure to 1. Infinite number
check all answers in the original equation before Putting both equations into slope-intercept
looking at the answers on page 124. form, you get y = –32x – 3 for both equations.
2. None
1. x2 – 6 = –5 The equations have the same slope but
different intercepts. The two equations in
2. c2 + 4c + 4 = 0 slope-intercept form are y = 0.75x – 12 and
y = 0.75x – 6.
3. 7m2 = 21m 3. Exactly one
If you divide the second equation by 2, you’ll
4. 3x2 + 12x + 12 = 0 see that both equations have the same y
intercept, but they have different slopes, so they
5. t2 + 9t – 10 = 3(3t – 2) will intersect. In fact, they’ll intersect at (0,–18).
4. To answer this question, you have to put the
second equation in slope-intercept form:
Summary y = 0.25x + 1.75. If the equation pair has no
solutions, the slopes must be equal (that is,
In this chapter you learned some ways to find solu- b = 0.25) and the y intercepts must be different
tions to systems of equations with two unknowns, (that is, a =/ 1.75; all other a’s will work). To get
and you also learned how to identify the solution re- an infinite number of solutions, the lines must
gion for systems of two inequalities. In addition, you be the same, so you must have b = 0.25 and
learned how to solve some quadratic equations. a = 1.75. The two equations will have exactly
one solution if b =/ 0.25.

Exercise 2
1. (–2.5,–1)
This one is easier to solve by elimination.
Adding the two equations yields a sum of –5y
= 5, so y = –1. Substituting –1 for y in the first
equation gives 6x –5 = –20 and x = –2.5.
2. (1.75,–0.25)
Here it’s easy to find from the first equation that
x = 1 – 3y. Then, using the second equation,
2(1 – 3y) – 2y = 4
2 – 6y – 2y = 4
–8y = 2
y = –0.25
Then, you can find x in the equation x = 1 – 3y
= 1 – 3(–0.25) = 1.75.

123
8th_GRDM_07.qxd:Layout 1 8/11/09 1:47 PM Page 124

–FINDING SOLUTIONS–

3. (3,–0.75) Exercise 4
Elimination is easier here because multiplica- 1. x = ±1
tion of the first equation by –2 will yield Add 6 to each side to get x2 = 1 and take the
–12x + 8y = –42, and adding this to the second square root of each side. Remember, the square
equation will give a sum of –5x = –15, from root of 1 can be 1 or –1.
which x = 3. Substituting 3 into either of the 2. c = –2
original equations leads to y = –43 or –0.75. This is a quadratic with a single root, because it
factors to (c + 2)2 = 0. This equation will work
Exercise 3 only if c = –2.
1. No 3. m = 0,3
Using the first inequality, 2(0) – 0 ≤ 6, but 3(0) Moving the 21m to the left-hand side gives 7m2
+ 2(0) is not greater than or equal to 12, so the – 21m = 0, which factors to 7m(m – 3) = 0.
point is not in the solution region. Setting each factor equal to 0 gives the roots 0
2. No and 3.
Using the first inequality, 2(5) – 0 ≤ 6 is false, 4. x = –2
so the point is not in the solution region. For this equation, divide both sides of the
3. Yes equation by 3 to get x2 + 4x + 4 = 0, which will
Using the first inequality, 2(3) – 3(3) ≤ 6 is have the same root as problem 2.
true, and 3(3) + 2(3) is greater than 12, so the 5. t = ±2
point is in the solution region. First, expand the right-hand side: t2 + 9t – 10 =
4. Yes 9t – 6. Then, move all terms to the left-hand
Using the first inequality, 2(0) – 3(6) ≤ 6 is side to get t2 + 9t – 10 – 9t + 6 = 0. Then,
true, and the point (0,6) lies on the line for 3x combine all terms on the left to get t2 – 4 = 0 or
+ 2y ≥ 12. Because the line is not dashed, the (t + 2)(t – 2) = 0.
point is included in the solution region.

124
8th_GRDM_07.qxd:Layout 1 8/11/09 1:47 PM Page 125

14
L E S S O N

ESTIMATING
AND CHECKING

L E S S O N S U M M A RY
When you’re trying to figure something out in real-life situations,
you often have to have a good estimate, and you have to know
whether your estimate is higher or lower than the exact answer. In
some of those real-life situations (and on all math tests), it’s also im-
portant to know the exact answer, and you have to know how to
check that your answer is exactly right. In this lesson, you’ll learn
how to estimate and check problems involving algebra.

I n this lesson, you’ll learn something about how to estimate times based on whether one variable is going up
or down when another changes, or when there is another input. You will also get some more practice check-
ing your work by going back to the original problem to see if your answer is the right one. You’ll also learn
how to do your work in a way that makes it easy to see where you have made a mistake when (not if) you have
made one.

125
8th_GRDM_07.qxd:Layout 1 8/11/09 1:47 PM Page 126

–ESTIMATING AND CHECKING–

Rate Problems: DAY INPUT OUTPUT


Direct and Inverse Variation TRIP WAS (TIME, (RATE, MILES
MADE HOURS) PER HOUR)

Suppose you have an equation that gives you the dis- Monday 2.5 48
tance traveled, depending on the speed and time Tuesday 2.25 ?
spent traveling. In fact, you saw this in Lesson 9:
Friday 2.9 ?
Distance = Rate × Time Saturday 2.1 ?

Now suppose that we hold the rate constant at 50 Which day’s rate would be the highest? Which the
mph and consider the distance to be a function of the lowest? Now, use a calculator to fill in the rate to the
time. Here’s an input/output table for the function. nearest hundredth. You’d start with the Distance =
Rate × Time formula, and because you want to find
INPUT OUTPUT Rate, you’d divide both sides of the equation by the
(TIME, HOURS) (DISTANCE, MILES)
Time, remembering that Distance is always 120:
1 50
120
2 100 Rate =
Time
5 250 You could also use function notation here: Call the
10 500 rate r(t) and define it as

120
As time gets longer, so does distance. Two variables, r(t) = t
in this case distance and time, are said to be in direct
proportion if one gets longer as the other gets longer. Now that you’ve done the calculation, you know that
The relationship is called direct variation. In direct the lowest rate was Friday’s 41.38 mph and the high-
variation, the ratio of the output to the input is est rate was Saturday’s 57.14 mph. In other words,
constant: the lower the time, the higher the rate. The relation-
ship of these two variables is called inverse varia-
1 2 5 10 tion. Here the product of the input and output
= = =
50 100 250 500 remains constant:
Now, suppose that, instead of holding the rate
constant, we hold the distance constant. In other 2.5 × 48 = 2.25 × 53.33 = 2.9 × 41.38
words, you might want to see what your average = 2.1 × 57.14
speed was on a certain 120-mile trip that you made
on four different days. (Note that the ratio is not always exactly constant
here because we rounded to the nearest hundredth. If
we took into account all decimal places, it would be
exactly constant.)

126
8th_GRDM_07.qxd:Layout 1 8/11/09 1:47 PM Page 127

–ESTIMATING AND CHECKING–

Exercise 1 So which is the worse estimate: 2.5 hours or 1


(Answers are on page 129.) hour? Obviously, 1 hour is closer to the actual time,
but what if you are supposed to open the pool at a
1. Hoshi and his little sister Gheeta are playing on certain time and you start with the hoses 1 hour
a seesaw. Gheeta weighs 45 pounds. The seesaw ahead of that time? You might have some angry cus-
is in exact balance when Hoshi sits 2 feet 6 tomers to deal with, and you might lose money if
inches from the center and when Gheeta sits 4 they go to a different pool because they were tired of
feet from the center, which is at the very end of waiting. On the other hand, if you start 2.5 hours be-
the board. How much does Hoshi weigh? fore opening time, the hot sun might make the water
(Hint: It’s a law of physics that there’s an so warm in the 1.3 hours before the pool opens that
inverse variation between the weight of the people will never come back to your warm-water
person and the distance from the center that a pool. The best estimate, if you don’t know how to fig-
person should sit on the seesaw.) ure something out exactly, always depends on the sit-
uation. Nevertheless, by the time you’re getting paid
2. Gheeta’s friend Ebony is about the same weight to make decisions that involve money, you’ll (hope-
as Gheeta. Can Hoshi balance them both if fully!) know enough algebra to calculate easy prob-
they sit together? If so, where should he sit and lems like this accurately.
where should the girls sit?

Using Your Own Work


Example of Bad Estimates to Check Your Own Work
You learn that one hose will fill up a swimming pool
in 2 hours and another will fill up the pool in 3 hours. When you are solving an equation, it’s important to
You also learn that you can turn both hoses on at the write down every step. Your teacher will tell you this,
same time with no loss of water pressure. About how of course, so she can see that you actually did the
long would it take to fill up the swimming pool with problem rather than just got an answer from your
both hoses turned on? Some would add 2 and 3 and friend. But it’s also important for you to do it in case
say 5 hours. Of course, when you think about it, you your checking procedure shows you that your answer
know that would be ridiculous; the number of hours doesn’t work with the facts of a problem.
it takes for the hoses working together has to be less Here’s an example. We’re given a problem like
than what it would take them to do it separately. this: “Automobile companies calculate two rates for
Some might take the halfway point between 2 and 3, mileage: one for city driving and one for highway
or 2.5. Would that make sense? Of course not: The driving. Otto’s car gets 34 miles per gallon in the
time with both hoses on has to be less than 2 hours. country and 21 in the city. He drove 410 miles on 16
So how about 3 – 2 = 1 hour? Well, that’s definitely gallons of gas. How many gallons did he use in the
a better estimate, but it’s not exactly right, either. The country and in the city? Give your answers to the
correct answer, done with algebra that’s beyond nearest tenth.” To solve this problem, we’ll set up
what we’ll do in this book, is 1.2 hours, or 1 hour and
12 minutes.

127
8th_GRDM_07.qxd:Layout 1 8/11/09 1:47 PM Page 128

–ESTIMATING AND CHECKING–

two equations and perform the following series of Now we’ll check again: 5.69 gallons × 34 miles per
calculations. gallon = 193.5, and 10.31 × 21 = 216.5. Now our total
mileage is 410, and our answers appear to be right.
Let x be the gallons used on the highway and y
Plus, our answers are, to the nearest tenth, 5.69 gal-
be the gallons used in the city.
lons in the country and 10.31 gallons in the city.
x + y = 16 Notice that the problem asked for answers to
x = 16 – y the nearest tenth, but we used answers rounded to the
nearest hundredth in checking, so that we’d get a
34x + 21y = 410
more accurate check.
34(16 – y) + 21y = 410
544 – 36y + 21y = 410
544 – 15y = 410 When you’re asked to give rounded answers,
–15y = 410 – 544 always use numbers that have been rounded to
–15y = –134 one more place to the right (if possible) until
y = 8.93 (rounded to nearest you’re checking your answer.
hundredth)
x = 16 – 8.93
x = 7.07 In the preceding problem, all steps were written
down, including the statement of what the variables
Now let’s see how we did. Going back to the original meant. It’s very important to know what the variables
problem, we plug in the numbers. If Otto used 7.07 mean. What if you forgot which variable represented
gallons on the highway, then at 34 miles per gallon, he highway gallons and which represented city gallons?
would have traveled 7.07 × 34 ≈ 240.4 miles in the You might multiply 10.31 times 34 and 5.69 times 21
country. And if he used 8.93 gallons in the city, he and get an answer that wasn’t close to 410. Then
traveled 8.93 × 21 ≈ 187.5 miles. This adds up to al- you’d look for the error in your calculations, but you
most 428 miles, which is too far off to be right, even wouldn’t find it there. By the time you finally did find
though we rounded our answers a little. So now we it, you’d have wasted a lot of time—not a good situa-
have to go back and find the error. Take a little while tion, especially when you are taking a test.
to find the error if you haven’t done so already.
If you found the error here, give yourself a pat
on the back. Write down everything you do while solving a
problem, especially the definition of variables.
34(16 – y) + 12y = 410
544 – 36y + 21y = 410

The 36y in the second line should, of course, be 34y.


Now we can go back and change everything else:

544 – 13y = 410


–13y = 410 –544
–13y = –134
y = 10.31 (rounded to nearest
hundredth)
x = 16 – 10.31
x = 5.69

128
8th_GRDM_07.qxd:Layout 1 8/11/09 1:47 PM Page 129

–ESTIMATING AND CHECKING–

Exercise 2 Answers
Spot the error or errors in the solution to the follow-
ing problem, and then provide the right answer. Exercise 1
1. 72 pounds
EasyShoot Photogs charged $3 per small photo The product of Gheeta’s weight and her
and $11 per portrait-sized photo at a recent distance from the center is 4 × 45 = 180, and
school dance. EasyShoot sold a total of 34 the product of Hoshi’s weight (we’ll call it w)
photos and received $214 for their trouble. and distance should be the same. 180 = 2.5 × w
How many of each kind of photo did and w = 180
2.5 = 72 pounds.
EasyShoot sell? Set up a system of two 2. To balance both girls, Hoshi must sit at the end
equations. of the board so that he can balance the larger
weight. The product of his weight and distance
Let s be the number of small photos and p the is now 72 × 4 = 288. Because the two girls
number of portrait-sized. together weigh 90 pounds, they will have to sit
288
90 = 3.2 feet from the center.
s + p = 34
s = 34 – p Exercise 2
The correct answers are 20 small photos and 14
11s + 3p = 214 portrait-sized photos. There are two errors in the so-
11(34 – p) + 3p = 214 lution given. The first, and probably more easily
374 – 11p + 3p = 214 caught, is that 214 – 374 is –160, not –120. Note that
–11p + 3p = 214 – 374 the error still gave an integer answer for p. Finding
–8p = –120 the error would have been much easier if the wrong
p = 15 answer hadn’t come out to be an integer, because you
s = 34 – p can’t sell a fraction of a photo. The other is more seri-
s = 34 – 15 ous, because it messed up the whole problem. The
s = 19 third line should have been 3s + 11p = 214, because
the s stands for small photos and should be multi-
Check: $3 × 19 = $57; $11 × 15 = $165; plied by 3, and the p stands for the more expensive
57 + 165 = $222, not $214. Oops. portrait-sized photos and should be multiplied by 11.
Mistakes like that are hard to find, but they’re much
more easily found if you have at least written every-
Summary thing down. And this includes writing down units, too.

In this lesson you have learned how to estimate using


direct and indirect variation. You have also been ad-
vised to write every step of your calculations down.
Whether you have learned that lesson probably won’t
be apparent until you have to solve a problem quickly
and accurately about 20 years from now.

129
8th_GRDM_07.qxd:Layout 1 8/11/09 1:47 PM Page 130
8th_GRDM_07.qxd:Layout 1 8/11/09 1:47 PM Page 131

15
L E S S O N

STRATEGIES

L E S S O N S U M M A RY
In this lesson you’ll learn how to approach some types of problems
that you’ll see in eighth grade and probably later on in life. If you
have an idea about how to begin these types of problems, you’ll
usually solve them more quickly and accurately. This lesson will
teach some of the strategies you’ll need.

I n this section you have learned how to graph and solve linear equations and inequalities, and you have
looked at how to apply what you have learned to the rate problems that you saw earlier, in Lesson 7. The
strategies you’ll learn in this lesson will help you to answer some of the basic questions of life: where, when,
how many, and how much. Some people—maybe you—think that word problems are hard, and some of them
are. But most of them are pretty easy if you know how to set them up right.

131
8th_GRDM_07.qxd:Layout 1 8/11/09 1:47 PM Page 132

–STRATEGIES–

Determining the Variables So, if you want to end up with gallons, your equation
has to look like this:
Take another look at the sample problem from Les-
Miles
son 14. Miles ÷ Gallon
= Gallons

Automobile companies calculate two rates for This means that the second equation will be
mileage: one for city driving and one for
h
highway driving. Otto’s car gets 34 miles per 34 + 21c = 16 or 1
34h
1
+ 21c = 16
gallon in the country and 21 in the city. He
drove 410 miles on 16 gallons of gas. How This equation looks much harder to solve by substi-
many gallons did he use in the country and in tution than the equations you used to find the num-
the city? Give your answers to the nearest tenth. ber of gallons driven. So you might want to change
your variables to find gallons used and then just use
Now, let’s look at another question that might be the rate formula to find the number of miles driven.
asked, given the same facts: How many miles did he
drive in the country and in the city? You have two Exercise 1
choices: You could name your variables h and c for Solve that problem using first this system of equations:
highway gallons and city gallons, and then figure out
the distances from that, or your variables could be h + c = 410
named for the distances, so that you have the answer 1
34h
1
+ 21 c = 16
as soon as you find the values of the variables. The
second choice is the usual one, because you don’t As a check, find the mileage, using the solution using
have to think about your choice too much. So now the number of gallons as calculated in Lesson 14. An-
you have the variables h and c, and your variable swer is on page 134.
statement will look like this: “Let h be the miles
driven on the highway and c be the miles driven in
the city.” (It’s always best to put the units you’re using One Equation or Two?
directly into the definition of the variables.) It’s possible to put almost any two-equation system into
Your first equation will be easy to set up: h + c = a single equation, but you can have only one variable
410. But your second equation will be a little more then. If you define your unknowns accurately in terms
complicated. You’d like some algebraic expression to of just one variable, though, that single equation might
equal 16, the number of gallons. Using an equation be easier to use. For example, in the system used in the
like 34h + 21c = 16 doesn’t make sense. Here the rate previous exercise, we could have solved for h = 410 – c,
is miles per gallon rather than miles per hour, so we and then solved for h in the second equation as
have to use a slightly altered rate formula to set up
our equation: ⎛ 1 ⎞
h = 34 ⎜ 16 − c ⎟
⎝ 21 ⎠
Distance = Rate × Time We could then have made those two expressions for h
equal to each other:
Put another way, it’s
⎛ 1 ⎞
Miles 410 − c = 34 ⎜ 16 − c ⎟
Miles = Gallon
× Gallons ⎝ 21 ⎠

132
8th_GRDM_07.qxd:Layout 1 8/11/09 1:47 PM Page 133

–STRATEGIES–

You could solve this equation for c. If you did solve it If we let d be the number of miles A travels be-
this way, your variable statement would have to read fore collision, then the time it will take A to get to the
d
“Let c = miles driven in the city and (410 – c) = miles spot of the collision will be 80 (from the rate formula).
driven on the highway.” The time B takes to get to the spot will be 200 –d
120 . Since
they left at the same time, we could set up the follow-
Exercise 2 ing equation:
Solve the following problem from Lesson 14 using
d 200 − d
just one equation (answer is on page 134): =
80 120

EasyShoot Photogs charged $3 per small photo We’ll solve this equation for d (if you don’t remember
and $11 per portrait-sized photo at a recent how, go back to Lesson 9) and get d = 80 miles, which
school dance. EasyShoot sold a total of 34 is the number of miles train A travels, meaning the
photos and received $214 for their trouble. collision happens 80 miles from station A. This an-
How many of each kind of photo did swers one of the questions (where?), but we still have
EasyShoot sell? to figure out when. Remember that the rate formula
tells us that time = distance 80
rate , so it took train A 80, or 1
hour to get to the collision site. Note that B also takes
120
Where and When? 120, or 1 hour to get there.

A classic algebra problem asks when and where two Exercise 3


moving vehicles will meet. Sometimes the vehicles are The Limited leaves Omaha at 4:00 A.M., traveling at
going in the same direction and sometimes in the op- an average of 75 miles per hour on a nonstop run to
posite direction. You’re usually given the time that Des Moines. The Corn King leaves Des Moines at
each vehicle begins (it may be the same time or differ- 4:30 A.M., traveling at an average of 100 miles per
ent times) and the rate at which each is traveling. The hour on a nonstop run to Des Moines. The stations
solution usually involves the amount of time before are 150 miles apart, and the trains are traveling on
they meet and the distance each covers. parallel tracks. What time will one train pass the
The usual way to solve problems like this is with other? How far from Omaha will they be when the
a single equation, because both vehicles have to reach passing occurs? Give answers to the nearest minute
the same spot. If the vehicles are going in opposite di- and mile. Answer is on page 135.
rections, their times will be equal (with some adjust-
ment). If train A leaves station A at 5:00 A.M. moving
at 80 mph and train B leaves station B at 5:00 A.M. How Many and How Much?
moving at 120 mph—both on the same track—when
will they collide? The stations are 200 miles apart: Problems that ask you to tell how many or how much
How far from station A will they be when they meet? are usually best solved by a system of two equations,
especially if they don’t involve rates. Here’s a fairly
? easy one, the kind you may have had to solve at an
d 200 – d
earlier age, just through guesswork, before you knew
any algebra.

A B
80 mph 200 miles 120 mph

133
8th_GRDM_07.qxd:Layout 1 8/11/09 1:47 PM Page 134

–STRATEGIES–

Seth is 10 years older than Natasha, and next Answers


year he will be twice as old as she is. How old
are Seth and Natasha? Exercise 1
Let h be the miles driven on the highway and c be the
Using algebra, we’ll let Seth’s age in years be s and miles driven in the city.
Natasha’s age be n. Then we’ll get this system of Use elimination to solve the system:
equations: Multiply the second equation by –21:

s – n = 10 (Seth 10 years older) –21


34 h
– c = –336
s + 1 = 2(n + 1) (Seth’s age will be twice + h + c = 410
Natasha’s in 1 year) 13
34h = 74
s = 10 + n 34 × 34h = 74 × 34
13

10 + n + 1 = 2n + 2 (substituting 10 + n for s) 13h = 2,516


11 – 2 = 2n – n h = 193.5
n = 9, s = 19 193.5 + c = 410
c = 410 – 193.5 = 216.5
Exercise 4
(Answers are on page 135.) Using the answers for numbers of gallons used from
Lesson 14 (5.7 and 10.3 for highway and city, respec-
1. A movie theater takes in $1,305 in admissions tively), you get roughly the same answers: 5.7 × 34 =
in one night. Adult tickets cost $9.00, and 193.8 and 10.3 × 21 = 216.3. The first value is slightly
children under 12 get in for half price. A total different due to rounding.
of 185 tickets were sold. How many adult
tickets and how many child tickets did the Exercise 2
theater sell that night? Use two equations in Let s = number of small photos and (34 – s) = num-
two unknowns. ber of portrait-sized photos sold.

2. Jorge has $9.80 in his pocket. If he had one 3s + 11(34 – s) = 214


more dime, he’d have 3 times as many dimes as 3s + 374 – 11s = 214
he has quarters. How many of each coin does –8s + 374 = 214
he have? –8s = –160
s = 20, (34 – s) = 14

Summary

In this lesson you have learned some strategies for


solving different classes of word problems. One of
these is to equate two ways of expressing the same
thing, such as time to reach a certain point. You also
learned different ways to set up and solve systems of
two equations with two unknowns.

134
8th_GRDM_07.qxd:Layout 1 8/11/09 1:47 PM Page 135

–STRATEGIES–

Exercise 3 Exercise 4
We start by observing that the time it takes the Omaha 1. Let a be the number of adult tickets and c be
train to get to the point that they pass will be distancerate , the number of child tickets sold.
d
or 75 , because we don’t yet know how far the train will a + c = 185
go. The Corn King will arrive at the passing point at 9a + 4.5c = 1,305
the same time, but it starts a half-hour, or 0.5 hours, a = 185 – c
later. How far will it have to travel? (150 – d), so the 9(185 – c) + 4.5c = 1,305
time it will take to get there is 150100– d. The time it takes 1,665 – 9c + 4.5c = 1,305
the Corn King to get there will be half an hour less, so –4.5c = 1,305 – 1,665
to make the two times equal, we have to add that half- –4.5c = –360
hour. The equation to solve is c = 80
a = 185 – 80
d 150 − d
= + 0.5 a = 105
75 100
2. Let q be the number of quarters and d be the
The best way to start solving this problem is by get- number of dimes.
ting everything on the right over a common denomi- 3q = d + 1 (because he needs
nator, so multiply 0.5 by 100 and combine with 150: 1 more dime to
have 3 times as
d 150 − d 50
= + many dimes as
75 100 100
quarters)
d 150 − d
= .25q + .10d = 9.80
75 100
d = 3q – 1
Then cross multiply and simplify: .25q + .10(3q – 1) = 9.80 (substituting 3q – 1
for d)
75(200 – d) = 100d .25q + .30q – .10 = 9.80
15,000 – 75d = 100d .55q = 9.80 + .10
15,000 = 175d .55q = 9.90
d = 85.7 miles (rounded to nearest q = 18
tenth) d = 3(18) – 1 = 53
53 dimes and 18 quarters
The time will be 85.775 = 1.14 hours from 4:00 A.M. To Note that if you end up with a non-integer, you
convert to minutes, multiply 1.14 times 60 to get 68 must have made a mistake, because it’s
minutes (rounding to the nearest minute). The time impossible to have a fraction of a coin.
they meet will be about 5:08 A.M. (4:00 + 1 hour and 8
minutes), and they will be about 86 miles from Omaha.
As a check, we have to make sure that the Corn
King will be (150 – 86 = 64) miles from Des Moines at
5:08. It will have been on the track for 38 minutes at
38
5:08, which is 60 = 0.633 hours. At 100 mph, it will have
traveled about 63.3 miles at 5:08, which is close enough,
because the difference is the result of rounding.

135
8th_GRDM_07.qxd:Layout 1 8/11/09 1:47 PM Page 136
8th_GRDM_07.qxd:Layout 1 8/11/09 4:52 PM Page 137

16 PUTTING
L E S S O N

IT ALL
TOGETHER

L E S S O N S U M M A RY
This lesson pulls together what you have learned about equations
and graphs in Lessons 9–15. You’ll use that knowledge to solve
problems and to write short and long answers that tell how you
solved those problems.

Y ou have now learned some of the algebra basics you’ll need to be successful in eighth grade. You have
also learned strategies for solving word problems based on real-life situations. Now it’s time to see
how well you know them.

What You Have Learned

Here’s a quick summary of the lessons in this section.

Lesson 9: Patterns, Variables, and Equations. This lesson reviewed finding rules for sequences and intro-
duced the use of variables and unknowns in equations. It also showed basic rules on how to solve equations.
Lesson 10: Introduction to Functions. You learned what functions are and why they’re useful. You
learned how to evaluate a function and how to use functions to make some kinds of decisions.

137
8th_GRDM_07.qxd:Layout 1 8/11/09 1:47 PM Page 138

– PUTTING IT ALL TOGETHER –

Lesson 11: Operations with Polynomials. You Practice


learned operations that can be done with combina-
tions of variables and numbers. You learned how to Short Answers
simplify algebraic expressions, which is often neces- (Answers are on pages 141–143.)
sary for solving equations.
Lesson 12: Graphing Linear Equations. You Complete the operations as indicated. (Calculators
learned how to graph linear equations with two un- permitted, especially for checking, but they won’t
knowns, and you learned how to find the equation of help you with most of these.)
a line, given two points in the coordinate graph sys-
tem. You also learned how to graph inequalities. 1. (a) Which of the following lines has a slope of
Lesson 13: Solutions. Here you learned how to –2? (b) Which has a slope of 3? (c) Which has a
find solutions to systems of two linear equations and slope of 0.5? (d) Which has a slope of 0?
inequalities. You also learned some basics for solving
quadratic equations. C
y A
Lesson 14: Estimating and Checking. You
learned some methods for estimating solutions for
D
and checking your answers to word problems involv-
ing algebraic operations.
Lesson 15: Strategies. You studied a general B B
procedure for solving types of word problems, such
x
as problems with rates that ask you to find out if,
where, and when two moving objects will meet. You
also got some tips on how to select variables when
solving word problems and situations where one D
equation might work better than two to find the same
answers. C

2. Find x in the equation x3 – 1 = –28.

3. Simplify the expression 8 z 2 − 4(z + 1) + (z − 3 )2


so that there are no parentheses and just one
term each for z and z2.

4. Evaluate the function f(x) = 3(2)x at x = –2,


x = 0, x = 2, and x = 4.

5. If f(x) = 2x3 – 4x2 – x + 3, find f(–2), f(0), f(2),


and f(4).

138
8th_GRDM_07.qxd:Layout 1 8/11/09 1:47 PM Page 139

–PUTTING IT ALL TOGETHER –

6. Find the equation of the line that contains the 10. Solve the following system of equations for x
points (0,3) and (–4,–4). Put the equation in and y, and graph both equations.
slope-intercept form.
y + 4 – 32x = 0
7. Find the equation of the line that contains the
2x + 7 = –2(2x – 3y) + 25
point (3,–2) and has a slope of 1.5. Put the
equation in slope-intercept form. y

8. Graphically indicate the region that solves the


following system of inequalities:

y ≥ –31x + 2

y≤x–1 x

11. A car-leasing business charges $79 per month


plus 2 cents per mile up to 90 miles in a month
and 8 cents per mile for any miles over 90.
Write a function f(m) that tells the customer
how much that would be in one month using
mileage as the independent variable. How
much would a customer pay for driving 125
miles in a month? How much for driving 60
9. Solve x – 3(4x – 2) ≤ x – 4(1 – x) for x. miles?

12. Solve for x:

4 x − 5 −25
=
10 2

139
8th_GRDM_07.qxd:Layout 1 8/11/09 1:47 PM Page 140

– PUTTING IT ALL TOGETHER –

Short Response Extended Response


Write a response that explains how you arrived at Write a response that fully explains how you used al-
your answer. (No calculator except to check.) gebra to arrive at your answer. Show all diagrams or
tables that you made as well as all calculations. (Note:
13. You have been given the function h(x) = 5 |x| –3 This is the same problem that was in Lesson 8, with
with a domain of –5 ≤ x ≤ 3. Make an input- numbers slightly changed. Here, however, you must
output table with at least five values and sketch set up an equation to solve the problem.)
a graph of the function.
_____________________________________ 14. A delivery truck leaves a printing company at
5:30 A.M. to deliver magazines to a customer
_____________________________________ whose warehouse is 220 miles away. The truck’s
_____________________________________ average speed is 50 miles per hour. At 5:45,
_____________________________________ someone at the printing company spots an
error on the cover. It is also discovered that the
_____________________________________ truck driver’s cell phone battery is dead. By
6:15 the error has been corrected and the
y
covers are reprinted. The new covers are put
into a car at 6:20. The car averages 60 miles per
hour over the same route as the truck. Will the
car catch the truck before it reaches its
destination? If so, what time will it be?

x _____________________________________
_____________________________________
_____________________________________
_____________________________________
_____________________________________
_____________________________________
_____________________________________
_____________________________________
_____________________________________
_____________________________________
_____________________________________
_____________________________________
_____________________________________

140
8th_GRDM_07.qxd:Layout 1 8/11/09 1:47 PM Page 141

–PUTTING IT ALL TOGETHER –

–4 – 3 7
Answers 6. The slope of this line is –4 – 0 = 4. At this point
you observe that the y intercept is 3 because the
1. (a) Slope of –2: line C, which is the only line line passes through (0,3). Thus the equation is
with a negative slope on the graph (that is, the y = 47x + 3. If you used the equation slope = y − y1 ,
x − x1
y values are falling as the x values increase). you’d get
y−3
(b) Slope of 3: line A, which is the line with the
7
4 = x −0 ⇒ 7x = 4y – 12 ⇒ y = 47x + 3
steeper positive slope (y values go up 3 for As a check, verify that (–4,–4) is a solution:
every 1 the x values increase). –4 = 47(–4) + 3 = –7 + 3 ✓
y − y1
(c) Slope of 0.5: line D, the less steep line with 7. Use the equation, slope = x − x1 :
y – (–2)
a positive slope.
3
2 = x–3 ⇒ 2y + 4 = 3x – 9 ⇒
3 13
(d) Slope of 0: B, the difference between any y = 2x – 2
two y values is 0. As a check, find the slope between the y
2. x = –3 intercept (0,–6.5) and (3,–2):
−2 − (−6.5) 4.5 = 1.5 ✓
x3 – 1 = –28 3− 0
=
3
x3 – 1 + 1 = –28 + 1 8.
x3 = –27
1 1 y
(x 3 ) = (−27)
3 3
y<
–x–1

x = –3
2
3. 9z – 2z(2 + 3 ) – 1
8z2 – 4(z + 1) + (z – 3 )2
8z2 – 4z – 4 + z2 – 2z 3 + 3 (expand terms) y>
1
–× + 2
– –3
(8z2 + z2) – (4z + 2z 3 ) + (3 – 4) (group terms)
9z2 – 2z(2 + 3 ) – 1 x
As a check, evaluate the unsimplified and the
simplified expression at z = 2.
8(2)2 – 4(2 + 1) + (2 – 3 )2 = 32 – 12 + 4 –
4 3 + 3 = 27 – 4 3
9(2)2 – 2(2)(2 + 3 ) – 1 = 36 – 8 – 4 3 – 1
= 27 – 4 3
Note that the 3 doesn’t have to be computed,
although it can be if you need to do a complete
computation.
4. At x = –2, f(–2) = 3(2)–2 = 43; at x = 0, f(0) = First, draw lines with each equation and test a
3(2)0 = 3 × 1 = 3; at x = 2, f(2) = 3(2)2 = 3 × 4 point in each so that you know which side of
= 12; and at x = 4, f(4) = 3(2)4 = 3 × 16 = 48. the line to shade. For y ≥ –(31)x + 2, you can use
5. f(–2) = 2(–2)3 – 4(–2)2 – (–2) + 3 (0,0), which doesn’t work, so shade all points
= 2(–8) – 4(4) + 2 + 3 on the other side from (0,0). For y ≥( x – 1, you
= –16 – 16 + 5 = –27 could use (0,0) again, but let’s test (–2,0) just
f(0) = 2(0)3 – 4(0)2 – (0) + 3 = 3 for a change. Then, 0 ≤ –2 – 1 is not true, so
f(2) = 2(2)3 – 4(2)2 – (2) + 3 = 16 – 16 shade the other side. The area where the
–2+3=1 shading overlaps is the solution region.
f(4) = 2(4)3 – 4(4)2 – (4) + 3
= 2(64) – 4(16) – 4 + 3 = 63
141
8th_GRDM_07.qxd:Layout 1 8/11/09 1:47 PM Page 142

– PUTTING IT ALL TOGETHER –

9. x ≥ 85 Because the lines intersect at the calculated


x – 3(4x – 2) ≤ x – 4(1 – x) solution, no further check is necessary.
x – 12x + 6 ≤ x – 4 + 4x 11. f(x) = 79 + .02x for 0 ≤ x ≤ 90
–11x + 6 ≤ 5x – 4 = 80.80 + .1(x – 90) for x > 90
–16x ≤ –10 (Note: The $80.80 is the amount paid for the
x ≥ 85 (signs reverse because of division by –16) first 90 miles: .02 × 90.)
Check for x = 1 in original: 1 – 3(4 – 2) ≤ 1 – For x = 125 miles, the fee is $80.80 + .1(125 –
4(1 – 1) ⇒ –5 ≤ 1 ✓ 90) = $80.80 + $3.50 = $84.30.
Check for x = (–1) in original (this should not f(60) = 79 + .02(60) = $80.20.
work): 12. x = –30
–1 – 3[4(–1) – 2] ≤ –1 – 4[1 – (–1)] ⇒ 4x – 5
10 = 25
–25

17 ≤ –9 not true, so ✓ 8x – 10 = –250


10. You could use either substitution or elimina- 8x = –240
tion, but here’s the solution using substitution. x = –30
4(−30) − 5
10 ÷ 5 = 2 ✓
⇒ –25 5 –25
First, note that y = 32x – 4, so you can substitute As a check, 10
2 13. Responses will vary, but using values for x
3x – 4 for y in the second equation:
2x + 7 = –2[2x – 3(32x – 4)] + 25 lower than –5 and larger than 3 will lose you
2x + 7 = –2(2x – 2x + 12) + 25 half-credit. Here’s a sample answer:
2x = –2(12) + 25 – 7
I know that the domain is between –5
2x = –6
and 3 (including –5 and 3), so I can make
x = –3
a table using five integers, because that’s
y = 32(–3) – 4 = –6
easier. I know that x can also be a decimal
To graph the equations, first put them in slope-
or a fraction.
intercept form:
y = 32x – 4
IN OUT
y=x–3
–5 22
y
–2 7

0 –3

2 7

3 12

(–3,–6)

142
8th_GRDM_07.qxd:Layout 1 8/11/09 1:47 PM Page 143

–PUTTING IT ALL TOGETHER –

The graph looks something like this. Its 14. Responses will vary; here is a sample. Note
low point is at (0,–3) and its high point is that this sample uses algebra to get the right
at (–5,22). I used a scale of 2 to 1 on the y answer. In eighth grade, you’ll be expected to
axis to fit the value of 22 onto it. It looks use algebraic solutions rather than guess-and-
like a backward check mark. check.

I know the speeds of the truck and the


y
20 car, and I know the truck leaves at 5:30
and the car leaves at 6:20. The other times
don’t matter. I know that at the end of
10 each hour the truck will be 50 miles
6
farther along, so at the end of each half-
4 hour it will be 25 miles farther. I wanted
2
x to set up an equation with distances for
–5 –2 0 1 2 3
–2 the car and the truck being equal, so I
–4
started with the distance/rate formula:
Distance = Rate × Time
The truck’s distance will be 50t, where t is
the time in hours that the truck traveled.
The car’s distance will be 60t, and it left 50
60
of an hour later. So in the equation, I have
60 × 60, or 50 miles, from the
to subtract 50
car’s side of the distance equation. Here’s
the equation:
50t = 60t – 50
–10t = –50
t = 5 hours
So they would meet 5 hours from 5:30, or
at 10:30. But at 10:30 the truck would
have traveled 5 × 50 miles, or 250 miles. It
would have reached its destination before
the car caught it.

143
8th_GRDM_07.qxd:Layout 1 8/11/09 1:47 PM Page 144

– PUTTING IT ALL TOGETHER –

IF YOU MISSED THEN STUDY

Question 1 Lesson 12

Question 2 Lessons 4, 9

Question 3 Lesson 11

Question 4 Lesson 10

Question 5 Lesson 10

Question 6 Lesson 12

Question 7 Lesson 12

Question 8 Lesson 12

Question 9 Lessons 11, 12

Question 10 Lessons 11, 13

Question 11 Lesson 10

Question 12 Lessons 9, 11

Question 13 Lesson 10

Question 14 Lessons 14, 15

144
8th_GRDM_08.qxd:Layout 1 8/11/09 3:22 PM Page 145

3
S E C T I O N

GEOMETRY

G eometry is the study of angles, shapes, perimeters, areas, and volumes. Geometry is perhaps the
most obvious way that the study of mathematics relates to the physical world, and its foundations
were discovered more than 2,000 years ago. Everything you have learned and will learn about
geometry fits together in a pattern that describes the world around us—and allows us to improve it from time
to time.
By the end of this section, you should have these basic skills:

■ Recognize different kinds of angles and find missing values in a set of angles.
■ Know the difference between regular and irregular polygons.
■ Find the areas and perimeters of various kinds of polygons and circles.
■ Know how to compute volumes of solids.
■ Know how scale factors affect length, area, and volume.
■ Know how to find areas of unusual shapes.

145
8th_GRDM_08.qxd:Layout 1 8/11/09 3:22 PM Page 146

–GEOMETRY–

WORDS YOU SHOULD KNOW

acute angle diagonal polygon

adjacent angles equilateral triangle Pythagorean theorem

angle intersect ray

arc irregular polygon regular polygon

base isosceles triangle right angle

bisect midpoint right triangle

chord obtuse angle scale factor

circumference parallel similar

complementary perpendicular supplementary

congruent point triangle

146
8th_GRDM_08.qxd:Layout 1 8/11/09 3:22 PM Page 147

17
L E S S O N

ANGLES AND
TRIANGLES

L E S S O N S U M M A RY
In this lesson, you’ll learn what angles are and how to refer to an-
gles and their parts. You’ll learn how they’re measured as well.
You’ll also learn about lines and triangles, and about interior an-
gles of triangles and other polygons.

A
three angles.
ngles are the building blocks of geometry. Without a way to describe angles, we wouldn’t have a
good way to describe the relationship between lines. In this lesson you’ll learn all about those won-
derful things called angles, and you’ll also learn about the triangle—the enclosed space between

147
8th_GRDM_08.qxd:Layout 1 8/11/09 3:22 PM Page 148

–ANGLES AND TRIANGLES–

Angles

What Is an Angle?
An angle is a set of points formed by two rays (called
sides) that share the same endpoint (called a vertex).
Instead, we use a unit called a degree, and we count
In this picture the vertex is point B and the sides are
how many of these units fit into the interior of the
ray BA and ray BC. Always list the vertex first when
angle. We read “m∠U” as “the measure of angle U.”
naming rays. Point I is said to be in the interior of the
angle, and point E is in the exterior.
E One degree:
A I

3
B C

The angle is called ∠ABC or ∠CBA. Note that U


the vertex is listed (and spoken) in the middle posi-
m∠U = 1°
tion of an angle. It would not be correct to call the
The measure of an angle is a number greater
angle ∠ACB because C is not the vertex. We could
than 0º and less than or equal to 180º.
also call it ∠3 or use the vertex alone and call it ∠B.
When several angles share a vertex, use three
letters to name the angle or use the little number in In the rest of this lesson we will assume that an-
the interior near the vertex. gle measures are in degrees, but we will usually omit
the degree symbol.
O
Congruent Angles
I
Congruent angles are angles that have the same
2 J
1
measure. So ∠1 ≅ ∠2 means m∠1 = m∠2. (Read the
N symbol “≅” as “is congruent to.”) The following dia-
gram shows two ways we indicate that angles are con-
It would cause confusion to name any angle in this
gruent. In both pictures, ∠1 ≅ ∠2 and ∠3 ≅ ∠4, but
figure ∠N.
because ∠1 and ∠3 are marked differently (e.g., by a
different amount of hatch marks), we do not know if
∠1 is another name for ∠JNO.
∠1 is congruent to ∠3.
∠2 is another name for ∠ONI.
There is no number name for ∠JNI.

Measuring Angles 1 1
2 2
When we measure angles, we are describing the size
of the interior of the angle. We cannot use linear 3 3
4 4
measure because the linear distance between the rays
is different at every point.

148
8th_GRDM_08.qxd:Layout 1 8/11/09 3:22 PM Page 149

–ANGLES AND TRIANGLES–

Names for Special Pairs of Angles In both pictures, ∠1 and ∠2 are adjacent angles. A is
Adjacent angles have a common side and a common the common vertex, ray AB is the common side, and
vertex, and no point is in the interior of both angles. no point is in the interior of both angles.
(In mathematics, common means shared, so a com- The following pairs of angles are not adjacent
mon side is a shared side.) (nonadjacent) angles:
Here are some examples of adjacent angles:
B

1
2 1 2
A A

∠1 and ∠2 are nonadjacent angles. They have


different vertices, they share only a part of a common P

side, and point P is in the interior of both angles.

1 2

∠1 and ∠CAD are nonadjacent because point P is C


located in the interior of both angles.
P

1
D
A

A
∠1 and ∠2 are nonadjacent because they do not
share a common side. The sides of ∠1 are rays AB
1 2
and AC, while the sides of ∠2 are AD and AE.

B C D E

Adjacent angles have a special (and fairly obvious) property related to their sums.

C We say m∠1 + m∠2 = m∠CAD.


B

1
2
D
A

149
8th_GRDM_08.qxd:Layout 1 8/11/09 3:22 PM Page 150

–ANGLES AND TRIANGLES–

Exercise 1
Vertical angles are congruent.
(Answers are on page 156.)

In the following diagram, if m∠1 = 10 and


Exercise 2
m∠2 = 40, then m∠CAD = _____.

Find x, y, and z.
B

1 2

C A D

Because ∠CAD is a straight angle, m∠1 + m∠2 70°

= 180. When the measures of two adjacent angles add x


to 180, we say they form a linear pair—because they z
form a line. y

Vertical angles are pairs of nonadjacent angles


formed when two lines intersect.

Complementary angles are two angles whose meas-


1 ures add to 90. Supplementary angles are two angles
2 4
whose measures add to 180. Do you see that the two
angles of a linear pair are supplements, because they
3
always add to 180?

TO DESCRIBE COMPLEMENTS AND


SUPPLEMENTS USING ALGEBRA
ITS ITS
ANGLE COMPLEMENT SUPPLEMENT
∠1 and ∠3 are a pair of vertical angles, and ∠2 and ∠4
are also vertical angles. x 90 – x 180 – x
It is possible to prove that vertical angles are
always equal (and congruent).
Because ∠1 and ∠2 form a linear pair, Exercise 3
m∠1 + m∠2 = 180. If m∠A = 65, how large is its complement? How large
Also, ∠2 and ∠3 form a linear pair: is its supplement?
m∠2 + m∠3 = 180, so m∠1 + m∠2 = m∠2 + m∠3.
By subtracting m∠2 from both sides, Exercise 4
m∠1 = m∠3. So ∠1 ≅ ∠3. An angle is twice as large as its supplement. How
By a similar argument, ∠2 ≅ ∠4. large is the angle?

150
8th_GRDM_08.qxd:Layout 1 8/11/09 3:22 PM Page 151

–ANGLES AND TRIANGLES–

Angles Formed by Angle Bisectors


An angle bisector is a ray located in the interior of an
angle that divides an angle into two equal angles. m

1
2 C
B
n
BD bisects ∠ABC

If ray BD is the angle bisector of ∠ABC, then all of p q


the following statements are true:
In a picture of two lines, the region between the
Ray BD bisects ∠ABC. lines is called the interior while the region outside the
∠1 ≅ ∠2 and m∠1 = m∠2. lines is called the exterior.
m∠1 = 12 m∠ABC and m∠2 = 12 m∠ABC.
2m∠1 = m∠ABC and 2m∠2 = m∠ABC.

Exercise 5 Exterior
In the following figure, if ray RE bisects ∠TRU, and
m∠1 = 80, then find m∠TRU and m∠2.

Interior
T

1
2
R
Exterior

Angles Formed by Transversals


If the two lines are intersected by a third line, t,
of Parallel Lines
t is called the transversal. The following terms are
If two lines cross, we call them intersecting lines. If
used to describe angles:
lines never intersect, they are called parallel lines.
We use lowercase letters for lines, and we mark par-
∠1, ∠2, ∠7, and ∠8 are said to be exterior
allel lines with arrows. The arrows indicate m || n
angles.
and p || q. “m || n” is read “m is parallel to n.” Lines n
∠3, ∠4, ∠5, and ∠6 are interior angles.
and p intersect.

151
8th_GRDM_08.qxd:Layout 1 8/11/09 3:22 PM Page 152

–ANGLES AND TRIANGLES–

Angles located on the same side of the transver- The other pairs of corresponding angles are ∠2
sal are called consecutive (or same-side) and ∠6 (upper right), ∠3 and ∠7 (lower right), and
angles. ∠4 and ∠8 (lower left).
Angles located on opposite sides of the transver-
sal are called alternate angles. Here is an important rule about alternate inte-
rior angles.
Certain pairs of these angles (one with vertex A
and one with vertex B) have names based on their po-
sition in the picture. If a pair of parallel lines is crossed by a transver-
sal, then each pair of alternate interior angles is
t congruent.

1 2
4 3

5 6
8 7 1 2
4 3

5 6
8 7
∠4 and ∠6 and also ∠3 and ∠5 are called pairs
of alternate interior angles.
∠1 and ∠7 and also ∠2 and ∠8 are pairs of al-
According to the rule and the above figure, ∠4 ≅
ternate exterior angles.
∠6. But vertical angles also are congruent, so ∠2 ≅
∠4 and ∠5 and also ∠3 and ∠6 are pairs of con-
∠4 and ∠6 ≅ ∠8. This means that ∠2 ≅ ∠4 ≅ ∠6 ≅ ∠8.
secutive interior angles (or same-side inte-
Using similar reasoning, ∠1 ≅ ∠3 ≅ ∠5 ≅ ∠7. If we
rior angles).
mark all the congruent angles in the figure, it be-
∠1 and ∠8 and also ∠2 and ∠7 are pairs of con-
comes clear that the following rule also holds.
secutive exterior angles (or same-side exte-
rior angles).
If a pair of parallel lines is crossed by a transver-
sal, then each pair of corresponding angles is
Upper Upper Upper Upper
Left Right Left Right congruent.
1 2 5 6
4 3 8 7
Lower Lower Lower Lower
Left Right Left Right

If the angles are located in the same relative


position—one at each vertex—they are called corre-
sponding angles. For example, ∠1 and ∠5 are both lo-
cated in the upper left position, so they are in
corresponding positions.

152
8th_GRDM_08.qxd:Layout 1 8/11/09 3:22 PM Page 153

–ANGLES AND TRIANGLES–

1 2
4 3 Acute angles have Right angles measure Obtuse angles Straight angles
measures between exactly 90º. The box have measures are exactly 180º.
0º and 90º. 0º and 90º. between 90º and
5 6 180º.
8 7

What can we say about the consecutive interior Triangles


angles in this figure? Because ∠4 and ∠3 form a
linear pair, m∠4 + m∠3 = 180. But because m∠4 = Types of Triangles
m∠6, we can substitute in the equation to get m∠6 + A triangle is a polygon with exactly three sides and
m∠3 = 180. This means ∠6 and ∠3 are supplements. three angles. A triangle might be called ΔABC. (The
By the same pattern of reasoning, ∠4 and ∠5 are sup- letters can be arranged in any order and they will still
plements. And this gives us the third rule. refer to the same triangle.)
A
If a pair of parallel lines is crossed by a transver-
sal, then each pair of consecutive angles are
supplements. B C

With a little experimentation, you will quickly find


Notice that the terms in the following figure are in al- that no triangle can have two right angles, two obtuse
phabetical order. angles, or a right angle and an obtuse angle. The rays
move apart and never connect to form a triangle, as
the following diagram illustrates. This means every
triangle has at least two acute angles.
1 2
Alternate Consecutive Corresponding
interior interior angles
angles angles

Exercise 6 Two right angles Two obtuse angles One right and
Suppose m || n and m∠1 = 100. Find m∠2, m∠3, m∠ one obtuse angle

4, m∠5, m∠6, m∠7, and m∠8.


t We can classify triangles into three groups based on
the size of the third angle.

1 8
m
2 7

3 6 n
4 5 An acute triangle A right triangle has An obtuse triangle
has three acute angles. one right angle has one obtuse angle
and two acute angles. and two acute angles.

Classifying Angles Based on Size


There are four types of angles. The types are based on
the measure of the angles: acute angles, right angles,
obtuse angles, and straight angles.
153
8th_GRDM_08.qxd:Layout 1 8/11/09 3:22 PM Page 154

–ANGLES AND TRIANGLES–

We can also classify triangles by the number of equal triangles are always equiangular, and equiangular tri-
sides. Hatch marks are used to indicate which sides angles are always equilateral. This rule, however, is
are equal. true only for triangles, not for other polygons. (See
Lesson 18 if you don’t know what a polygon is.)

Equiangular but not equilateral:

Scalene triangle

A scalene triangle has no equal sides.


A
Vertex

Leg Leg

Equilateral but not equiangular:

Base Angle Base Angle


B C
Base
Isosceles triangle

An isosceles triangle has at least two equal sides. The 2

equal sides are called legs and the third side is the
base. ∠B and ∠C are the base angles. ∠A is called a 1
vertex angle.
m∠1 = m∠2

The Sum of the Angles of a Triangle


Try this:

2
Equilateral triangle 2

An equilateral triangle has three equal sides. Because 1 3


Cut out any triangle.
three sides is also at least two sides, equilateral trian-
1 3 2
gles are also isosceles. Tear off corners.
1 3
Line up the edges
and vertices.

No matter what kind of triangle we experiment with


in this way, two of the sides will always make a
straight angle. This means that m∠1 + m∠2 + m∠3
= 180.
Equiangular triangle

An equiangular triangle has three equal angles. The sum of the three angles of a triangle is al-
If you think equilateral triangles look exactly ways 180.
like equiangular triangles, you are correct. Equilateral
154
8th_GRDM_08.qxd:Layout 1 8/11/09 3:22 PM Page 155

–ANGLES AND TRIANGLES–

Exercise 7 m∠A + m∠ABC + m∠C + m∠ADC


Find the value of x in each figure. = m∠A + m∠1 + m∠2 + m∠C + m∠4 + m∠3
= m∠A + m∠1 + m∠3 + m∠C + m∠4 + m∠2
1. A = 180 + 180 = 2(180), or 360

The next diagram shows how we can use the


same method to find the sum of the measures of the
30° x
angles of any polygon.
B C

2. D

70

5 Sides 6 Sides 7 Sides


3 Triangles 4 Triangles 5 Triangles
Sum of angles Sum of angles Sum of angles
x is 3(180). is 4(180). is 5(180).

E F
Have you noticed a pattern? The number of tri-
angles is always two less than the number of sides, so
The Sum of the if there are n sides there will be n – 2 triangles. So the
Angles of a Polygon sum will be (n – 2)180.

To find the sum of the measures of the angles of a


convex polygon, we divide the interior of the polygon The sum of the angles of a convex polygon of n
into a set of triangles that cover every part of the inte- sides is S = (n – 2)180.
rior of the polygon yet do not overlap each other. The
easiest way to do this is by drawing all the diagonals
from one vertex, as pictured. This rule does not work for concave polygons
(see Lesson 18) because in concave polygons at least
A B one angle opens to the exterior rather than to the in-
1
2
terior of the polygon. Using the formula on such a
concave polygon would produce too large an answer.

Exercise 8

3 1. Find the sum of the measures of the angles of a


4
D C polygon with 10 sides.

The sum of the four angles of the quadrilateral 2. Find the measure of each angle of a regular
equals the sum of the measures of the six angles of polygon with 15 sides.
the two triangles. Because m∠ABC = m∠1 + m∠2
and m∠ADC = m∠3 + m∠4,

155
8th_GRDM_08.qxd:Layout 1 8/11/09 3:22 PM Page 156

–ANGLES AND TRIANGLES–

Summary Exercise 5
Because ray RE bisects ∠TRU, m∠1 = m∠2, and be-
In this lesson, you have learned definitions for angles cause m∠1 = 80, m∠2 = 80. Also, m∠1 = 12 m∠TRU,
and triangles. You have also learned the parts of both so 80 = 12 m∠TRU and m∠TRU = 160.
angles and triangles, and you have learned how to
find missing angle measures given information about Exercise 6
their supplements and complements. Because ∠1 and ∠8 form a linear pair, they are sup-
plements, so m∠1 + m∠8 = 180. But m∠1 = 100, so
100 + m∠8 = 180. Using algebra, m∠8 = 80. Because
Answers m || n, we can use our three rules about the angles
formed when parallel lines are crossed by a transver-
Exercise 1 sal (see boxes), and say m∠1= m∠7 = m∠3 = m∠5 =
m∠CAD = 50. 100, and m∠8= m∠2 = m∠6 = m∠4 = 80.

Exercise 2 Exercise 7
Because we have a linear pair, 1. Because the three angles of a triangle add to
70 + x = 180 180,
x = 110 m∠A + m∠B + m∠C = 180
Vertical angles are congruent, so y = 70. Likewise, By substituting, 90 + 30 + x = 180
z = 110. 120 + x = 180
x = 60
Exercise 3 2. Because the three angles of a triangle add to
If m∠A = 65, its complement is 90 – 65 = 25, and its 180,
supplement is 180 – 65 = 115. m∠D + m∠E + m∠F = 180
And because m∠E = m∠F,
Exercise 4 70 + x + x = 180
120 70 + 2x = 180
Let x = the angle and 180 – x = its supplement. Now 2x = 110
translate the English to mathematics. x = 55

An angle is twice as large as its supplement.

x = 2× (180 – x)

x = 2(180 – x)
x = 360 – 2x
3x = 360
x = 120

So the angle is 120 and its supplement is 180 – 120


= 60.

156
8th_GRDM_08.qxd:Layout 1 8/11/09 3:22 PM Page 157

–ANGLES AND TRIANGLES–

Exercise 8 2. 156
1. 1,440 First, we need to find the sum of all the angles.
It is faster to use the formula than to draw the Then, because all the angles of a regular
polygons. Because n = 10, polygon are equal, we can divide by the
S = (n – 2)180 number of angles to find the size of each angle.
S = (10 – 2)180 S = (n – 2)180
S = (8)180 S = (15 – 2)180
S = 1,440 S = (13)180
S = 2,340
Each angle = 2,340
15 = 156

157
8th_GRDM_08.qxd:Layout 1 8/11/09 3:22 PM Page 158
8th_GRDM_08.qxd:Layout 1 8/11/09 3:22 PM Page 159

18
L E S S O N

POLYGONS
AND CIRCLES

L E S S O N S U M M A RY
In this lesson you’ll learn some of the properties of polygons. You’ll
learn about area and perimeter for several types of polygons.
You’ll also learn about the perimeter and area of the circle, and
about parts of circles.

N ow that you have learned some properties of angles, it’s time to move on to the properties of closed
two-dimensional shapes. This lesson will review concepts you already know—area and perimeter—
and will introduce some other properties. And it will show you how to find dimensions you don’t
know, using the ones you do know. (You haven’t forgotten Section 2, have you?)

159
8th_GRDM_08.qxd:Layout 1 8/11/09 3:22 PM Page 160

–POLYGONS AND CIRCLES–

Polygons and Their Parts We name the vertices of a polygon with capital
letters. The vertices of the following polygon are A, B,
First, let’s look at some pictures. These are polygons: C, and D. Some books will name a side by listing its
endpoints and drawing a straight line above the two
letters. Don’t let this confuse you! In the following di-
agram, AB, BC, CD, and DA are the sides of the poly-
gon. Because it does not matter which endpoint we
name first, AB and BA name the same side. Occasion-
As you can see, a polygon is made of straight line seg- ally, we will name a side with a single lowercase letter.
ments (called sides) joined at their endpoints (called AB, for example, could be called p.
vertices) in such a way that each vertex is connected
A
to exactly two segments.
These are not polygons:
Curved sides are not allowed!

B
D

We name a polygon by listing the vertices in the


order we would cross them as we trace the polygon
with our finger. So we might name this polygon
ABCD, CBAD, or even CDAB. We could not name it
Only one segment is connected to point B:
ACBD because A is not directly joined to C. In tracing
B the polygon, we must pass either B or D before we can
reach A from C.

Convex and Concave Polygons


All polygons are either convex or concave (sometimes
called non-convex).
Three segments are connected to point A:
These polygons are convex:
1
A
2

If you lassoed a convex polygon and pulled the


rope tight, the rope would touch every part of the
polygon.

160
8th_GRDM_08.qxd:Layout 1 8/11/09 3:22 PM Page 161

–POLYGONS AND CIRCLES–

If you lassoed a concave polygon and pulled the


Lasso rope tight, the rope would not touch every part of the
polygon.

Lasso

These are concave polygons:

Lasso does not


touch all four sides.

In geometry we spend most of our time study-


ing convex polygons.

Other Types of Polygons


Two polygons with the same number of sides often
have the same characteristics, so we study them in
groups based on the number of sides. This chart
shows some of the more common polygons we study.

TRIANGLE QUADRILATERAL PENTAGON HEXAGON OCTAGON N -GON

3 sides 4 sides 5 sides 6 sides 8 sides n sides

When we are unsure how many sides a polygon has The boxes are linked by either solid or dashed
we call it an n-gon, where n is a variable for the num- paths. Those linked by solid paths belong to the par-
ber of sides. allelogram family, and those linked by dashed paths
are trapezoids. A shape that belongs to one of the cat-
Well-Known Convex Quadrilaterals egories listed also belongs to every category listed
In the previous lesson we discussed the different kinds above it and it always possesses every one of the prop-
of triangles we might encounter. In this lesson we will erties listed in those boxes. A shape sometimes belongs
investigate many types of quadrilaterals, namely paral- to the categories listed below it and sometimes pos-
lelograms, rectangles, rhombuses, squares, and trape- sesses the properties listed in those boxes. Shapes
zoids. Each box in the chart on the following page linked only by solid lines never belong to the cate-
contains the name of a category of quadrilaterals, some gories linked only by dashed lines and never possess
examples, and a listing of the properties every quadri- any of the properties listed in those boxes. The reverse
lateral in that category possesses. is also true. Shapes on dashed lines never belong to
the categories linked by only solid lines and never
possess any of the properties listed in those boxes.

161
8th_GRDM_08.qxd:Layout 1 8/11/09 3:22 PM Page 162

–POLYGONS AND CIRCLES–

Use the chart frequently in practice and try to Then, if you need the information, you can quickly
reproduce it from memory, writing just the names of sketch the chart.
the shapes. Occasionally, try to include the pictures.

Convex Quadrilateral
Example:

Properties:
4 sides and 4 angles

Parallelogram Trapezoid
Example: Example:

Properties: Properties:
Two pairs of opposite sides are parallel. Exactly one pair of sides is parallel.
Two pairs of opposite sides are equal.
Two pairs of opposite angles are equal.
Diagonals bisect each other.
Isosceles Trapezoid
A B
Each pair of consecutive
Example:
angles are supplements.

D C
m∠A + m∠B = m∠B + m∠C = 180 Properties:
m∠C + m∠D = m∠D + m∠A = 180 Nonparallel sides are equal.

Rectangle Rhombus Two sets of base


angles are equal.
Example: Example:

Properties: Properties:
All sides are equal.
All 4 angles are right angles. The diagonals are
The diagonals are equal. perpendicular.
A C
The diagonals
bisect Key:
the angles. To solve sometimes, always, or never
D AB = CD B problems choose

Always, if moving up
Square
Sometimes, if moving down
Example:

Properties: Never, if moving from


Four equal sides to
Four equal angles or from
to

162
8th_GRDM_08.qxd:Layout 1 8/11/09 3:22 PM Page 163

–POLYGONS AND CIRCLES–

Exercise 1 Exercise 3
Write sometimes, always, or never in the blank. An- Find the perimeter of each polygon (measurements
swers are on page 169. in meters, or m). Answers are on page 169.

1. A rectangle is _____________ a quadrilateral. 1.


6m

2. A rectangle is _____________ a square. 10 m

5m
3. A rectangle is _____________ an isosceles
trapezoid.
12 m

4. A rectangle is _____________ a rhombus.


2. 5

Exercise 2 5 5
Write sometimes, always, or never in the blank. An-
swers are on page 169.
5 5
1. A rectangle _____________ has four right
5
angles.
Because 6(5) = 5 + 5 + 5 + 5 + 5 + 5, we could write
2. The diagonals of a trapezoid _____________
P = 6(5) = ?
bisect each other.
Number of Length of
3. The opposite angles of a rhombus are equal sides each side
_____________ equal.
So now we have a formula.
4. The diagonals of a rhombus are
_____________ equal.
If all the sides of a polygon are equal, then the
Finding the Perimeter of a Polygon formula for the perimeter is P = ns
The perimeter of any shape is the sum of the lengths where n = the number of sides and s = the
of its sides. Notice the word rim in the middle of the length of each side. Squares and rhombuses
word peRIMeter. It reminds us to add all the numbers have four equal sides, so the formula for their
on the rim of the shape. We use the capital letter P for perimeters is P = 4s.
perimeter.

163
8th_GRDM_08.qxd:Layout 1 8/11/09 3:22 PM Page 164

–POLYGONS AND CIRCLES–

Exercise 4 This system works great for small right-angled


(Answers are on page 169.) polygons, but what about parallelograms? How do we
count all those little bits of squares?
1. Find the perimeter of a polygon with 32 equal
sides, each 6 inches long. 6 units

2. Find the perimeter of a rectangle with length 4


units and width 10 units. 4 units

Exercise 4 leads us to the following formula.

Or what if a rectangle has a length or width that


The perimeter of a rectangle with length l and is a decimal? Or what if the width is 100 units long?
width w is Counting squares quickly becomes impractical. This
P=2×l+2×w is exactly why people have come up with formulas
By factoring, we get P = 2(l + w). for area.
Did you notice that, instead of counting squares
in that problem with the 4 by 6 rectangle, we could
Exercise 5 have just calculated 4 × 6 to get 24? It turns out that
Find the perimeter of the following rectangle. An- the easy way to find the area of a rectangle is to mul-
swers are on page 170. tiply the length and the width. One formula for the
2x + 5 area of a rectangle is A = lw where l is length and w
is width.
x–3 x–3
We could state this formula another way. No-
tice that in our 4 by 6 rectangle one side of the rec-
tangle is 6 units long and each stack of square units
2x + 5
is 4 squares in height. So we could say the formula
for the area of a rectangle is A = bh where b is the
Area of Triangles and Polygons length of one side (the base) and h is the height.
This is very useful because the base and the height
Area is the measure of the inside of a polygon. We are key players in finding the areas of triangles and
measure area by counting how many square units fit convex polygons!
inside the shape. A square unit is a square that is one
unit long on each side. For example, the following di- Recognizing the Base and the Height
agram shows a 4-unit by 6-unit rectangle that has of Polygons
been cut into square units. By counting the small Any side of a polygon can be a base and each base is
squares, we find that the area is 24 square units. connected to a height. Because the base and the
6 units height are always perpendicular, look for the box that
indicates perpendicular objects.

one square unit


4 units

24 square units

164
8th_GRDM_08.qxd:Layout 1 8/11/09 3:22 PM Page 165

–POLYGONS AND CIRCLES–

In ΔABC the right-angle box tells us that if CB is


used as the base, then AD is the height. The height of a polygon can be located inside
the polygon, on the polygon, or outside the
A
polygon. The base is always a side of the poly-
gon. The height is always perpendicular to the
h line containing the base.
To save time, we often just say the word
base when we mean the length of the base and
C D B
just say height when we mean the length of the
In parallelogram SMIE, the box tells us that SE is the height.
base and either SD or IL can be used as the height.
(This is true because the distance between parallel
lines is constant, so SD = IL.) Using the Formula for the Area of
a Rectangle to Find Other Useful
D M I Area Formulas
They say that a picture is worth a thousand words.
These “proofs without words” show us how we discov-
ered the formulas for the areas of various polygons
and they are helpful in remembering the formulas.

S E L The Area of a Triangle


b b
When a triangle is a right triangle, the two sides
forming the right angle can be used as the base and
height. The right-angle box indicates that AE and EY h h h h h h
can be the base or the height. If the hypotenuse, AY,
were used as the base, then ES would be used as the
height. Notice it is often helpful to draw the height as b b b b

a dotted line. Two identical Flip one To make a


triangles of them rectangle
A

or
h

E Y

h h h

b b c d
Two identical triangles b=c+d Cut one apart Move the parts to
make a rectangle

165
8th_GRDM_08.qxd:Layout 1 8/11/09 3:22 PM Page 166

–POLYGONS AND CIRCLES–

Because the formula for the area of a rectangle is A = bh and since the triangle is exactly half the area of the rectan-
gle, the formula for the area of a triangle is A = 12bh.

The Area of a Parallelogram

b Cut apart Move the small part . . . To make a


rectangle

Because nothing is lost if we cut off a triangle from one end of a polygon and move it to the other end, the area
of the polygon must have the same area as the rectangle. Therefore, the formula for the area of a parallelogram
is A = bh.

The Area of a Trapezoid


The two heights are equal because the distance between two parallel lines is the same no matter where we meas-
ure. The two parallel bases are marked b1 and b2: A good way to think of them is as base one and base two.
b1 b1

h h +
h h

b2 Draw a diagonal b2

Area of a trapezoid = 12b1h + 12b2h


A = 12h(b1 + b2)

Using Formulas to Find Area Exercise 6


Finding the areas of triangles and quadrilaterals Find the area of each triangle. Answers are on page 170.
means identifying the shape in order to find the right
formula and then substituting the correct data into 1. R

the formula. When no picture is given, it is helpful to 13 m


draw one. Also, problems involving area often contain 5m

extra numbers that are never used. Distinguishing be-


tween useful and useless data is an important part of Q 3m T 9m S

problem solving. In these problems, remember that


recognizing where the height is measured helps us 2. R
find the good data: the length of the base and the
height.

15 13
12

T 9 M 5 S

166
8th_GRDM_08.qxd:Layout 1 8/11/09 3:22 PM Page 167

–POLYGONS AND CIRCLES–

Writing out the formula as you begin solving a prob- Exercise 9


lem will help you memorize the formula, and it helps Find the area of this shape (assume that the lines that
you to remember that all formulas are equations—a look parallel really are parallel):
point crucial to solving problems like the next one.
3

Exercise 7 3 7
Find the height of a triangle if the base is 12 cm and
the area is 78 cm2. Answers are on page 170. 6

Exercise 8
14
Find the area of each quadrilateral. Answers are on
page 170.
Circles
1.
8
12 A circle with center C and radius r is the set of all
15 points whose distance from C is r. When you join all
those points, you get a continuous line, and thus a
closed shape. In the next picture, the radius is 5. Each
point on the rim of the circle is exactly 5 units away
2. 6
4
from the center C. CD is called the radius and AB is a
5
2 diameter. Because both A and B are 5 units from the
14
center, AB = 10. So the diameter is twice the radius, or
d = 2r.
3. A x+2 B
A

x 5
5
C D
D C
5
Finding the Areas of
Concave Polygons B
The following is an important fact about area.
Circumference of a Circle
The perimeter of a circle is called the circumference.
If the area of a polygon is divided into several The formula for the circumference has had to be found
regions that do not overlap, the sum of the ar- experimentally, because there are no handy straight
eas of the regions is equal to the area of the lines to measure. It has been found to be C = πd, where
polygon. π is the value that’s been found experimentally and d
is the diameter. That value can be computed out to
many places; to the nearest hundredth, it’s 3.14. Be-

167
8th_GRDM_08.qxd:Layout 1 8/11/09 3:22 PM Page 168

–POLYGONS AND CIRCLES–

cause the radius is exactly half the diameter, the for- 2. Find the circumference of an arc of a circle that
mula is often written as C = 2πr. has a diameter of 28 cm if the central angle is
Part of a circle is called an arc. 48 degrees.

arc
E

r
48°
60°
r d = 28 cm
C D

Because you need to know the radius of a circle to Area of a Circle


find the circumference, you also need to know the The formula for the area of a circle is
length of the radius of the arc. And you need to know
the angle between the two lines from the center of the A = πr2
circle that mark the boundaries of the arc (CE and
CD in the picture). Why? Remember from Lesson 17 The area of a circle with radius 5.2 feet is A = π(5.2 ft.)2
that there are 360 degrees in a circle. So, if the angle is = π × 27.04 ft.2.
60 degrees, the arc will be 16 of a circle, and its circum- Finding the area of part of a circle, called a sec-
ference will be 2πr tor, is like finding the circumference of an arc. You
6 . The general formula is
just need to know the angle of the arc and the area of
angle of arc the circle (or the length of its radius). Then, divide
Carc = arc length = 360
× 2π r
the angle by 360 degrees and multiply that fraction
times the area of the circle:
Exercise 10
angle of arc
Asector = 360
× πr2
1. Find the radius of a circle with circumference
of 30π cm. Exercise 11
Find the area of a sector with radius 15 inches if the
C = 30π cm
sector’s central angle is 120 degrees.

120°

r = 15"

168
8th_GRDM_08.qxd:Layout 1 8/11/09 3:22 PM Page 169

–POLYGONS AND CIRCLES–

Summary 3. Opposite angles are equal is a property of


parallelograms. Because a rhombus is always a
The most common types of polygons are the triangle parallelogram, choose always.
and the quadrilateral. If you know the properties of 4. The diagonals are equal is a property of
the different quadrilaterals, you’ll be able to find rectangles, and because a rhombus is
missing lengths and angles, as well as perimeters and sometimes a rectangle (if it is a square) choose
areas. In eighth grade and beyond, you’ll be combin- sometimes.
ing your algebra skills with geometry skills to do this.
Exercise 3
1. 33 meters
Answers P = Sum of the lengths of the sides
P = 6 m + 10 m +12 m + 5 m
Exercise 1 P = 33 m
1. To move from the box containing the word 2. 30 meters
rectangle to the box containing the word P = Sum of the lengths of the sides
quadrilateral, we must move up, so use the P=5+5+5+5+5+5
word always. P = 30
2. Rectangle is above square, so use the word
sometimes. Exercise 4
3. The rectangles are on the solid line and the 1. 192 inches
isosceles trapezoids are on the dashed lines, so Since all 32 sides are equal, P = ns
never is the correct choice. P = 32 × 6 inches
4. A bit more thought is required here, because P = 192 inches
we must move both up and down. Because a 2. Draw a picture of a rectangle and write the
square is always a rectangle and always a lengths on the sides.
rhombus, the rectangle is sometimes a 10

rhombus.
4 4

Exercise 2
1. The box containing the word rectangle includes 10
the property that a rectangle contains four P = 10 + 4 + 10 + 4
right angles. The correct choice is always. P = 28
2. The words diagonals bisect each other appear Or, since the numbers in the problem repeat,
under the properties of parallelograms. By P = 2 × 4 + 2 × 10
looking at the chart, we see that trapezoids are P = 8 + 20
never parallelograms, so choose never. P = 28

169
8th_GRDM_08.qxd:Layout 1 8/11/09 3:22 PM Page 170

–POLYGONS AND CIRCLES–

Exercise 5 Write the formula and then substitute in the data.


Since the shape is a rectangle, use the formula A = 12bh
P = 2l + 2w 78 cm2 = 12(12 cm)h
P = 2(2x + 5) + 2(x – 3) 78 cm2 = (6 cm)h
P = 4x + 10 + 2x – 6 78 cm 2
=h
6 cm
P = 6x + 4
13 cm = h
Or, to decrease the number of computations and thus
decrease the number of opportunities to make an er-
Remember that length is in units and area is in square
ror, use
units. When you divide cm2 by cm, you get cm. (If
P = 2(l + w)
you have forgotten why, review Lesson 4.)
P = 2[(2x + 5) + (x – 3)]
P = 2(3x + 2)
Exercise 8
P = 6x + 4
1. The shape is a parallelogram, so use the
formula A = bh. The box tells us that b = 8 and
Exercise 6
h =12.
1. 22.5 m2
A = bh
The right-angle box indicates that the height is
A = (8)(12)
5. Because the height is outside the triangle, we
A = 96 square units
follow the line QT to the side of the triangle.
2. The shape is a trapezoid, so use the formula
The base is TS, or 9.
A = 12h(b1 + b2). The bases are parallel in a
A = 12bh
trapezoid, so they are 6 and 14. The right-angle
A = 12(9 m)(5 m)
symbol tells us that h = 2.
A = 22.5 square meters, or 22.5 m2
A = 12h(b1 + b2)
2. 84 units2
A = 12(2)(6 + 14)
The box tells us that TS is the base, so b = 9 + 5
A = 20 square units
= 14 and the height is 12, the length of RM.
3. The shape is a rectangle, so the formula is
A = 12bh
A = bh. The box indicates that the height is x
A = 12(14)(12)
and the base is DC. But how long is DC?
A = 84 square units
Because a rectangle is a parallelogram, the
(When no units are given in the problem, we
opposite sides are equal, and DC = AB = x + 2.
use square units as a part of the answer.)
A = bh
A = (x)(x + 2)
Exercise 7
A = x2 +2x square units
Draw a picture and include all data.

A = 78 cm2

12 cm

170
8th_GRDM_08.qxd:Layout 1 8/11/09 3:22 PM Page 171

–POLYGONS AND CIRCLES–

Exercise 9 Exercise 10
81 square units 1. 15 cm
First, we divide the shape into regions and then find Here we know the circumference C but not the
the area of each region. Finally, we add the areas. radius. But we can start with the same formula:
We could divide the area this way. The top shape C = 2πr.
is a square and the bottom one is a trapezoid. 30π cm = 2πr
30π
Opposite 2π cm = r
sides of a
square are 15 cm = r
3 equal 3
2. 11.71 cm
3 7 3 7 The angle of 48 degrees means that the arc
3 48
10
covers 360 of the circle’s circumference. So the
6 6 arc length is
48
Carc = arc length = 360 × 2πr
14 14 The radius is 14 cm, because it’s half of the
A=b×h
A=3×3 = 9 diameter. So the arc length is
Carc = arc length = 48(2π )14 = 3.73π cm ≈ 11.71 cm
1
A=—
2 h(b1 + b2)
1 360
A= —
2
× 6(10 + 14) = 72
Add 9 + 72 to get 81

Could we have divided this differently? Yes, Exercise 11


there are several other ways. One is shown next. Can For a quick calculation, notice that 120 is exactly one-
you think of others? third of 360.
3 3 Asector = 13 × π(15 inches)2
3 3 3 A = bh = A = 3 × 3 = 9 Asector = 13 × 225π inches2 = 75π inches2 ≈
R T 7 S
10
235.6 inches2
6 1 1
6 A =— bh = — 4 × 6 =12
2 2
A = bh = 10 × 6 = 60
4 Then add: 9 + 12 + 60 = 81
A B 14 C

Cut into a triangle


and a rectangle
6

Since AC = 14 10
and RS = BC = 10
AB = 4

171
8th_GRDM_08.qxd:Layout 1 8/11/09 3:22 PM Page 172
8th_GRDM_09.qxd:Layout 1 8/11/09 1:53 PM Page 173

19
L E S S O N

VOLUME AND
SIMILAR SHAPES

L E S S O N S U M M A RY
In order to be successful at geometry, you will need to learn some
basic definitions and formulas. By understanding the properties of
three-dimensional objects and by learning how to calculate them,
you will be able to figure out many real-life problems. You will also
learn about similar two-dimensional and three-dimensional shapes.

Volume

When landscape architects remove dirt to make a pond, how do they measure how much dirt was removed? This
is an important question because it takes time and money to dispose of the dirt. In fact, the proposal for a new
project will include plans for repositioning the soil somewhere on the site. Landscape architects even have a term
for the process. They call it cut and fill. How do they know how much dirt will be removed? Mathematicians
solved this problem ages ago. They use the term volume to describe the amount of dirt that will be removed.

What Is Volume?
To measure volume we use cubic units. A cubic unit is the volume of a cube that is one unit long on each side. A
cubic foot would be a cube 1 foot long on each side.

173
8th_GRDM_09.qxd:Layout 1 8/11/09 1:53 PM Page 174

–VOLUME AND SIMILAR SHAPES–

Base

Height Height
1 ft.

or or

1 ft.
Base
1 ft.
Base
1 cubic foot Height

Just as it was impractical to find area by filling a shape If the bases are circles instead of polygons, then
with squares, it is impractical to measure something the solid is called a cylinder. Cylinders can be right or
like a pile of dirt by filling up cubic feet with the dirt. oblique.
Instead, we rely on formulas. In this lesson we will in-
vestigate the formulas for the volumes of four solids: Base
the prism, the cylinder, the regular pyramid, and the
right cone.

The Prism and the Cylinder


Imagine two parallel planes, each containing an iden-
tical polygon. If we connect the polygons by joining
their vertices with line segments, we produce a shape
called a prism. All the sides are polygons and we call
them the faces. The two polygons in the parallel
planes are called the bases. If a face is not a base, then Right Base Oblique
cylinder cylinder
it is known as a lateral face. The segments joining the
polygons are called lateral edges. Because the lateral faces are rectangles, the height
of a right prism is the length of the lateral edge. The
Base height of any prism or any cylinder is a segment that
Lateral joins the parallel planes containing the bases and is
face
perpendicular to at least one base. (Look for the small
Lateral
box in an angle’s corner that indicates a right angle.)
edge The word altitude is a synonym for height.

Base Right prism Oblique prism


Right prism Oblique prism

If the lateral edges make right angles with the edges of Height
Height
the polygon, the prism is a right prism. Otherwise, it
is called an oblique prism. In a right prism all the lat-
eral faces are rectangles. If the bases are also rectan-
gles, then any pair of opposite faces can be chosen as
Height
bases.

174
8th_GRDM_09.qxd:Layout 1 8/11/09 1:53 PM Page 175

–VOLUME AND SIMILAR SHAPES–

The Formula for the Volume of a Prism Method 2: Because the base is a rectangle and B =
or Cylinder lw, we can rewrite the formula for volume. (We use lw
Advanced mathematics—calculus—is required to de- here rather than bh because we are already using the
velop the formula for volume, so we will just state it. variable h for the height of the prism.)
We find the volume by finding the area of the base
V = Bh
and multiplying by the height.
V = lwh (Substituting for B)
V = 10(8)(3)
FORMULA FOR THE V = 240 cu. ft., or 240 ft.3
VOLUME OF A PRISM This method is shorter to write and requires fewer
OR CYLINDER key strokes on the calculator.

V = Bh where B is read as “the area of the base.” Exercise 1


This formula is used for prisms and cylinders, The landscape architect would like to use the dirt from
right or oblique. the 10 foot × 8 foot × 3 foot pool to fill some cylindri-
cal planters that will decorate the property. The dimen-
sions of the interior of the planters are given. The top 3
Let’s use the formula to solve a problem. A land- inches will not be filled to allow room for the dirt
scape architect plans to remove dirt to make way for a around the plant roots. How many planters will be
rectangular pool that will be 10 feet × 8 feet × 3 feet. filled? Use π = 3.14. Answer is on page 181.
What volume of dirt will be removed?
It helps to draw a sketch of the pool. All the
faces of the pool are rectangles so all edges are per-
pendicular and the pool is a right prism. 3"

3'
Dirt
3'

2'
8'

10'

Method 1: Find B and then find the volume. The The Regular Pyramid and the Cone
base is a rectangle with length = 10′ and width = 8′. Vertex

B = lw The face
is an
B = 10′(8′) isosceles
B = 80 sq. ft. triangle
V = Bh
V = 80(3)
V = 240 cu. ft., or 240 ft.3
Base
Regular Cone
pyramid

175
8th_GRDM_09.qxd:Layout 1 8/11/09 1:53 PM Page 176

–VOLUME AND SIMILAR SHAPES–

As shown in the preceding figure, the base of a regu- Proving the formulas for the cylinder, the cone,
lar pyramid is a regular polygon. This means the and the pyramid requires calculus, but we can verify
polygon is equilateral, equiangular, and convex. The some relationships by filling hollow solids with water.
base of a cone is a circle. The point at the top is called If a cone and a cylinder have congruent bases and
the vertex. A cone or a regular pyramid can be right equal heights, the formula tells us that the volume of
or oblique. If the cone or the pyramid is right, the the cone will be 13 the volume of the cylinder. This
height is the length of the segment joining the vertex means that if we used the hollow cone to transfer wa-
to the center of the base. The lateral faces of a right ter to the cylinder, we would have to fill the cone
regular pyramid are congruent isosceles triangles. three times before we filled the cylinder.
The height from the vertex to the midpoint of
the base of one of these isosceles triangles is the slant
height of the pyramid. The slant height of the cone
joins the vertex to the rim of the circle.

Height is
Slant height perpendicular
to the base.

h
h
Exercise 2
C (Answer is on page 181.)
A

Slant height is B 1. A candy company plans to make foil-wrapped


perpendicular to AB. chocolate pyramids to sell at museums as part
of a traveling exhibit on Egypt. The dimensions
Volume of a Regular Pyramid or a Cone are shown in the following figure. What volume
To find the volume of both the right regular pyramid
and the cone, take 13 the product of the area of the
base and the height.
3"

4
FORMULA FOR THE
VOLUME OF THE REGULAR
PYRAMID OR THE CONE 2"

V = 13Bh, where B is the area of the base 2"


This formula can be used for right or oblique
of chocolate will be needed for the pyramids?
cones and pyramids.

2. Find the maximum volume of a cone-shaped


paper cup if the depth is 4 cm and the slant
height is 5 cm. Use π = 3.14.

176
8th_GRDM_09.qxd:Layout 1 8/11/09 1:53 PM Page 177

–VOLUME AND SIMILAR SHAPES–

r This is because we have lined the triangles up so the


corresponding parts are in the same position.
A
4 cm

P
5 cm

C B

M K
Similar Figures
This arrangement visually matches the correspon-
ding parts—the largest to the largest, the smallest to
When we enlarge a photograph, the new picture is
the smallest, and so on. All the points, sides, and an-
just like the old one, only larger. When an architect
gles are lined up in the same way. We often use the
designs a building, he or she may build a scale model.
double arrow to indicate corresponding parts:
The scale model is just like the building he or she
plans to build, only smaller. One of the purposes of (∠A ↔ ∠P is read as angle A corresponds to
mathematics is to quantify and explain relationships angle B.)
found in the real world. Mathematicians say that the ∠A ↔ ∠P AB ↔ PK
larger and smaller objects are similar. This means ∠B ↔ ∠K BC ↔ KM
that all the corresponding angles are equal, and all the ∠C ↔ ∠M AC ↔ PM
corresponding sides are proportional. To illustrate
this, let’s look at two similar triangles. All this information can be packed into one short
statement: ΔABC ↔ ΔPKM. This is called a matching
What Are Corresponding Parts? of the vertices because we matched up the vertices of
C corresponding angles. As the following list reveals,
lining up the angles automatically lines up the sides:
P
ΔABC ↔ ΔPKM
ΔABC ↔ ΔPKM
ΔABC ↔ ΔPKM

B A If the two triangles are similar, we write “ΔABC ~


ΔPKM.” This is read as ΔABC is similar to ΔPKM.
M K The corresponding sides are not congruent, but
The corresponding parts of two polygons are parts there is a relationship. It looks as though all of the
(one from each polygon) matched both by size and sides of the smaller triangle have been stretched by
related position in the polygons. Triangles ΔABC and the same amount to produce the larger triangle.
ΔKPM are similar, but the arrangement conceals the Mathematicians say the sides of the triangle are pro-
relationship. If we line them up so the congruent an- portional, which means that the ratios of correspon-
gles are in the same position, they look more alike. ding sides are equal.

177
8th_GRDM_09.qxd:Layout 1 8/11/09 1:53 PM Page 178

–VOLUME AND SIMILAR SHAPES–

Δ ABC ~ Δ PKM MEANS Scale Factor


∠A ≅ ∠P
When sides are proportional, the ratios of the corre-
∠B ≅ ∠K and AB = BC = AC sponding parts all reduce to the same number. In Exer-
PK KM PM

∠C ≅ ∠M cise 6 the ratios reduced to 23. The number 23 is the scale


factor for the two similar triangles. We usually repre-
Exercise 3 sent the scale factor by the symbol ab where ab is the ratio
Are the triangles similar? Answer is on page 181. of the corresponding sides reduced to lowest terms.

S
Circles

15
A Two circles are always similar and the scale fac-
9
tor equals the ratio of their radii.
5
3 T U
12

B C
4

Exercise 4 6 8
Are the rectangles similar? Answer is on page 181.
A B
R S

8 If the radii of the two circles are 6 and 8, then the


3
scale factor is 68 = 34.
F 6 T D 10 C

The Importance of Scale Factor


Exercise 5
Are the polygons similar? Answer is on page 181.
If the scale factor for two similar polygons is ab,
then the ratio of the corresponding heights is
also ab.
8 4

If ΔABC ~ ΔEDF, find x, the height from E to


10 5
side DF.
Exercise 6 A

The triangles are similar. Find x, y, z, and w. Answer is E


on page 182. 10
8
5 x
z
y
8 9
6 B CD F

w 30 The scale factor for the two similar triangles is 105 = 21.
x 15 This means the ratio of the two corresponding
heights is also 21. So, 8x = 21 and x = 4. Did you notice

178
8th_GRDM_09.qxd:Layout 1 8/11/09 1:53 PM Page 179

–VOLUME AND SIMILAR SHAPES–

that the numerators of all the fractions were related to Exercise 7


ΔABC while the denominators were related to ΔEDF? The two triangles in this figure are similar. Find the
The most spectacular thing about scale factor is perimeter of the smaller triangle.
its usefulness in working with area, perimeter, and
volume. Let’s look at how the perimeters and areas of
6
two similar triangles compare. Suppose these two tri-
angles are similar.

10
15 20
24

26
25

15 Exercise 8
A contractor is using a blueprint with a scale of 1 inch
36 = 10 feet. He figures that the area of a room in the
blueprint is 12 square inches. Find the area of the ac-
39 tual room in the building.

We can find the scale factor by finding the ratio of a Similar Prisms, Cylinders,
pair of corresponding sides: 10 2
15 = 3. So the scale factor
Cones, and Pyramids
is 32.
Similar three-dimensional shapes have congruent
Now let’s look at the ratios of the perimeters: angles and proportional lengths. Suppose two prisms
or two cylinders are similar. How do their scale fac-
Perimeter of first triangle 10 + 24 + 26 60 2
= = = tors compare?
ond triangle 15 + 36 + 39 90 3
Perimeter of seco
The two prisms are similar.
This is the same as the scale factor. And at the
areas:
1 (10)(24) 14
Area of first triangle 2 120 4 22 7
= =
1 (15)(36) 270
= =
Area of second trianglle 2
9 32
4
3
In other words, the ratio is the scale factor squared. 8
This suggests a rule. 6

Because 14 6 8 2 2
7 = 3 = 4 = 1, the scale factor is 1. Now com-
If two triangles are similar and the ratio of a pair pare the volumes.
of corresponding sides is ab, then
the ratio of the perimeters is ab
2
and the ratio of the areas is a 2 .
b

179
8th_GRDM_09.qxd:Layout 1 8/11/09 1:53 PM Page 180

–VOLUME AND SIMILAR SHAPES–

Volume of first prism Bh lwh The results for two similar pyramids would be the
= 1 1 = 1 1 1
Volume of second prism B2h2 l2w 2h2 same. This suggests the following rule, which elimi-
6 × 8 × 14 8 ⎛ 2 ⎞
3 nates a lot of work.
= = = ⎜ ⎟ = (Scale factor)3
3× 4 × 7 1 ⎝ 1⎠

Let’s look at some cylinders. The two cylinders If the scale factor for two similar three-
are similar. dimensional figures is ba , then the ratio of their
3
a
volumes is b 3 .

Exercise 9
6
12 If the pyramids in this figure are similar and the vol-
ume of the larger pyramid is 160 cm3, what is the vol-
ume of the smaller pyramid?

The ratio of the corresponding lengths is 96 = 12 8


= 32, so
the scale factor is 32. The ratio of the volumes is:
Volume of first cylinder π × 62 × 8 8
= = 4 cm
Volume of second cyllinder π × 92 × 12 27
3
23 ⎛ 2 ⎞ 3 cm
= = ⎜ ⎟ = (Scale factor)3
3 ⎝ 3⎠
3

These two cones are similar. V=?


V = 160 cm3

Summary

6
In this lesson you were introduced to a number of ex-
2 tremely important geometric concepts that you will
use as a foundation for more complicated concepts
you’ll encounter in eighth grade and beyond. Your
1
3 understanding of shapes like regular pyramids,
prisms, and cones in terms of volume and their natu-
The ratio of the corresponding heights is 26 = 31 and the
ral properties will allow you to begin applying your
ratio of the radii is 31, so the scale factor is 31.
geometric knowledge to the world around you.
1 π 32 × 6
Volume of first cone 3 27
= 1 π 12 × 2
=
Volume of second cone 3
1
3
33 ⎛ 3 ⎞
= = ⎜ ⎟ = (Scale facttor)3
1 ⎝ 1⎠

180
8th_GRDM_09.qxd:Layout 1 8/11/09 1:53 PM Page 181

–VOLUME AND SIMILAR SHAPES–

Answers Exercise 3
Because ∠A ≅ ∠S, ∠B ≅ ∠T, and ∠C ≅ ∠U, all the
Exercise 1 corresponding angles are congruent. Now, we will
Six planters look at the ratios of the corresponding sides. Because
First, we will find the volume of the planter and then the order matters with ratios, we will put the sides of
we will divide the 240 cu. ft. of dirt by the answer. Be- ΔABC in the numerators and the sides of ΔSTU in the
cause the base of the planter is a circle, B = πr2, r = 2′, denominators.
h = 3′, and V = Bh.
AB 3
ST = 9 = 13
V = Bh
BC 4
V = πr2h TU = 12 = 13
V = (3.14)(22)(3) AC 5
= 13
SU = 15
V = 37.68 cu. ft.
240 cu. ft. Because all the fractions reduce to 13, we say,
37.68 cu. ft. = 6.3, so we will have enough dirt for six
planters. AB BC AC 1
ST = TU = SU =3

Exercise 2
which means the sides are proportional. In fact, the
1. One cubic inch
sides of the smaller triangle were tripled.
Because the base is a square, B = 2(2) = 4 in.2.
So
Exercise 4
1
V = Bh All the angles are right angles, so all the correspon-
3
ding angles are congruent. The opposite sides of a
⎛ 1⎞ ⎛3 ⎞
V = ⎜ ⎟ (4 in.2 )⎜ in.⎟ rectangle are congruent, so we only need to check two
⎝ 3⎠ ⎝4 ⎠
ratios:
V = 1 in.3
2. Depth is the word we use for height when the DC
FT = 106 = 35 but BC
RF = 38
cone is upside down. The base is a circle, so
B = πr2, but the radius is not given. The radius,
Because 53 ≠ 38, these rectangles are not similar.
height, and slant height always form a right
triangle, so we can use the Pythagorean
Exercise 5
theorem to find r2.
Although these sides are in proportion (105 = 21 and
a2 + b2 = c2 8 2
4 = 1, the angles are not congruent, so the polygons
r2 + 42 = 52
are not similar.
r2 + 16 = 25
r2 = 25 – 16
r2 = 9
V = 13Bh
V = 13πr2h
V = 13 (3.14)(9)(4)
V = 37.68 cm3

181
8th_GRDM_09.qxd:Layout 1 8/11/09 1:53 PM Page 182

–VOLUME AND SIMILAR SHAPES–

Exercise 6 Exercise 7
When the problem is as vague as this one, we assume The smallest sides of the two triangles are 15 and 6, so
that what looks related is related. So z = 90 as they are the scale factor is 156 = 25. The perimeter of the larger
the two largest angles, and w = 30 as these are the two triangle is 20 + 15 + 25 = 60. Since the ratio of the
smallest angles. Because the sides are proportional: perimeters is equal to the scale factor,

Smallest Medium Largest Perimeter of larger triangle 60 5


sides sides sides = =
Numerators are Perimeter of smaaller triangle x 2
6 8 x sides of first triangle.
– = – = —–
9 y 15 Denominators are So x = 60(2)
5 , and x = 24.
sides of second triangle.

Proportions are solved two ratios at a time (see Les- Exercise 8


son 9), so we have three choices: Because the floor of the building will be similar to the
picture on the blueprint, the scale on the blueprint is
6 8 8 x 6 x
= , = , and = 1
the scale factor. So the scale factor is 10 and (scale fac-
9 y y 15 9 15 2 1
tor) = 100.
You can’t solve the second equation, but you have
Area on the blueprint 12 square inches
your choice of the first or third. So let’s begin by solv- =
ing for y using the first equation: Area of the room in thee building x square feet

6 8 And the scale factor squared is


=
9 y
12 square inches 12 1
y = 9(8) = 2=
x square feet 10 100
6
y = 12 Then 12x = 100
1
, so x = 120 square feet.

Once we have y, we need to choose the next propor- Exercise 9


tion. If we choose the second proportion and our 67.5 cm
value for y is incorrect, our value for x will also be in- First, find the scale factor: 43. Next, set the ratios of
correct. Choose the third proportion to solve. the volumes equal to the cube of the scale factor.
6 x
= Volume of larger pyramid 160 4 3
9 15 = = 3
Volume of smaller pyyramid x 3
x = 15(6)
9 160 64
=
x = 10 x 27
64 x = 27(160)
Finally, check to see that the ratios of the correspon- 27(160)
ding sides are all equal: 96 = 23, 12
8
= 23, and 10 2 x=
15 = 3. This in- 64
dicates that our computations are correct. x = 67.5
Because the height of the smaller pyramid is in cen-
timeters, its volume is 67.5 cm3.

182
8th_GRDM_10.qxd:Layout 1 8/11/09 1:54 PM Page 183

20 COORDINATE
L E S S O N

GEOMETRY—
DISTANCES

L E S S O N S U M M A RY
You were introduced to the coordinate system in Lessons 10 and 12.
Now you’ll learn how to find distances from one point to another
with the Pythagorean theorem, which is one of the building blocks
of the math that you’ll learn in high school. You’ll also learn how to
compute the coordinates of the midpoint of a line, and you’ll be
able to tell what kind of shape a given set of points describes.

R ené Descartes was a soldier and mathematician. One of the problems in war is figuring out where
the enemy is and how long it takes to get there. Descartes solved the problem by inventing a new
branch of mathematics, coordinate geometry, a tool that changed map making and warfare forever.
Coordinate geometry enables us to compute distances (a tool also vital to navigation systems) as well as areas
and volumes.

183
8th_GRDM_10.qxd:Layout 1 8/11/09 1:54 PM Page 184

–COORDINATE GEOMETRY—DISTANCES–

A right triangle has one right angle that is This led to the following theorem.
formed by two sides called legs. The third and longest
side is the hypotenuse.
THE PYTHAGOREAN
Hypotenuse THEOREM

If a and b are the lengths of the legs of a right


triangle and c is the length of the hypotenuse,
then
a2 + b2 = c2 or
Legs
(first leg)2 + (second leg)2 = (hypotenuse)2

The Pythagorean Theorem


Here’s an example. Find x in this triangle:
In this section we will see how the Pythagorean theo-
rem helps us to find lengths in the coordinate plane. 5
The Pythagorean theorem was known to builders in
12
ancient Egypt and Greece. Builders and architects still
use it today. It is surprising how many professionals
x
use the theorem. For example, interior decorators—
persons you might not expect to use much math—
use the theorem to place lights in offices. Because 12 and 5 are legs and x is the hypotenuse:
The ancient Egyptians and Greeks noticed that
if you build a square on each side of a right triangle (first leg)2 + (second leg)2 = (hypotenuse)2
and divide each square into square units, the sum of 52 + 122 = x2
the areas of the squares built on the legs equals the 25 + 144 = x2
area of the square built on the hypotenuse. 169 = x2
x = ± 169 = ±13

Therefore, x = 13, because lengths are always positive,


and the hypotenuse is 13.

Exercise 1
Find x. Answer is on page 188.

25 cm
x

20 cm
9 + 16 = 25

184
8th_GRDM_10.qxd:Layout 1 8/11/09 1:54 PM Page 185

–COORDINATE GEOMETRY—DISTANCES–

The Distance Formula As an example, find the distance from –125 to 37 on


the number line.
The distance from A to B is the shortest possible path
from A to B. We abbreviate the distance from A to B Distance = |–125 – 37| = |–162| = 162
as AB.
Did that surprise you? Here’s a sketch to verify the
B A
answer.
–2 –1 0 1 2 3 4 5 6 7 8 9 10
125 units + 37 units = 162 units
What is the distance from A to B on the number line?
Method 1: Just count the units from A to B. –125 0 37

B7 6 5 4 3 2 1 A This formula is very useful in coordinate geometry.


–2 –1 0 1 2 3 4 5 6 7 8 9 10 y

There are 7 units from A to B. Notice we counted (3,5)


y
jumps, not marks, on the number line.
This method works well if A and B are close to-
(3,5)
gether, but what if they are far apart? Some people (0,0) x 5
x
or
will visualize the picture and get the answer, but visu- –3 –2 –1 0 1 2 3 4 0 3

alization is hard for many, so mathematicians devel-


oped a formula. Here is how they probably did it:
(3,–25)
Method 2: The distance from A to B is –25
(3,–25)

AB = 9 – 2 = 7

If the problem had asked us to find the distance


In this figure, how far apart are points A and B?
from B to A, some may have worked it like this: 2 – 9
Points A (3,5) and B (3,–25) are on the same vertical
= –7. Since distance is never negative, we would have
line. Only the y-coordinate is changed. We can use the
to erase the negative sign. Mathematicians have a way
formula for the distance between points on a line.
to remind people to erase the negative sign. It is called
absolute value. (See Lesson 1.) AB = |y1 – y2|
Since |9 – 2| = |7| = 7, |2 – 9| = |9 – 2|, and it AB = |5 – (–25)| = |30| = 30 units
does not matter which number is written first within
the absolute value sign. We have the following rule. Now let’s see how to compute the distance of
two points that are on the diagonal by developing a
formula.
The distance between two points is the short-
est positive distance from one point to the
other.
AB = |coordinate of A – coordinate of B|
or |x1 – x2| where x1 is the coordinate of A
and x2 is the coordinate of B.

185
8th_GRDM_10.qxd:Layout 1 8/11/09 1:54 PM Page 186

–COORDINATE GEOMETRY—DISTANCES–

A A
(x1,y1) (x2,y2)

B I B
(x2,y2) (x1,y2) (x2,y2)

To find the distance from A to B, draw a right triangle.

We will label the vertex of the right angle as I. Do you The Midpoint of a Segment
see that the coordinates of I are (x1, y2)? By the
Pythagorean theorem, The midpoint of a segment is the point exactly in the
middle of the segment. In this figure M is the mid-
(AI)2 + (IB)2 = (AB)2
point of segment AF if AM = MB.
Since A and I are on the same vertical line, AI = A M B
|y1 – y2| and IB = |x1 – x2|, so

Midpoint
( y1 − y 2 )2 + (x1 − x 2 )2 = AB
The Coordinates of the Midpoint
Are you wondering what happened to the absolute
The coordinate of A is 15 and the coordinate of B is 37.
value signs? Because squaring also produces only pos-
itive numbers, we no longer need them to remind us A M B
to erase any negative values in the answer! 0 15 37

THE DISTANCE FORMULA Finding the coordinate of the midpoint, M, is exactly


the same as finding the number halfway between 15
The distance from points A (x1,y1) to B (x2,y2) is and 37. To do this, we find the average of 15 and 37:
given by the formula Add and divide by 2. So the coordinate of the mid-
AB = ( y 1 − y 2 ) + ( x1 − x 2 )
2 2 point = 15 +2 37 = 522 = 26.
To check, find AM and MB. If our coordinate is
right, AM should equal MB. AM = |26 – 15| = 11 and
MB = |37 – 26| = 11. This demonstrates that M is ex-
Exercise 2 actly in the middle of A and B.
(Answers are on page 188.)

1. Find the distance between (4,3) and (–5,15).

2. Find the distance between (–6,2) and (4,–2).


186
8th_GRDM_10.qxd:Layout 1 8/11/09 1:55 PM Page 187

–COORDINATE GEOMETRY—DISTANCES–

THE COORDINATE OF Using Slope to


THE MIDPOINT ON Identify Quadrilaterals
THE NUMBER LINE
You learned in Lesson 12 that slope is the measure of
If two points have coordinates x1 and x2 on the the slant of a line. Horizontal lines have zero slant, so
number line, the coordinate of the midpoint is the slope of a horizontal line is zero. Vertical lines
the average of x1 and x2: have infinite slope, and no number reaches infinity,
x +x so we say they have no slope. The slope of all other
xm = 1 2
2 lines is computed using the formula
y 2 − y1
slope =
This idea can be extended to find the midpoint of a x 2 − x1
segment on the coordinate plane. Parallel lines never cross, so they must have ex-
actly the same slope. As you learned in Lesson 13,
THE COORDINATES OF parallel lines have equal slope.
THE MIDPOINT OF Perpendicular lines are lines that cross to form
ANY SEGMENT right angles. The slopes of perpendicular lines are op-
posite reciprocals. In other words, if the slope of a
The coordinates of the segment with endpoints line is ba, then the slope of a line perpendicular to it is
(x1,y1) and (x2,y2) are (xm,ym ), where – ab. There is one exception to this rule: Horizontal
x +x y + y2 lines, even though they have zero slope (and zero has
x m = 1 2 and y m = 1
2 2 no reciprocal), are perpendicular to vertical lines.
We can use information about slope and dis-
tance to identify quadrilaterals when we know only
Exercise 3 the coordinates of their vertices. Try the following ex-
Find the midpoint of the segment joining (2,7) and ercise; give it a good try, based on what you know, be-
(4,1). fore looking at the solution.

y
Exercise 4
The coordinates of quadrilateral ABCD are A(5,4),
7
(2,7) B(7,–2), C(–2,–5), and D(–4,1). Is this quadrilateral a
6 parallelogram, rectangle, square, rhombus, trapezoid,
5
4 m (xm, ym) isosceles trapezoid, or none of the above?
3
2
(4,1)
1 x
1 2 3 4 5
Summary

In this lesson you have learned some useful methods


for finding out information about lines on graphs us-
ing the Pythagorean theorem. You have also learned
to apply what you have learned in Lessons 17 and 18
to the coordinates of a graph.

187
8th_GRDM_10.qxd:Layout 1 8/11/09 5:07 PM Page 188

–COORDINATE GEOMETRY—DISTANCES–

Answers Exercise 4
First, draw a quick sketch to see how the points are
Exercise 1 arranged. Sometimes, the answer will be clear from
15 cm your picture alone. This sketch tells us the sides are
The legs are x and 20, and the hypotenuse is 25. AB, BC, CD, and DA.

(leg)2 + (leg)2 = (hypotenuse)2


x2 + 202 = 252 A
(5,4)
x2 + 400 = 625
x2 = 225 (–4,1)
x = ± 225 D

x = 15
B
(7,–2)
Exercise 2 (–2,5)
1. 15 C
Use distance = ( y1 − y 2 )2 + (x1 − x 2 )2 :
(3 − 15)2 + (4 − −5)2 = (−12)2 + (9)2
= 144 + 81 Next, use slope to decide if the quadrilateral is a par-
allelogram, a trapezoid, or an ordinary quadrilateral.
= 225
A parallelogram has two pairs of parallel sides, and a
2. 2 29 trapezoid has exactly one pair of parallel sides. Some
Use distance = ( y1 − y 2 )2 + (x1 − x 2 )2 : quadrilaterals have no parallel sides.
(2 − − 2)2 + (−6 − 4)2 = (4)2 + (−10)2 Here are the slopes of all four sides:

= 116
Slope of AB = 45–––2
7 = –3
= 4 × 29 Slope of BC = 7 – –2 = 31
–2 – –5

= 2 29 –5 – 1
Slope of CD = –2 – –4 = –3
Slope of DA = 5 – –4 = 31
4–1

Exercise 3
The midpoint is (3,4) Since we have two pairs of sides parallel, we have a
x1 + x 2 y1 + y 2 7 + 1 parallelogram. Also, the slopes are –3 and 31 (opposite
2+4
xm = 2 = 2 = 3; y m = 2 = 2 reciprocals), so the nonparallel sides are perpendicu-
= 4. lar, indicating that the shape is also a rectangle.

188
8th_GRDM_10.qxd:Layout 1 8/11/09 5:07 PM Page 189

–COORDINATE GEOMETRY—DISTANCES–

But could it be a square? To be a square it Because 40 ≠ 90 , we don’t need to simplify the


needs to have four equal sides. We can use the dis- answers. We know that AB is not the same length as
tance formula to see how long the sides are. Since BC, and the quadrilateral cannot be a square. There-
the opposite sides of any rectangle are equal, we fore, it’s a rectangle.
only have to find the length of two nonparallel If we had used graph paper to make our original
sides. Let’s find AB and BC, using the distance for- sketch, we might have detected that the shape was not
mula, distance = ( y1 − y 2 )2 + (x1 − x 2 )2 : a square without solving the last two problems. In
Lesson 22 you’ll see how to improvise very accurate
AB = (5 − 7)2 + (4 − −2)2 BC = (7 − −2)2 + (−2 − −5)2 graph paper and find an efficient strategy for answer-
AB = (−2)2 + (6)2 BC = (9)2 + (3)2 ing questions like these.

AB = 40 BC = 90

189
8th_GRDM_10.qxd:Layout 1 8/11/09 1:55 PM Page 190
8th_GRDM_11.qxd:Layout 1 8/11/09 1:56 PM Page 191

21
L E S S O N

ESTIMATING
AND CHECKING

L E S S O N S U M M A RY
In this lesson you’ll learn about making estimates for geometry.
Many of these estimates involve drawing a picture, but some just
involve straight calculation. You’ll get tips on how to think about
scale factor problems and on how to solve similarity problems
quickly and accurately.

S ome people finish tests more quickly than others. Sometimes, it is because they are really smart or have
worked really hard to learn the material. Sometimes, they are good test takers who have learned tech-
niques designed to solve problems using few steps. The purpose of this lesson is to alert you to some
typical errors in geometry so that you can avoid them, to teach some shortcut methods of solving certain geom-
etry problems, and to show ways of checking others.

191
8th_GRDM_11.qxd:Layout 1 8/11/09 1:56 PM Page 192

–ESTIMATING AND CHECKING–

Estimate the Answer It looks like the area is six times larger. Problem
solved—though you still can check with some made
Estimating and checking are closely related. Estimat- up dimensions. Let’s say a garden is 4 feet long and
ing can be a form of checking or a process used when 2 feet wide (area = 8 ft.2). Then doubling the length
only a good guess is needed. and tripling the width will give you an 8 × 6 garden:
Estimating an answer saves time even if you use area = 48 ft.2, which is 6 × 8.
a calculator, since fewer key strokes are needed. There are limits to what drawing a picture will
If a question requests an exact answer, the estima- do. It doesn’t always work to give the final answer. If
tion will help you check a decimal point error. For ex- you had to double the radius of a circle, the picture
ample, a good estimation of 100 warns you that an wouldn’t help except to perhaps estimate the area. It
answer of 1,016.18 is too large and 10.1618 is too small. looks in this figure like three or four small circles
would fit in the larger one.
Exercise 1
The area of a circle with a radius of 5.8 inches is
about

a. 30 inches r 2r
b. 75 inches
c. 110 inches
d. 125 inches

Draw a Picture Guess and Check

Sometimes, a picture is the answer to the problem. This is how you might use guess and check to solve
For example: the problem of doubling the radius.

Double the radius of a circle. What change will


The Smiths have decided to enlarge their
occur in the area?
rectangular garden. They plan to double the
length and triple the width. What change will Pretend the radius of the small circle is 3. Then
occur in the area? the radius of the large circle is 6. Now, find the areas
of both circles.
Carefully sketch two pictures, one of the original
shape and the other with the length doubled and the
width tripled. It doesn’t matter what the original
length and width are, and the problem doesn’t tell
you, anyway. 3 6

3w

w
A = π32 = 9π
l
2l A = π62 = 36π = 4(9π)
192
8th_GRDM_11.qxd:Layout 1 8/11/09 1:56 PM Page 193

–ESTIMATING AND CHECKING–

So the new circle is 4 times larger than the original Exercise 2


circle. In the following figure, the triangles are similar. Set
Of course, the substitution of just one number up proportions to find the values of x and y.
isn’t always enough to prove that you got the right an-
swer. If you’re uncomfortable with this solution, you
8 6
can check it another way by using 3 for π to get:

y
A = π32 = 9π ≈ 9 × 3 ≈ 27
A = π62 =36π ≈ 36 × 3 ≈ 108 9 x

Since 108 ÷ 27 = 4, the new circle is 4 times larger


than the original circle. 15

Check by Using Look for Multiples and


an Alternative Method Common Factors When
Doing Proportions
Since all circles are similar to one another, we can use
scale factor to solve the radius-doubling problem.
When triangles are similar, there are often common
Original radius r 1r 1 factors to make your work easier. For example, we
= = = could rewrite the proportion from Exercise 2 by re-
New radius 2r 2r 2
ducing 69 to 23. This shows us that the scale factor is 23.
So the scale factor is 12.
The 3 in the numerator means that all three numbers
We can use what you learned in Lesson 19 about
in the numerator are multiples of 3, and the 2 in the
the scale factor:
denominator means that all three of the numbers in
Original radius r 1r 1 the denominators are multiples of 2. So we get:
= = =
New radius 2(2r ) 4r 4 3 3(5) 3(?)
= =
So the new area is four times as large as the original 2 2(?) 2(4)
area.
For all these fractions to reduce to 23, the ? in the
second fraction must be 5, and the ? in the third frac-
tion must be 4:
Keep the Numbers in Order
When Writing Proportions 3 3(5) 3(4)
= =
2 2(5) 2(4)
You need to be very careful when you are matching
Going back to the original proportion (see the answer
the numbers in a proportion, because if it is set up in-
to Exercise 2), y = 2(5) = 10 and x = 3(4) = 12. With a
correctly, every answer will be wrong. Here’s an ex-
ample using similar triangles that could easily go
wrong.

193
8th_GRDM_11.qxd:Layout 1 8/11/09 1:56 PM Page 194

–ESTIMATING AND CHECKING–

bit of practice, this technique is faster than solving


the proportions. Even if you decide to solve the pro-
portions, remember this: Every fraction in the pro- 1 2
portion reduces to the scale factor. So, check your 4 3
work by reducing the fractions. If they reduce to the 5 6
same number, your work is probably correct. 8 7

Exercise 3
Donna says she found the answers to a problem Exercise 4
about similar triangles and the sides were 10, 26, 22 In a figure, the parallel lines are cut by a transversal. If
for one triangle and 15, 36, 39 for the other. Is she ∠1 is 125, how large is ∠5?
correct? Answer is on page 196. a. 50
b. 60
c. 55
Use This Picture When d. 125
Dealing with Parallel Lines The parallel lines are cut by a transversal. If m∠5 +
m∠7 = 88°, what are m∠1 and m∠2?
The picture that was used in Lesson 17 is a good one
to draw anytime you have parallel lines cut by a trans-
versal. The picture can help you estimate and help Know Your Pythagorean Triples
you to check. Unless a transversal cuts the parallel
lines at right angles, any angle whose measure is >90 To keep the arithmetic simple, test writers use the
is drawn as one of the obtuse angles: 1, 3, 5, or 7. And same numbers over and over. It pays to look for them.
those angles are all equal to one another. Their sup- The usual suspects are the 3-4-5 right triangle and the
plements—2, 4, 6, and 8—are also equal to one an- 5-12-13 right triangle. Because 32 + 42 = 52 and 52 + 122
other. If you get a number larger than 90 for any of = 132, we say (3,4,5) and (5,12,13) are Pythagorean
the acute angles, you know you should look for an er- triples. It is also true that all multiples of Pythagorean
ror in your computation. triples are Pythagorean triples. Why? Because if all the
sides are multiples, the triangles must be similar. (Re-
view Lesson 19 if you’re not sure about this.)
When right triangles appear in the picture, look
for Pythagorean triples to save time. But be careful to

PYTHAGOREAN TRIPLES

(3, 4, 5) (5, 12, 13)


(2 × 3, 2 × 4, 2 × 5) = (6, 8, 10) (2 × 5, 2 × 12, 2 × 13) = (10, 24, 26)
(3 × 3, 3 × 4, 3 × 5) = (9, 12, 15) (3 × 5, 3 × 12, 3 × 13) = (15, 36, 39)
(4 × 3, 4 × 4, 4 × 5) = (12, 16, 20) (4 × 5, 4 × 12, 4 × 13) = (20, 48, 52)

194
8th_GRDM_11.qxd:Layout 1 8/11/09 1:56 PM Page 195

–ESTIMATING AND CHECKING–

remember that the multiples of 3 and 4 are the legs, Summary


and the multiples of 5 are the hypotenuse!
Note, however, that distances obtained from the In this lesson you have learned some quick ways to
distance formula will often have the square root of a estimate and check problems in geometry. These in-
number that’s not a perfect square, such as 5 . You clude drawing pictures, substituting values to solve a
can’t count on test writers to always give you answers general problem, labeling triangle sides to keep you
that are integers, especially if they let you use calcula- from solving similarity problems incorrectly, and
tors on the test. learning some Pythagorean triples.

Exercise 5
Answers
1. Find the third side of the right triangle. Answer
is on page 196. Exercise 1
c. The exact answer is not required, so use 3 for π
and 6 for the radius. The formula is A = πr2.
A = 3 × 62
A = 3 × 36
300
A = 108, so the correct choice is c.

400

2. Find the third side of the triangle. Answer is on


page 196.

400
300

3. Find the third side of the triangle. Answer is on


page 196.

35
28

195
8th_GRDM_11.qxd:Layout 1 8/11/09 1:56 PM Page 196

–ESTIMATING AND CHECKING–

Exercise 2 Exercise 4
You will get this one wrong if you set up your propor- The answer to the first part is c. With parallel lines we
tion for x as know that m∠1 = m∠3 = m∠5 = m∠7 and that
m∠2 = m∠4 = m∠6 = m∠8. Also, all the small and
6 8
x=9 large angles add to 180. This means we can rule out
125 as too large, and we can rule out 50 and 60, since
Why? Because x in the first triangle isn’t the shortest adding them to 125 will result in a number ending in
side, but 6 in the second triangle is the shortet side. 5, not 180.
Choose one triangle for all the numerators and the The answers to the second part are 136° and 44°.
other for the denominators, and each fraction should Because m∠5 and m∠7 are equal, we have 2(m∠5) =
match a pair of corresponding sides. Sometimes, it 88, and m∠5 = 44. That means that m∠2 is also 44.
helps to mark the sides with the words small, Then m∠1 = 180 – 44 = 136.
medium, and large.
Exercise 5
8
1. 500
m 6
Sm

iu The multiples of 3 and 4 on the legs indicate


ed
al

M
l

Large the possibility of a 3-4-5 right triangle. Since


y
300 = 3(100) and 400 = 4(100), then the
9 Me x hypotenuse must be 5(100) = 500. This is
l

diu
al
Sm

m much faster than using the Pythagorean


Large theorem!
15 2. 100 7
Here the hypotenuse is a multiple of 4, not 5, so
Then write the proportion.
we can’t use the Pythagorean triple. However,
we can take advantage of the multiple of 100
Smallest Largest Medium
and just work with 3 and 4 and then multiply
sides sides sides
the answer by 100. Using the Pythagorean
Sides of larger triangle → 9 15 x
= = theorem, and letting a = the length of the
Sides of smaller triangle → 6 y 8
missing side,
42 = a2 + 32
Exercise 3 42 – 32 = a2
Carefully arrange the numbers in the proportion. a= 7
Because the other sides were 3 × 100 and
Smallest Largest Medium 4 × 100, the missing side is 100 7 .
Fiirst triangle → 10 26 22 3. 21
Second triangle → 15 = =
39 36 Since 35 = 7(5) and 28 = 7(4), think 3-4-5
right triangle. The missing side is a leg, so 7(3)
18 ≠ 3, one of the numbers 22 or 36 is incorrect.
Since 11 2

(Or it could be that all the other numbers are incor- = 21 is the answer.
rect, if she’s sure that 22 and 36 are correct.)

196
8th_GRDM_12.qxd:Layout 1 8/11/09 1:57 PM Page 197

22
L E S S O N

STRATEGIES

L E S S O N S U M M A RY
In this lesson you’ll learn something about how to apply geome-
try formulas to certain word problems, and you’ll also learn how
to start solving two-step problems involving geometry. You’ll
also get some tips about how to start solving geometry prob-
lems in general.

A rchitects, carpenters, bricklayers, carpet layers, city planners, park service personnel, landscape ar-
chitects, and many others encounter problems involving area, perimeter, and volume. For example,
when a lawn service wants to provide a customer with an estimate of the cost of applying fertilizer
or other products to a lawn, it has to compute the area of the lawn. Suppose a prospective customer requests an
estimate. The company will send someone to produce a diagram of the lot including measurements.

197
8th_GRDM_12.qxd:Layout 1 8/11/09 1:57 PM Page 198

–STRATEGIES–

The diagram might look like this: shapes are in the picture? If the picture is tiny,
150'
draw a larger copy. If there is no picture, use the
Tree
words of the problem to draw one.
2. Read the problem carefully. Look for words
10'
Garden
such as area, perimeter, volume, triangle, rectan-
70'
80' gle, and supplementary. Use these words along
20' House
with the picture to choose a formula.
Driveway 3. Define your variables. Find out what you do
30'
know and what you are asked for; then define
18'
80' those values with variables.
Notice that the lot is a bit irregular, so perhaps 4. Write down both sides of the formula (or
the customer lives at the end of a cul-de-sac. In this equation) using variables. Remember that
problem there are a trapezoid, two rectangles, a circle, some problems are much easier to work if both
and a semicircle, so this is clearly an area problem. sides of the formula are written down.
One plan (not the only one) for finding the area of 5. Replace the variables in the formula with
the lawn would be to find the area of the lot and sub- numbers. Mark them in the picture and plug
tract the areas that are not lawn. Here is a diagram of them into the formula. Use the picture and the
the plan: words of the problem to find values for the vari-
ables. Some values will be easy to find. Others
Lot House Driveway Patio Garden will require you to look for clues. Are there
boxes indicating right angles or slash marks to
A= – – – –
indicate that a pair of sides or angles are equal?
A=  h(b1 + b2) – bh – bh – ? – πr2 Look for key words (like isosceles, equilateral, bi-
A= (80)(150 + 80) – 70 × 20 – 18 × 30 – ? – ?
sector, vertical angles) that tell us certain things
A= 9,200 – 1,400 – 540 – ? – ?
in the picture are equal.
In this lesson we will discuss some basic strategies 6. Solve the equation using algebra.
that are useful in solving a variety of problems, and 7. Make sure you actually answered the question.
along the way we’ll solve this one as well. If possible, check your answer by plugging it
back into the facts from the original problem. If
that is not possible, at least make sure that your
Problems Involving Formulas answer is sensible.
and Equations from Geometry

The following strategies are helpful in solving prob- Two-Step Problems


lems involving area, perimeter, volume, and other
problems involving formulas or equations. Can you Step 6 can be dangerous to omit. Standardized tests
identify where we have already used the first two often require you to do more than one operation be-
strategies in the preceding area problem? We’ll do fore choosing your answer—and at least one of the
steps 4 through 6 in other parts of this lesson. wrong answers is the answer to the first step. It’s
sometimes very easy to choose the correct formula,
1. Study the picture—or draw your own. Are any solve it correctly, and find the value of x in the set of
items marked with slashes or arcs to show they choices. But if it’s a two-step problem, something
are equal? Are measurements given? What more is required, and if you don’t notice that, you will
198
8th_GRDM_12.qxd:Layout 1 8/11/09 1:57 PM Page 199

–STRATEGIES–

get the wrong answer. Exercise 1 shows how this Bushes and Flowers
could happen.
There is another type of two-step problem. It Tree

involves solving a first equation in order to find the


data to solve a second equation. When you’re working
with perimeter, area, or volume, the first equation
will often involve the Pythagorean theorem or even
another formula for area, perimeter, or volume. Exer-
cise 2 demonstrates a problem of this type. C = 19'
In Exercise 3, you’ll begin to solve the lawn
estimate.
Other Tips to Remember
Exercise 1
■ If possible, use the formula, not the method
x + 20 used to develop the formula. The point of for-
mulas is to put you into autopilot. Plug in the
numbers and crank out the answer. (But if you
forget the formula, and it’s not given to you for
2x x
the test, you should be able to go back and fig-
ure it out.)
Find the measure of the largest angle of the triangle.
■ Draw a picture when none is provided. Some-
Answer is on page 201.
times the picture alone provides the answer. At
the least, it is a visual reference when you are
a. 40
finding distances and computing slopes.
b. 60
■ Things that look the same length on a test or on
c. 80
a homework assignment may not actually have
d. 100
the same length, and things that look unequal
e. none of these
may actually be equal. They may be optical il-
lusions. Never say two things are equal unless
Exercise 2
you have a rule or some other information to back
Suppose a lot is an isosceles trapezoid with dimen-
it up.
sions of 60′, 50′, 120′, and 50′. Find the area of the lot.
■ If necessary, make your own graph paper. Draw-
Answer is on page 202.
ing graphs without graph paper makes it diffi-
cult to come to a conclusion. Hold two sheets of
Exercise 3
lined paper to create workable graph paper. Of-
In the problem at the start of this lesson the landscape
ten a problem is more easily solved with a graph
architect needs to find the area of the circle garden.
than with computation.
Because a large tree is located in the center of the cir-
■ Follow an organized procedure when you are
cle, she was unable to find the radius or the diameter,
asked to identify a quadrilateral by using the co-
but she was able to measure the circumference of the
ordinates of its vertices.
circular garden surrounding the tree. It was 19′. What
is the area of the circle, to the nearest square foot? An-
swer is on page 202.

199
8th_GRDM_12.qxd:Layout 1 8/11/09 1:57 PM Page 200

–STRATEGIES–

Remember from Lesson 18 that a polygon Exercise 4


shares all the properties of every family to which ABCD is a rhombus with diagonals AC and DB. Line
it belongs. Refer to the chart in Lesson 18 if you TB is 5 units in length. How long is DB? Answer is on
have forgotten. page 202.
■ When asked to find areas of a fraction of circles,
find the area of the whole circle and then multi- Exercise 5
ply by a fraction.
A B

Draw a picture. 5
The shape may be obvious.
List the 4 sides.
T

Find the slope of the 4 sides.

D C
Two pairs of Exactly one pair No parallel
parallel sides of parallel sides means it Rhombus ABCD
means it is a sides means is an ordinary
parallelogram. it is a trapezoid. quadilateral.
The patio of the house in our problem is a semicircle
with a diameter of 10′. Find the area of the patio
Are the slopes Use the distance formulas.
negative reciprocals? Are the legs equal? rounded to the nearest square foot. (Note: a semicircle
yes no yes no
is half of a circle.)
It might It is an It is an
It is a
be a isosceles ordinary
rectangle.
rhombus. trapezoid. trapezoid.

Use the Use the


distance distance
formula: formula:
10' 10'
Are 2 of the Are 2 of the
non-parallel non-parallel
sides equal? sides equal?
yes no yes no

It is an
It is a It is a It is a
ordinary
square. rectangle. rhombus.
parallelogram.
Irregular Shapes

Finding the area of an irregular shape requires divid-


ing the shape into smaller polygons with known for-
mulas and then adding (or subtracting) the areas of
these smaller pieces.
This is exactly what we are doing in the lawn
problem. Using the information from Exercise 3 and

200
8th_GRDM_12.qxd:Layout 1 8/11/09 1:57 PM Page 201

–STRATEGIES–

Exercise 4, we can now solve the landscape architect’s Answers


problem.
Use the formula for the area of a trapezoid to Exercise 1
find the area of the whole lot: The correct answer is c. Since the sum of the angles of
a triangle is 180, add the three angles together to get
A = 12(80)(80 + 150) = 40(230) = 9,200 ft.2 180:
2x + x + (20 + x) = 180
Use the formula for the area of a rectangle to find the 4x + 20 = 180
area of the house and the driveway: 4x = 160
x = 40
A (house) = 70(20) = 1,400 Notice that 40 is one of the choices. If you stopped
A (drive) = 30(18) = 540 here and selected choice a, you would miss the prob-
Lot House Driveway Patio Garden lem. The question was, “Find the measure of the
largest angle.” To decide, plug the 40 in for the x in
A= – – – –
the expression for each angle:
A= 9,200 – 1,400 – 540 – 39 – 29 x = 40
2x = 2(40) = 80
A (total) ≈ 7,192 square feet
x + 20 = 40 + 20 = 60
The correct answer is 80, choice c. The key to avoid-
ing this problem is to go back once you have solved
Summary
for x and ask yourself, “What is x, and is it what I was
asked to find, or do I have to do something more?”
In this lesson you solved an area problem that en-
compassed several general rules about solving geom-
etry problems. It was a multistep problem that
required an organized plan of attack and the correct
application of formulas—things you need to keep in
mind when you’re trying to find the answers in
geometry.

201
8th_GRDM_12.qxd:Layout 1 8/11/09 1:57 PM Page 202

–STRATEGIES–

Exercise 2 Exercise 3
2
3,600 ft. To find the area of the circle, you need the radius of
First draw a picture of the lot. the circle, but you only know the circumference. First,
use the circumference formula to find the radius, and
D 60' A D 60' A then find the area.
C = 2πr
50' 50'
50' 50'
19 = 2πr
19
2π = r
C 120' B C 30' F 60' E 30' B 3.024 = r
Remember to round to more places than your final
Since it is an isosceles trapezoid, the two unequal sides
answer.
are the parallel bases and the two equal sides are the
A = πr2
legs. The formula for the area of a trapezoid is A =
1
A = π(3.024)2
2h(b1 + b2). Here, b1 = 60, and b2 = 120, but the height A = 28.73
is unknown. To find the height, draw two vertical seg-
Rounded to the nearest square foot, the area of the
ments as shown in the second picture to create congru-
circle is about 29 square feet.
ent right triangles ΔCDF and ΔAEB and rectangle
AEFD. Since the opposite sides of a rectangle are equal,
Exercise 4
FE = 60 and CF = EB = 120 –2 60 = 30. Now, use the
10 units
Pythagorean theorem to find DF, which is h, the height
Look at the diagram in Lesson 18. None of the
of the trapezoid.
properties listed for a rhombus apply, so go up the
c2 = a2 + b2
chart. A rhombus is also a parallelogram. One of
502 = 302 + h2
the properties of a parallelogram is that the diago-
2,500 = 900 + h2
nals bisect each other. This means that DT = TB = 5.
1,600 = h2
Since 5 + 5 = 10, DB = 10.
±40 = h
Since lengths are positive, h = 40. So the height is 40′.
Exercise 5
(Or you could recognize the Pythagorean triple 30,
39 ft.2
40, 50 and say the height is 40′.) Next,
The diameter of the semicircle is 10′, so the radius is
A = 12h(b1 + b2)
5′. Use the area formula to find the area of a circle
A = 12(40)(60 + 120)
with a radius of 5 feet and take half of that. We can do
A = 20(180)
this in one step.
A = 3,600 square feet
A (of semicircle) = 12 × area of circle = 12πr2
= 12π(5)2
= 39.26991
So the area of the patio is about 39 square feet.

202
8th_GRDM_13.qxd:Layout 1 8/11/09 1:58 PM Page 203

23 PUTTING
L E S S O N

IT ALL
TOGETHER

L E S S O N S U M M A RY
This lesson pulls together what you have learned about geometry
in Lessons 17 through 22. You’ll use that knowledge to answer
problems and to write short and long responses that explain how
you solved those problems.

Y ou have now learned some of the basics of geometry that you’ll need to be successful in eighth grade.
You have also learned strategies for solving word problems based on real-life situations. Now it’s time
to see how well you know them.

What You Have Learned

Here’s a quick summary of the lessons in this section.


Lesson 17: Angles and Triangles. In this lesson you learned definitions for angles and triangles. You also
learned the parts of both angles and triangles, and you learned how to find missing angle measures given infor-
mation about their supplements and complements.

203
8th_GRDM_13.qxd:Layout 1 8/11/09 1:58 PM Page 204

– PUTTING IT ALL TOGETHER –

Lesson 18: Polygons and Circles. You learned Practice


about the most common types of polygons, the
quadrilaterals, as well as other types of polygons. You Short Answers
learned how to find missing lengths and angles, as Most of the short answers in this section are multiple-
well as perimeters and areas. You also learned about choice. That doesn’t mean that they’re easy, though.
the perimeters and areas of circles and partial circles. Answers begin on page 210.
Lesson 19: Volume and Similar Shapes. In this
lesson you learned about the volumes of solids, such as 1. The two parallel lines are cut by a transversal. If
regular pyramids, prisms, and cones. You also learned angle 6 is 70 degrees, find the measure of angle 3.
about similar shapes and how to apply proportions
and scale factors to find unknown dimensions.
Lesson 20: Coordinate Geometry—Distances.
You learned about the Pythagorean theorem and how 1 2
4 3
to use it to find distances between points. You also
learned how to find the midpoint of a line segment 5 6
8 7
given its endpoints, and you learned some of the geo-
metric properties of slopes.
Lesson 21: Estimating and Checking. You
learned some methods for estimating solutions for a. 70°
and checking your answers to word problems involv- b. 100°
ing concepts of geometry. c. 110°
Lesson 22: Strategies. In this lesson you solved d. 125°
a multistep problem that required an organized plan e. 200°
of attack and correct application of formulas—things
that you need to keep in mind when you’re trying to 2. Find the measure of angle B.
find the answers in geometry.
100°

110°

a. 30
b. 60
c. 75
d. 100
e. 110

204
8th_GRDM_13.qxd:Layout 1 8/11/09 1:58 PM Page 205

–PUTTING IT ALL TOGETHER –

3. The perimeter of the rectangle is 5. Find the coordinates of the midpoint of the
segment joining (5,6) and (10,24).
10 meters
24
y (10,24)
23
8 meters 22
21
20
19
18
17
a. 18 m 16
15
b. 36 m 14
c. 45 m 13
12
d. 40 m
11
e. 80 m 10
9
8
4. If ΔABC ~ ΔDEF, then how large is angle F? 7
(5,6)
6
A 5
4
100° 3
2
1
x
–4 –3 –2 –1 0 1 2 3 4 5 6 7 8 9 10

50° a. (7.5,15)
B C
b. (2.5,8)
E
c. (3,10)
d. (8,2.5)
e. (15,7.5)

D F

a. 22
b. 30
c. 35
d. 40
e. 50

205
8th_GRDM_13.qxd:Layout 1 8/11/09 1:58 PM Page 206

– PUTTING IT ALL TOGETHER –

6. Find x. 8. The triangles are similar. Find x.


x
9

14
6

60°
10

a. 4
b. 913
a. 30
c. 10.5
b. 60
d. 17
c. 90
e. 21
d. 100
e. 120
9. What is the approximate volume of a cylinder
with a radius of 6 inches and a height of
7. The width of a rectangle doubles. The length
10 inches?
remains unchanged. What happens to the area
a. 60 cubic inches
of the rectangle?
b. 359 cubic inches
a. The area stays the same.
c. 620 cubic inches
b. The area doubles.
d. 1,100 cubic inches
c. The area triples.
e. 1,500 cubic inches
d. The area is 10 times larger.
e. The area is 100 times larger.
10. Find the length of a diagonal of a rectangle if
the sides are 9 and 12.
12

a. 7
b. 10
c. 15
d. 21
e. 200

206
8th_GRDM_13.qxd:Layout 1 8/11/09 1:58 PM Page 207

–PUTTING IT ALL TOGETHER –

11. The circle has a diameter of 10 inches. The area 13. The two cones are similar, and the smaller cone
of the semicircle is approximately which of the has a height of 10 inches. Find the approximate
following? volume of the larger cone.

5 15

a. 40 square inches a. 700 cubic inches


b. 78 square inches b. 2,100 cubic inches
c. 100 square inches c. 4,200 cubic inches
d. 150 square inches d. 5,000 cubic inches
e. 300 square inches e. 7,000 cubic inches

12. The sum of the measures of the five angles of 14. Find the perimeter of the triangle.
this polygon is

15
12

a. 180
b. 360
c. 465 a. 9
d. 540 b. 36
e. 800 c. 43
d. 90
e. 108

207
8th_GRDM_13.qxd:Layout 1 8/11/09 1:58 PM Page 208

– PUTTING IT ALL TOGETHER –

15. How do the volumes of the cone and the y


cylinder compare?
A
(5,6)

10
10 x
0

B (–7,–3)
5
5

a. The volumes are the same.


b. The volume of the cylinder is twice as large
as the volume of the cone.
c. The volume of the cylinder is 3 times the
volume of the cone. 16. In the graph above, the distance from A to B is
d. The volume of the cylinder is 4 times the a. 2
volume of the cone. b. 8
e. The volume of the cylinder is 9 times the c. 11
volume of the cone. d. 5 2
e. 15

17. Two lines are perpendicular. The equation of


one is y = 43x + 4. Which of the following
equations could be the equation of the other?
a. y = – 43x + 1
b. y = 43x + 6
c. y = 3x + 4
d. y = 43x + 4
e. y = – 43x

208
8th_GRDM_13.qxd:Layout 1 8/11/09 1:58 PM Page 209

–PUTTING IT ALL TOGETHER –

18. Find the area of the parallelogram. Extended Response


Write a response that fully explains how you arrived
at your answer. Show all diagrams or tables that you
made, as well as all calculations.
12 10

20. A park district plans to plant a circular garden


with a diameter of 10 feet. They will surround
14 the garden with a 4-foot-wide concrete
walkway. They will also build a 20 × 4-foot
a. 70
walkway from the parking lot to the garden.
b. 120
Before they can order the cement to build the
c. 140
walkways, they must find the total area to be
d. 168
covered with cement. Round your answer to
e. 175
the appropriate square footage.
Short Response
Write a response that explains how you arrived at
your answer. (Use calculator only to check.)

19. Angle B is four times larger than its comple- Statue

ment, angle A. How large is angle B?


4'

Trees
20 ft.

Parking lot 4 ft.

209
8th_GRDM_13.qxd:Layout 1 8/11/09 1:58 PM Page 210

– PUTTING IT ALL TOGETHER –

Answers 11. a. Because the diameter is 10 inches, the radius


is half that, or 5 inches. The formula for a
1. c. Angle 3 is the supplement of ∠6, and circle is A = πr2, and using π = 3 for an
180 – 70 = 110. approximation, the area of the whole circle
2. b. The sum of the angles of a quadrilateral is is 3 × 25 = 75, and half of that is about 37.5.
360°. The other three angles are 110°, 100°, Choice a is the closest number to 37.5.
and 90°, which add to 300°. (Note: If another choice had been close to
3. b. Just use the formula 2l + 2w = 2(10) + 2(8) 37.5, such as 35, you would have had to use
= 36. the approximation 3.14 for π, and you
4. b. Similar triangles have congruent angles. would have found that the area of the circle
The angle opposite the longest side of both is about 78.5, half of which is about 39.25.)
triangles is 100°, and the angle opposite the 12. d. The formula for the total degrees of the
second longest is 50°, so the other angle angles is 180 × (number of sides – 2). This
must be 180 – 150 = 30°. polygon has 5 sides, so the total number of
5. a. The coordinates of the midpoint are the degrees is 3 × 180 = 540.
average of the x’s and the average of the y’s. 13. e. Because the scale factor of the small cone to
In this case x = 10 2+ 5 = 7.5 and y = 24 2+ 6 the large cone is 13, the height of the large
= 15. cone is 3 × 10 = 30. The volume of a cone is
6. a. Opposite angles are equal, so one of the given by V = 13πr2h. Using π = 3 for an
interior angles of the right triangle is 60°. approximation, we get 13(3)(15)2 ≈ 225(30)
The other, the one with x degrees, is 30°. ≈ 6,750. Answer e, 7,000, is the closest
7. b. A = lw. New A = (2l)w = 2lw. choice.
8. e. 69 = 14x , so 6x = 126 and x = 21. 14. b. This is a (4×) (3-4-5) right triangle, so the
9. d. Using the formula for the volume of a missing side is 9. Use the 9 to find the
cylinder with π = 3, V = 3(6)2(10) perimeter: 9 + 12 + 15 = 36.
= 3(36)(10) = 108(10) = 1,080 ≈ 1,100. 15. c. The heights and the radii of the two figures
10. c. The diagonal of a rectangle forms a right are identical, so the only difference will
triangle, with the two sides as the legs. come in their respective formulas. The
Letting the length of the diagonal be c, volume of a cone is 13πr2h, and the volume
of a cylinder is πr2h, so with h and r
c 2 = (12)2 + (9)2
identical, the volume of the cylinder is three
c = (12)2 + (9)2 times that of the cone.
c = 225 = 15 16. e. Use the distance formula to get
distance = ( y1 − y 2 )2 + (x1 − x 2 )2
= (−3 − 6)2 + (−7 − 5)2
= 81 + 144
= 155

210
8th_GRDM_13.qxd:Layout 1 8/11/09 1:58 PM Page 211

–PUTTING IT ALL TOGETHER –

17. e. If two lines are perpendicular, their slopes 20. Responses will vary; here is a sample:
will be negative reciprocals of each other. There are two separate areas that will
The given equation has a slope of 43, as you need to be paved. The first is the
can tell from its slope-intercept form. rectangular path leading to the garden,
(Review Lesson 12 if you have forgotten and the area of that is
this.) All of the choices are also in the slope- 4 × 20 = 80 ft.2
intercept form, and only one of them has a The area of the circular walkway is harder
–43 as its coefficient of x. The y intercept isn’t to find, because it’s like a donut—it doesn’t
relevant in determining whether the lines include the garden. So the first thing to
are perpendicular. do is find the area of the whole circle that
18. c. Use the formula for the area of a parallelo- includes the garden and the walkway.
gram: 14 × 10 = 140.
Walkway
19. Responses will vary. Drawing a picture might
help here, but the problem can easily be solved
with algebra, so a picture isn’t necessary in this
short response. Here’s a sample answer: 4' 10' 4'

The question asks for the measure of an Garden

angle. I know that it’s 4 times its


complement, and I know that complement
means that the two angles add up to 90.
So I let b = the measure of angle B and
(90 – b) is angle A. Then I use the As you can see from my drawing, the
equation diameter of the whole circle is 18 feet,
b = 4(90 – b) so the radius is 9 feet, and the area is
b = 360 – 4b π × 92 = 81π
5b = 360 The area of the garden with radius of 5 is
b = 72 25π. And the area of the walkway is
So, the answer is 72°. To check, I took 41 of 81π – 25π = 56π = 56 × 3.14159
72, which is 18, and added them together. = 175.929
They add to 90, and everything checks. Rounding this up to 176 and adding to
the 80 for the path:
176 + 80 = 256 square foot total
[Note: Because the right angles of the rectangle
and the circular walkway will overlap a bit, the
256 square feet is a little bit high. If you notice
something like that in a question about
geometry, note it in your response. However,
you’d probably receive full credit even without
that. The 256 ft.2 is an appropriate response,
because you would probably want to have a
little extra cement on hand anyway.]

211
8th_GRDM_13.qxd:Layout 1 8/11/09 1:58 PM Page 212

– PUTTING IT ALL TOGETHER –

IF YOU MISSED THEN STUDY

Question 1 Lesson 17

Question 2 Lesson 18

Question 3 Lesson 18

Question 4 Lesson 19

Question 5 Lesson 20

Question 6 Lesson 17

Question 7 Lessons 18, 19

Question 8 Lesson 19

Question 9 Lessons 19, 21

Question 10 Lessons 18, 20

Question 11 Lessons 18, 22

Question 12 Lesson 18

Question 13 Lesson 19

Question 14 Lessons 18, 22

Question 15 Lesson 19

Question 16 Lesson 20

Question 17 Lesson 20

Question 18 Lesson 18

Question 19 Lessons 17, 21, 22

Question 20 Lessons 18, 22

212
8th_GRDM_14.qxd:Layout 1 8/12/09 10:28 AM Page 213

4
S E C T I O N

MEASUREMENT

I t’s very likely that you’ll have to do some kinds of measurement in the jobs you’ll be doing as an adult. It’s
even more likely that you’ll be making both precise and approximate measurements in your daily life. For
example, you’ll have to know the dimensions of the rooms in your home if you want to make improvements
on them; you’ll need to know how to measure and convert quantities for recipes; and you’ll need to be able to
estimate distances if you want to know how long it will take to get somewhere. You won’t learn how to do all of
that here, but you will get a good start on the measurement skills you’ll need for eighth grade.

213
8th_GRDM_14.qxd:Layout 1 8/12/09 10:28 AM Page 214

–MEASUREMENT–

By the end of this section, you should have these basic skills:

■ Reading the fractions of an inch on a ruler.


■ Converting measurements in both standard U.S. and metric systems.
■ Converting temperatures between Fahrenheit and Celsius.
■ Making good estimates of measurements using everyday objects, including your body.
■ Solving non-measurement problems using what you have learned about conversion fractions.

WORDS YOU SHOULD KNOW

capacity dimension metric system

Celsius Fahrenheit pint

cubic units gallon quart

cup gram square units

customary system liter

degrees meter

214
8th_GRDM_14.qxd:Layout 1 8/12/09 10:28 AM Page 215

24
THE METRIC
L E S S O N

AND
CUSTOMARY
SYSTEMS

L E S S O N S U M M A RY
In this lesson you’ll learn how to read fractions on a ruler, and you’ll
learn how to do conversions in both the customary and metric sys-
tems of measurements. You’ll also learn how to use scale factors
for square and cubic measures. Finally, you’ll convert temperatures
with formulas.

M ost of the world uses the metric system to measure everything: lengths, areas, volumes, electric
charges, and on and on. The nice thing about the metric system is that it’s based on 10’s, just like
the rest of our number system. Here in the United States, though, we use the same system that
1
we’ve used for centuries. We call it the customary or standard system. Our rulers are divided into 16 ’s of an inch,
our pounds are divided into 16 ounces, and our gallons are divided into 8 pints and 16 cups. Our freezing point
is 32° instead of 0°, and our boiling point is 212° instead of 100°. And our mile is divided into 1,760 yards and
5,280 feet. Yet most Americans remember all these measurements, and you will as well when you’re a student
and especially when you’re an adult. You’ll also very likely have some area of your life that will require you to
know the metric system as well, and how to convert between the two.

215
8th_GRDM_14.qxd:Layout 1 8/12/09 10:28 AM Page 216

–THE METRIC AND CUSTOMARY SYSTEMS –

How to Read For most purposes (in construction, for example),


Length Measurements measuring to the nearest inch is not good enough. So
the ruler is cut up into smaller subdivisions.
Being able to take and record measurements is an 1
– 112– 2 12–
essential tool in many jobs and professions. Meter read- 2 1 2 3

ers, scientists, carpenters, nurses, and bricklayers need 1 2 3


to be able to make careful measurements and record
them accurately. In everyday life, nearly all people
make measurements to buy such things as drapes,
carpets, and even food. Look at these measures:
A measuring device such as a ruler will be di-
221″ means 2″ + 21″ more.
vided into units such as inches, and each inch is
321″ means 3″ + 21″ more.
marked with a number.
As you learned in Lesson 2, a number like 3 12 is
called a mixed number. The 3 is the whole number,
1 2 3
the 1 is the numerator of the fractions, the 2 is the de-
nominator.

If the item being measured ends exactly at the 2-inch


3 12–
Numerator
Whole number
mark, record 2″. If it falls between the 1″ and 2″ Denominator
marks, then make your best guess. Is it closer to 1″ or
closer to 2″? Most rulers have lots of vertical marks. How do
you read those? Each of the smaller marks is a frac-
Exactly 2" tion. To read a mark on the ruler, first decide which of
the inch marks it falls between. Record the smaller
1 2 3
number. That is the whole number part of the answer.

The mark falls between


Ends closer to 1", so record as 1" 5 5 and 6. Choose the
smaller, 5.
1 2 3

5 6 7
Ends closer to 2", so record as 2"

1 2 3
The denominator tells how many spaces between the
inch marks. So count the spaces between the inch
marks and record that number in the denominator.
In the ruler like the following one there are 4 spaces
between the edge and the 1-inch mark, 4 spaces be-
tween the 1- and the 2-inch marks, and so on.

216
8th_GRDM_14.qxd:Layout 1 8/12/09 10:28 AM Page 217

–THE METRIC AND CUSTOMARY SYSTEMS –

4 spaces between inches. And here’s how to find the reading at B:



Put 4 in the denominator: 5 4

1 2 3 4 7 ← B is between 7 and 8. 7 is smaller.


5 6 7
74 4 spaces between inch marks.

7 43 ← B is 3 spaces past 7.
5–
4
The numerator tells how many spaces you have gone
past an inch mark. In this case we are 2 spaces past
the 5-inch mark, so 2 goes into the numerator of the
fraction.
More Marks
On this ruler, there are 8 spaces between the inch
2 spaces past the 5, so put
2 in the numerator: 5 24– marks. Every denominator will be an 8, unless it can
be reduced.
1 2

5 6 7
A B C D

Finally, decide if the fraction can be reduced. Most 1 2 3

rulers provide a good hint. The longer marks be-


tween inch marks can always be reduced. This mark
is not one of the smallest marks, so it can be reduced:
542 = 521. Exercise 1
Now let’s read this ruler: what are the readings Read the ruler above at the spots marked A, B, C,
at A and B? and D. Answer on page 223.

A B

Even More Subdivisions


In the typical 1-foot ruler, each inch is cut into 16 small
6 7 8
sections, so every denominator will begin as a 16.

Exercise 2
Here’s how to find the reading at A: Read the ruler at the points marked A and B. Answer
on page 223.
6 ← A is between 6 and 7. Choose 6. It's smaller.
A B

6 4 ← 4 spaces between inch marks.

6 14 ← A is 1 space past the 6. 1 2 3

217
8th_GRDM_14.qxd:Layout 1 8/12/09 10:28 AM Page 218

–THE METRIC AND CUSTOMARY SYSTEMS –

Inch Rulers with Decimals A and B could also be said to be at 4 millimeters and
1
If each unit is cut into 16 sections, the measure- 17 millimeters, respectively, because a millimeter is 10
ments are recorded as decimals. On a decimal ruler, of a centimeter.
the vertical marks indicating the end of a subdivi-
sion are almost all the same size. Because we are
The techniques used to read rulers can be used
working with decimals, there is no need to reduce.
to read many other measuring instruments.
Only the inch marks and the halfway marks are
larger. This is to help with the counting.

Exercise 3
Read the ruler at the spots marked A and B. Answer Converting between Units
on page 223.
A system of measures is a group of measurement
A B units that are related. Our standard system of units is
sometimes called the English system because it was
devised in England. Here are some of the English
1 2 3 equivalents for units of length.

ENGLISH UNITS OF LENGTH


12 inches = 1 foot

Metric Rulers 3 feet = 1 yard


The next figure is a metric ruler marked in centime-
5,280 feet = 1 mile
ters. The 1 represents 1 centimeter. Each centimeter is
cut into ten subdivisions.
The units (inches, feet, yards, and miles) are related
A B because miles are made of feet (or yards), and feet
and yards are made of inches.
It is often necessary to convert from one unit of
1 2 3
measure to a different unit. We may do this because
the original unit is so small and we have so many of
them that we cannot understand the amount. For ex-
ample, suppose someone added a column of meas-
As an example, let’s read the ruler at the spots marked
ures and the total was 1,080 inches. How big is that?
A and B. A is between 0 and 1, so the whole number
Or perhaps someone needs to mark the boundaries
part is 0. It is 4 spaces to the right of zero, so we
of a race and they need 1.25 miles of tape—but the
record 0.4 cm for A. B is between 1 and 2, so the
rolls of tape have 500 feet of tape to the roll. How
whole number part is 1. It is 7 spaces to the right of 1,
many rolls are needed?
so we record 1.7 cm for B. As you may already know,
The good news is that conversions are relatively
easy to do. You just have to remember a few facts
about operations with fractions:

218
8th_GRDM_14.qxd:Layout 1 8/12/09 10:28 AM Page 219

–THE METRIC AND CUSTOMARY SYSTEMS –

1. If the numerator and denominator are equal, 1,080 inches × 1 foot = 90 feet
1 12 inches
the fraction equals 1. So aa = 1.
2. Multiplying by 1 does not change the value of a Exercise 4
number. So 1 × a = a × 1 = a. (This is called the The last 1.25 miles of a marathon will be marked by
Identity Law of Multiplication, and you used it tape to keep the crowd off the race path. Tape comes
in Lesson 2 to find common denominators.) 500 feet to the roll. How many rolls of tape will be
3. Whole numbers can be changed to fractions by needed? Answer on page 224.
putting a 1 under them. So 5 = 15.
4. Conversion equations can be rewritten as frac- Converting Metrically
tions worth 1. The most frequently used units in the metric system
As an example, let’s change the conversion are the liter, gram, and meter. The liter is used for liq-
equation 12 inches = 1 foot to a fraction: uid measure; the gram, for weight; and the meter, for
distance. Instead of having totally different names for
12 inches = 1 foot the subdivisions of these units (such as yard, foot,
and inch), the metric system uses different prefixes
Divide both sides by 12 inches:
(kilo, hecto, deka, deci, centi, and milli, for example)
12 inches 1 foot
= with the same basic unit.
12 inches 12 inches
Conversion can be accomplished exactly the
1 foot
1= same way in the metric system. Let’s look at a conver-
12 inches
sion table and try a few problems.
Or divide both sides by 1 foot:
12 inches 1 foot COMMON METRIC ABBREVIATIONS
=
1 foot 1 foot PREFIX ABBREVIATION
12 inches
=1 kilo k
1 foot
Now, write conversion fractions worth 1, using the hecto h

conversion equation deka da

5,280 feet = 1 mile deci d

The conversion fractions are centi c

5,280 feet and 1 mile milli m


1 mile 5,280 feet
CONVERSIONS
Both fractions equal 1, because 5,280 feet and 1 mile
1 kiloliter = 10 hectoliters
both represent the same length. Notice that either num-
ber can be placed in the numerator or the denominator. 1 hectoliter =10 dekaliters
Now let’s take a look at a useful conversion: 1 dekaliter =10 liters
Change 1,080 inches to feet. First, put the amount to
be changed over 1: 1,080 1inches . Then, choose another 1 liter =10 deciliters

fraction with inches in the denominator so that it will 1 deciliters = 10 centiliters


cancel out the inches in the numerator of the first
1 centiliter =10 milliliters
fraction: 121inches
foot
. Then, just multiply (you’re multiply-
ing by 1, so the value won’t change).
219
8th_GRDM_14.qxd:Layout 1 8/12/09 10:28 AM Page 220

–THE METRIC AND CUSTOMARY SYSTEMS –

CONVERSION TABLE BASED ON Here is a list of the same units of metric weight,
THE MEANING OF PREFIXES* using scientific notation:
kilo is 1,000 1 kg =1,000 g
KILOGRAM GRAM MILLIGRAM
hecto is 100 1 hg =100 g
103 1 10–3
deka is 10 1 dag = 10 g

deci is .1 1 dg = .1 g Convert by moving the decimal point right if you


need a smaller unit of measure and move left if you
centi is .01 1 cg = .01 g
want a larger unit.
milli is .001 1 mg = .001 g As an example, let’s change 5.4 kg to milligrams.

*Meters or liters can be substituted for grams in this list. 5.4 kg 1, 000 1, 000 mg
× × = 5.4 × 103 × 103 mg
1 kg g
The metric units most commonly used in the United = 5.4 × 106 mg
States are the kilometer, meter, centimeter, and millime-
ter for length; the kilogram, gram, and milli-gram for This is the same as moving the decimal point 6 places
weight; and the liter and milliliter for liquid volume. to the right: 5.400000 × 106 ⇒ 5,400,000.
As an example, let’s convert 2,500 ml to liters.

2,500 ml 1 liter
1 × 1,000 ml = 2.5 liters In the metric system, move the decimal point
right to convert to smaller units and move left
Or you can do metric conversions with decimals: to convert to larger units.

2,500 ml × 0.001 liter = 2.5 liters


1 ml Here’s a table of the most common prefixes, along
Exercise 5 with their corresponding scientific notations.
(Answers on page 224.)
PREFIX VALUE EXAMPLES
1. Convert 65 m to cm. giga- 109 Gigabyte (GB)

2. Convert 432 mm to meters. mega- 106 Megabyte (MB)

kilo- 103 kilogram (kg), kilometer (km)


A Faster Way to Do Metric centi- 10–2 centimeter (cm)
Conversions
Did you notice that, when we changed 2,500 ml to 2.5 milli- 10–3 milligram (mg), millimeter (mm),
millisecond (ms)
liters, the numbers looked the same except that we
moved the decimal point three places left? Because micro- 10–6 micrometer (µm), microgram (µg),
the metric system is based on 10’s, we can convert microsecond (µs)

from one unit to the other just by moving the decimal


point. For example,

1 g = .001 kg 1 m = 1,000 mm

220
8th_GRDM_14.qxd:Layout 1 8/12/09 10:28 AM Page 221

–THE METRIC AND CUSTOMARY SYSTEMS –

Exercise 6 Square and Cubic Units


(Answers on page 224.)
How many square feet are in a square yard? How
1. Convert 64 centimeters to millimeters. many cubic feet are in a cubic yard? Drawing pictures
can help solve this problem.
2. Convert 8 milligrams to kilograms.
1 yard 3 feet

3. Subtract 18 millimeters from 9 centimeters.


Give the answer in centimeters and meters.
1 yard = 3 feet

Converting Measures
That Combine Units
1 square yard = 9 square feet

A measure like 5′3″ is a combination of feet and


But sometimes there are too many squares to count.
inches. Sometimes, we need to convert it to just one
How many square inches in a square foot?
unit of measure. The key is to remember that 5′3″
means 5′ + 3″. If we want inches, we convert the 5′ to 1 ft. 12 inches
inches. If we need feet, we convert the 3″ to feet. For
example:
1 ft. = 12 inches
Change 5′3″ to inches.

We need to change the 5′ to inches and then add the


answer to the 3 inches. A = 1 × 1 = 1 sq. ft. A = 12" × 12" = 144 sq. in.

1 sq. ft. = 144 sq. inches


5 feet × 12 inches = 60″ and 60″ + 3″ = 63″
1 1 foot How many cubic inches in a cubic foot?
Converting from a Single Unit to a
Mixed Unit
Many scales give digital readings. For example, a
bathroom scale may give a digital reading of 95.6
1 ft. 12 in.
pounds. How many ounces is .6 pounds? Just convert
the 0.6 pounds to ounces using the usual method.
1 ft. 12 in.
1 ft. 12 in.
0.6 pounds × 16 ounces = 9.6 ounces V=l×w×h
1 pound V = 1' × 1' × 1' V = 12" × 12" × 12"
V = 1' cu. ft. V = 1,728 cu. ft.
So the answer is 95 pounds and 9.6 ounces. Isn’t the 1 cu. ft. = 1,728 cu. in.

decimal system an easier way to think of measure-


ments?

221
8th_GRDM_14.qxd:Layout 1 8/12/09 10:28 AM Page 222

–THE METRIC AND CUSTOMARY SYSTEMS –

A third way to solve the problem is to use scale factor. Notice that there’s a pair of parentheses in the first
In Lesson 19 you learned that the ratio of the areas of equation but not in the second. Forgetting this can
two similar figures is equal to the square of the scale lead to some strange conversions.
factor, and volume is equal to the cube of the scale Why is converting between different tempera-
factor. All squares are similar, and so are all cubes. ture systems so different from converting between
The conversion fractions for square inches are other units? Why is a formula necessary? There is a
major difference between temperatures and other
(12 inches)2 144 inches 2
= measuring systems. In other systems, 0 is the starting
(1 foot)2 1 foot 2 point. When we measure length, quantity, volume,
and area, or weight we begin at 0. Zero is the first possible
measure on the measuring device. Thermometers do
(1 foot)2 1 foot 2
= not start at 0, though. It is possible to measure a neg-
(12 inches)2 144 inches 2 ative temperature. It is never possible to weigh a neg-
The conversion fractions for cubic inches are ative number of pounds, to be a negative number of
inches tall, to drink a negative number of bottles of
(12 inches)3 1, 728 inches soda, to have a negative area or a negative volume.
=
(1 foot)3 1 foot 3 The formulas for temperature conversion were de-
and signed to adjust for the different starting points. In
fact, there is a temperature system in which 0 really is
(1 foot)3 1 foot 3
= a temperature with absolutely no heat. It’s called the
(12 inches)3 1, 728 inches 3 Kelvin system, and its 0 is –273° C and –459° F.

Exercise 7 Exercise 8

1. How many square centimeters in a square 1. A comfortable summer day is about 75°F.
meter? Convert this to Celsius, to the nearest tenth of
a degree.
2. Convert 3 square meters to square centimeters.
2. Is 35° C a pleasant summer day? Convert to
Fahrenheit and see.
Converting Temperatures
The two most common systems of temperature are
Fahrenheit and Celsius. Converting between them is
somewhat complicated. Although water boils at 212° F Summary
and 100° C and it freezes at 32° F and 0° C, these tem-
peratures cannot be used to make conversion fractions. In this lesson you have learned something about the
C ↑ 0° C . The zero in the denominator
212° F 32° F two systems of measurements most commonly used
For example, 100°
is also a problem. Instead of conversion fractions, we in the United States. You have also learned how to
use the following formulas to convert between the two read fractions on a ruler, and how to do conversions
systems, with C and F standing for Celsius temperature in both systems. You also learned to convert tempera-
and Fahrenheit temperature, respectively. tures with formulas.

C = 95(F – 32) F = 59C + 32

222
8th_GRDM_14.qxd:Layout 1 8/12/09 10:28 AM Page 223

–THE METRIC AND CUSTOMARY SYSTEMS –

Answers Exercise 2
For point A:
Exercise 1 1 ← A is between 1 and 2. 1 is smaller.
For point A: 1 16 ← 16 spaces between 1 and 2.
0 ← A is between 0 and 1, 0 is smaller.
1 163 ← A is 3 spaces past 1.
0 8 ← 8 spaces between inches. A is at one of the smallest marks, so it does not re-
0 3 ← A is 3 spaces past 0.
3
duce. The answer is 116 .
8
A is at one of the smallest marks, so it does not re- For point B:
duce. The answer is 083, or 83 ″. 2 ← B is between 2 and 3. 2 is smaller.
For point B: B is exactly on the 1-inch mark. 2 16
← 16 spaces between 1 and 2.
The answer is 1″. 10 ←
2 16 B is 10 spaces past the 2.
For point C:
B is at one of the larger marks, so it does reduce. The
1 ← C is between 1 and 2.
answer is 258.
1 8 ← 8 spaces between inches.
1 68 ← C is 6 spaces past 1. Exercise 3
C is not at one of the smallest marks, so it should be A: 1.3, B: 2.5
reduced. 186 = 143 ″. To read the spot marked A, first decide which of the
For point D: inch marks it falls between. It falls between 1 and 2.
2 ← D is between 2 and 3. Record 1 because it is the smaller number. Place a
2 8 ← 8 spaces between inches. decimal point to the right of the 1 and then count
how many spaces there are between the 1-inch mark
2 48 ← D is 4 spaces past 2.
on the ruler and A. There are 3 spaces, so 1.3 inches is
This is not at one of the smallest marks, so reduce:
the reading for A.
248 = 221″.
B is at 2.5 inches because it is between the 2-
and the 3-inch marks and it is exactly 5 spaces to the
right of the 2-inch mark.

223
8th_GRDM_14.qxd:Layout 1 8/12/09 10:28 AM Page 224

–THE METRIC AND CUSTOMARY SYSTEMS –

Exercise 4 Exercise 6
First, look at the conversions that will be needed: 1. 640 mm
We have to move the decimal one step to the
5,280 feet 1 mile
5,280 feet = 1 mile, so 1 mile = 5,280 feet = 1 right, and 64.0 becomes 640. As a check, note
1 roll 500 feet that your answer has to be bigger because
1 roll of tape = 500 feet so 500 feet = 1 roll = 1
you’re using a smaller measure for the same
Next, set up the problem and solve. length.
2. 0.000008 kg
1.25 mile × 5,280 feet × 1 roll = 13.2 rolls Here you’re going from a smaller measure to a
1 1 mile 500 feet
larger measure for the same weight, so the
See how the miles cancel and the feet cancel. The only number of units has to go down. You’re going
unit left is rolls, so then all you have to do is the arith- from 10–3 to 103, so move the decimal 6 places.
metic (you’ll probably need a calculator, or at least a 3. 7.2 cm; 72 mm
pencil): 1.25 × 5,280 ÷ 500 = 13.2. First, convert the 9 cm to millimeters, so that
Next, round the answer. Would you round down you can work with integers for the subtraction
to the nearest whole roll? No. You’ll need 14 rolls, be- problem: 9 cm ⇒ 90 mm. 90 mm – 18 mm =
cause you need more than 13 rolls to do the job. 72 mm = 7.2 cm.

Exercise 7
When you are choosing conversion fractions, 1. 10,000 cm2, or 104 cm2
choose the fraction with a denominator that has (100 cm)2 10,000 square cm
(1 m)2
= 1 square m
. Note that (102)2 = 104.
the same unit of measure as the unit of measure
(See Lesson 4 if you’ve forgotten why.)
in the numerator of the previous fraction. This
2. 30,000 cm2. Using answer 1,
allows you to cancel units and simplify. 1,000 square cm 3 square m = 30,000 cm2.
×
1 square m 1

The beauty of this system is that we can use as many Exercise 8


fractions as necessary. 1. 23.9° C
To convert from Fahrenheit to Celsius, use the
Exercise 5 formula C = 95(F – 32).
C = 95(75 – 32)
1. 65 meters × 100 centimeters = 6, 500 cm C = 95(43)
1 1 meter
432 millimeters 1 meter 2. Is 95° F a nice summer day? Maybe, if you have
2. × = 0.432 m air-conditioning. This time use the formula
1 1,000 millimeters
F = 59C + 32.
F = 59(35) + 32
F = (9 × 7) + 32
F = 95

224
8th_GRDM_15.qxd:Layout 1 8/11/09 4:20 PM Page 225

25
L E S S O N

ESTIMATING
AND CHECKING

L E S S O N S U M M A RY
In this lesson you’ll learn to use your body to estimate measure-
ments, and how to decide what unit might be appropriate for a
particular measurement. You’ll also be given a list that you should
memorize, and you’ll get some tips on checking measurement
problems.

B ■


y the time you reach adulthood, you should have a relatively good feel for the common units of meas-
ure. An adult should be able to do the following:

Select an appropriate unit when measuring an object.


Make accurate measurements.
■ Estimate common measures.
■ Convert from one unit of measure to another.

Lesson 24 concentrated on reading measurements and converting from one unit to another. This lesson
will emphasize estimating, checking, and choosing appropriate units.

225
8th_GRDM_15.qxd:Layout 1 8/11/09 4:20 PM Page 226

–ESTIMATING AND CHECKING–

Standard and away; measurement devices based on the model were


Non-Standard Units developed; and the use in business transactions of
anything but the standard unit was made illegal.
Historically, the units of our customary English sys- From time to time problems arose and the standards
tem were actually based on the lengths of our body were readjusted until they became the ones we have
parts. For example: today. As new problems occur, new adjustments will
be made. However, at any point in time, the exact
Estimation value of a standardized unit is defined by law.
How does knowing this history help us get bet-
ter at measuring? Knowing history may not make us
better at measuring, but it can help us to be better at
estimating measures. Because our bodies are always
1 span with us, we can use the body as a device for estimat-
ing measures. A good exercise is to take a ruler and
find parts of the body that have the same length as a
standard unit. For example, the first joint of the
1 ft. 1 cubit
thumb is about one inch for most people, and the fin-
gernail of the pinky finger is about 1 cm across. It’s
These units were not, of course, exact. As a result,
also useful to know that the hand-and-finger span is
problems occurred.
about 8 inches, the hand is about 4 inches, and pencil
lead is about 1 mm thick.
or
1 mm

8"

1 cm

Whose foot would you want to use


if you were buying land? 1 inch

Some people felt they were being cheated. Quar-


First joint Pinky Pencil
rels erupted. Citizens complained to the king. Eventu- of thumb fingernail Span lead
ally, the king and his court began to make laws
standardizing some units of measure. For example, Once you find these measures on your own body,
standardizing the foot meant that a foot could no it is easy to use them for real measurement. For exam-
longer be the length of just anyone’s foot, but it would ple, you can make quick estimates of the size of furni-
be, by law, a certain length. An exact model of the ture in stores by counting how many spans fit along the
unit was made and locked away in a safe place. Copies edges of the item. Making the different measurements
were also made and distributed so the people could of your body or of common, everyday objects gives a
make measures. These copies eventually developed frame of reference for what unit is appropriate. For ex-
into our rulers and tape measures. ample, if you were asked whether millimeters would be
Standardization occurred with every type of a good unit to measure a picture, you could visualize a
measure. The standard, or official, model was locked line of little pieces of lead (each 1 mm across). This

226
8th_GRDM_15.qxd:Layout 1 8/11/09 4:20 PM Page 227

–ESTIMATING AND CHECKING–

should lead you to say that millimeters would not be Some rules of thumb may not be clever, but
appropriate. If asked to estimate the length of a new they create a picture in our mind to help us remem-
pencil, you could compare it to the hand-and-finger ber. For example,
span or see how many thumb joints fit along it.
Now go find a ruler and find some spots on ■ A yard extends from the tip of a person’s nose to
your hand that equal the tip of the finger on the outstretched hand. A
meter goes from the far shoulder to the tip of
1. 1 inch finger.
2. 1 centimeter
3. 4 or 5 inches
4. 1 decimeter

Next, measure your hand-and-finger span and


your foot. But remember, you are probably still grow-
ing, so you should repeat this exercise once a year or
so until you reach your full height.

Rules of Thumb
A rule of thumb is a rough, but practical, rule based
1 yard 1 meter
on experience rather than a scientific calculation.
Rules of thumb about measurements are often good ■ A bag of brown sugar is about 2 pounds or 1
estimates of size, and they are often worded in a kilogram.
clever way that helps us remember. Here are some ex- ■ A liter of soda is barely more than a quart.
amples along with their meanings:
Rules of thumb give rough estimates and help us vi-
■ “A pint’s a pound the world around.” (A pint of sualize the situation.
water weighs about a pound.)
■ “A liter is a kilogram when you’re in a metric Exercise 1
jam.” (A liter of water weighs about 1 kilogram.) Would a pail of water weigh 2 pounds or 8 pounds?
■ “Change to metric, a dieter’s delight. Lose half Answer on page 229.
your weight.” (One kg is about 2 pounds, so a
person who weighs 150 lbs would be about 75
kg, a number half as big.) A Caution
■ “A meter measures 3 foot 3. It’s longer than a Rules of thumb are fine to use when you are estimat-
yard, you see.” (Actually, the meter is about 3 ing the amount needed, but if exact values are re-
feet plus 3.5 inches (or 39.5 inches), but 3′3″ is quired for the purpose of buying and selling, it is
not a bad estimate.) better to use an official equivalent. Some official
equivalents for converting to metric are 1 inch = 2.54
cm, 1 liter = 1.057 quarts, and 1 kg = 2.205 pounds.
Many tables of exact equivalents are available at
your local library and on the Internet.

227
8th_GRDM_15.qxd:Layout 1 8/11/09 4:20 PM Page 228

–ESTIMATING AND CHECKING–

Visualization for Checking Memorize the Most


Some people have trouble when they convert units Useful Conversions
metrically by moving the decimal point. Should they
move it right or left? Sometimes they slip up in other Keep a little list of those conversions that you fre-
conversions as well. One check on conversions is to quently use in class. In algebra you often need to
visualize one big tower and think, “One big tower know the following conversions, so it pays to memo-
converts to lots of small bricks.” This helps you to re- rize them.
member that when you change a big unit to a small
unit, you will get lots of those small units. Also, it 12 inches = 1 foot
takes lots of small units to make a big one. 3 feet = 1 yard
5,280 feet = 1 mile
16 ounces = 1 pound
1 inch = 2.54 centimeters
1 quart = 0.946 liter
2.2 pounds = 1 kilogram

Remembering that 1″ = 2.54 cm, 1 quart is about 1


liter, and 2.2 pounds is about 1 kg will make it easy to
estimate conversions between metric measurements
and customary units.
Also, if you do any cooking or handling of food
at all, you should memorize these units of capacity:

3 teaspoons (tsp.) = 1 tablespoon (tbsp.)


1 Big tower = Lots of small bricks 2 tbsp. = 1 fluid ounce (fl. oz.)
8 fl. oz. = 1 cup (c.)
2 c. = 1 pint (pt.)
Exercise 2 2 pt. = 1 quart (qt.)
(Answers on page 229.) 4 qt. = 1 gallon (gal.)

1. Which is likely to be wrong: 200 cm = 2 m or Can you see why they have to be memorized? There’s
200 cm = 20,000 m? not really a system that’s obvious; it’s all based on tra-
dition.
2. Suppose you are changing 50 kiloliters to liters,
but you can’t remember whether to move the Exercise 3
decimal point 3 places right or 3 places left. Convert 5 yards to meters without a calculator. Then
What can you do? do it with a calculator. Answer on page 229.

228
8th_GRDM_15.qxd:Layout 1 8/11/09 4:20 PM Page 229

–ESTIMATING AND CHECKING–

Some Ways to Check Answers

1. Visualize the size of the units. Which is larger? Exercise 1


Ask yourself, “Should the answer get smaller or 8 pounds is the better guess. Milk is sold in 21 pint car-
larger?” Then check to see if your answer meets tons in the lunchroom. An ordinary pail would hold
your expectations. 15 or 16 cartons, or about 8 pints. A pint is a pound,
2. Look for the usual suspects. Whenever you so 8 pints is about 8 pounds. Eight pounds would be
make an error, try to figure out what went the better guess.
wrong. Was it something you did not under-
stand, or did you make some little error that Exercise 2
you really should not have made? Look for a 1. Lots of small units make one big unit.
pattern. Do you make that error frequently? For Centimeters are smaller than meters, so there
example, some people often reverse two digits should be lots of centimeters and few meters.
in a number. (They write “42” instead of “24.”) The second answer must be wrong because
So people who do that should double-check there are a lot more big units, not a lot more
their numbers before they start the arithmetic. small ones.
3. Be sure the conversion fractions are not upside 2. Think, “Kiloliters are larger. A large item is
down. All the units except the one you want to made of lots of small parts. I am breaking each
arrive at should cancel each other out. And kiloliter into little pieces, so the number of
there should be no unit canceling the unit you pieces should get larger. Moving the decimal
want in your final answer. point right produces 50,000 and moving it left
4. Use an alternative method to check. You have produces .050, so it should go right. Therefore,
been doing that with every other area of math, 50 kiloliters = 50,000 liters.”
but you might think it’s harder with measure-
ments. But in this lesson and Lesson 24, you Exercise 3
have learned multiple ways to do many different A yard is a little less than a meter, so the conversion to
kinds of problems. Learn at least two ways to do meters will give you a number a little less than 5. Us-
a problem, and then do the problem one way ing “A meter measures 3 foot 3,” you can figure that
1
and check by using a different method. each meter is about 10 more than a yard, so you can
1
subtract 10 from each of the 5 yards and get an esti-
mate of 4.5 meters. Calculating,
Summary 5 yards
1 × 361 inches
yard
× 2.54
1 inch × 100 cm
cm 1m

5(36)(2.54)
= 100 m = 4.572 m
In this lesson you have learned to use your body and
some everyday objects to estimate measurements and
to decide what unit might be appropriate for a given
measurement. You have memorized a list of useful
conversions, and received some tips on checking
measurement problems.

229
8th_GRDM_15.qxd:Layout 1 8/11/09 4:20 PM Page 230
8th_GRDM_15.qxd:Layout 1 8/11/09 4:20 PM Page 231

26
L E S S O N

STRATEGIES

L E S S O N S U M M A RY
In this lesson you’ll learn how to measure with rulers that don’t
start exactly at 0. You’ll also learn how to apply conversion frac-
tions to word problems, even when they involve a mixture of stan-
dard measurements and other ratios. And you’ll learn how to start
solving word problems that give data in one form and ask for an-
swers in another.

W hen you measure an object, you are comparing the object to a number line that has been
marked in convenient units, and each unit is cut into a number of equal subdivisions. As you
learned in Lesson 24, the largest marks on the number line indicate whole units, such as inches.
They are always expressed as whole numbers. The smaller marks indicate fractions. The denominator of a frac-
tion is the number of spaces between the inch marks. The numerator is the number of spaces between the inch
mark to the left and the spot on the ruler that is being read.
The number that’s being read, though, is the length of your measurement only if you start measuring at 0.
Read on to see what to do if you can’t start measuring at 0.

231
8th_GRDM_15.qxd:Layout 1 8/11/09 4:20 PM Page 232

–STRATEGIES–

Reading Damaged get you the correct answer, and you would have all the
or Unusual Rulers fun of doing some extra math operations as well!

Measurements are supposed to be exact. Examine a


You can begin measuring at any spot on a ruler
ruler before measuring with it. Sometimes, the ruler
by recording the starting and ending readings
is damaged at the left side and its corner is missing,
and then subtracting the smaller reading from
making it difficult to start measuring.
the larger to get the actual length.

The number line on some rulers does not begin


1 2 on the left edge, but a bit over. (See picture.) Take care
to start measuring at zero and not the left edge.
If the end of the ruler is damaged, start measuring at the
one-inch mark, but remember to subtract 1 from your total. Line up the edge of the object being measured with zero,
not the edge of the ruler.

1 2
0 1 2

Some rulers don't start measuring at the far left edge.

In this case start measuring at the 1-inch mark. Make


Exercise 1
a reading at the other end of the object and then sub-
tract 1 from your measurement before recording it.
1. Chuck starts measuring a cut on a damaged
ruler at the 181 -inch mark. The board he is
1" 3  "
measuring reaches the 421 -inch mark. How long
is the board?
1 2 3 4
2. Josephine stretches a piece of string from the
1
3  " – 1" = 2  " so the line is 2  " long. 2 -inch mark on a damaged ruler. It stretches to
7
the 216 -inch mark. How long is the string?
To measure the line, begin by placing one end of the
object at the 1″ mark and take a reading at the other
end of the object. Let’s say we read 3 58 . Subtract 1 from
Rates and Conversion Fractions
3 58 to get 258 as the length of the line.
When you are solving problems involving conver-
Why did we pick 1″ to start measuring at? Because
sions, you can often incorporate some non-standard
it’s easy to subtract 1 from just about any number, even
conversions into your calculations. In Lesson 24 we
if it has a fraction in it. To keep your number operations
used the non-standard conversion “1 roll of tape
sharp, though, you might want to begin the measure-
equals 500 feet.” This is non-standard in the sense
ment at some other number, like 187, and then subtract
that not all rolls of tape will be 500 feet. Also, a prob-
that number from the reading you get. That would still
lem might involve a rate such as “$2 per gallon” or “24

232
8th_GRDM_15.qxd:Layout 1 8/11/09 4:20 PM Page 233

–STRATEGIES–

miles per gallon.” The word “per” means division, so Exercise 2


$2
we can change $2 per gallon to 1 gallon , for example, Abbie is having a party with 25 guests. She figures
when we are doing conversions. each guest will need about two servings of soda. One
serving of soda is 355 ml. She will be buying 2-liter
bottles. One 2-liter bottle costs $1.50.
When you’re asked to give an answer in a partic-
ular unit, make sure that all units cancel—except
1. How many 2-liter bottles must she buy?
for the unit(s) you want your answer in.

2. How much money should she budget for soft


In this estimation $2 = the cost of 1 gallon of drinks?
$2
gas so 1 gallon = 1 for the purpose of this example.

Eddie is planning a trip of 465 miles. His car Word Problems with
averages 20 miles to the gallon of gas. He Multiple Conversions
estimates he will be paying about $2 per gallon
on the trip. How much money must he be Sometimes you’re given information that’s not all in
prepared to spend? the same units. For example, you may want to know a
speed in meters per second (m/s, one of the most
We set up the conversion fractions and multiply: common metric measurements of speed), but the
only data you have might be how many miles were
465 miles 1 gallon $2 $(465 × 2) traveled in a certain number of hours. Let’s say it was
× × = = $46.50
1 20 miles 1 gallon 20 25 miles in 1.5 hours. Then you would set up a series
Eddie should budget $46.50 (or more) for the gas. of multiplications like this, using 1,609 meters as the
Now read this warning before trying Exercise 2. equivalent for a mile:

1,609 meters 25 miles 1 hour 1 minute


× × ×
1 mile 1.5 hours 60 minutes 60 seconds
When you are computing prices of items that
come prepackaged, you must first compute the Every unit cancels except for meters/second, so you
quantity needed and round up if necessary to know that if you do your calculations correctly, you’ll
find the number of packages needed. Then have the right answer. In this case it’s 1,609 × 25 ≈
1.5 × 60 × 60
you can compute the total cost by multiplying 7.45 m/s.
the price of one package by the number of
packages needed. Exercise 3
It is usually a good idea to think about how The local 5-k (5-kilometer) race this year was won by
to round the quantity to be purchased before a runner who covered the distance in 16 minutes.
computing the price. What was his speed in (1) meters per second (m/s)
and (2) miles per hour (mph)? Give answers to the
nearest hundredth. (Use 1,609 meters = 1 mile.) Then
(3) figure out a good way to check your answer.

233
8th_GRDM_15.qxd:Layout 1 8/11/09 4:20 PM Page 234

–STRATEGIES–

Rounding When Answers


Using a Calculator
Exercise 1
Remember that, when you have a series of operations 1. 338 inches
that you’re doing on a calculator, rounding is allowed Here you just have to subtract fractions. (See
only in the last step of a problem. So keep values in Lesson 2 if you’ve forgotten how.) Because 21 is
your memory, use parentheses, and enter in the larger than 18 , it’s easy just to subtract the
whole problem all at once, or record as many digits fractions and then the whole numbers:
1 1 3
past the decimal point as possible, if you have to 2 – 8 = 8 and 4 – 1 = 3.
15
record intermediate values. 2. 116 inches
Convert the mixed numbers to improper
fractions, find a common denominator, and
Summary 39 1
subtract: 16 – 2 = 16 39
– 168 = 16
31
= 115
16.

In this lesson you learned a good method for dealing Exercise 2


with rulers that don’t start at 0. You also learned how 1. 9 bottles
to apply conversion fractions to problems that don’t This problem may seem easy to solve with just
necessarily involve measurements, and how to start a series of arithmetic operations, but using
solving word problems that give data in one form and conversion fractions helps to keep the units
ask for answers in another. straight. And we can cancel non-standard units
like guests and servings just the same as we can
cancel standard units. We’ll start with 25 guests:
25 guests 2 servings
1 × 1 guest × 1355 ml 1 liter
serving × 1,000 ml

× 2 × 3555
× 12bottle
liters
= 251,000 × 2 bottles
This calculation comes to 8.875 bottles. The
grocery store doesn’t sell bottles in units of
0.875 bottles, though, so to accommodate all
her guests, Mary will have to buy nine 2-liter
bottles.
2. $13.50
If you had used the conversion fraction
$1.50
1 bottle at the end of the string of conversions
in question 1, your total would have been
8.875 × $1.5 = $13.31, which would not be
enough, because she has to buy nine bottles.
Her budget for soda should be 9 × $1.5 =
$13.50.

234
8th_GRDM_15.qxd:Layout 1 8/11/09 4:20 PM Page 235

–STRATEGIES–

Exercise 3 3. Here are two ways of checking. First, we did (1)


1. 5.21 m/s and (2) both by starting from the original facts.
Here we have to get kilometers to meters and If we start (2) by using the m/s answer we got
minutes to seconds, so we need only two in (1), it should be the same. If it’s not, there’s a
conversion fractions: mistake in either (1) or (2). So we’ll start with
5 kilometers 1,000 meters
× 1 kilometer × 601 minute 5.208333 m/s.
16 minutes seconds
5.208333 meters
5,000
= 6 × 60 = 5.208333 meters per second second
× 3,6001 hour
seconds 1 mile
× 1,609 meters
2. 11.65 mph = 11.6532 miles per hour ✓
Because we have the equivalent of hours given Another way to do a rough check is to estimate.
in meters, we’ll also have to convert to We know that the race was exactly 5,000 meters
kilometers, and we’ll also have to convert and it was run in about 60 × 16 seconds, or
minutes to hours. close to 1,000 seconds. So the answer for (1)
5 kilometers
× 11,000 meters 60 minutes 1 mile
× 1 hour × 1,609 should be about 5 m/s, which it is.
16 minutes kilometer meters
= 11.6532 miles per hour

235
8th_GRDM_15.qxd:Layout 1 8/11/09 4:20 PM Page 236
8th_GRDM_15.qxd:Layout 1 8/11/09 4:20 PM Page 237

27 PUTTING
L E S S O N

IT ALL
TOGETHER

L E S S O N S U M M A RY
This lesson pulls together what you have learned about data and
probability in Lessons 24 through 26. You’ll use that knowledge to
answer problems and to write short and long responses that ex-
plain how you solved different problems.

Y ou have now learned some of the basics of measurement you’ll need to be successful in eighth grade.
You have also learned strategies for solving word problems based on real-life situations. Now it’s time
to see how well you know them.

What You Have Learned

Here’s a quick summary of the lessons in this section.


Lesson 24: The Metric and Customary Systems. You learned how to read a ruler, and how to set up con-
version fractions to solve any conversion problem. You also learned some shortcuts in converting measurements
within the metric system, and you learned how to convert temperatures from one system to another.
Lesson 25: Estimating and Checking. You learned some methods for estimating measurements and for
checking your answers to measurement problems.

237
8th_GRDM_15.qxd:Layout 1 8/11/09 4:20 PM Page 238

– PUTTING IT ALL TOGETHER –

Lesson 26: Strategies. You learned how to use 3. Give the reading, in inches, for A, B, C, and D.
defective rulers, how to solve problems without meas-
urements that nevertheless use conversion fractions, A B C D

and how to convert a series of units.

5 6 7

Practice

Short Answers 4. Read the ruler at point A.


Answer the questions as indicated. Answers begin on
page 240. A

A B C D 1 2 3

1 2 3
5. Bob weighs 95 pounds. What is a good
estimate of his metric weight?
a. 19 kg
1. Which point represents 143 ? b. 47 kg
a. A c. 470 g
b. B d. 190 mg
c. C
d. D 6. Tracy thinks that if the temperature is 20° C he
will need to wear a heavy winter coat. Change
20° C to Fahrenheit to see if he’s right.
A B C D

7. Frank measured his hand’s span and discov-


ered it was 7″. Later he measured a table and
5 6 7
discovered it was about 12 spans long and
about 3.5 spans wide. What was the approxi-
mate area of the table?
7
2. Which point represents 5 16 ?
8. Change 6 yards, 2 feet to feet.
a. A
b. B
9. The bathroom scale gave Kelly’s weight as
c. C
125.7 pounds. What is her weight in pounds
d. D
and ounces? Round to the nearest ounce.

238
8th_GRDM_15.qxd:Layout 1 8/11/09 4:20 PM Page 239

–PUTTING IT ALL TOGETHER –

10. Corinne was using a special measuring device Extended Response


in shop class, and it was marked like the Write a response that fully explains how you arrived
picture below. She started measuring at the at your answer. Show all diagrams or tables that you
lowest point showing here on the ruler. What is made as well as all calculations.
the length of a piece of wood that extends to
point A? 14. The playing surface of a football field measures
160 feet by 360 feet. The maintenance company
A
charges by the acre. How many acres is this
field, to the nearest tenth of an acre? Use the
2 3
conversion table that follows.
1

LAND MEASURE CONVERSIONS

9 sq. ft. = 1 sq. yd.


a. 0.4
b. 151 30.24 sq. yd. = 1 sq. rod

c. 165 160 sq. rods = 1 acre


d. 0.9

11. Laura is 5′4″. What is her height in centimeters,


to the nearest tenth of a centimeter?

12. How many square feet are in 5 square yards?

Short Response
Write a response that explains how you arrived at
your answer.

13. The leaders of the boys and girls club are


taking two busloads of students to Chicago.
There will be 40 students on each bus. Halfway
there, they plan to give each student one cereal
bar. The bars come six to the package and
packages cost $2.50. What will be the total cost
of the bars?

239
8th_GRDM_15.qxd:Layout 1 8/11/09 4:20 PM Page 240

– PUTTING IT ALL TOGETHER –

Answers 13. $35


Answers may vary, but give yourself extra credit
1. c. Other measurements: A, 43 ; B, 183 ; D, 283 . if you used conversion fractions. Here’s a sample.
2. a. Other measurements: B, 5 87 ; C, 6 41 ; D, 7. To find out the total cost, you need to
3. A, 413 3 3 3 know how many boxes of candy bars will
16 ; B, 516 ; C, 58 ; D, 54 .
4. 1.3. (Units aren’t given.) be needed, because the price per box is
5. b. A kilogram is close to 2 pounds, so Bob’s the only money figure given. To find that
weight in kilograms would be only about out, you can start a series of conversion
half of what it is in pounds, and half of 95 fractions from the number of two
is 47.5. busloads to the number of packages:
6. 68°—not usually considered weather in which 2 busloads
× 401 busload
students 1 bar
× 1 stu
1 dent
to wear a heavy coat. Use the conversion 1 pack × 40
formula F = 59 C + 32. Then × 6 bars = 62packs
F = 59 × 20 + 32 80 ÷ 6 = 1313 packs, so round up to 14 so
F = 36 + 32 everybody could get a bar. And 14 × $2.50
7. About 2,058 square inches. The table was = $35.
about 12 × 7″ = 84″ long and 3.5 × 7″ = 24.5″ 14. 1.3 acres
wide. The area was approximately 84″ × 24.5″ Answers may vary, but give yourself extra
= 2,058 in.2. credit if you used conversion fractions. Here’s a
8. 20 feet sample.
There are 3 feet in each yard, and 18 + 2 = 20 First, draw a picture.
feet.
9. 125 pounds, 11 ounces
7
There are 16 ounces in 1 pound, so 10 = 16x and
10x = 112. So .7 of a pound is 11.2 ounces.
10. b. It looks like she started measuring at the 35
(or 0.6) mark, and A is at 145 , or 1.8. 145 – 35 =
115.
11. 162.6 cm
Laura is (5 × 12)″ + 4″ = 64″ tall, and there are
2.54 cm/inch.

1 × 1 in. = 162.56 cm
64 in. 2.54 cm
360

2
12. 45 ft.
(3 feet)2 9 feet 2
= is the conversion factor.
(1 yard)2 1 yard 2
5 yards 2 9 feet 2
So 1 × 1 yard 2 = 45 feet 2 .

160

240
8th_GRDM_15.qxd:Layout 1 8/11/09 4:20 PM Page 241

–PUTTING IT ALL TOGETHER –

Unfortunately, that didn’t help much because IF YOU MISSED THEN STUDY
you need to convert to larger units, not smaller
ones, so you can’t divide up the field. So I Question 1 Lesson 24

started by figuring out how many square feet Question 2 Lesson 24


the field was. It was 360 ft. × 160 ft., because of
Question 3 Lesson 24
the formula for the area of a rectangle. 360 ×
160 = 57,600 sq. ft. Then I used conversion Question 4 Lesson 24
fractions based on the table:
Question 5 Lesson 25
57,600 sq. ft. 1 sq. yd. 1 sq. rod
1
× 9 sq. ft.
× 30.24 sq. yd.
Question 6 Lessons 24, 25
× 1601sq.
acre
rods
= 1.3228 acres
Question 7 Lesson 25
As a check, I know that the number of acres has
Question 8 Lesson 24
to be a lot smaller than the square feet, because
square feet are a lot smaller than acres. In fact, Question 9 Lesson 24
the number would have to be smaller by a
Question 10 Lesson 26
factor of about 10 × 30 × 160 = 48,000, and 48 57

is just over 1. Question 11 Lesson 24

Question 12 Lesson 24

Question 13 Lesson 26

Question 14 Lessons 24, 26

241
8th_GRDM_15.qxd:Layout 1 8/11/09 4:20 PM Page 242
8th_GRDM_16.qxd:Layout 1 8/11/09 4:22 PM Page 243

5
S E C T I O N

DATA AND
PROBABILITY

D ata are all around us. Businesses make decisions on data they have collected, and even the decisions
of school boards are often made on the basis of data about how many kids are in the school system,
what they and their parents expect from the system, and what other local people, such as employers,
expect from the system. In fact, you make daily decisions based on data as well. If you’re watching your weight,
for example, your data would be the reading on your scales and the calorie information on the labels of the food
you eat. Your success in reaching your weight goals would, in part, depend on your ability to interpret the data.
In this section you’ll learn how data can be interpreted and presented.
You’ll also learn about probability in this section. Calculating the probability that an event will occur is
presented in Lesson 29. Probability, when calculated, is always accurate—if the facts are well-interpreted. Know-
ing how to calculate probabilities is very important if you want to make wise decisions in life, but it’s no guar-
antee that those decisions are going to turn out well.

243
8th_GRDM_16.qxd:Layout 1 8/11/09 4:22 PM Page 244

–DATA AND PROBABILITY–

By the end of this section, you should have these ■ constructing a line from a scatter plot
basic skills: ■ calculating probabilities of single and multiple
events
■ identifying important parts of a data set ■ strategies for solving problems that involve data
■ understanding how best to display data for and probability
certain purposes

WORDS YOU SHOULD KNOW

combination mean random

correlation median range

data mode scatter plot

factorial outlier stem-and-leaf plot

frequency permutation

histogram probability

244
8th_GRDM_16.qxd:Layout 1 8/11/09 4:22 PM Page 245

DATA:

28 DESCRIBING,
L E S S O N

DISPLAYING,
AND
INTERPRETING

L E S S O N S U M M A RY
In this lesson you will review some terms to describe sets of data.
You will also learn some ways of presenting data in graphs and ta-
bles and how to choose which to use. You’ll also learn how to
make a scatter plot to help interpret how useful data are.

D ata are bits of information. When you put them together, you get a data set. To interpret a data set,
though, you need to know certain things about it, such as where it begins and ends and what’s in the
middle. Then, you might need to display that data set in a meaningful and accurate way. And some-
times you might want to see how one set of data is related to another set, and you’ll learn a way to do that, too.

Describing Data Sets

A data set consists of bits of information, or data. These data are sometimes called statistics. If a data set consists
entirely of numbers, such as grades on a science test, we have some terms that help us describe the data set. As
an example, here’s an unordered list of the 21 final exam grades in Ms. Stewart’s science class:

67, 78, 56, 97, 100, 92, 65, 88, 75, 48, 84, 88, 88, 72, 70, 66, 92, 88, 73, 80, 83

245
8th_GRDM_16.qxd:Layout 1 8/11/09 4:22 PM Page 246

–DATA: DESCRIBING, DISPLAYING, AND INTERPRETING–

The first possibly useful attribute of this data set is The median is the middle value in the data set
the range. The range is found by subtracting the low- when the values are arranged in numerical order. The
est value, or minimum, in the set from the highest median of the data is 80, which is the 11th number in
value, or maximum. In this set, the maximum is 100, rearranged order, counting from the minimum up.
and the lowest value is 48:
48, 56, 65, 66, 67, 70, 72, 73, 75, 78, 80, 83, 84,
range = 100 – 48 = 52 88, 88, 88, 88, 92, 92, 97, 100

The range in this set tells us that that there’s a large Here we have an odd number of data (21). What if we
difference between the best performance and the added another 88, so that we had had 22 values? Then
worst on this particular test. For example, in this data our set would look like this:
set, it might tell us that there’s a wide gap in the
amount of work done by the person with the highest 48, 56, 65, 66, 67, 70, 72, 73, 75, 78, 80, 83, 84,
score and the person with the lowest, or in their rela- 88, 88, 88, 88, 88, 92, 92, 97, 100
tive science intelligences, or maybe in their health
over the last couple of weeks. But the data by them- Now we have an even number of values (22). So if
selves don’t tell us any of those things; data are only we count from the minimum up and from the max-
useful if used with other factors. imum down, we get two different values (80 and
Another possibly useful attribute is the mode. 83), and the median is the average of those two val-
The mode is simply the value that appears most often ues: 80 +2 83 = 81.5.
in the data. In this case, the mode is 88, because it ap- With this data set, the mean and median are
pears four times. As in most small data sets, the mode very close together, so they’re probably both fairly
doesn’t appear to be very helpful in this data set, but good measures of the center. But take a look at this set
it might be—and it often appears on standardized of exam scores (in order):
tests, so you should remember how to find it.
The mode is sometimes called a measure of cen- 45, 49, 82, 87, 90, 91, 94, 97, 97
tral tendency, meaning that it tells us something
about where the center of the data is—something The mean is 81.3, but the median is 90. In fact, seven
that’s usually worth knowing. Two better indications of the nine students scored higher than the mean,
of where the center lies are the mean and the median. which was pulled way down by just the two poor
The mean is the average value of the data. To find the scores. In this case, the median is a more effective way
mean, you have to add all the values together and of looking at the center if you want to see how the
then divide by the total number of values. You nor- class as a whole did on the exam.
mally need a calculator to find the average, especially
if the numbers are large. The sum of the 21 numbers
in the data set is 1,650. Before you divide by 21 to get
the mean, make an estimate of what the mean is. Now
use your calculator. Did you come within 5 points? If
so, you have a good sense of numbers. The answer is

mean = 1,650
21 = 78.57 (rounded to nearest
hundredth)

246
8th_GRDM_16.qxd:Layout 1 8/11/09 4:22 PM Page 247

–DATA: DESCRIBING, DISPLAYING, AND INTERPRETING–

Exercise 1 A pictorial way to look at frequencies is with a his-


Give the range, mode, mean, and median of each togram, which is a bar graph with the category listed
of the following data sets, rounded to the nearest under the bar and the relative frequency shown on
hundredth. Answers are on page 252. the y axis.

.60
1. 4.25 3.6 3.9 8.18 3.72 3.07 2.84 5.6 4.75
.50
2. 58 61 64 62 62 64 58 57 58 60

Relative Frequency
.40

3. 406 233 492 445 445 448 .30

.20
Frequencies
Although data sets often consist of numbers, they .10
may also be anything else that’s descriptive. For ex-
ample, a survey of your friends’ eye colors might yield Brown Blue Green Gray
a data set like this:

blue, brown, blue, brown, green, brown, gray,


Exercise 2
Find the relative frequencies (to the nearest hun-
brown, brown, green
dredth) in the eye-color survey shown in the follow-
ing table and draw a histogram to show the data. (You
In math, of course, we like numbers, so we like to de-
may use a calculator or a spreadsheet program.) An-
scribe data sets like this with numbers, too. We use
swers are on page 252.
the word frequency to make sense of this data. Fre-
quencies tell us how many from a class or category
RELATIVE
with certain attributes there are in the given survey. EYE COLOR FREQUENCY FREQUENCY
The term relative frequency gives us the proportion
(the same as percentage without multiplying by 100; blue 223 ?

see Lesson 3) for each class. A frequency table for the brown 341 ?
preceding data would look like this.
green 199 ?

EYE COLOR RELATIVE gray 44 ?


( CATEGORY ) FREQUENCY FREQUENCY
Total 807 1.0
blue 2 0.2

brown 5 0.5

green 2 0.2

gray 1 0.1

total 10 1.0

247
8th_GRDM_16.qxd:Layout 1 8/11/09 4:22 PM Page 248

–DATA: DESCRIBING, DISPLAYING, AND INTERPRETING–

Displaying Data This provides a pretty clear picture of how much was
raised in each grade, but you might also want the in-
You’ve just seen some ways to display data, but there formation in graphical form.
are many others. Some use tables, and others use Here’s a line graph of the same data.
graphs. In Lesson 10, you used Input/Output tables to
help define functions. Certain data can be displayed $450
in the same way. Here’s a table showing how much 400

money each grade raised in the school fundraiser. 350


300
250
GRADE $ 200
150
K 320 100
50
1 389
K 1 2 3 4 5 6 7 8
Grade
2 430

3 487
This graph shows at a glance that the money raised
peaks in the middle grades 3–4; you would have had
4 485 to read all the numbers in the table to see that pattern
5 396 without the graph, so the graph is effective for con-
veying patterns quickly. However, it’s hard to know
6 345 the exact numbers from the graph, so the table would
7 276 be more effective if you had to do calculations with
the exact numbers.
8 350
Here’s another set of data: the amounts spent by
a school board, in millions of dollars, broken down
into five different categories.

ACADEMIC
ACADEMIC SUPPORT ADMINISTRATION MAINTENANCE OVERHEAD

26.3 6.9 11.9 6.1 5.5

248
8th_GRDM_16.qxd:Layout 1 8/11/09 4:22 PM Page 249

–DATA: DESCRIBING, DISPLAYING, AND INTERPRETING–

Here’s one possibility for displaying the data, a bar 1’s digit), and the stem, which consists of the remain-
chart: ing digits. Here are the scores of a fictional basketball
team through its first 20 games:
Expenditures in Millions of $
30

25 91 67 93 101 88 94 91 107 118


95 94 89 94 98 96 110 83 90
$ (millions)

20

15 81 99
10

5 And here’s the stem-and-leaf plot.


0
ic

d
or

STEMS LEAVES
nc
io

a
m

he
pp

t
de

na
ra

er
Su

ist

te
a

Ov
Ac

in

ain
ic

m
m

6 7
M
Ad
ade
Ac

7
This bar chart makes the money spent on academic
8 1, 3, 8, 9
pay look much larger than the other categories, but
what if you wanted to show that academic pay was 9 0, 1, 1, 3, 4, 4, 4, 5, 6, 8, 9
less than half of the school budget? You could use a
10 1, 7
circle graph like this:
11 0, 8
Expenditures FY 2009–2010 (million $)
Overhead
5.5 The score 67 is represented by the 6 in the Stems col-
Maintenance Academic umn and the 7 in the Leaves column. The score of 110
6.1

Academic
Academic Support is indicated by the 11 in the Stems column and the 0
26.3
Administration in the Leaves column. From this table, it’s easy to see
Maintenance
Administration
11.9 that the team usually scores in the 90s, and the 94,
Overhead
which occurred three times, is the mode. It’s also easy
Academic Support
6.9
to find the median in a stem-and-leaf, because the
numbers are already in order. Just count up 10 num-
This graph still shows the same facts, but now anyone bers from the minimum or down 10 numbers from
who looks at the graph can see that teacher pay is less the maximum, and you’ll see that the median is 94.
than half the budget without having to add up the The mean, however, still has to be calculated; there’s
other categories. usually no shortcut to finding the mean in a data set
with more than four or five numbers.
Stem-and-Leaf Plot
The stem-and-leaf plot is one method to display
scores in groups. It gives a picture of the distribution
of scores, as well as the exact data in compact form.
Each number is broken into a leaf, which is the digit
with the least value (in the following example, it’s the

249
8th_GRDM_16.qxd:Layout 1 8/11/09 4:22 PM Page 250

–DATA: DESCRIBING, DISPLAYING, AND INTERPRETING–

Exercise 3 (1) What was the approximate amount of


(Answers on pages 252–253.) money awarded in 2000? (2) How much did
the money increase between 2001 and 2004?
1. Look at this line graph and answer the (3) What is the approximate percentage
following questions. increase between 2001 and 2004? (4) What is
the approximate percentage decrease between
Scholarship Money Awarded at Little College 2004 and 2007?
200

2. Arrange the following results of lab tests for


$ (Thousands)

150
percent of solution (to the nearest hundredth)
100 in a stem-and-leaf plot. What is the median of
this data set? (Hint: The hundredths place
50
should be the leaf.)
2000 2001 2002 2003 2004 2005 2006 2007
Grade 9.76, 10.13, 9.60, 9.92, 10.01, 10.15, 9.79,
10.03, 9.56

Scatter Plots

Suppose we take the 20 basketball scores from the preceding stem-and-leaf plot and put them alongside the
margin of victory or defeat for the team. (A margin with a negative number was a loss for the team; the team’s
record was 12 wins and 8 losses.)

91 67 93 101 88 94 91 107 118 95 94 89 94 98 96 110 83 90 81 99

9 –11 24 –4 6 3 10 –1 6 –8 9 10 7 –6 3 –9 1 14 –10 –7

We’ll now plot the scores on the x, or horizontal, axis (92,24)


25
of a graph, and the margins on the y axis. We’ll get 20
Margin

what’s called a scatter plot, which consists of a dis- 15


10
connected points with two coordinates—in this case
5
team scores and margins of victory/defeat. 0
60 70 80 90 100 110
–5 (101,–4)
–10
(67,–11)

Scores

250
8th_GRDM_16.qxd:Layout 1 8/11/09 4:22 PM Page 251

–DATA: DESCRIBING, DISPLAYING, AND INTERPRETING–

The object of a scatter plot is to find correlation, or a


relation that tells if and how two variables (in this
case team score and margin of victory/defeat) are
connected. You can find correlation by drawing a
straight line through most of the data, if that’s possi-
Positive Negative No
ble. In this plot, we see that most of the dots fall into a Correlation Correlation Correlation
pattern, but to draw a line that shows the general
trend, we’ll have to ignore some points: (67,–11), Note: Correlation is not the same as cause-and-
(81,–10), (92,24), and (118,6). Notice that the line effect. A team’s inability to win when it scores more
drawn through the center of the data goes down as it points does not say that scoring more points causes
goes to the right. the team to lose. There may be any number of other
reasons for this correlation, and one would have to be
25
(92,24) an expert in basketball to sort out which causes were
Outliers
20 more valid.
Margin

15
10
5 Exercise 4
60 70 80 90 100 110
(Answers on page 253.)
–5 (101,–4)
–10
(67,–11) 1. Here is a table showing years of teacher
Outliers experience in the top row and scores on a
Scores certain standardized seventh-grade social
studies test in District 4 in the bottom.
If you remember Lesson 12, you’ll know that this line Construct a scatter plot and identify whether
has a negative slope. That is, as our team’s point total there’s a positive or a negative correlation
gets higher, its margin of victory goes down; and, in between experience and higher scores. Identify
fact, the team is even more likely to lose! Thus, we say the outliers (points that don’t seem to fit the
that there’s a “negative correlation between points trend), if any.
scored and margin of victory.”
Here are some scatter plots showing different 2 14 1 6 8 3 16 10 6 2 2
kinds of correlations. When the line has a positive 74 85 70 85 87 80 77 89 82 75 79
slope, we say there’s a positive correlation, and when
the slope is negative, we say the correlation is nega-
tive. If the line goes more or less straight across—that
is, the slope is 0—we say there’s no correlation.

251
8th_GRDM_16.qxd:Layout 1 8/11/09 4:22 PM Page 252

–DATA: DESCRIBING, DISPLAYING, AND INTERPRETING–

2. Here is a table showing total acreage (in whether there’s a positive or a negative
hundreds) planted in corn (top row) and correlation between high corn production and
wheat in AnyCounty, Midwestern state, during high wheat production. Identify the outliers, if
the 1990s. Construct a scatter plot and identify any.

1990 1991 1992 1993 1994 1995 1996 1997 1998 1999

38 46 44 50 49 51 47 45 48 45

9 8 12 7 7 4 10 6 3 5

Summary Exercise 2
RELATIVE
In this chapter you learned what data are and some EYE COLOR FREQUENCY FREQUENCY
reasons that data sets and statistics are important. blue 223 0.28
You also learned something about describing sets of
data and how to present data in graphs and tables— brown 341 0.42

and to interpret data already in graphs and tables. You green 199 0.25
also learned how to make scatter plots to determine
gray 44 0.05
the correlation of two variables.
.50

Answers .40
Relative Frequency

.30
Exercise 1
1. The range is 8.18 – 2.84 = 5.34. There is no .20

mode, because no values are repeated. The


.10
mean is 39.91 ÷ 9 = 4.43. The median is the
middle value, 3.9.
Brown Blue Green Gray
2. The range is 64 – 57 = 7. The mode is 58,
which occurs three times. The mean is 604 ÷ Exercise 3
10 = 60.4, and the median is the average of the 1. (1) about $76,000; give yourself credit if you’re
two middle values (60 and 61): 60 +2 61 = 60.5. within a thousand. (2) The money increased
Note that this data set is very closely bunched, from $75,000 to about $150,000, or $75,000.
and that the mean and median are very close to (3) Recalling Lesson 3, the way to find percentage
each other. increase is to divide the increase by the original:
75,000 = 1, and the percentage increase is 1 × 100,
3. The range is 492 – 233 = 259. The mode is 445, 75,000

which occurs twice. The mean is 2,4696 = 411.5, or 100%. (4) The percentage decrease is the
and the median is the middle value, 445. Note amount of decrease (which is about $150,000 –
that a single low number (233) makes the $125,000 = $25,000) divided by the original
25,000
mean much lower than the median. amount, $150,000: 150,000 = 61, or about 16.67%.

252
8th_GRDM_16.qxd:Layout 1 8/11/09 4:22 PM Page 253

–DATA: DESCRIBING, DISPLAYING, AND INTERPRETING–

2. STEMS LEAVES 2. There is a slight negative correlation between


the corn and the wheat acreage. In other words,
9.5 6
when more corn is planted, less wheat is
9.6 0 planted. This makes sense when the total
number of acres is more or less stable. That is,
9.7 6, 9
the farmers of this county can’t take advantage
9.8 of high prices in a given year by planting more
9.9 2
of everything, because they can’t fit more crops
into the available land.
10.0 1, 3

10.1 3, 5

Wheat acres
Perhaps the hardest part of constructing this 10
plot is remembering to include both a 10.0 and 5

a 10.1 as stems. Remember that the leaf can be 5 10 15 20 30 40 50


just one digit, so the stems have to carry all the Corn acres

other information. The median is 9.92, which


is fifth from both the top and the bottom.

Exercise 4
1. There is definitely a positive correlation
between teacher experience and higher scores
in this data set—at least between two years and
ten years. The point (16,77) is definitely an
outlier, and you might also call (1,70) and
(14,85) outliers as well.

85
80
75

1 2 3 4 5 6 7 8 9 10 11 12 13 14 15 16

253
8th_GRDM_16.qxd:Layout 1 8/11/09 4:22 PM Page 254
8th_GRDM_17.qxd:Layout 1 8/11/09 4:23 PM Page 255

29
PROBABILITY,
L E S S O N

PERMUTATIONS,
AND
COMBINATIONS

L E S S O N S U M M A RY
In this lesson you will learn what probability is and how to deter-
mine it if all the facts are known. You will also learn something
about interpreting surveys and how to calculate the probabilities
that multiple events will occur.

P robability, roughly speaking, is the likelihood that something will happen. Probability is expressed as a
decimal, and it’s defined as the number of ways the event will occur divided by the number of events
that can occur:

P( A) =
number of ways A can occur
total number of event that can occur
Note that no probability can be greater than 1, because the denominator will always be at least as big as the nu-
merator. In this lesson, you’ll learn how to count the number of events that can occur: That’s the key to calcu-
lating probability. You’ll also learn about permutations, or ways to arrange things, and combinations, which are
ways to group things when their order isn’t important.

255
8th_GRDM_17.qxd:Layout 1 8/11/09 4:23 PM Page 256

–PROBABILITY, PERMUTATIONS, AND COMBINATIONS–

The Whole and the Parts outcomes. But if your outcomes were defined as two-
digit numbers, always taking the red die first, then
No doubt you have thrown a pair of six-sided dice in (1,5) would be different from (5,1).
some game or other. You know that each die has six Sometimes, the number of outcomes is too high
sides, and each is equally likely to come up when you to count. What if you were rolling five dice? How
throw the die. Remember, though, that here and in many possible outcomes would you have? The first
other similar examples we’ll look at, we assume that die could come up any of 6 possibilities, and so could
each side really is equally likely to come up; in practice the second, so could the third, and so on. Note that
this may not always be true. for two dice we counted 6 × 6 = 36 possibilities. If all
of the dice are independent of each other, meaning
that what you roll with one doesn’t affect the others at
Probability calculations are valid only if assump- all, then with five dice there are 6 × 6 × 6 × 6 × 6 = 65
tions about fairness are true. = 7,776 possibilities, Don’t worry; you won’t have to
list them all here. But you might have to figure out a
probability with five dice.
The possible outcomes when you throw one die are
(6), (5), (4), (3), (2), and (1). Let’s list the possible
Exercise 1
outcomes with two dice. We’ll start with 1 and the re-
Give the probabilities of the following possible results
sult on the first die and proceed systematically:
of throwing two fair dice. Answers are on page 261.

(1,1) (1,2) (1,3) (1,4) (1,5) (1,6)


1. 7
(2,1) (2,2) (2,3) (2,4) (2,5) (2,6)
(3,1) (3,2) (3,3) (3,4) (3,5) (3,6)
2. 8
(4,1) (4,2) (4,3) (4,4) (4,5) (4,6)
(5,1) (5,2) (5,3) (5,4) (5,5) (5,6)
3. 9
(6,1) (6,2) (6,3) (6,4) (6,5) (6,6)
4. 5
This makes a total number of 36 possible dice
rolls using two dice. If we want to find the probability
5. 12
of throwing a 10 with two dice, we next have to list all
the combinations that add up to 10: (4,6), (5,5), and
3
Give the possible number of outcomes for the
(6,4). So the probability of throwing a 10 is 36 , or .083.
following:
Note that the probability of throwing a 10 is limited
by the fact that you can’t throw a 1, 2, or 3 at all and
6. throwing three fair dice
still get a 10. The probability of getting a 6 with two
dice is much higher. Let’s count the ways: (1,5), (2,4),
7. picking a card from a standard deck
(3,3), (4,2), and (5,1). The probability of getting a 6 is
thus 365
≈ .139.
8. tossing a coin three times to see if it lands
The hardest part of figuring probabilities is of-
heads or tails.
ten finding the total number of outcomes, but some-
times it’s also counting the event you want. For
9. What are the chances that you get exactly two
example, if you threw two dice that were red and
heads when you toss a coin three times?
green, respectively, you would still have 36 different

256
8th_GRDM_17.qxd:Layout 1 8/11/09 4:23 PM Page 257

–PROBABILITY, PERMUTATIONS, AND COMBINATIONS–

Random Surveys Exercise 2


A related probability problem is the survey. Using ra- A poll is taken in Chicago asking people at random
tios (see Lesson 3 if you have forgotten them), you who will win the National League baseball pennant.
might be able to figure out what a whole group will Subjects are selected at random from around
do, based on a survey of just a part of that group. For Chicago, and it turns out that the number of men and
example, if you have taken a statistically valid survey women are balanced, and that ethnic ratios are about
and found that 48% of the people surveyed will vote the same as in the city as a whole. In the poll, the
for your candidate, how many votes can you reason- Chicago Cubs are picked by 43%, the Los Angeles
ably expect your candidate to get when 25,000 people Dodgers by 19%, the Colorado Rockies by 17%, the
actually vote? The answer is 0.48 × 25,000 = 12,000. Milwaukee Brewers by 12%, and other teams split the
To be valid, a survey has to be remaining 9%. Is this poll accurate? Answers are on
page 261.
■ random, meaning that the subjects are chosen
without prejudice.
■ large enough to be statistically significant. Probability of Serial Events

Surprisingly, perhaps, it’s been mathematically proven Sometimes, you want to know the probability of two
that a survey doesn’t have to take in a very high per- events occurring at the same time. Sometimes, you
centage of the total population of interest to be valid. may want to know the probability that two events will
The hard part is keeping it random. For example, any occur in a certain order. Standardized tests very often
survey should be chosen completely at random and want to know that you understand how to calculate
then checked for representation among all possible these probabilities. This lesson will teach you how.
groups. Asking members of the Beyonce Fan Club
obviously would not be a good sample if you were Outcome Trees
trying to find out who the student body’s favorite You may remember the factor trees from Lesson 1.
singer was. Nor would asking only the basketball Outcome trees look very much the same, except that
team be a good way to find out what students’ fa- they’re turned on their sides. They can be used to
vorite sport was, or even their favorite movie. compute how many possible outcomes there are for a
In your American history book, you may have given event, and they can also be used to find the
seen President-elect Harry Truman holding up a probability of a certain sequence of events. For the
newspaper whose headline read, “Dewey Defeats tree to be valid, each level of it has to concern an
Truman.” That newspaper was basing Dewey’s vic- event that is independent of the event that preceded it.
tory on a poll that was conducted by readers of a For example, here’s the outcome tree for simul-
certain magazine who had mailed in their choices. taneously flipping a coin and rolling one 6-sided die.
As it turned out, the headline was wrong. Nowadays
newspaper writers have become smart enough to
know that polls in which the participants are self-
selected are not good indicators of how they should
write their headlines.

257
8th_GRDM_17.qxd:Layout 1 8/11/09 4:23 PM Page 258

–PROBABILITY, PERMUTATIONS, AND COMBINATIONS–

1 Here’s the tree:


2
Green
3
H
4

5
Green
6

2
Green
3
T
4

6 Red

There are 12 possible outcomes. Let’s use this tree to


find the probability that you will get heads on the
Red
coin flip and throw a 4 or higher on the die. Follow
the tree to H and then count the number of outcomes
connected to the H that have a value of 4 or higher on Green
Green
the die. You’d count three possibilities, and your Blue
Green
3 Red
probability would be 12 = .25. How about the proba- Red
bility of getting tails on the coin flip and a 2 on the Blue
Green
die? That’s right, you would find only one possible Green
1 Green
leaf on the tree, and your probability would be 12 . Blue
Red
Let’s do one more. For this one, you’ll have a box Red
Blue
that contains 3 green balls, 2 red balls, and 2 blue balls.
You’re going to take the balls out one at a time, and
you won’t be putting the ones you take out back in. In this tree, the second level is drawn only for the blue
balls. You can see that the first level has seven choices,
because you could pick any one of the seven balls. If
you pick a blue one first, then you’ll have just six possi-
bilities left: 3 greens, 2 reds, and only 1 blue. You would
get a six-forked tree for any ball you picked first, so the
number of possibilities for picking two balls at random
from the box is 42. Look at the tree to see the probabil-
ity of choosing a blue ball followed by a red ball. There
are four possibilities for picking a blue first and a red
second, so the probability is 424 , or .095.
But you can already see that we’re having to do a
lot of drawing, and that it would be hard to find prob-

258
8th_GRDM_17.qxd:Layout 1 8/11/09 4:23 PM Page 259

–PROBABILITY, PERMUTATIONS, AND COMBINATIONS–

abilities by constructing a tree every time. So here are 6. In the box described in question 5, what is the
a couple of rules to help you evaluate probabilities. probability of drawing first a red ball, then a
green ball, and then a blue ball?
■ To calculate the probability that event A and in-
dependent event B will occur, you multiply the 7. In the box described in question 5, what is the
individual probabilities: probability of drawing first a red ball, then a
P(A and B) = P(A) × P(B) green ball, and then a blue ball or a green ball?
■ To calculate the probability that event A or inde-
pendent event B will occur, you add the individ-
ual probabilities: Permutations and Combinations
P(A or B) = P(A) + P(B)
Permutations are the different ways things can be
Let’s try these rules on the examples we’ve just done. arranged. For example, drawing first a green ball, then a
To find the probability of tossing a coin to be tails and red, and then a blue is one of the possible permutations
throwing a 2 on the die is P(T and 2) = 12 × 16 = 12 1
. To of drawing balls from the box described in the last part.
find the probability of tossing a coin to be heads and As another example, how many two-letter words of
a die to be a 4, 5, or 6, we use P(heads and (4, 5, or 6)) the letters w, x, y, and z can you construct, using each
= 1 × ⎛⎜ 1 + 1 + 1 ⎞⎟ = ⎛⎜ 3 ⎞⎟ = 3 . Both of these computations one of the letters only once? (Of course, these won’t be
2 ⎝ 6 6 6⎠ ⎝ 6⎠ 12
agree with our counting leaves on the tree. The com- real words that you’d find in the dictionary, but they
putation for a blue ball followed by a red ball from the will be unique two-letter sequences.) First, let’s think it
box of 3 greens, 2 reds, and 2 blues will be P(1 blue through logically to figure out the answer, and then
and 1 red) = 72 × 26 = 42
4
. The 72 describes the probability we’ll list them all as a check. Any one of the four letters
of drawing a blue ball on the first try, and the 26 de- could come first in the combination. Then there will be
scribes the probability of drawing a red ball from the only three letters left, and any one of them could come
six left on the second try. next. Then we have finished, because we’re talking
about two-letter permutations. So you would have 4 × 3
Exercise 3 = 12 possibilities. Let’s check it out:
(Answers are on pages 261–262.)
wx wy wz xw
1. Draw an outcome tree for four consecutive xy xz yw yx
tosses of a coin. yz zw zx zy

2. How many outcomes are possible in this tree? Can you think of others? Not likely—12 is the logical
and visual answer. Now let’s think of how many
3. How many of the outcomes have exactly three three-letter words we can make from w, x, y, and z,
heads? without repetition. First, we have four letters to choose
from, then three, and then two, so our answer should
4. How many outcomes have exactly two heads? be 4 × 3 × 2 = 24. Let’s list them; we do have 24.

5. In the box with 3 green balls, 2 red balls, and 2 wxy wxz wyx wyz wzx wzy
blue balls, how many possible outcomes are there xwy xwz xyw xyz xzw xzy
for drawing 3 balls without replacing the balls? ywx ywz yxw yxz yzw yzx
zwx zwy zxw zxy zyw zyx
259
8th_GRDM_17.qxd:Layout 1 8/11/09 4:23 PM Page 260

–PROBABILITY, PERMUTATIONS, AND COMBINATIONS–

Now, something that might seem a bit strange hap- Exercise 4


pens if we ask how many four-letter permutations we (Answers are on page 262.)
can make from the same letters. The answer is 4 × 3 ×
2 × 1 = 24—the same number as the three-letter per- 1. From the digits 01234, how many different
mutations. That’s because there was only one choice two-digit numbers can you make, if no
left for each of the three-letter permutations. Here are numbers are used twice?
the four-letter words, in the same order as the three-
letter permutations: 2. From the digits 0123456789, how many two-
digit numbers can you make, if numbers are
wxyz wxzy wyxz wyzx wzxy wzyx allowed to be used twice?
xwyz xwzy xywz xyzw xzwy xzyw
ywxz ywzx yxwz yxzw yzwx yzxw 3. From the digits 0123456789, how many two-
zwxy zwyx zxwy zxyw zywx zyxw digit numbers can you make, if you can’t start
any of them with 0, and no number can be
A combination is an arrangement of things in used twice?
which the order doesn’t matter. For our two-letter
arrangements of w, x, y, and z, wx and xw were differ- 4. From the digits 56789, how many three-digit
ent. If we’re considering only combinations, wx and xw numbers can you make that are even, if no
count as the same thing. Put another way, if you’re play- digit is used twice? (Hint: Work backwards.)
ing a card game where your opponent can’t see your
hand, and you’re holding a king and an ace, it doesn’t 5. From the digits 56789, how many three-digit
matter whether you got the ace first or the king. numbers can you make that are even, if
So how many two-letter combinations do we numbers are allowed to be used twice or even
have for our four letters? We can again put any of four three times?
in the first place, and any of three in the second, but
then exactly half of our words will be repetitions, and 6. From the digits abcd, how many two-digit
there are only six words we can count as combina- words can you make, if no numbers are used
tions. Here’s the list: twice and the order of the numbers is
disregarded?
wx wy wz xw
xy xz yw yx
yz zw zx zy Summary

Similarly, we have to cross out 20 of the three- In this lesson you have learned how to count the
letter words if order isn’t important, leaving us with number of possibilities that an event will occur, as
only four three-letter combinations: well as all the possibilities that a similar but different
event will occur. This is the secret to calculating prob-
wxy wyz wxz xyz abilities. Once you had mastered that task, you
learned to figure out probabilities of A and B and of A
And there’s only one four-letter combination possi- or B. Finally, you learned how to figure the number of
ble: wxyz. permutations in a given situation.

260
8th_GRDM_17.qxd:Layout 1 8/11/09 4:23 PM Page 261

–PROBABILITY, PERMUTATIONS, AND COMBINATIONS–

Answers Exercise 3
1. H
H
Exercise 1 T
H
1. There are six ways to get 7: (1,6), (2,5), (3,4), H
(4,3), (5,2), and (6,1). So P(7) = 36 6
≈ .17. T
T
2. There are five ways to get 8: (2,6), (3,5), (4,4), H
H
(5,3), and (6,2). So P(8) = 36 5
≈ .14. H
3. There are four ways to get 9: (3,6), (4,5), (5,4), T
T
and (6,3). So P(9) = 364
≈ .11. H
T
4. There are four ways to get 5: (1,4), (2,3), (3,2),
T
and (4,1)). So P(5) = 36 4
≈ .11. H
5. There is only one way to get 12: (6,6). So H

P(12) = 361
≈ .03. H
T
H
6. The three dice are independent from one
T
another, so the number of possibilities is T
T
6 × 6 × 6 = 216. H
H
7. There are 52 cards in a standard deck, so you
T
would have exactly 52 ways of picking one T
H
card. T
8. If you toss a coin three times, your possibilities T

are (HHH), (HHT), (HTH), (HTT), (THH), 2. 16; count the individual leaves
(THT), (TTH), and (TTT). Because there are 3. 4
only two possibilities each time, and each toss (HHHT), (HHTH), (HTHH), (THHH). The
is independent of the one that went before, you easiest way to find these is to look at the leaves
would have 2 × 2 × 2 = 8 possibilities; this way, and trace a path back to the start.
you wouldn’t need to count, except as a check. 4. 6
9. Of the eight possibilities listed in the answer to (HHTT), (HTTH), (HTHT), (THHT),
question 8, three of them contain exactly two (THTH), and (TTHH).
heads, so the probability is 83, or .375. 5. 210
There are seven possible outcomes for the first
Exercise 2 ball, six for the second round, and only five for
This is not a trick question. The fact that the poll was the third round, because one ball has been
taken in Chicago is enough to make it invalid—after taken out for each round. Because these are
all, people usually root for the home team. independent events, you can multiply 7 × 6 × 5
to get the total number of outcomes.
6. In this case you have a probability of 72 on the
first round, 36 on the second round (three of the
remaining six balls are green), and 25 on the
third round (two of the remaining five balls are
blue). Because these are independent events,
the probability is 72 × 36 × 25 = 35
2
(canceling and
reducing), or .057.

261
8th_GRDM_17.qxd:Layout 1 8/11/09 4:23 PM Page 262

–PROBABILITY, PERMUTATIONS, AND COMBINATIONS–

7. The probabilities of the first two draws are the 4. 24


same as in question 6. However, the probability To begin this problem, you have to first realize
for the third draw is 25 + 25, because there are 2 that the three-digit number has to end with 6
green balls and 2 blue balls left among the or 8, the only two even numbers allowed. So
remaining 5 balls, and you can have green or the number will look like this if it ends in 6:
blue. So the probability is 72 × 36 × 55 = 35
4
__ __ 6
(canceling and reducing), or .114. Notice that You have only four choices for the first number
there is a greater probability in this question (5, 7, 8, 9) and only three of those can be used
than in question 6, which makes sense because for the second number. So there are 12
you increased the number of valid options in possibilities that end in 6. You can use the same
this question. reasoning to find that there are 12 possibilities
that end with 8. Because the question is asking
Exercise 4 for the possibilities for the number ending in 6
1. 20 or 8, you can add 12 + 12 to get 24.
You can select 5 numbers for the first place and 5. 50
any of the remaining 4 numbers for the second Like the previous problem, you have to find
place: 5 × 4 = 20. how many possibilities there are for both 6 and
2. 100 8 and add them. If 6 is the last number, you
You select any of the 10 numbers for the first have 5 choices for the first digit, and you still
digit, and you still have 10 to choose from for have 5 choices for the next, because repetition
the second digit: 10 × 10 = 100. is allowed. So you have 5 × 5 possibilities for 6
3. 81 as the final digit, and the same number for 8 as
Because 0 can’t come first, you have only 9 the final digit: 5 × 5 + 5 × 5 = 50.
numbers to use as the first digit. For the second 6. 6
digit, you have already used a number, but you You would do this the same way that it’s done
get to put 0 back into the mix, so again you in the lesson for the letters wxyz. First, list the
have 9 numbers to choose from: 9 × 9 = 81. As 12 two-digit words that you can make with the
a check, think of the 19 two-digit numbers that different orders, and then cross out the ones
you can’t use here but you could use in that are repeats, if order isn’t important. For
question 2: 00, 01, 02, 03, 04, 05, 06, 07, 08, 09, example, you would have to cross out either ab
11, 22, 33, 44, 55, 66, 77, 88, 99. or ba (but not both!).

262
8th_GRDM_18.qxd:Layout 1 8/11/09 4:24 PM Page 263

30
L E S S O N

ESTIMATING
AND CHECKING

L E S S O N S U M M A RY
In this lesson you will learn some methods for estimating probabil-
ities from probability trees, which are improved outcome trees.
You’ll also learn some formulas that will help you check whether
you’ve counted permutations and combinations correctly.

I n Lesson 29 you learned how to draw probability trees so that you could count the outcomes. In this lesson
you’ll learn how to use those probabilities to find probabilities directly. You can use these probability trees as
checks on your work or as tools for estimation. Also in Lesson 29, you learned how to count permutations
and you were introduced to combinations. As a check on your ability to count these patterns, in this lesson
you’ll learn how to use two formulas for permutations and combinations.

Finding Probability with Trees

In Lesson 29 you saw part of a tree for counting the outcomes when selecting colored balls from a box that con-
tains 3 green balls, 2 red balls, and 2 blue balls. It was easy enough to show the possibilities from the first draw,
but the second draw had so many possibilities that you would need a large sheet of paper to accommodate all of

263
8th_GRDM_18.qxd:Layout 1 8/11/09 4:24 PM Page 264

–ESTIMATING AND CHECKING–

them. If you’re interested only in probabilities, though, G


there’s a way to draw a tree that will show the probabil- 
ities and not just the total number of outcomes. 
G R
If we concentrate on the possible events rather

than on all outcomes, there are only three possibili-
ties in the first draw: You can draw a green ball, a red
 R
ball, or a blue ball. But these events have different
G
probabilities:


Green ball: 73  
R R
Red ball: 72 
Blue ball: 72
R

So, the beginning of our new tree looks like this: G

G

B R

 

R

R
Notice that all the probabilities on this round are
based on what was drawn in the first round. Thus, the

denominator of all the probabilities in the second
round is 6, because there are only six balls left, no
B
matter what color has already been drawn. Also, the
probabilities coming out of any of the event circles
If you now want to know the probability of any- add up to 1:
thing on a second drawing from the same box, you
don’t need to draw all 42 possible outcomes; you can From G: 62 + 62 + 62 = 1
just use the probabilities, like this: From R: 36 + 61 + 62 = 1
From B: 36 + 62 + 61 = 1

In fact, this is important to remember in estimating


or checking probabilities:

If counted correctly, all the probabilities that an


event will happen add to 1.

264
8th_GRDM_18.qxd:Layout 1 8/11/09 4:24 PM Page 265

–ESTIMATING AND CHECKING–

Now let’s use the tree to find some probabilities. Exercise 1


What’s the probability that a green ball will be picked Using the tree for seven balls in a box (3 green, 2 red,
first from the box, followed by a blue ball? That is 2 blue), give the following probabilities. Answers are
easy to find if the tree is drawn correctly: 73 × 62 = 71. on page 267.
What’s the probability that a blue ball will be picked
first from the box, followed by another blue ball? 1. Drawing a red ball followed by a blue
7 × 6 = 21. So far we’ve just multiplied the probabili-
2 1 1

ties, because we were considering one event and an- 2. Drawing a green ball followed by a green
other event. Now, let’s do one event or another event.
As a very easy problem, what’s the probability that a 3. Drawing a green ball on the second draw
green ball or a red ball will be taken on the first draw?
The answer is 73 + 72 = 75. Now, let’s try something 4. Drawing a blue or green ball on the second
harder: What is the probability that the second ball draw
drawn is blue, regardless of what the first is? To an-
swer this question, find all the second-round circles Exercise 2
with a B. There will be three of them. Then trace the With this experiment, you’ll be picking balls from the
path back to the start to find the probability of each. same box as before: 3 green, 2 red, and 2 blue. Then
The top B—the one at the end of the G–B path—has you’ll be flipping a fair coin (even chance of heads or
a probability of 73 × 62 = 71, as we have already calculated. tails). If the ball you picked was red, you’ll flip the
The one at the end of the R–B path has a probability coin again. Answers are on page 267.
of 72 × 62 = 21
2
. And the one at the end of the B–B path
has a probability of 72 × 16 = 21 1
. Because any one of 1. Draw the probability tree. Place a check mark
these paths will satisfy the condition that a blue ball at the end of all paths that have no heads, or H,
be picked second, the probability of a blue ball being in them.
picked second is
2. What is the probability that no coin flipped
1 2 1 3 2 1 6 2
+ + = + + = = will come up heads?
7 21 21 21 21 21 21 7
Amazingly, picking a blue ball on the second draw has
the exact probability of picking a blue ball on the sec- Permutations and Combinations
ond try! This is probably not something you would
have guessed, so it’s usually better to calculate a serial The math in the following lesson may not be needed
probability by another method rather than just seeing in eighth grade, depending on your teacher. But it’s a
if your first method produced what seems like a rea- good way to check your answers on permutations and
sonable answer. combinations, and it might be useful to you in real
life, particularly if you like to play cards.

265
8th_GRDM_18.qxd:Layout 1 8/11/09 4:24 PM Page 266

–ESTIMATING AND CHECKING–

The Permutation Formula We couldn’t use cab, for example, as an answer, be-
In the example from Lesson 29, where you were mak- cause it’s the same letters as abc just arranged in a dif-
ing two-letter words out of a set of four letters, the ferent order.
problem could be stated as counting permutations, As you might expect, the number of combina-
taking 2 out of 4. As a general statement, it’s counting tions is always less than the number of permutations,
permutations, taking r out of n. And the formula for and if you look at the formula, you’ll see why: It’s al-
that is most the same as the permutation formula, except
that it has an added factorial in the denominator:
n!
P(n,r) =
r !(n − r )! C(n, r ) =
n!
r !(n − r )!
where n!, called n-factorial, is n × (n – 1) . . . × (1). As
an example, 6! is defined as 6 × 5 × 4 × 3 × 2 × 1. The number of three-letter combinations from the 26
Using the formula to compute the number of letters of the alphabet is much smaller than the num-
permutations possible by choosing two letters out of ber of permutations:
four, we’d get
26 ! 26 × 25 × 24 × 23!
C(26, 3) = =
4! 4 × 3 × 2 ×1 3!(26 − 3)! 3! × 23!
P(4, 2) = = = 12
(4 − 2)! 2 ×1 26 × 25 × 24
= = 2, 600
3× 2
This is the same answer we got when we listed the
words in Lesson 29, and when we just used common Note: The number of combinations of 13 from
sense to get four (possibilities for the first letter) × 52 cards would be computed as C(52,13), and this
three (possibilities for the second letter) = 12. The would tell you how many different hands you could get
formula comes in handy when you need to compute in a game of bridge, or any other card game in which
large numbers of permutations, such as how many the whole deck is dealt out to four players. That num-
three-letter words you can make out of all 26 letters ber is about 6.35 × 1011—not a small number. If you
of the alphabet. The answer is wanted to calculate probabilities based on getting dif-
ferent hands, you would figure out how to form that
26 ! 26 × 25 × 24 × 23!
P(26, 3) = = hand the same way you did serial probabilities in Les-
(26 − 3)! 23! son 29 (or earlier in this lesson) to get your numera-
= 26 × 25 × 24 = 15, 600 tor and then use 6.35 × 1011 as your denominator.

Formula for Combinations Exercise 3


In Lesson 29 you learned that combinations were al- A meeting of a club at school has ten people attend-
most like permutations except that order doesn’t ing. They decide that the club’s officers (president,
matter. For instance, how many three-letter combina- secretary, and treasurer) will be named from among
tions can you make from the letters a, b, c, and d if the the ten present. But no one wants to volunteer, so
order isn’t important? The answer is 4: they decide to hold a lottery. How many ways could
the three officers be chosen if no one is allowed to
a a a b c hold more than one office? Answers are on page 267.
b c b d c d d

266
8th_GRDM_18.qxd:Layout 1 8/11/09 4:24 PM Page 267

–ESTIMATING AND CHECKING–

Exercise 4 Exercise 2
How many different hands of three cards can you be 1. H

dealt from a 24-card deck, if each card is different in
G
some way? Answer is below. 

 T  H


H 
Summary  T
R
  H
In this lesson you have learned some methods for es-  T 
timating probabilities from enhanced outcome trees. H T

You also learned some formulas that will help you
B
check whether you have counted permutations and 
combinations correctly, and which you can use if the T
numbers are too big to count. 2. 3
or .43
7
To answer this question, just add up the
probabilities of the three paths you checked
Answers that have no heads. These are G–T, R–T–T, and
B–T. Their respective probabilities are 73 × 12
Exercise 1 = 14, 7 × 2 × 2 = 14
3 2 1 1 1
, and 72 × 12 = 14
2
. (Because we
21 or .095. Follow the R–B path to get 7 × 6.
2 2 2
1. have to add them, we’re not reducing the last
7 or .14. Follow the G–G path to get 7 × 6.
1 3 2
2. fraction yet.) The answer is
3
3. 7 or .43. There are three paths that can lead to 3 1 2 6 3
a G on the second ball: G–G, R–G, and B–G. + + = =
14 14 14 14 7
The probabilities of those are, respectively, 71,
Exercise 3
7 × 6 = 7, and 7 × 6 = 7. The total of the three is
2 3 1 2 3 1

the probability that a green ball will be drawn 720


second. This is a permutation problem, because it asks how
5
4. 7 or .71. You’ve already calculated the each of the officers can be chosen, so it does make a
probability for both green (73) and blue (72) in difference whether someone is chosen president or
previous problems, so you just have to add secretary, for example. Using the formula P(10,3), we
them together. Of course, you can just count get
the leaves on the probability tree at a certain 10 ! 10 × 9 × 8 × 7 !
point. If you did count the leaves for this P(10, 3) = = = 720
(10 − 3)! 7!
question, you would count 3 G’s and 3 B’s and
get a probability of 69. However, the probabili- Exercise 4
ties of all nine leaves at this stage are not equal, 2,024
so you have to calculate the probability of each The order in which you receive the hand doesn’t matter,
24!
path. so you would use C(24,3), which is 3!21! = 2,024.

267
8th_GRDM_18.qxd:Layout 1 8/11/09 4:24 PM Page 268
8th_GRDM_19.qxd:Layout 1 8/11/09 4:26 PM Page 269

31
L E S S O N

STRATEGIES

L E S S O N S U M M A RY
In this lesson you will learn how to attack some types of data and
probability problems that you’ll see in real life and on tests: (1) The
problem of raising and lowering averages, (2) the problem of inter-
preting data from graphs and other visual arrangements of data,
and (3) the problem of determining how many people were sur-
veyed, given some results of that survey.

T his lesson’s focus is on getting you through tests that you might face in eighth grade. Between this les-
son and Lesson 30, you’ll cover most of the types of word problems related to probability that you’ll
see in eighth grade. The types you covered in Lesson 30 involved lots of computation; the ones in this
lesson will involve more logical thinking.

269
8th_GRDM_19.qxd:Layout 1 8/11/09 4:26 PM Page 270

–STRATEGIES–

The Whole and the Parts: The diagram shows the circles overlapping, which indi-
Another Look cates that some students have both a laptop and a PC.
Yet the circles don’t fill up the entire sample space—so
In Lesson 29 you learned that probability is com- some students have neither a laptop nor a PC.
puted as: A typical data-related problem you might see on
a test (or that you might even have to figure out in real
number of ways A can occur life) is to figure out how many are in the sample space
P( A) =
total number of events that can occur or how many are in the intersection of those two cir-
Sometimes, you need to know the total even if you’re cles, depending on the information you’re given.
not figuring out probability. If you were to take a sur- As an example, all the students in the seventh
vey of who would vote for someone in an election, for grade at Franklin Middle School were asked two
example, you might have a picture of the respondents questions: (1) Do you have one or more dogs at
that looks like this. home? (2) Do you have one or more cats at home? So
the possibilities for answering the two questions are
(Yes, Yes), (Yes, No), (No, Yes), and (No, No). There
were 129 students who said yes to the dog question,
Candidate Candidate and 104 students said yes to the cat question. The sev-
Whole sample space
X Y enth grade at Franklin has 261 students, and 74 said
that they owned neither a dog nor a cat. How many
No opinion Other
students owned at least one dog and at least one cat?
Here’s a picture, with a “?” in the space we want to
know:
Those voting for Candidate X are represented inside
of one circle, those for Candidate Y in another, and
those who said they would vote for someone else in Students with neither (74)

another. But the rectangle represents everyone who


was surveyed. The others might have said they were Dog Cat 261 Students
?
undecided or that they wouldn’t vote at all. You can (129) (104)

see that even if everyone who says they’ll vote for Y


comes through, it’s still possible that Y will lose if all
the others somehow decide to vote for X.
Here’s another sketch of a sample space using a We know how many answered Yes to the first
Venn diagram, something you’ve probably seen in question, how many answered Yes to the second, and
your reading or social studies courses. It’s from a sur- how many answered No to both. The only thing we
vey that asked how many students had a laptop com- don’t know is how many answered Yes to both ques-
puter and how many had a personal computer (PC). tions. To solve this problem, we start by adding up the
three responses to the question: 129 (for dogs) + 104
(for cats) + 74 (for neither) = 307. Because there are
only 261 students in the seventh grade, we have to ac-
count for the extra 46. And where are they? They’re
Laptop PC All students
in the space where the circles intersect, and there are
307 – 261 = 46 of them. That’s how many have at least
one dog and at least one cat.

270
8th_GRDM_19.qxd:Layout 1 8/11/09 4:26 PM Page 271

–STRATEGIES–

When you get this kind of situation, there’s a Average Mileage


Legend Ethanol made with Process 1
formula that you might want to memorize, but you’re 36
Ethanol made with Process 2
probably better off reasoning through it anyway. 33
30
Total number = number in circle 1 27

Miles per gallon (mpg)


+ number in circle 2 + number not in circles 24
– number in intersections of circles 21
18
Exercise 1 15
(Answers are on page 273.) 12
9
1. In the laptop/PC example, there were 120
6
students surveyed, with 75 saying they had a
3
PC and 18 saying they had a laptop. There were
32 who had neither. How many had both a Car A Car B Car C Car D
laptop and a PC?
First, notice the different parts of the graph, specifi-
2. The same laptop/PC survey was conducted at cally the axis labels. What is the y axis measuring? As
Jefferson Middle School. At this school, 89 of it clearly says, it’s indicating miles per gallon. The x
the eighth graders had a laptop, 169 had a PC, axis is a little trickier; it’s just labeled with different
47 had neither, and 16 had both. How many car identifiers, with two bars each, and you don’t
students were in Jefferson’s eighth grade class? know what they mean without the legend. If a graph
has a legend—and many don’t—it’s important that
3. The pet survey in the Franklin seventh grade you read it. This one shows two different shadings to
was also given to the eighth graders at Franklin. indicate what the bars mean. Notice that there are
There are 243 students in the eighth grade, and numbers on the y axis, as you’d expect, because miles
45 had both a dog and a cat. Of the 243 per gallon is measured by numbers. If you see a graph
students, 83 said they had neither a dog nor a like this on a test, the best strategy is to try to under-
cat, and 120 said they had at least one cat. How stand the graph a little before trying to answer the
many had at least one dog? questions. Here it seems that four cars have driven
with one type of fuel for awhile and with another
type for awhile, and that each has had its gas mileage
Data Interpretation recorded with each type of fuel. Now, let’s look at
some questions that might accompany a graph like
Here’s a graph that shows the difference in perform- this.
ance between different cars, each of which used dif-
ferent kinds of fuel over different periods of time.

271
8th_GRDM_19.qxd:Layout 1 8/11/09 4:26 PM Page 272

–STRATEGIES–

Exercise 2 To solve this problem, you have to realize that the av-
(Answers are on pages 273–274.) erage is the total of all scores divided by the number
of tests. So the first thing we have to do is to find that
1. How much did the mileage increase for Car A total. We know that
when it ran on the Ethanol made by Process 2?
total
average = number
2. Which car’s mileage increased the most by 87 = 5
total
switching to Process 2? total = 5(87) = 435

3. What was the mean increase for all four cars? If Gerry wants an average of 89, he needs a total of
6 × 89 points, or 534. Because he now has 435 total
4. What was the average percentage increase? points, he would need to get a 99 on the last exam
(Round to nearest hundredth of a percent.) to get his average up just 2 points. Remember that,
when you think your early grades in a course aren’t
5. If your family car averages 18 miles per gallon all that important!
using fuel made by Process 1, what does the
graph show it might average using fuel made Exercise 3
by Process 2? (Answers are on page 274.)

Axis Labels 1. Chris Sprintgirl has 100-meter times of 12.4


When axis labels are given to you for a problem, they seconds, 12.8 seconds, 12.9, and 13.3. She has
will generally be clear, unless you’re supposed to im- to get an average of 12.75 seconds or lower on
prove the graph. But when you’re asked to construct a five races to make the traveling team. What
graph, be sure to give some thought to your axis la- does she have to get on her fifth try to make
bels. They must: the team?

■ have a scale that will make the graph readable. 2. Hank Basketball is averaging 16.25 points per
■ make the variables clear to anyone who looks at game after 8 games. He’ll probably make the
the graph. all-star team if he averages 15 or better per
game after his first 12 games of the season.
How many points per game does he have to
Raising and Lowering average over the next four games?
an Average

This is a popular type of question for math teachers Summary


and standardized tests, because it tests your knowl-
edge of how to find a mean and your knowledge of In this lesson you have learned how to solve problems
how to find an unknown. involving the calculation of the whole and the parts.
Gerry Algebra’s scores on five tests so far this You have also learned a bit more about how to answer
semester give him an average of 87. What does questions involving the interpretation of graphs, and
he have to get on his sixth and final test to get his you have learned to solve some problems that ask for
average up to 89, if all tests are equally weighted? how an average will be affected by adding more values.

272
8th_GRDM_19.qxd:Layout 1 8/11/09 4:26 PM Page 273

–STRATEGIES–

Answers Exercise 2
1. 4.5 miles per gallon
Exercise 1 The bar for Process 2 is about halfway between
1. Five students had both. 27 and 30, so a good estimate would be 27 +2 30
You start by adding 75 + 18 + 32 = 125. = 28.5. 28.5 – 24 = 4.5 mpg.
Because there are only 120 students, 125 – 120 2. Car C
students must have answered Yes to both Car B increased by 3 mpg (30 – 27), Car C by
questions. 6 mpg (31 – 25), and Car D by 4 mpg (30 – 26).
2. 289 students are in the eighth grade at Jefferson. Even though these are not exact because you
You add up the numbers of the laptops (89) have to estimate the exact spot these fall on the
and the PCs (169) and neither (47) to get 305. y axis, it’s clear that Car C’s increase is largest.
But in doing that, you’ve counted everyone 3. 4.375 miles per gallon
who had both twice (once with the laptop We already have the numbers, so just add them
people and once with the PC people), so you and divide by 4: 4.5 + 3 + 6 + 4 = 17.5, and
17.5 3
have to subtract that number: 305 – 16 = 289. 4 = 4.375 (or 4 8). (Note: mean and average
Or you just use the formula directly: N = 89 + are interchangeable terms.)
169 + 47 – 16 = 289. 4. 17.16%. We can do this either by computing
3. 85 students have at least one dog. each percentage increase and then using the
Here it might be easiest to just use the formula, mileage from Process 1 to find the average
with the unknown D representing the number percentage increase, or by using the average
of dog owners: increase (which we already know from
243 = D + 120 + 83 – 45 question 3) and then dividing it by the average
243 – 120 – 83 + 45 = D mileage of all four cars to get the average
D = 85 percentage increase. The average increase is
You could also reason that, because 83 students 4.375 mpg, and the average mileage from
have neither, you could subtract that number Process 1 fuel is 24 + 27 + 25 + 26 = 102 = 25.5 mpg.
4 4
from the total to get 160. Of those, you could The percentage increase is:
subtract the number of cat people to get 40, 4.375
× 100 = 17.16%
but then you’d have to add back in the people 25.5
who had both, because they would qualify as As a check, we can calculate it using fractions:
dog people, too: 40 + 45 = 85. 35 51 35 2 35
÷ = × = = .1716 ✓
8 2 8 51 204
(This is a check of the calculation only, not the
whole problem.)

273
8th_GRDM_19.qxd:Layout 1 8/11/09 4:26 PM Page 274

–STRATEGIES–

5. If you get a question like this as a multiple- Exercise 3


choice question on a standardized test, it’s best 1. She has to run the race in 12.35 seconds or less.
just to answer using a 17.16% increase First, we’re looking for an upper limit here;
multiplied by the 18 mpg your car already gets: Chris has to attain a certain time or lower to
.1716 × 18 = 3.09. Then add the 3.09 to get make the team. Also, we don’t really need to
21.09. One of the multiple answers will know her current average, just her current
probably be “21.” However, as you learned in total, which is 12.4 + 12.8 + 12.9 + 13.3
Lesson 28, you don’t have enough information = 51.4 seconds. Because she’ll have five races
from this graph to tell you any information to identify her status, she’ll need 5 × 12.75 total
about the mileage your car (or a friend’s car, for (or less) to make the team. 5 × 12.75 = 63.75.
that matter) might get with Process 2 fuel. So if Subtracting her current total from 63.75 gives
your teacher asks you this question, your us 12.35 seconds. In other words, Chris will
answer might run something like this: “The have to run her fastest 100 meters yet in order
cars may be very different from mine, or they to make the team.
may be almost the same. They may have had 2. 12.5 points per game
drivers who are very interested in saving fuel, Hank has scored 16.25 × 8 = 130 points in his
and my parents aren’t that interested in saving first eight games. After twelve games, he will
fuel. They may never have gone faster than 50 need at least 12 × 15 = 180 points to be an all-
mph on the highway, but my dad always drives star. In the next four games he has to score
fast. There are too many unknown variables, 180 – 130 = 50 points, which is an average of
and no proof that the sample is valid.” 50
4 = 12.5 points per game.

274
8th_GRDM_20.qxd:Layout 1 8/11/09 4:27 PM Page 275

32 PUTTING
L E S S O N

IT ALL
TOGETHER

L E S S O N S U M M A RY
This lesson pulls together what you have learned about data and
probability in Lessons 28 through 31. You’ll use that knowledge to
answer problems and to write short and long responses that tell
how you solved different problems.

Y ou have now learned some of the basics of data and probability you’ll need to be successful in eighth
grade. You have also learned strategies for solving word problems based on real-life situations. Now
it’s time to see how well you know them.

What You Have Learned

Here’s a quick summary of the lessons in this section.


Lesson 28: Data: Describing, Displaying, and Interpreting. You learned what data are, and some reasons
why data sets and statistics are important. You also learned something about describing sets of data and how to
present data in graphs and tables—and to interpret data already in graphs and tables. You also learned how to
make scatter plots.

275
8th_GRDM_20.qxd:Layout 1 8/11/09 4:27 PM Page 276

– PUTTING IT ALL TOGETHER –

Lesson 29: Probability, Permutations, and 1. What is the median score for the class? What is
Combinations. You learned how to count the number the mode? What is the range?
of possibilities that an event will occur, as well as all
the possibilities that a similar but different event will 2. Do you think the mean will be higher or lower
occur. Once you mastered that task, you have learned than the median? Now calculate the mean, then
to figure out probabilities of A and B and of A or B. explain why your guess was right or wrong.
Lesson 30: Estimating and Checking. You
learned some methods for estimating solutions for 3. A score of 245 is considered a passing score on
and checking your answers to probability problems. this test. What percentage of Ms. Myers’ class
Methods involve the use of probability trees and per- passed the test? What percentage failed?
mutation and combination formulas.
Lesson 31: Strategies. You studied a general 4. A score of 275 or higher is considered
procedure for solving different types of word prob- “outstanding” on this test. What percentage of
lems, such as problems that ask you to interpret data Ms. Myers’ students had scores in the
displays and problems that ask for solutions with “outstanding” range?
overlapping responses to surveys. You also learned
how to figure what’s needed to raise or lower averages 5. Here is a graph of average monthly rainfall, in
by a certain amount. inches, in Carroll City.

Average Rainfall (inches)


12
Practice
10

Short Answers 8

Answer the questions as indicated (rounding to the


6
nearest hundredth). (Calculators permitted, espe-
cially for checking. Answers are on pages 279–281.) 4

The first four questions concern this stem-and- 2

leaf plot that shows the scores from Ms. Myers’ class
0
on a standardized math test. Jan Feb Mar Apr May June July Aug Sep Oct Nov Dec

What is the approximate range of the monthly


STEMS LEAVES
rainfalls? Which is higher, the mean or the
23 3, 3, 6, 7 median?
24 0, 1, 8

25 1, 3, 8, 9

26 0, 0, 0, 1, 1, 3, 4, 4, 6, 8, 9

27 9

28 8

276
8th_GRDM_20.qxd:Layout 1 8/11/09 4:27 PM Page 277

–PUTTING IT ALL TOGETHER –

6. The following graph shows the results of a 8. How many ways can you roll a total of 4 using
survey at Jefferson Middle School in 2009, two fair six-sided dice? What is the probability
along with the relative frequencies of the (to the nearest thousandth) that you will roll a
answers. How many students answered that total of 4?
orange was their favorite color?
9. If the graph in question 6 were turned into the
Favorite Colors, Spring 2009, 560 Students
background for a spinner, what would be the
Purple
7% approximate probability that the spinner
Blue
Red
13%
32% would land on green on the first two spins?
a. .02
b. .03
Green
15% c. .075
Yellow d. .225
4%
Orange
29%

10. A fair coin is tossed three times, and a head or


7. The following graph plots the average scores on tail is recorded every time. What is the
physics exams for one semester against the self- probability that the coin comes up heads
reported number of hours students studied per exactly two times?
week during the semester.
11. A survey taken of the senior class (total 2,200
100
students) showed that 925 said they had at least
one brother living at home and 597 said they
Average test scores

90
had at least one sister living at home. In the
80
senior class overall, 801 said they had no
70 siblings living at home. How many students in
the class have a brother and a sister living at
60
home?

1 2 3 4 5 6 7 8 9 10 11 12
12. A bag of marbles contains 12 blue marbles and
Hours studied, weekly
1 white marble. If you reach into the bag
Is there a positive correlation, a negative without looking, what is the probability of
correlation, or no correlation? Identify at least selecting the white marble on the first or
two outliers by coordinates. State what the second draw?
correlation means in terms of number of hours
students studied.

277
8th_GRDM_20.qxd:Layout 1 8/11/09 4:27 PM Page 278

– PUTTING IT ALL TOGETHER –

Short Response Extended Response


Write a response that explains how you got your an- Write a response that fully explains how you used al-
swer (no calculator except to check). gebra to arrive at your answer. Show all diagrams or
tables that you made as well as all calculations.
13. Given a box with 3 red balls, 2 yellow balls, and
1 green ball, draw a tree to show the number of 14. Patty and Ivan each took surveys to try to see
possible outcomes for randomly drawing 2 who would win the election for class president.
balls without replacing the first one. Then, draw There are 433 students in their class. There are
a probability tree showing the probabilities of three candidates. Ivan took his 60-student
each outcome. What is the probability that the survey during lunch periods in the cafeteria.
green ball will be drawn on the second try? (The school has an open campus at
lunchtime.) Patty works for the yearbook, so
she had access to a list of all students. She chose
60 names completely at random, but she was
able to interview only 50 of those because the
marching band was on a trip during the survey
period. Here are the results of the surveys and
the final vote (note that the percentages have
been rounded to the nearest whole number).

NUMBER OF MARY’S NUMBER OF ACTUAL VOTES NUMBER OF


BILL’S SURVEY VOTES (%) SURVEY VOTES (%) IN ELECTION VOTES (%)

Rachel 16 (27%) Rachel 14 (28%) Rachel 138 (32%)

Pablo 30 (50%) Pablo 15 (30%) Pablo 159 (37%)

Devontre 10 (17%) Devontre 17 (34%) Devontre 127 (29%)

Undecided or other 4 (7%) 4 (8%) 9 (2%)

Total 60 50 433

Whose poll was more accurate, Ivan’s or Patty’s? What might have improved the surveys?
___________________________________________________________________________________
___________________________________________________________________________________
___________________________________________________________________________________

278
8th_GRDM_20.qxd:Layout 1 8/11/09 4:27 PM Page 279

–PUTTING IT ALL TOGETHER –

Answers 5. 9.5 inches (give yourself credit if you’re within


half an inch); mean is higher than median. The
1. 260, 260, 55 maximum rainfall (in July) is about 10.5
The median is 260, which you can find by inches, and the minimum rainfall (February) is
counting either up or down. Because there are about 1 inch, so the range is about 9.5 inches.
24 scores (an even amount), the median is the To determine whether the mean or the median
average of the 12th and 13th scores, and both is higher, first find the approximate median.
the 12th and 13th are 260. The mode is also The 6th-lowest month seems to be November,
260 because it occurs three times, more than which is a little less than 3, and the 7th-lowest
any other individual score. The range is seems to be May, which is about the same. So
288 – 233 = 55. the median rainfall is around 3 inches, maybe
2. The mean, which is the sum of all scores less. For the mean to be above 3, the total
divided by 24, is lower than the median. The monthly rainfall would have to be above 36,
reason for this is that, aside from all the scores because you find the mean by dividing the total
at the 260 level, there are a lot more scores at by 12. The total for June, July, and August alone
the 230, 240, and 250 levels than at the 270 and is around 25 or 26. A quick inspection of the
280 levels, so you’d expect the mean to be lower graph shows that six other months are at 2
than 260. In fact, it’s 256.33, not a lot lower but inches or above, which puts the answer over 36
lower nevertheless. already, and there are still three months to add.
3. 75% of students passed; 25% failed So the mean is definitely higher than the
Six students got scores lower than 245, and 24 6
is median.
.25, or 25%. The other 18 passed, meaning that 6. 162 students prefer orange. The key to solving
18
24 = 75% passed.
this problem is noticing that the number of
4. 8.33% students is in the title: 560 × 0.29 = 162.4.
Only two students had scores higher than 275, Because you can’t have 0.4 student, the number
and 242
= .0833, or 8.33%. must be rounded to the nearest whole, or 162.
7. There’s a very clear positive correlation. The
outliers are at (1,93) and (11,70). You might
also have said that (8,70) is an outlier. This
positive correlation between hours studied
shows that, in general, exam grades go up as
the amount of studying goes up.
8. There are three ways to roll a total of 4; P(4) = 363
1
or 12 or .083
You may remember that there are 36 ways to
roll a pair of dice. If you didn’t remember that,
you can figure the number because each die is
independent of the other, so there are 6 × 6
possible outcomes. There are three ways to get
a 4: (1,3), (2,2), and (3,1), and the probability
3
is thus 36 or .083.

279
8th_GRDM_20.qxd:Layout 1 8/11/09 4:27 PM Page 280

– PUTTING IT ALL TOGETHER –

9. a. .02 13. Responses will vary. Here’s a sample answer:


You know the probability of hitting green is
.15, because it’s 15% of the area. Because you R R Y
Y
want to find the probability of hitting green G

and then green again, the probability of doing R R


R R
both is .15 × .15 = .0225 ≈ .02. Y Y
3 Y Y
10. 8 G
G
You could draw a probability tree to answer
R
this, but let’s just make a list of the possible R
results, because that’s easy with so few results R R
to consider. Each flip can have only one of two Y

outcomes: H or T. After two flips, you’d have Y


R
G
just four outcomes: HH, HT, TH, or TT. Each
of these would generate two more outcomes
depending on the flip of the third: HHH, HHT, Y R
R
HTH, HTT, THH, THT, TTH, and TTT. So the
R
number of possible outcomes is eight. Looking Y Y
at the outcomes, you can see that exactly three G
G
have two H’s, so the probability is 38.
R
11. 123
R R
With 2,200 students, 801 of whom have no R R
Y
siblings at home, the class has 1,399 students G R
Y
who do have siblings at home. Because R G
R
925 + 597 = 1,522 have brothers or sisters, Y Y R

there must be 1,522 – 1,399 = 143 who have at


I drew an outcome tree that shows all 30
least one sister and one brother living at home.
25 possible outcomes. As you can see, it’s pretty
12. 156 or .16
The probability of selecting the white marble cramped because there are so many outcomes
1
on the first draw is 13 . If you select a blue one to get in.
instead, the probability of selecting the white
1
one on the second draw is 12 . Because this
outcome can be satisfied by one
1
event (with a probability of 13 ) or another
1
event (with a probability of 12), you need to
add these probabilities:
1 1 12 13 25
+ = + =
13 12 156 156 156

280
8th_GRDM_20.qxd:Layout 1 8/11/09 4:27 PM Page 281

–PUTTING IT ALL TOGETHER –

Here’s the probability tree, which is easier to 14. Responses will vary, even on the main question
read. of whose survey is more accurate. Here’s a
sample answer:
R You could say that Ivan’s survey is more
 accurate, because he got the winner right,
 and the winner is all that counts. But if
R Y
Pablo had believed Ivan’s survey, he might

have stopped campaigning because he was
 G so far ahead.
I think Patty’s survey is more
R
 accurate because she shows that the race is
very close. And her survey is based on
 
Y Y picking random people.
 It looks like, for some reason, Pablo
has more friends who eat in the cafeteria,
 G
and Devontre and Rachel have more
R friends who leave campus during lunch,
 which is what messed up Ivan’s survey.
 Patty’s survey would have been okay, but
G Y
(just guessing), Rachel and Pablo might

be in the marching band, or at least have a
G lot of friends there. Even though Patty’s
survey could have been random, the
The ways to draw green on the second try are marching band is not a random group at
R–G and Y–G. These chances are 36 × 15 = 30
3
for all, and it was totally excluded.
R–G and 6 × 5 = 30 for Y–G. Adding these two, I
2 1 2
5
got 30 or 16.

281
8th_GRDM_20.qxd:Layout 1 8/11/09 4:27 PM Page 282

– PUTTING IT ALL TOGETHER –

IF YOU MISSED THEN STUDY

Question 1 Lesson 28

Question 2 Lessons 28, 31

Question 3 Lesson 28

Question 4 Lesson 28

Question 5 Lessons 28, 31

Question 6 Lesson 28

Question 7 Lesson 28

Question 8 Lesson 29

Question 9 Lesson 29

Question 10 Lessons 29, 30

Question 11 Lesson 31

Question 12 Lessons 29, 30

Question 13 Lessons 29, 30

Question 14 Lessons 28, 31

282
8th_GRDM_21_post.qxd:Layout 1 8/11/09 4:28 PM Page 283

Posttest

C ongratulations! You’ve finished all of the lessons in this book and have dramatically improved your
math skills. This posttest will give you a chance to measure your new level of math success.
The questions on this test are different from the pretest, but the format is the same. Take the
test, using as much time as you need. Then grade yourself, or have someone else do it, and compare your score
with your pretest score. If you have a much better score, give yourself a pat on the back—you’ve significantly
improved your skills. If your score is only a little better, there are probably some lessons you should review. Is
there a pattern to the types of questions you got wrong? Do they all seem to deal with the same strategies? Did
you remember to study every passage actively? There’s an answer sheet to use on the next page, or you can sim-
ply circle the correct answers or write the numbers 1–40 on a piece of paper and record your answers there.
When you finish, check your answers against the key on page 295. The key also tells you which lesson covers the
strategy in each question. Good luck!

283
8th_GRDM_21_post.qxd:Layout 1 8/11/09 4:28 PM Page 284
8th_GRDM_21_post.qxd:Layout 1 8/11/09 4:28 PM Page 285

–LEARNINGEXPRESS ANSWER SHEET –

1. a b c d 15. a b c d 29. a b c d
2. a b c d 16. a b c d 30. a b c d
3. a b c d 17. a b c d 31. a b c d
4. a b c d 18. a b c d 32. a b c d

5. a b c d 19. a b c d 33. a b c d

6. a b c d 20. a b c d 34. a b c d
7. a b c d 21. a b c d 35. a b c d
8. a b c d 22. a b c d 36. a b c d
9. a b c d 23. a b c d 37. a b c d
10. a b c d 24. a b c d
11. a b c d 25. a b c d
12. a b c d 26. a b c d
13. a b c d 27. a b c d
14. a b c d 28. a b c d

285
8th_GRDM_21_post.qxd:Layout 1 8/11/09 4:28 PM Page 286
8th_GRDM_21_post.qxd:Layout 1 8/11/09 4:28 PM Page 287

–POSTTEST–

Section 1: 4. Circle A’s radius is equal to twice the radius of


Multiple-Choice Questions Circle B. What is the relationship of the area of
Circle A to the area of Circle B?

Circle A Circle B
1. List the following numbers from greatest to
least.
3
2, 2 , 2p, 2.7
2
p 2r r
3 2.7
a. 2> 2 > 2> 2
2.7 p
b. 2 > 2> 2 > 32
p 3 2.7
c. 2>2> 2 > 2
p 2.7 3
d. 2> 2 > 2 > 2
a. Circle A’s area is twice the area of Circle B.
2. What is the value of x in the following b. Circle A’s area is four times the area of
equation? Circle B.
x −1
3
= −3 c. Circle A’s area is equal to Circle B’s area
1
a. 9 multiplied by p.
b. 9 d. There’s not enough information to
c. –9 determine the relationship between the
d. – 13 areas of Circle A and Circle B.

3. Which of the choices below is another way of 5. The Acela Express high-speed train from New
expressing the following product? York City to Washington, D.C., travels at an
34 × 52 × 45 average speed of 80 miles per hour and covers a
a. 35 × 53 distance of approximately 220 miles. The train
b. 313 × 510 also makes a 15-minute scheduled layover in
c. 37 × 52 Philadelphia. If Tony wants to arrive in
d. 36 × 53 Washington, D.C., at 2:00 P.M., which of the
following departure times should he choose for
his trip?
a. 10:45 A.M.
b. 11:00 A.M.
c. 11:15 A.M.
d. 11:30 A.M.

287
8th_GRDM_21_post.qxd:Layout 1 8/11/09 4:28 PM Page 288

–POSTTEST–

6. Jennifer has received an average grade of 87 in 10. Which of the following equations represents
six history exams so far this semester. What the relationship between the values of x and y
should Jennifer’s grade on the seventh and final in the table?
exam be for her to receive a final grade of 88, if
all seven exams are weighed equally in x y
computing the final grade? 1 2
a. 89
b. 91 2 6

c. 93 3 12
d. 94
4 20

7. Which property allows us to determine that 5 30


the product of the expression (4 × 5) × 6 is
equivalent to the product of the expression a. y = 2x2
4 × (5 × 6)? b. y = 4x – 2
a. Associative property c. y = 6x – x2 – 2
b. Commutative property d. y = x2 + x
c. Additive identity property
d. Distributive property 11. In 1999, the Hubble Space Telescope estimated
that there were 125 billion (125,000,000,000)
8. Which of the following is the best estimate of galaxies in the universe. How is this number
the quotient below? written in scientific notation?
419.360914 ÷ 6.98934856 a. 125 × 109
a. 70 b. 1.25 × 1011
b. 65 c. 1.25 × 108
c. 60 d. 12.5 × 108
d. 55
12. Determine the value of x if 3(2x – 5) < (5 + x)
9. Ashley is holding a fair six-sided die and a fair a. x < 4
coin. If Ashley rolls the die once and then flips b. x > 4
the coin, what is the probability that the die c. x > 2
will show a 5 and the coin will come up tails? d. x < 2
1
a. 12
b. 18
1
c. 10
1
d. 7

288
8th_GRDM_21_post.qxd:Layout 1 8/11/09 4:28 PM Page 289

–POSTTEST–

13. ABCD is a square. A diagonal line connecting A 16. The graph of Line A contains the points (5,–3)
to D creates two equivalent right triangles, and (2,–9). What is the slope of a line that is
Triangle ACD and Triangle DBA. If the length perpendicular to Line A?
of AD is 128 inches, what is the perimeter of a. – 12
ABCD? b. –2
c. 12
A B
d. 2

17. Last year, Janice’s weekly allowance was $25.


128
This year, she receives a daily allowance of $4.
What is the percentage increase in Janice’s
allowance?
a. 3%
b. 21%
C
c. 15%
D
d. 12%
a. 64 inches
b. 32 inches 18. ∠A and ∠B are complementary angles. Which
c. 128 inches of the following cannot be true?
d. 96 inches a. ∠A ÷ ∠B = 2
b. ∠A + ∠B = 180°
14. What is the area of the trapezoid if WX = 14 c. ∠A – ∠B = 60°
cm, YZ = 22 cm, and the height of the d. ∠A = ∠B
trapezoid is 5 cm?
19. The Chicago Bulls and the New York Knicks
W 14 X are playing a five-game playoff series, in which
the first team to win three games wins the
h=5 series. Assuming that each team has an equal
chance of winning any given game, what is the
probability that the Chicago Bulls can come
Y 22 Z
back from a 0–2 deficit to win the series?
a. 180 cm2 a. 12
b. 110 cm2 b. 18
c. 70 cm2 c. 16
d. 90 cm2 d. 14

15. Solve for x in the following equation: 2x = |–6|


+ 3(4 – |–2|)
a. 12
b. 0
c. 6
d. 7

289
8th_GRDM_21_post.qxd:Layout 1 8/11/09 4:28 PM Page 290

–POSTTEST–

20. The Great Pyramid of Giza is a regular 24. The rabbit population at the local zoo doubles
pyramid that reaches a height of approximately every day. If there are 6 rabbits at the zoo on
140 meters, and its base is a square with each Monday, how many rabbits will there be on the
side measuring about 230 meters. According to following Sunday?
these measurements, which of the following is a. 67
the approximate volume of the Great Pyramid b. 6 × 26
in scientific notation? c. 27
a. 7.5 × 106 cubic meters d. 2 × 67
b. 750 × 104 cubic meters
c. 25 × 106 cubic meters 25. A random survey conducted before local
d. 2.5 × 106 cubic meters elections found that 376 out of the 1,200
registered voters asked were planning to vote
21. Kevin has 43 times the number of baseball cards for Mr. LaGuardia for mayor. If 600,000 people
as Dustin has. Together, they have 140 baseball vote in the election, which of the following is
cards. How many cards would Kevin have to the best prediction for the number of votes Mr.
give Dustin for the two friends to have an equal LaGuardia will receive in the election?
number of baseball cards? a. 376,000
a. 10 b. 24,000
b. 15 c. 50,000
c. 20 d. 188,000
d. 30
26. Triangle ABC is an isosceles triangle that is
22. The volume of a cube-shaped box is 420 m . 3
similar to Triangle XYZ. If ∠B is 58°, what is
Which of the following is closest to the length the measurement of ∠X?
of each side of the box?
X
a. 20 meters
b. 12.5 meters
A
c. 5 meters
d. 7.5 meters

23. Which of the following measures is the highest


for the data sample given below?
{2, 4, 7, 3, 8, 4, 6, 5, 5, 7, 7}
a. Mode
58°
b. Median
B C Y Z
c. Mean
d. Range a. 58°
b. 32°
c. 64°
d. 122°

290
8th_GRDM_21_post.qxd:Layout 1 8/11/09 4:28 PM Page 291

–POSTTEST–

27. The following scatter plot shows the relation- 29. The following table shows the number of
ship between the average rainfall in a given visitors to the World’s Smallest Museum
month and the number of car accidents in that during the first six days of a particular week.
month.
VISITORS TO THE WORLD’S
Car Accidents
SMALLEST MUSEUM
2,000
Number of Car Accidents

DAY OF NUMBER
1,600 THE WEEK OF VISITORS
1,200
Monday 286
800
Tuesday 293
400

0 Wednesday 295
0 2 4 6
Average Monthly Rainfall (Centimeters)
Thursday 295

Which of the following best describes the trend Friday 300


shown in this scatter plot?
Saturday 290
a. Negative correlation
b. Positive correlation
If only 90 tourists visit the museum on Sunday,
c. No correlation
which measure of this data will change the
d. Correlation cannot be determined from the
most?
information available.
a. the mode
b. the mean
28. The wheels of Emma’s bicycle measure 24
c. the range
inches in diameter. If she travels a distance
d. the median
of 1,570 feet, approximately how many full
rotations will each wheel make? (Use 3.14
30. Evaluate the following: (– 52)–3 ÷ 14
for π.)
a. 15
24
a. 250 32
b. – 125
b. 500
c. 125
32
c. 65.42 2
d. – 125
d. 130.83

31. The price of oil increased by 20% from January


1 to February 1. However, it decreased by 10%
from February 1 to March 1. What was the
percentage increase in the price of oil from
January 1 to March 1?
a. 10%
b. 15%
c. 8%
d. The percentage increase cannot be
determined without knowing the original
price of oil.

291
8th_GRDM_21_post.qxd:Layout 1 8/11/09 4:28 PM Page 292

–POSTTEST–

32. Two parallel lines are cut by a transversal as 36. The Venn diagram below shows the number of
shown in the figure below. first-year students at Acme University who
own laptop computers and MP3 players.

1 2
3 4 MP3 Players Laptops

5 6
7 8 650 800

If ∠4 is 130°, what is the sum of ∠6 and ∠7?


If 400 of the 1,600 first-year students at Acme
a. 50°
own neither a laptop computer nor an MP3
b. 25°
player, how many first-year students own both
c. 40°
items?
d. 100°
a. 250
b. 150
33. The equation 3x + 2y = –6 represents a line
c. 400
that can be plotted on graph paper. What are
d. 300
the coordinates of the x-intercept of this line?
a. (–2,0)
37. Identify the value of x:
b. (–3,0)
4x + |–3| > –|–9| + x
c. (0,3)
a. x < –3
d. (0,–2)
b. x > –3
c. x < –4
34. In his backyard Larry has a pool in the shape of
d. x > –4
a rectangular prism. The pool is currently 3
meters wide, 8 meters long, and 2.5 meters
deep. If Larry increases the depth of his pool to
3 meters, how much will the volume of the
pool increase?
a. 16 cubic meters
b. 12 cubic meters
c. 8 cubic meters
d. 24 cubic meters

35. Between which two consecutive integers is


1
(420) 4?
a. 20 and 21
b. 10 and 11
c. 100 and 101
d. 4 and 5

292
8th_GRDM_21_post.qxd:Layout 1 8/11/09 4:28 PM Page 293

–POSTTEST–

Section 2: 38. In 18 years, Michael’s father will be twice as old


Short-Response Questions as Michael. If the sum of Michael’s current age
and his father’s current age is 54, how old is
Directions: In the space below each problem, show Michael now? How old is his father? Show your
how you worked the problem. You need not write work.
complete sentences, but you should show the steps
you take to solve the problem. Your solution should
include a drawing or diagram if you used one. Be sure
that your final answer and all the steps are clear.

TYPICAL SCORING RUBRIC


SCORE LEVEL DESCRIPTION

0 No answer is given. The answer


given is incorrect in terms of
strategy and result.

1 The answer given is partially


correct.

2 The answer given is completely 39. A standard deck has 52 playing cards,
correct. The strategy used in
solving the problem is clearly containing 13 cards in each of the four suits
explained, the work is shown, (spades, hearts, diamonds, and clubs). If Noah
and the result is accurate.
picks a card out of the deck at random and
then picks a second card out of the deck
(without replacing the first card), what is the
probability that both cards will be of the same
suit? Show your work.

293
8th_GRDM_21_post.qxd:Layout 1 8/11/09 4:28 PM Page 294

–POSTTEST–

Section 3: Extended-Response Question

Directions: In the space below each problem, show how you worked the problem. You need not write complete
sentences, but you should show the steps you take to solve the problem. Your solution should include a drawing
or diagram if you used one. Be sure that your final answer and all the steps are clear.

TYPICAL SCORING RUBRIC


SCORE LEVEL MATHEMATICAL KNOWLEDGE, STRATEGIC KNOWLEDGE, AND EXPLANATION

0 No answer is given. There is no clear strategy provided for the answer. No attempt is made to
give an explanation of the solution process.

1 Shows little or limited understanding of the mathematical concepts involved. Strategy used to
answer is wrong, irrelevant, or inappropriate for the question. Gives minimal explanation of
the solution process and does not adequately explain the steps taken.

2 Shows some understanding of the mathematical concepts and principles relevant to the
question. Some evidence of a strategy to solve the problem is presented. An explanation of
the solution process is given, but it is inadequate or does not match the solution of the
problem; it may also be vague or difficult to understand.

3 Shows a high level of understanding of the mathematical concepts and principles relevant to
the question. The student successfully identifies most of the important elements of the
problem and the strategy presented to solve the problem is almost comprehensive. The
student gives a nearly complete explanation of the solution process, clearly explains what was
done, and addresses the question of why it was done to some extent.

4 Shows full understanding of the mathematical concepts and principles relevant to the
question. The student successfully identifies all of the important elements of the problem, and
the strategy presented to solve the problem is comprehensive. The student gives a complete
explanation of the solution process and clearly explains what was done and why it was done.

40. Ms. Lucas bought 6 violins, 9 oboes, 4 drums, and 2 cellos for the school orchestra, spending a total of
$1,380. If each violin costs 1.5 times as much as an oboe, each drum costs $30 more than a violin, and a
cello costs as much as a violin and a drum combined, how much does each instrument cost?

294
8th_GRDM_21_post.qxd:Layout 1 8/11/09 4:28 PM Page 295

–POSTTEST–

Answers

If you missed any of the questions, you can find help with that kind of question in the lesson(s) shown to the
right of the answer.

QUESTION ANSWER LESSON QUESTION ANSWER LESSON

1 c 1, 2, 18 21 a 13, 15

2 a 4, 9 22 d 4, 19

3 d 4 23 a 28

4 b 18, 24 24 b 10

5 b 7, 15 25 d 28, 31

6 d 28, 31 26 c 19

7 a Intro 27 a 28

8 c 6 28 a 15, 18, 26

9 a 29, 30 29 c 28

10 d 10 30 b 2, 4

11 b 4, 6 31 c 3, 6

12 a 12 32 d 17, 22

13 b 18, 22 33 a 11, 12

14 d 18, 22 34 b 20, 24

15 c 1, 9, 11 35 d 4, 6

16 a 12, 20, 22 36 a 31

17 d 3, 7 37 d 9, 12

18 b 17 38 Michael: age 12; 13, 15


Father: age 42
12
19 b 29 39 51 29, 31

20 d 19 40 Violin: $60; 13, 15


Oboe: $40;
Drum: $90;
Cello: $150

295
8th_GRDM_21_post.qxd:Layout 1 8/11/09 4:28 PM Page 296

–POSTTEST–

Short Answer 39. Solution: The question concerns the


38. Solution: There are two unknowns in the computation of a probability in the case of a
question: Michael’s age and Michael’s father’s dependent event. There are two ways to
age. Let us designate: calculate the probability that both cards picked
x = Michael’s age will be of the same suit.
y = Michael’s father’s age
Now, we can express the information given in 1. We can start by calculating the probability
the question in the following equations: that the two cards picked will both be from a
Equation 1: x + y = 54 particular suit, such as diamonds. Since there
Equation 2: 2(x + 18) = y + 18 are 13 diamonds in the deck of 52 cards, the
Simplifying the second equation, we get: probability that the first card will be a dia-
13
2x + 36 = y + 18 mond is 52 , which can be simplified to 41.
2x + 18 = y Since the first card is not put back into the
Plugging this information into Equation 1, we deck, there will be 51 cards left in the deck
see that: before the second card is picked, and 12 of
x + (2x + 18) = 54 those (13 – 1) will be diamonds. So, the
3x + 18 = 54 probability that the second card will be a
3x = 36 diamond is 12 51. Multiplying these two
x = 12 probabilities, we find the probability that
Thus, Michael is 12 years old. Since the sum of both cards will be diamonds:
Michael’s age and his father’s age is 54, we get: P(diamonds) = 14 × 12 51
12 + y = 54 However, there are three other suits in the
y = 42 deck, and the same calculation can be made
Thus, Michael’s father is 42 years old. to calculate the probability that both cards
Michael is 12 years old, and his father is 42 will be spades or hearts or clubs. To find the
years old. probability that the two cards will be of the
same suit, we need to add all of those prob-
To the Grader abilities together:
Scoring: If the student gives no answer or fails to P(same suit) = (14 × 12 51) + (4 × 51)
1 12

provide a strategy for solving the problem (or if the + (14 × 12


51) + (4 × 51)
1 12

strategy provided is completely wrong/irrelevant), the This is equal to:


corresponding score on the rubric should be 0. If P(same suit) = 4 × (14 × 12 12
51) = 51

the student shows an understanding of how to set up Thus, the answer is 12 51.

the two equations with the two unknowns and pro-


vides a strategy for solving the problem, but applies
the strategy incorrectly or makes a computation error
to arrive at a wrong answer, the score given should be
1. If the student shows an understanding of the rele-
vant equations, explains the strategy clearly, and ar-
rives at the correct result by applying the strategy to
the given question, the answer should be scored 2 ac-
cording to the rubric.

296
8th_GRDM_21_post.qxd:Layout 1 8/11/09 4:28 PM Page 297

–POSTTEST–

2. A simpler way to arrive at the answer is to Extended Response


realize that we’re only interested in the 40. Solution
probability that the suit of the second card Variables:
matches the suit of the first card. Whatever v = Price of a violin
suit the first card picked out of the deck b = Price of an oboe
may show, there will be 12 cards of that suit d = Price of a drum
left in the deck of 51 cards, so the probabil- c = Price of a cello
ity that the suit of the second card matches Thus, the information given in the question
the suit of the first card must be 12
51. This is can be expressed in the following equation:
equal to the probability that both cards are Equation 1 6v + 9b + 4d + 2c = 1,380
of the same suit, so the answer is 12
51. There are four unknowns in this equation, and
a single equation with four unknown variables
To the Grader cannot be solved to arrive at a unique solution.
Scoring: If the student gives no answer or fails to pro- However, the information given about the
vide a strategy for solving the problem (or if the strat- relative prices of each of the instruments can
egy provided is completely wrong/irrelevant), the also be expressed as equations.
corresponding score on the rubric should be 0. If the Equation 2 v = 32 b
student shows an understanding of probabilities and Equation 3 d = v + 30
provides a strategy for solving the problem, but ap- Equation 4 c = v + d
plies the strategy incorrectly or makes a computation The next step is to simplify all of these
error to arrive at a wrong answer, the score given equations.
should be 1. If the student shows an understanding of Plugging Equation 2 into Equation 3, we get:
probability calculations, explains the strategy clearly, Equation 5 d = 32 b + 30
and arrives at the correct result by applying the strat- Plugging Equation 3 into Equation 4, we get:
egy to the given question, the answer should be Equation 6 c = v + (v + 30) = 2v + 30
scored 2 according to the rubric. c = 2v + 30
Plugging Equation 2 into Equation 6, we get:
Equation 7 c = 2 × (32 b) + 30 = 3b + 30
We have now expressed the price of every other
instrument in terms of the price of an oboe, as
follows:
v = 32 b
d = 32 b + 30
c = 3b + 30

297
8th_GRDM_21_post.qxd:Layout 1 8/11/09 4:28 PM Page 298

–POSTTEST–

Using these values, Equation 1 can be which can then be solved for the value of b (i.e., the
expressed as follows: price of an oboe). After finding the price of an oboe,
6(32 b) + 9b + 4 × (32 b + 30) + 2 × (3b + 30) the student can use the information given in the ques-
= 1,380 tion (expressed in Equations 2, 3, and 4 above) to find
Further simplification gives us: the prices of the other three instruments.
9b + 9b + 6b + 120 + 6b + 60 = 1,380 Alternatively, the student can choose to use
30b + 180 = 1,380 the price of an instrument other than the oboe as
30b = 1,380 – 180 = 1,200 the “main” unknown variable and express the price
b = 1,200
30 = 40 of the other three instruments in terms of that in-
b = 40 strument by setting up different equations to express
So the price of an oboe is $40. Now, we can use the same information. If the student sets up the equa-
Equations 2, 3, and 4 to find the price of every tions correctly and arrives at the correct answers,
other instrument. such a route is perfectly acceptable. For example, tak-
Violin: v = 32 b = 32 (40) = 60 ing the price of a violin as the main unknown vari-
A violin sells for $60. able, the equations would look like this:
Drum: d = v + 30 = 60 + 30 = 90
The price of a drum is $90. b = 23 v
Cello: c = v + d = 60 + 90 = 150 d = v + 30
The price of a cello is $150. c = 2v + 30

Answer Therefore:
Violin: $60
Oboe: $40 6v + 9(23 v) + 4 × (v + 30) + 2 × (2v + 30) =
Drum: $90 1,380
Cello: $150 6v + 6v + 4v + 120 + 4v + 60 = 1,380
20v + 180 = 1,380
To the Grader 20v = 1,200
Scoring: The student’s score should depend on how v = 60
many of the arithmetic equations needed to solve the
problem are set up and solved correctly. The key to And then:
solving the question correctly is identifying that three
of the four unknown values can be expressed in terms b = 23 v = 40
of one of the other unknown variables, which means d = 30 + v = 90
that a total of four equations are necessary for the four c = 2v + 30 = 150
unknown variables in the problem. Once the four
equations are set up correctly, the rest of the question
requires the student to simplify each one and insert
the information from all of them into Equation 1,

298
8th_GRDM_21_post.qxd:Layout 1 8/11/09 4:28 PM Page 299

–POSTTEST–

To the Grader nation given is inadequate or does not match the solu-
Regardless of the instrument price used as the main tion of the problem, or if the answer is vague and dif-
variable, if the student sets up each equation correctly, ficult to understand, the answer should be scored 2. If
and shows a full understanding of linear equations the student shows little to no understanding of the rel-
and how to solve them, the answer should be scored 4 evant mathematical concepts and principles, presents
according to the rubric. If the student shows a high a strategy that’s wrong or inappropriate for the ques-
level of understanding of the relevant mathematical tion, and doesn’t explain the solution process ade-
concepts, but makes an error in setting up the equa- quately, the answer should be scored 1 according to
tions or commits an arithmetic error in solving for the rubric. If no clear strategy is provided for the an-
the unknown values, the answer should be scored 3 swer and no attempt is made to give an explanation of
according to the rubric. If the student shows some the solution process (or no answer is given at all), the
understanding of the relevant mathematical concepts answer should be scored 0 according to the rubric.
and gives a strategy to solve the problem, but the expla-

299
8th_GRDM_21_post.qxd:Layout 1 8/11/09 4:28 PM Page 300
8th_GRDM_22_glos.qxd:Layout 1 8/11/09 4:29 PM Page 301

Glossary

absolute value The absolute value of x, denoted |x|, is the positive value of x. In mathematical terms, |x| = x
if x ≥ 0, and |x| = –x if x < 0. Examples: |–3| = 3; |3| = 3.
acute angle An angle with a measurement less than 90 degrees. An acute angle is sharper than a right angle.
adjacent angles Two angles that have a common side and a common vertex.
algorithm A set of rules in a specific order that always leads to a correct solution. Even though algorithms
always lead to a specific solution, there are often multiple algorithms that will lead to a given solution.
angle (∠) Region between two rays sharing a common endpoint (vertex). Angles are measured in degrees
and, based upon those measurements, they are categorized as acute, right, or obtuse.
approximately equal to (≈) Almost, but not exactly, the same in value. The expression is indicated with the
symbol ≈. Example: 13 ≈ 3.333.
arc Curved part of a circle between two given points on the circle.
base of a polygon One side of a polygon, often refers to the bottom side. It is one-dimensional and is
measured by a unit of length.
base of a solid A surface of a solid, usually the surface on which the solid is thought to rest. It is two-dimensional
and is measured in square units.
binomial A polynomial with exactly two terms.
bisect To divide into two equal parts.

301
8th_GRDM_22_glos.qxd:Layout 1 8/11/09 4:29 PM Page 302

–GLOSSARY–

canceling When a common factor occurs in both the numerator and the denominator, the common factors
cancel each other out. Eliminating common factors is known as canceling. Example: In 75 × 52, the 5’s
cancel (actually, they both become 1’s), and the result is 72. This is possible because you’re really just
dividing the numerator and denominator by the same number (5). Similarly, two terms on either side of
an equal sign in an equation will cancel—except that in this case, both terms become zero. Example: In
9b + 6 = 3a + 9b, the 9b’s cancel each other, leaving 6 = 3a. This is possible because you’re subtracting 9b
from both sides of the equation. Canceling simplifies problems, making them easier to solve.
capacity A general term for how much a container can hold. The basic unit for capacity in the metric system
is the liter, and the common customary system measures for capacity include the cup, pint, quart, and
gallon.
Celsius (° C) A system of measuring temperature based upon the freezing temperature (0° C) and boiling
temperature (100° C) of water. The United States is one of very few countries that does not use this
system.
chord A straight line joining two points on the circumference of a circle.
circumference The perimeter of a circle.
coefficient The constant by which a variable is multiplied. In the expression 3x + y, 3 is the coefficient of x,
and 1 is the coefficient of y.
combination A group of elements taken from a larger set without regard to order.
composite number A composite number has more than two factors, and therefore is not a prime number.
Example: 10 is a composite number; its factors are 1, 2, 5, and 10. 11 is not a composite number; it is a
prime number and has only two factors: 1 and 11.
congruence (@) Having the same shape and size. Two or more polygons are congruent if they are the same
shape and size; that is, if their sides and angles are congruent.
constant Any quantity in an algebraic term or expression that is not a variable. Example: In the expression
5.4 + 6yz, 5.4 and 6 are the constants.
coordinate graph A visual representation of the relationship between specific numbers. Points are plotted
on a coordinate grid along two axes. The horizontal axis is the x axis; the vertical axis is the y axis. Points
on the grid are identified using one x coordinate and one y coordinate, expressed as (x,y).
correlation The degree to which one variable is dependent on another. Correlation between two variables
may be positive, negative, or nonexistent.
cubic units (3) The standard of measurement for volume. Cubic units are calculated using the formula
B (area, in square units) × h (height) = V (volume).
cup (c.) A unit of measurement common in the United States. It is equal to half of a pint, or eight fluid
ounces.
data Pieces of information. Any information may qualify as data, from size and history of astronomical
bodies to sports statistics to what you ate for dinner.
data set Any collection of information.
decimal A number based on multiples of tenths. A decimal point (the . symbol) is used in a base-ten place
system to separate the parts of a number that are greater than or equal to 1 from the fractional part of the
number (less than 1). The distance from the decimal point indicates the value of the number. Example:
In the number 3.47, the 3, to the left of the decimal point, is in the ones place; the 4, to the right of the
decimal point and therefore less than 1, is in the tenths place; and the 7 is in the hundredths place.
degrees (°) A unit of measurement applied to, among other things, angles and temperature.
diagonal A line connecting two vertices in a polygon or polyhedron that are not on the same side of the
figure.

302
8th_GRDM_22_glos.qxd:Layout 1 8/11/09 4:29 PM Page 303

–GLOSSARY–

dimensions Measurements of length, width, and/or height used to determine the size of an object.
dividend In a division problem, the number that will be divided. Example: 36
9 = 4. The dividend is 36.
divisor In a math problem, the number that divides another number. Example: 36 9 = 4. The divisor is 9.
elapsed time Amount of real time taken by a specified activity or event. Example: Omar left his house at
8:25 and arrived at school at 9:05; the elapsed time was 40 minutes.
equation A mathematical statement that the values on either side of an equal sign are equivalent. Examples:
2 + 8 + 9 = 19; 3x + 2 = 14.
equilateral triangle A triangle in which all three sides and all three angles are congruent to one another.
equivalent fractions Fractions that have equal decimal value are called equivalent fractions. Another way to
tell if two fractions are equivalent: ba and dc are equivalent if ad = bc. Example: 21 and 36 are equivalent
fractions: 1 ÷ 2 = 3 ÷ 6 = 0.5, and 1 × 6 = 2 × 3.
evaluate To evaluate an expression or function is to find the value of the expression or function by using a
particular value for the unknown. For example, you can evaluate the expression 2x + 3 for x = 3 as 2 × 3
+ 3 = 9. You can evaluate the function f(a) = 4a – a2 for a = 0.5 as f(a) = 4 × 0.5 – (0.5)2 = 2 – 0.25 = 1.75.
exponent The power to which a base number is raised. Examples: In x6, x is the base and 6 is the exponent,
and the expression is evaluated as x × x × x × x × x × x. In 34, 4 is the exponent, and the expression is
evaluated as 3 × 3 × 3 × 3 = 81.
expression A meaningful combination of mathematical symbols that includes a sum or difference of terms.
In algebra, an expression must contain at least one variable. Examples: 4 + 7 is an expression; and 4x + 7
is an algebraic expression.
factorial (n!) A function with input positive integer n that multiplies all positive integers less than n:
n(n – 1)(n – 2) × . . . × (1).
Fahrenheit (° F) A system of measuring temperature invented by German instrument maker Gabriel
Fahrenheit. In the Fahrenheit system, water freezes at 32° F and boils at 212° F. Fahrenheit is used
primarily in the United States, and a few other countries, for non-scientific purposes.
frequency The number of times that an event occurs in an experiment in a survey. Examples: the numbers
of people with different hair colors in a given survey.
function Expresses a dependent relationship between the value on one side of an equation and the depend-
ent variable on the other side. There is only one output result for each input value. Example: 4x + 3 = y.
In this function, if x = 2, y must be 11. If x = 4, y must be 19. The value of x determines the value of y;
so y is the dependent variable.
gallon (gal.) A unit of liquid measurement, used primarily in the United States, that is equal to four quarts.
gram (g) The basic unit of mass, or weight, in the metric system.
greatest common factor The largest factor that two numbers share. Example: Find the greatest common
factor of 24 and 36. Factors of 24 include 1, 2, 3, 4, 6, 8, 12, 24. Factors of 36 include 1, 2, 3, 4, 6, 9, 12, 18,
36. Common factors are 1, 2, 3, 4, 6, and 12. The greatest common factor is 12. If you know the greatest
common factor of the numerator and denominator of a fraction, you can reduce the fraction to its lowest
common denominator by dividing numerator and denominator by the greatest common factor.
histogram A form of bar graph that displays frequencies.
improper fraction A fraction in which the numerator is greater than, or equal to, the denominator.
Examples: 57 (57) and 33 (33).
inequality An expression that contains a relationship of inequality, such as greater than (>),
less than (<), greater than or equal to (≥), or less than or equal to (≤). Example: 3x > 2y.
integers All natural numbers, the negative counterparts of those numbers, and 0. Example: –3, –2, –1, 0, 1, 2,
3 are integers, but 13, 2.35, and 3 are not.

303
8th_GRDM_22_glos.qxd:Layout 1 8/11/09 4:29 PM Page 304

–GLOSSARY–

interest The amount of money, usually expressed as a rate, a borrower is charged by a lender. The interest is
often a percentage of the principal (the amount of money actually borrowed) charged over a specific
period of time. Interest rate calculations can become very complex. This equation can help you calculate
simple interest: Total Interest = (Principal)(rate)(number of periods). Computing compound interest
involves exponential functions, of the form A = P(1 + i)n, where A is the amount of money accumulated, P
is the principal, or original amount, i is the annual interest rate, and n is the number of periods in which
the interest has been compounded.
intersecting lines Two lines that meet at a point.
irrational number A decimal number that continues indefinitely, does not repeat, and cannot be expressed
by a fraction. An irrational number can’t be expressed as a ratio of two integers. Examples: π and 3 .
irregular polygon A polygon whose sides are of differing lengths and/or whose interior angles are unequal.
isosceles triangle A triangle with two sides of equal length, and two acute angles of equal measurement.
iteration The repetition of a mathematical operation. Iterations often involve inputting the output of the
previous iteration into the next operation. Example: You can iterate the expression 2x + 3. Start by
inputting 0 for x, and you get 3; then input 3 (the output from the first iteration) into the next iteration
and you get 9. Input 9 into the next iteration and so on.
least common multiple The smallest multiple, not including 0, that two or more numbers have in common.
Example: The least common multiple of 6 and 8 is 24. Multiples of 6 are 0, 6, 12, 18, 24, 36, . . . Multiples
of 8 are 0, 8, 16, 24, 32, . . . Because 0 cannot be the least common multiple, the answer is 24. The least
common multiple is what you use to determine the least common denominator, used for adding and
subtracting fractions.
linear equation A mathematical expression in which each term is either a constant or the product of a
constant and a single variable. Both a linear equation and its solution must be expressed in a single line,
so a linear equation cannot contain exponents or division. A linear equation in two unknowns is an
algebraic equation of the form y = mx + b, where m is the slope of the line and (0,b) is the point where
the line crosses the y axis.
liter (l) The basic metric unit of measuring volume. A liter is equivalent to one cubic decimeter. Although the
metric system is still not widespread in the United States, many liquids are now sold as liters.
lowest terms A fraction is in its lowest terms, or simplest form, when the greatest common factor of the
numerator and the denominator is 1. Example: 31 is in its lowest terms, but 26 is not.
mean (arithmetic average) A measure of central tendency in a data set. The mean is the sum of the
numbers in a set divided by the number of numbers in that set. Example: For the set 3, 2, 4, 7, 9, the
mean is computed by adding 3 + 2 + 4 + 7 + 9 = 25, then dividing 255 = 5.
median A measure of central tendency in a data set. In a set with an odd number of data, the median is the
middle number; in a set with an even number of data, it is the mean of the two middle numbers.
Examples: For the set 2, 4, 6, 8, 10, the median is 6; for the set 2, 4, 6, 7, 8, 10, the median is 6.5.
metric prefixes Prefixes that apply a scale factor (always with a base of 10) to the basic units in the metric
system. A table of the most common of these can be found in Lesson 24, and complete lists are available
in many reference books and on the Internet.
metric system A system of measuring that is based on 10. It is used as a standard in most countries of the world.
midpoint A point at the center of a line segment that divides it into two equal segments.
miles per hour (mph) Measurement of speed that assesses the distance covered (in miles) over a period of
time (in hours).
mixed number A number consisting of both an integer and a proper fraction. Examples: 232 and 885.
mode The most frequently occurring number in a set. Example: For the set 1, 5, 3, 6, 3, 8, 3, 1, the mode is 3.

304
8th_GRDM_22_glos.qxd:Layout 1 8/11/09 4:29 PM Page 305

–GLOSSARY–

monomial An algebraic expression with exactly one term.


multiple The product of a specified whole number and an integer. Example: 8, 12, 16, and 20 are all
multiples of 4.
multiply/multiplication (* or • or ×) A mathematical operation in which a number is added to itself a
specified number of times. Multiplication is expressed using the * or • or × symbols placed between the
number and the multiplier. Example: 6 + 6 + 6 = 6 * 3 = 18.
natural number Any positive whole number not including 0. Example: 1, 2, 3, 4, . . . Also called the
counting numbers.
negative exponent The negative power to which a base number is raised. Example: In x–3, “–3” is the
1
negative exponent. A negative exponent x–3 can also be expressed as 3 .
x
negative number A number with a value less than 0. Example: –2.045.
nth term The term in a sequence that expresses the pattern of the sequence. Example: If you have a sequence
2, 4, 8, 16, 32, 64, you must figure out the pattern of the sequence in order to calculate the nth term. In
this case, each term is multiplied by 2 to get the next term. For this sequence, therefore, the nth term is 2n.
obtuse angle An angle with a measurement greater than 90 degrees but less than 180 degrees.
order of magnitude A general measure of how large anything, especially a number, is. It’s generally used to
mean the number of powers of 10 that a given quantity contains. For example, 3.2 × 1015 is said to be six
orders of magnitude larger than 5.1 × 109, and 6.7 × 10–2 is said to be of the same order of magnitude
as 4.4 × 10–2.
order of operations Rules of sequence to be followed when simplifying mathematical expressions. When
you are solving an algebra equation, the acronym PEMDAS indicates the correct order in which to
proceed. Numbers in parentheses should be handled first, then exponents, followed by multiplication
and division moving from left to right, then addition and subtraction from left to right.
outlier In a scatter plot, a point that lies a significant distance from a line that represents the general
direction of the other points.
parallel (||) Two lines, or rows of objects, existing in the same plane that may extend indefinitely and never
intersect are considered parallel. Planes may also be parallel.
per In mathematics, a term that means “for each,” as in, “miles per hour” and “dollars per ounce.”
percent (%) Number compared to 100; indicated by the symbol %. Example: If 10% of gumballs are purple,
then you may expect 10 out of every 100 gumballs to be purple. Also, if you examine 25 gumballs and
find that 7 of them are orange, then you divide 7 by 25 to get 0.28. The percentage is then found by
multiplying by 100. Percents may also be written as decimals.
perimeter The sum of the lengths of all sides of a closed shape.
permutation An arrangement of elements in a specific order.
perpendicular (^) When one line or plane intersects another line or plane at a right angle, it is considered
perpendicular to that line or plane.
pint (pt.) An English unit measuring volume. In the United States, one pint is equal to 16 fluid ounces.
place value The value assigned to a digit according to its place relative to the decimal point. Examples: In the
number 25.04, the 2 has a value of 20 because it is two places to the left of the decimal point, and the
4
number 4 has a value of 0.04, or 100 . In the number 278, the 2 has a value of 200 because it is three places
to the left of the decimal point.
polygon A closed figure in a plane that is made of three or more line segments.
polynomial A mathematical expression, composed of constants and variables, that may involve addition,
subtraction, multiplication, or positive integer exponents. Example: 3a + 2ab – c8 is a polynomial, but a3
+ 2b – c–8 is not.

305
8th_GRDM_22_glos.qxd:Layout 1 8/11/09 4:29 PM Page 306

–GLOSSARY–

prime number Natural number only divisible by itself and 1. Example: 3, 5, 7, 11, 13, . . .
probability A way of expressing belief that an event will occur. It is defined as a ratio of the number of ways
the event can occur and the total number of events of the same class that can occur. Example: There are
36 separate outcomes possible with the simultaneous roll of two dice, and two of those possible outcomes
2
give a total of 3. So the probability of rolling a 3 with two dice is 36 .
proportion An equation that shows two ratios are equal. Example: 4 = 12 1
48. Proportions can be used to
calculate an unknown variable. Example: 41 = 48x : by cross-multiplying, you find 4x = 48 and x = 12.
Pythagorean theorem A theorem, named after Greek mathematician Pythagoras, that states that, in a right
triangle, the sum of the squares of the two sides making up the right angle is equal to the square of the
opposite side, or hypotenuse. It is often expressed as a2 + b2 = c2, in which c is the hypotenuse.
quadratic equation An equation in one unknown in which the unknown is raised to the second power. It
has a maximum of two solutions and results in a parabola when graphed. The general form is ax2 + bx +
c = 0 where a, b, and c are constants.
quart (qt.) A unit measuring volume equal to two pints and one-fourth of a gallon.
quotient The result when one number is divided by another. Example: When 24 is divided by 4, the quotient is 6.
radicals Expressions involving the symbol, indicating the root of a number. If there is no number at the
front of the symbol, it1 represents a square root. If there is a number 1
n, then the exponent of the base is 1n.
Examples: 18 = 18 2 = square root of 18 ≈ 4.24; 5 −32 = (–32)5 = –2.
random Lacking a predetermined order. More specifically, a random survey is one in which the subjects are
chosen without being limited by some factor, such as self-selection or living in only one kind of neighbor-
hood when there are other kinds of neighborhoods in the area that is being surveyed. A random number
is one that is chosen randomly, usually by a computer nowadays.
range The difference between the largest and smallest numbers, or amounts, in a data set.
ratio (“:”or “to”) The measurable comparison of one variable to another. Example: If there are 4 boys and
5 girls in the class, the ratio of boys to girls is expressed as 4:5, or 4/5, or 54.
rational number A number that can be expressed as a fraction, and in which the denominator is not 0.
A rational decimal must either be terminal or repeating. Examples: 31 and 1.25.
ray Part of a line that extends infinitely in just one direction.
reciprocals Two numbers that, when multiplied together, equal 1. Examples: 18 × 81 = 1; 65 is the reciprocal of 65.
regular polygon A polygon in which all angles are of equal measure and all lines are of equal length.
Example: An equilateral triangle is a regular polygon.
relation Any ordered relationship between sets of information. Example: A pairing of the eye colors of the
students in a classroom with the individual students. This ordered relation is not a function because
certain eye colors will be paired with multiple students. (The opposite pairing—students paired with eye
colors—would be a function because each student would usually have only one eye color.)
right angle An angle of 90°. It is represented by a box, instead of an arc, at the vertex.
right triangle A triangle containing one right angle.
scale drawing Drawing of an object created at a specified ratio to the original size of the object. Example:
House plans are scaled drawings.
scale factor The constant that applies a scale to another number. Example: For similar polygons with corresponding
sides 9 and 6, the scale factor is 23; for a scale drawing where 1 cm represents
1
1 km, the scale factor 10 .
5

scalene triangle A triangle in which all three sides are of different lengths.
scatter plot A plotting of data on a coordinate graph. Scatter plots are a graphic way of illustrating to what
degree one variable depends on another.

306
8th_GRDM_22_glos.qxd:Layout 1 8/11/09 4:29 PM Page 307

–GLOSSARY–

scientific notation A method of writing numbers as the product of a number between 1 and 10 and a power
of 10. It’s especially useful in representing very large or very small numbers. Examples: 7,000,000,000
could be written 7 × 109; and .0000085 could be written 8.5 × 10–6.
sequence An ordered arrangement of objects or events. The order is important, and a pattern can be figured
out from the order of the sequence. Example: 1, 4, 9, 16, 25 (these are the square numbers), and 1, 1, 2, 3, 5,
8, 13 (the Fibonacci sequence, in which the next term is derived by adding together the previous two terms).
significant digits Digits in a number that help the user of the number to judge the number’s accuracy.
Significant digits do not include leading or trailing zeros, unless the trailing zeros appear after the
decimal point. Examples: In 0.0004, there is only one significant digit, the 4. In 5,600, there are two
significant digits, the 5 and the 6. In 43.790, there are five significant digits, because the trailing zero
follows the decimal point and indicates accuracy to three decimal places.
similarity (~) A figure that is the same shape, but perhaps a different size, from another figure is considered
similar to that figure. For two similar figures, all sides have the same ratio.
simplest terms (of a fraction) A fraction is in simplest terms if the greatest common factor of the numera-
tor and denominator is 1.
slope The measure of a line’s steepness, often described as the ratio of “rise to run.” In the coordinate graph
y −y
system, the slope of a line between two points (x1,y1) and (x2,y2) is equal to x − x .
2 1

2 1

square units (2) Way of expressing measurements of area. Example: A rectangle of height 5 cm and width 2
cm has an area of 10 cm2 (5 cm × 2 cm = 10 cm2).
stem-and-leaf plot A way of arranging numbers in a data set so that part of the number (say, the 10’s and
100’s places in three-digit numbers) are in the left-hand column of a table and the 1’s place of the
number is in the right-hand column. Example, with numbers ranging from 121 to 139:

12 1, 1, 2, 2, 2, 3, 5, 7, 7, 7, 8

13 0, 1, 1, 1, 1, 2, 3, 3, 4, 5, 9

system of equations One or more equations in two or more unknowns that involve the same variables. A
solution to the system is one in which the values of the variables satisfy all the equations.
term A constant or a constant multiplied by a variable.
triangle (D) A three-sided polygon.
unknown A value represented by a variable in an equation that can be discovered by solving that equation.
Example: 2x + 8 = 20. Solving the equation gives the value for the unknown, x; in this case x = 6.
variable A symbol, usually a letter of an alphabet, that is capable of taking on any one of a defined set of
values.
vertex A point that describes the intersection of lines of the corners of polygons.

307
8th_GRDM_22_glos.qxd:Layout 1 8/11/09 4:29 PM Page 308

– NOTES –
8th_GRDM_22_glos.qxd:Layout 1 8/11/09 4:29 PM Page 309

– NOTES –
8th_GRDM_22_glos.qxd:Layout 1 8/11/09 4:29 PM Page 310

– NOTES –

Vous aimerez peut-être aussi